EXAM 4

Pataasin ang iyong marka sa homework at exams ngayon gamit ang Quizwiz!

down syndrome

Trisomy 21 genetic disorder

dissatisfaction with communicated illness

1. disrespectful attitude 2. breaking bad news in insensitive manner 3. withholding information 4. changing a treatment coarse without preparing the child and family

What is ankylosis?

Joint immobility.

chronic and complex diaseases difference from special needs kids

-complex, high level skills required for daily health -continous nature -potential volatility of the condition

Iron Deficiency Anemia assessment

1. Pallor 2. Weakness and fatigue 3. Low hemoglobin and hematocrit levels 4. Red blood cells that are microcytic and hypochromic

Which total of the Glasgow Coma Scale (GCS) is appropriate for the patient that is opening the eyes only in response to pain, is in abnormal flexion motor posture, and is uttering incomprehensive words? Record your answer using a whole number.

7 Rationale As per GCS, the score for ocular response is 2, the score of motor response is 3, and the score for verbal response is 2. The total GCS score is 2 + 3 + 2 = 7.

ecchymosis

Escape of blood into the tissues:

Complicated fracture

Fracture in which bone fragments cause damage to surrounding organs or tissues:

components is traction

traction (forward force) countertraction (backward force) friction

Skeletal limb deficiency

- Amelia = no limb at all - Meromelia = partial limb - Phocomelia = seal limbs - Usually congenital - Prosthesis should begin at 3-6 months to ensure natural progression and replaced as child grows

Cobb's Method of Scoliosis Evaluation

Landmarks: locate the superior and inferior extremes of scoliosis, draw a parallel line through the superior end plate of the extreme superior vertebra and through the inferior end plate of the extreme inferior vertebra. Intersecting perpendicular lines are drawn and the angle is measured. This is the preferred method of scoliosis evaluation

Diagnostic evaluation is important for early recognition of scoliosis. Which of the following is the correct procedure for the school nurse conducting this examination? A. View the child who is standing and walking fully clothed, to look for uneven hanging of clothing B. View all children from the left and right side to look mainly for asymmetry of the hip height C. Completely undress all children before examination D. View the child, who is wearing underpants, from behind when the child bends forward at the hips

D. View the child, who is wearing underpants, from behind when the child bends forward at the hips

A child falls from a two-story building and presents to the emergency department appearing drowsy. Which assessment should be performed by the nurse first? Glasgow coma scale Test deep tendon reflexes Assess gait, balance, and coordination Assess overall muscle strength and tone

Glasgow coma scale The Glasgow coma scale should be used to assess a child's level of consciousness and indicate next steps for evaluation and treatment.

satisfaction with health care

HCP available demonstrate competence engage child and parents in care decision making

The thirteen-month-old child had prenatal microsurgery for a myelomeningocele. Which assessment finding indicates the surgery was not completely successful? Increased head circumference Child is unable to walk or stand up A decreased ability to pick up objects Child is only saying sounds like "bah bah"

Increased head circumference Increased head circumference can indicate hydrocephalus which results from the myelomeningocele complications, meaning the surgery was not completely successful in repairing the neural tube defect.

kidney stones

That + urinary stasis - Prevention: hydration, mobilizing as much as possible, assessing urine output, treating UTIs promptly

True/False The application of technical aids makes it possible for older persons with CP to eventually function in their own apartments and can be extended into the workplace

True

denial for a child

allows to maintain hope

Corticosteroid administration in muscular dystrophy has been shown to do all of the following except: a. increase muscle bulk and muscle power b. improve pulmonary function c. prolong ambulation d. increase incidence of scoliosis

d. increase incidence of scoliosis

neuropathies

disease or degeneration of the peripheral nerves gullain-barre syndrome

Balanced suspension traction

doesnt exerts any traction directly on a body part

granulocytes produced in bone marrow app 6-8 hrs HL

neutrophils basophils eosinophils

Which nursing interventions should be included in the plan of care while Madison is in traction?

-assess toes for capillary refill and edema -ensure that the amount of weight remain consistent

Ataxic CP

Ataxic Cerebral Palsy - problems with balance and coordination. rapid repetitive movements; wide gait, unable to hold onto objects

Down Syndrome (Trisomy 21) physical problems

cause unknown congenital heart malformation (septal defect) respiratory tract infections hypotonicity of chest and abdominal muscles dysfunction of the immune system thyroid dysfunction ( especially hypothyrodism) increased incidence of leukemia

Ortolani maneuver

check hips for congenital dislocation, done until 1 yr old, should be smooth with no sounds , abnormal= feels like a clunk as head of femur pops back into place- positive ortolani sign

Which nursing intervention should be included in the plan of care to prevent this complication?

cleanse around the pin site with half-strength hydrogen peroxide

In response to the child with chronic illness or disability, parents may be affected by

feelings of inadequacy and failure; excessive demands on their time, energy, and financial resources; and strain on the marital relationship.

clinical manifestations of bacterial meningitis/neonates/non-specific signs

hypothermia or fever jaundice irritability drowsiness seizures respiratory irregularities or apnea cyanosis weight loss

Types of traction

manual, skin, skeletal

orthoses

must fit each body curvature bony prominences re observed for pressure or irritation and are padded

Which other task should be delegated to the LPN at this time?

spend time with Madison to distract her from the discomfort

migrane with aura

aura (may be visible) hemiparetic ( tingling and numbness of lips, lower face) aura less than 1 hr, followed by throbbing unilateral or bilateral headache coupled with nausea, vomiting, pthotophobia, phonophobia

Immobility effects on the skin

- Skin breakdown R/T moisture, friction, shear, and pressure, reduced circulation healing capacity is diminished - mechanical appliances, friction during turning, poor circulation, negative nitrogen balance, anemia, due to difficulty performing hygiene

How can nurses assist in injury prevention?

- Take advantage of opportunities to talk to parents and children about risky activities and behavior and how to follow safety guidelines to prevent injury

Siblings q

1) may simultaneously feel guilt, anger, or jealousy toward their ill sibling. 2) at risk for negative psychological effects 3) encourage parents

mixed CP

4. Mixed Cerebral Palsy - symptoms of more than one type of CP.

The primary diagnostic tool used in developmental dysplasia of the hip (DDH) in a newborn is: A. A radiograph B. An ultrasound C. MRI - magnetic resonance imaging D. the Barlow and Ortolani maneuvers

D. The Barlow and Ortolani maneuvers

Buck extension traction

Lower extremity, legs extended, no hip flexion

Which granulocyte would be most active during an acute bacterial pneumonia? Basophils Neutrophil Eosinophil Macrophages

Neutrophil Neutrophils are the primary defense against bacterial infections. Neutrophils are an important part of the innate immune system and will be most active during an acute bacterial infection.

Sports R/T highest injury rates

Tackle football, basketball, hockey, soccer, and cheerleading

Knee-ankle-foot orthosis (KAFO)

Used to prevent buckling of the knee, to support the extremity when there is paralysis or marked weakness of the knee extension or quadriceps muscle:

Spina bifida cystica

Visible defect with external saclike protrusion Two types: Meningocele Myelomeningocele

consciousness

ability to respond to sensori stimuli and have subjective experiences. has 2 aspects: 1. alertness -narousal waking state and ability to repond to stimuli 2. cognition - ability to process stimuli and produce verbal and motor responses

Bacterial Meningitis

acute inflammation of the meninges and CSF most common in children under 1 yo secondary 16-23 yo

hydrocephalus decription

an imbalance of CSF absorption or production caused y malformations, tumors, hemorrhage, infections, orr trauma results in head enlargement and increased ICP

while playing Madison throws some toys on the floor and scolds her imaginary friend for that. Her mother says that Madison cannot play if she will be throwing toys How should the nurse respond to this situation?

support the mother's decision to hold Madison accountable for her own misbehavior

meningococcal meningitis tramsmission

the only type redily transmitted by droplet infection from nasopharengyal secretions has potential to occur in outbreaks

central nervous system

the part of the nervous system that is composed of two cerebral hemispheres, the brain stem, the cerebellum, and the spinal cord

For which problem should Madison's parents be instructed to contact the healthcare provider after dischardge from the hospital ( compound fracture)

warm spots are felt on the cast

SIADH

scant urine, hyponatremia, hypoosmalaality, overhydration manifestations

Lordosis

- Inward curve of the lumbar and cervical spine

Hemiplegia

Most common form of spastic CP; motor deficit greater in upper extremity; one side of the body affected

Prosthetics

The fabrication and fitting of artificial limbs:

Which traction is a type of skeletal traction?

90-90 Femoral

divelopmental disability

any significant lag or delay in a child's physical, cognitive, behavioral, emotional or social development when compaired against developmental norms

Continuous pressure on the radial nerve may result in: a. footdrop b. wristdrop

b. wristdrop

Comprehensive care involves

building parent-professional partnerships that can support a family's adaptation to the many changes that may be necessary in everyday life, defining expectations of and for the child, and providing a long-term perspective

Trough crutches

Allows the weight to be assumed by the elbow:

Buckle, or torus, fracture

Appears as a raising or bulging at the site of the fracture:

Classification neuromuscular dysfunction

Diseases of anterior horn cells Neuropathies Neuromuscular junction disease Diseases of muscles

What about meningitis? What is meningitis?

Is it viral, bacterial, or can be either one? Either one. What is the definitive diagnosis for meningitis? Lumbar puncture.

Diplegia

Type of spastic CP; all extremities affected- lower more than upper

What is a VP shunt?

Ventriculoperitoneal shunt (pages 1163-1164) Drains excess CSF/fluid from ventricles into peritoneum. What are the biggest complications with a VP shunt? Infection and malfunction. Those are the big things. It can also you can have hemorrhage with it...if the tube migrates you can have hemorrhage with it.

diagnostic evaluation

lumbar puncture - definitive fluid pressure is measured ( elevated) - difficult if child is crying so give medazolam, fentanyl samples for culture (causative organism) gram stain blood cell count determination of glucose (reduced) and proteine levels CT scan elevated WBC blood culture nose and throat cultures

The nurse teaches a patient about brain structure and function. Which statement by the nurse is true regarding the child's brain anatomy and physiology? "Cerebral spinal fluid reduces injury to the brain in the case of a fall." "The subarachnoid space maintains a constant pressure of the cerebral spinal fluid." "The epidural space contains a blood brain barrier to reduce pain signals that reach the brain." "The blood brain barrier (BBB) prevents harmful medications from entering the brain."

"Cerebral spinal fluid reduces injury to the brain in the case of a fall." One of the functions of CSF is to reduce trauma to the brain by providing a cushion.

Nursing care directed toward non-surgical management in a teenager with scoliosis primarily includes: A. Promoting self-esteem and positive body image B. Preventing immobility C. Promoting adequate nutrition D. Preventing infection

A. Promoting self-esteem and positive body image

Treatment of vWD

Infusion of von Willebrand protein concentrate DDAVP infusion before surgery, during menses, or to treat bleeding episode

So how do we feed these kids then until they have surgery?myelomeningocele

Turn their head to the side. Yeah, holding these kids is very difficult because of that sac and we don't want to damage it so we feed them with their head turned to the side. When do they get surgery? How quickly do they get it done? Within 72 hours of life unless they are leaking fluid. If they are leaking fluid it is done as soon as possible - within the first 24 hours.

transverse fracture

crosswise at right angles to the long axis of the bone

Idiopathic Thrombocytopenic Purpura (ITP)

An acquired hemorrhagic disorder characterized by Thrombocytopenia—excessive destruction of platelets - easy bruising, mucosal bleeding, petechaie, menorrhagia) Purpura—discoloration caused by petechiae beneath the skin Normal bone marrow with usual increase in large immature platelets Know this Refering to platlets 50,000 below referred to thrombocy so give platelet 20,000 below referred to life threating and platelet transfusing so give So no rectal temps Im injections: hold for the bleeding issue Fall precautions

Spastic CP

The most common clinical type of CP; physical signs of increased stretch reflexes, hypertonicity, poor control of posture, balance, and coordinated motion; impairment of fine and gross motor skills

nursing care management of the immobilized child

1. assessment - injured part and functioning systems 2. encourage the child to be as active as possible or provide passive exercise 3 pressure reduction matress 4. frequent position chnages 5. DVT prevention - change position, dorsiflex feet, rotate ancle, sit in bedside chair, ambulate, use antiembolic socks 6.possible anticoagulant therapy 7. transporting child outside the room, visitors are good, recreational planning 8 asap wear street clothing and resume school allow parent or siblings stay overnight, minimize family disruption over hospitalization 9. help with participation in self-care - the most useful - select diet, clothing, etc

assessment of factors affecting family adjustment

1. available support system (status of mariatl relationships, alternate support system, ability to communicate) 2. perception of the illness or disability (previous knowledge of disorder, imagined cause of disorder, effects of illness or disability on family) 3. coping mechanisms (reaction to previous crises, reactions to the child, childbearing practices, influence of religion, attitudes) 4. available resources 5. concurrent stresses

Immobilization causes what effect on metabolism? a. Hypocalcemia b. Decreased metabolic rate c. Positive nitrogen balance d. Increased levels of stress hormones

ANS: B Immobilization causes a decreased metabolic rate with slowing of all systems and a decreased food intake. Immobilization leads to hypercalcemia and a negative nitrogen balance secondary to muscle atrophy. Decreased production of stress hormones occurs with decreased physical and emotional coping capacity.

MCV

average volume or size of a single RBC

The method of fracture reduction is not determined by: a. The child's age b. The manner in which the fracture occurred c. The degree of diplacement d. The amount of edema

b. The manner in which the fracture occurred

chronic sorrow

feelings of sorrow and loss that recur in waves over time

arachnoid tuberculae

fibrious fillaments that provide protection by helping to anchor the brain

spondylolisthesis

forward slipping of one vertebra over another

Chiari malformation

part of cerbellum herniates down through foramen magnum

Simple, or closed fracture

Fracture that has not produced a break in the skin:

diagnostic procedures of unconscious child

lab - for cause of unconsciousness EEG - potential for seizures, to determine brain death lumbar puncture - spinal fluid pressure to rule out subdural effusion auditory and visual CT, MRI - lesions, tumors, hematoma subdural tap to rule out subdural effusion ventricular puncture - to releive pressure ECG - intracranial lesions RTUS ( real time ultrasouography) - high resolution anatomic visualization radiography - fractures, dislication PET - blood volume and flow in the brain, metabolic activity DSA - vascular abnormalities SPECT - blood flow functionng

Types of Hemophilia B

Hemophilia B Also known as Christmas disease Caused by deficiency of factor IX Accounts for 15% of cases of hemophilia

Which medical conditions may affect the patient's ability to produce red blood cells? Select all that apply. Lung cancer Osteomyelitis Fibromyalgia Osteosarcoma Chronic renal failure

Osteomyelitis Red blood cells are produced in the bone marrow. Osteomyelitis can lead to a decreased production of red blood cells. Osteosarcoma Red blood cells are produced in the bone marrow. Osteosarcoma can lead to a decreased production of red blood cells. Chronic renal failure Erythropoietin is produced in the kidneys. Kidney failure can decrease the production of the hormone, and lead to decreased ability to produce red blood cells.

The pediatric patient with a moderate brain injury demonstrates decreased ability to walk normally. Which finding indicates to the nurse a severe and progressive change in intracranial pressure (ICP)? Patient often lies in the fetal position. Upper extremities are drawn in toward chest. Feet exhibit decreased sensation when touched. Patient is unable to eat without experiencing GI upset.

Upper extremities are drawn in toward chest. Flexion and decorticate posturing indicate increased ICP.

postmeningitic complications in neonates

ventriculitis which results in cystic, walled-off areas of the brain with fluid accumulation and pressure permanent impairment of vision or hearing and other nerve palsies ( especially deafness) cerebral palsy cognitive impairment learning disorders attention-dificit/hyperactivity disorder seizures

The priority nursing action for Johnny (in item 30) is to: a. elevate the affected extremity. b. administer oxygen. c. administer pain medication. d. start an IV infusion of heparin.

b. administer oxygen.

Which of the following would be expected in the infant with hypotonia? a. When held in horizontal suspension, the infant responds by slightly rasing the head b. When pulled to the sitting position, the infant demonstrates head lag that is quickly corrected to a normal position c. When placed in horizontal suspension position, the infant's head droops over the examiner's supporting hand and the infant's extremities hang loosely d. The infant has a slower weight gain but a good sucking reflex

c. When placed in horizontal suspension position, the infant's head droops over the examiner's supporting hand and the infant's extremities hang loosely

detect early signs of cardiomyopathy

weight loss nausea vomiting cough increased fatigue on performingADL orthopnea

parental empowerment

1) recognizing, promoting, and enhancing competence. 2) Advocating for the child and developing parent-professional partnerships are part of taking charge.

nursing goal

assess whivh families are at risk for succumbing to the effects of the crisis

Immobility effects on the neurosensory system

- Loss of innervation and sensory and perceptual deprivation R/T pressure results in things like foot drop and wrist drop peripheral nerves fo not degenerate with disuse, but loss of innervation happens if nerves are damaged by pressure or if blood supply to them disrupted need passive or active range of motion to stimulate circulation and keep nerves from becoming pinched

A child is brought to the emergency department for altered mental status. Which would be the most concerning assessment finding? The child reacts angrily to being awakened. The child does not recall the trip to the hospital. The child believes the triage room is the bedroom. Parents state that the child does not recognize them.

Parents state that the child does not recognize them. A child who does not recognize the parents is classified a disoriented level of consciousness. More serious than confusion and delirium, disorientation indicates that the child is suffering from a more severe decrease in level of consciousness, and this would be the most concerning assessment.

When do I want to start antibiotics on bacterial [meningitis]?

Now. The sooner the start, the better the outcome. Doctor may tell you to start empirics before labs are back.

Match each component of the musculoskeletal system with the connecting tissue that helps it to function.

Tendons Connect muscle to bone Ligaments Connect bone to bone Skeletal muscles Contractile structure Articular cartilage Shock absorbing structure

intellectual disability

(formerly referred to as mental retardation)

cervical traction

-halo brace or halo vest: how most cervical traction is accomplished -Gardner-Wells tongs -inserted through burr holes in the skull with weights attached to the hyperextended head -as neck muscles fatigue, vertebral bodies gradually separate so the spinal cord is no longer pinched between the vertebrae

Hydrocephalus Diagnosis

-measure head - CT, MRI -Ultrasonography can determine size of ventricles

brain dead children

1. coma and apnea 2. brainstem function absent 3. fully dilated pupils, do not respond to light 4. absence of spontaneous eye movements 5. absence of cough, gag, sucking, rooting reflexes 6. absence of repiratory movements 7. not signoficatly hypothermic or hyperthermic 8. flaccid tone and absence of spontaneous movements

home care goals

1. normalize the life of a child, including those with technologically complex care, in a family and community context and setting 2. minimize the disruptive impact of the child's condition of the family 3. foster the child's maximum growth and development

nursing care management/ bacterial meningitis

1. precautions to protect themselves and others from the infection ( washing hands) 2. keep room as quiet as possible and environmental stimuli at a minimum ( because they are sensetive to noise, light and other external stimuli) 3. limit number of visitors 4. use pillows between knees and along side to help with nuchal regidity 5. avoid lifting child's head 6. evaluate pain 7. ensure safety - fall prevention 8. observe - vitals, neurological signs, LOC, urinary output 9. frequent assessment of fontanels 10. fluids and nourishment 11 most challenging - adequate antimicrobial therapy (IV)

When does rapid growth in the fetal life occur

15-20 weeks 30-birth How is brain growth reflected head circumference When does complete ossification of the skull bones occur? 2 years

Calculate the cerebral perfusion pressure (CPP) if the mean arterial pressure (MAP) is 100 mm Hg and intracranial pressure (ICP) is 10 mm Hg. Record your answer using a whole number.

90 Rationale The formula to calculate the cerebral perfusion pressure (CPP) is: CPP = MAP - ICP. Therefore, CPP = 100 - 10 = 90 mm Hg.

The nurse is preparing to admit a 7-year-old child with an upper motor neuron syndrome. What clinical manifestations of an upper motor neuron syndrome should the nurse expect to observe? (Select all that apply.) a. No flexor spasms b. Babinski reflex present c. No wasting of muscle mass d. Marked atrophy of atonic muscle e. Hyperreflexia with tendon reflexes exaggerated

ANS: B, C, E Clinical manifestations of an upper motor neuron syndrome include Babinski reflex present, no wasting of muscle mass, and hyperreflexia with tendon reflexes exaggerated. No flexor spasms and marked atrophy of atonic muscle are manifestations of a lower motor neuron syndrome.

The nurse cares for a child with rapidly progressing paralysis due to Guillain-Barré syndrome (GBS). After supportive care fails, the nurse anticipates what treatment as the next step in this patient's care? Consult palliative care Administer corticosteroids Teach about plasmapheresis Administer IV immunoglobulin

Administer IV immunoglobulin A child with rapidly progressing paralysis due to Guillain-Barré syndrome should be given high-dose IVIG. The IV immunoglobulin interferes with autoantibodies causing GBS and helps restore function in the patient.

Iron Rich foods

Breads and cereals dark green, leafy vegetables dried fruits egg yolks iron enriched infant formula and cereal kidney beans legymes liver meats molasses nuts potatoes prune juice raisins seeds shellfish tofu whole grains

Which is the main action involved in the movement of muscle? Spasticity Relaxation Contraction Articulation

Contraction Movement of the skeletal muscles are produced by contraction of the elongated fibers.

urinary output specific gravity serum sodium

DI: increased SIADH: decreased DI: decreased SIADH: increased DI: increased SIADH: decreased

contusion

Damage the soft tissue subcutaneous structures and muscles

Match the clinical manifestations of trisomy 21 with its description.

Disproportionate shortness of the head Brachycephaly Singular transverse creases across the palm and the sole of the foot Palmar crease Top of the pinna falls below the outer canthal line and is considered "Low set" ears

List the most common side effects of medications (baclofen, dantrolene sodium, and diazepam) administered to children with CP to decrease overall spasticity

Drowsiness, fatigue, and muscle weakness.

Managing Hemarthrosis

During bleeding episodes, elevate and immobilize the joint Ice Analgesics ROM after bleeding stops to prevent contractures PT Avoid obesity to minimize joint stress Know this Don't force that range of motion

Manifestations vWillebrand

Easy bruising Epistaxis Gingival bleeding, nose bleeding from mucous membranes Excessive bleeding with lacerations or surgeries Menorrhagia exsessive period risk of post labor hemorrhage

Which fractures are likely for a child to be hospitalized?

Femur and distal supracondylar area of the distal humerus

Complete fracture

Fracture in which fracture fragments are separated:

Comminuted fracture

Fracture in which small fragments of bone are broken from the fractured shaft and lie in surrounding tissue:

Anencephaly

Most serious NTD Absence of cerebral hemispheres Portion of brainstem function may be intact Incompatible with life Few hours to few weeks Referral to neonatal palliative care ASAP Death due to respiratory failure ? Organ donation

A school-age child is admitted with osteomyelitis. What assessments would the nurse expect to perform if erythrocyte sedimentation rate (ESR) and C-reactive protein (CRP) levels are elevated? Vital sign assessment Patient allergy history Evaluation of patient dressing Patient history of recent falls

Patient allergy history The given laboratory results, along with patient symptoms, indicate infection, which will require IV antibiotics. Prior to antibiotic administration, the nurse should take a patient history to evaluate allergies and any problems the child experienced during previous antibiotic administration.

Hip-knee-ankle-foot orthosis (HKAFO)

Provides support for the knee, ankle, and hip; used for flail lower limb and paralysis:

Orthotics

The fabrication and fitting of braces:

Classification and Diagnosis of neuromuscular dysfunction

Upper motor neuron lesions Weakness/spasticity Increased DTRs and abnormal superficial reflexes Primarily cerebral palsy Lower motor neuron lesions Weakness, atrophy of skeletal muscles, hypotonia Usually symmetric Gradual or sudden onset indicates causation Cerebral palsy number 1 neuro disorder

Ankle-foot orthosis (AFO)

Used to prevent foot drop due to bed rest, trauma to the foot, or paralysis of muscles that flex the foot:

subarachnoid space

located between the pia mater and the arachnoid membrane; filled with CSF, which acts as a protective cushion for the brain tissue

agranulocytes

monocytes lymphocytes

headache

primary - migraine, tension-type headache, trigeminal autonomic sephalalgias secondary caused from another condition - brain tumor, brain infection, withdrawal, hunger, medical overuse etc

basila-type migrane

recurrent attacks headache typically occipital symptoms may include dysarthria, verttigo, diplopia, vomiting, altered consciousness

When assessing the family's adjustment to a child's chronic illness, disability, or death, the following should be noted:

the availability of a support system, the family's perception of the event, their coping mechanisms, their concurrent stressors, and their response to the child.

meninges

the membrnes that cover and protect the brain; the dura mater, arachnoid membranes; and pia mater

parents support

the need extra support on various milstones, because their kid is a little different with CP The parents need extra support. There are certain milestones in the child's development that they hit that hit the parents extra hard. Can any of you guys think what those might be? : Time of diagnosis, exacerbation of the child's physical symptoms, crises that increase parental care, require medical intervention or hospitalization, start of school, beginning to walk (major milestones). Those first big milestones in a kid's life hits the parents really hard and cause depression and they need a little extra support.

The condition recognized in the infant with limited neck motion, in which the neck is flexed and turned to the affected side as a result of shortening of the sternocleidomastoid muscle, is: a. b. brachial plexus palsy c. lordosis d. kyphosis

torticollis

water and electrolytes for smortsmen

water loss - perspiration, urination, evaporation from the perspiratory tract water recommneded 5-8 oz every 15-20 min best water - cold, los osmolality, large volume no carbonated drinks no salt needed and sports drinks are not good weight loss shouldn't be more than 1.5% of total body weight ( 1-2 lb a weeek) optimum diet - all essential food groups and adjusted to energy requirements

nonreduceable contractures

wedge casting surgical reduction

Hemophilia transmission

x-linked ressesive disorder 60% - gene mutation most frequent pattern - unaffected male, trait carrier female

Iron Deficiency anemia Description

1. Iron stores are depleted, resulting in a decreased supply of iron for the manufacture of hemoglobin in red blood cells. 2. Commonly results from blood loss, increased metabolic demand, syndromes of gastrointestinal malabsorption, and dietary inadequacy.

Most common competitive sports-related injury?

Ankle

What is spina Bifida? What exactly does that entail?

Spina two types: SB Occulta (defect not visible externally) and SB Cystica (visible defect with external saclike protrusion. SB Cystica has two forms: I. Meningocele -which encases meninges and spinal fluid but no nerves. Neurologic deficit not associated with meningocele. II. Myelomeningocele which contains meninges, spinal fluid, and nerves. Varying, often serious neurologic deficits seen with myelomeningocele.

The nurse is caring for a child who has a loss of respiratory muscle strength and who is unable to cough. Which nursing intervention does the nurse perform to help promote airway clearance? Ask the child to suppress the cough. Restrict fluid intake for the child. Splint the chest while the child is coughing. Administer antibiotic drugs to the child.

Splint the chest while the child is coughing. Rationale In case of respiratory muscle weakness, there is difficulty in coughing. The nurse should support the child's chest by splinting with a pillow so that it is easier to cough. Coughing is a defense mechanism of the body that removes foreign irritants from the respiratory tract. Thus the nurse should encourage the child to cough. The child should be provided adequate fluids to prevent thickening of chest secretions. Antibiotic drugs should be administered only if the child shows signs and symptoms of infection.

Trends in the treatment of children with chronic illness or disability focus on

developmental age, the child's strengths and uniqueness, family-centered care, normalization, early discharge, home care, mainstreaming, and early intervention.

thomas splint

extends from the groin to midair above the foot

Curves of the spine

less than 10 degrees - postural variation 10-25 mild no treatment

nerve compression syndrome

may be at the time of injury or develop in the process reaalignment, or arise from use of immobilization apparatus can cause carpal tunnel tarsal tunnel footdrop, wristdrop check always for sensory - pinprick, motor strength symptoms - pain, discomfort, muscular weakness, burning sensation, limitation of motion, altered sensation treatment - alleviation of pressure on nerve

Waterhouse-Friderichsen syndrome

overwhelming septic shock disseminated intravascular coagulation massive bilateral adrenal hemorrhage purpura child who develops petechial or purpural rash may have meningococcemia

Which are some major conditions contributing to childhood death? Select all that apply. Cancer Asthma Trauma Cystic fibrosis Muscular dystrophy

Cancer Trauma Cystic fibrosis Muscular dystrophy Rationale Major conditions contributing to childhood death include cancer, trauma, cystic fibrosis, and muscular dystrophy. Asthma is a chronic condition but generally does not contribute directly to death for children.

peripheral nervous system

part of the nervous system that is composed of the cranial nerves that arise from or travel to the brainstem and the spinal nerves that travel to or from the spinal cord and which may be motor ( efferent) or sensory (afferent)

external fixation devices

monolateral taylor spatial frame ilizarov

resolution of crisis depends on

available support system perception of the event coping mechanisms reaction to the child available resources concurrent stresses within the family

What nursing interventions should be considered for a child who is immobilized following a femur fracture? Select all that apply. Antiembolism stockings Correct body alignment Passive range-of-motion activities Supine posture whenever possible Play activities involving all extremities

Antiembolism stockings Correct body alignment Passive range-of-motion activities Rationale Nursing interventions for the child who is immobilized include the use of antiembolism stockings, correct body alignment, and passive range-of-motion activities. The child should be placed in an upright, rather than supine, position when possible. Play activities should be planned to use the child's uninvolved extremities rather than the involved extremity.

What characterizes an infant's concept of death? (Select all that apply.) a. Death is seen as temporary. b. Death is seen as a departure, a kind of sleep. c. Death has no significance before 6 months of age. d. They believe that death is a consequence of their thoughts. e. Anxiety is not created by death but by loss, even temporary, of the parent.

ANS: C, E Infants have no concept of death before six months and anxiety is not created by death but by loss, even temporary, of the parent. Death seen as temporary, a departure, or a belief that death is a consequence of thoughts are characteristic of a preschool child's concepts of death.

When communicating with dying children, what should the nurse remember? a. Adolescent children tend to be concrete thinkers. b. Games, art, and play provide a good means of expression. c. When children can recite facts, they understand the implications of those facts. d. If children's questions direct the conversation, the assessment will be incomplete.

ANS: B Games, art, and play provide children a way to use their natural expressive means to stimulate dialogue. Adolescent children are abstract thinkers. Children may not understand the implication of facts just because they can recite them. The assessment is more complete when children's questions direct the conversation.

Cerebral Perfusion Pressure (CPP)

CPP = MAP - ICP

The healthy sister of a 9-year-old male with cystic fibrosis approaches the nurse and states, "I feel so guilty my brother is sick and I'm not." Which response by the nurse is appropriate? "Could you tell me a little more about how you are feeling?" "Feeling guilty is normal for someone your age, it will go away." "I know you feel bad, but you should be thankful that you aren't sick." "It's okay to feel guilty, you are able to do things your brother can't."

"Could you tell me a little more about how you are feeling?" This response provides the sibling with the ability to express feelings to the nurse and work through the guilt being experienced. This will assist the sibling in resolving the feelings and not carrying them forward into the adult years.

Septic arthritis

- A bacterial infection in the joint, usually spread from hematogenous (blood) or adjacent cellulitis or osteomyelitis - Usually S. aureus - Usually elbow, hips, knees, ankles - Sx: Severe joint pain, swelling, warmth, erythema, resistance to moving the joint - Dx: Aspiration of joint fluid for culture, MRI, CT - Hip septic arthritis is 911 because we don't want compromised blood supply to the head of the femur - IV antibiotics, pain management, PT if the kid was immobilized in a cast or traction

siblings of "different" brother/sister

- guilt, anger, jelousy toward ill sibling - secondary losses - ability to participate in extracirriculum activities

As the nurse assumes care for the client, which actions are most important for the nurse to take? (select all that apply)

-stabilize the injury -assess neurovascular status every hour -place an ice pack over the injury -elevate the affected extremity

What should the nurse do during assessment of a kid's fracture?

1. Calm the parents and child 2. As long as there is no hemorrhaging, immobilize the limb with a splint if it is not already 3. Ask everyone what happened 4. Have the child point to the area that hurts 5. Have the child wiggle their toes or fingers that are distal to the injury 6. Explain everything to the child before you do it to keep them calm

The nurse receives an intershift report on four assigned patients with iron deficiency anemia (IDA). After a review of each patient's history, combined with the shift report information, which patient should the nurse see first? A 5-year-old with headache and fatigue A 16-year-old who menstruates for an average of 5 days each month A 3-month-old with an intestinal malformation and slight pallor An 11-year-old with joint pain who has grown 4 inches in the past 6 months

A 3-month-old with an intestinal malformation and slight pallor Premature infants and those with gastrointestinal impairment are at increased risk for IDA due to the prevalence of immature red blood cells and the inability to absorb iron. Pallor indicates inadequate perfusion and must be further assessed and treated.

The clinic nurse is assessing infant reflexes. What assessment indicates a persistence of primitive reflexes? a. Tonic neck reflex at 8 months of age b. Palmar grasp at 4 months of age c. Plantar grasp at 9 months of age d. Rooting reflex at 3 months of age

ANS: A Persistence of primitive reflexes is one of the earliest clues to CP (e.g., obligatory tonic neck reflex at any age or nonobligatory persistence beyond 6 months of age and the persistence or even hyperactivity of the Moro, plantar, and palmar grasp reflexes). The palmar grasp disappears by 6 months, the plantar grasp disappears by 12 months, and the rooting reflex disappears at 4 months, so these are normal findings.

The nurse is caring for a 14-year-old child with systemic lupus erythematous (SLE). What clinical manifestations should the nurse expect to observe? (Select all that apply.) a. Arthralgia b. Weight gain c. Polycythemia d. Abdominal pain e. Glomerulonephritis

ANS: A, D, E Clinical manifestations of SLE include arthralgia, abdominal pain, and glomerulonephritis. Weight loss, not gain, and anemia, not polycythemia, are manifestations of SLE.

When taking the history of a child hospitalized with Reye syndrome, the nurse should not be surprised if a week ago the child had recovered from what? a. Measles b. Influenza c. Meningitis d. Hepatitis

ANS: B The etiology of Reye syndrome is not well understood, but most cases follow a common viral illness, typically influenza or varicella.

A child with sickle cell anemia (SCA) develops severe chest and back pain, fever, a cough, and dyspnea. What should be the first action by the nurse? a. Administer 100% oxygen to relieve hypoxia. b. Notify the practitioner because chest syndrome is suspected. c. Infuse intravenous antibiotics as soon as cultures are obtained. d. Give ordered pain medication to relieve symptoms of pain episode.

ANS: B These are the symptoms of chest syndrome, which is a medical emergency. Notifying the practitioner is the priority action. Oxygen may be indicated; however, it does not reverse the sickling that has occurred. Antibiotics are not indicated initially. Pain medications may be required, but evaluation by the practitioner is the priority.

What explanation best describes how preschoolers react to the death of a loved one? a. Grief is acute but does not last long at this age. b. Children this age are too young to have a concept of death. c. Preschoolers may feel guilty and responsible for the death. d. They express grief in the same way that the adults in the preschoolers' life are expressing grief.

ANS: C Because of egocentricity, the preschooler may feel guilty and responsible for the death. Preschoolers may need to distance themselves from the loss. Giggling or joking and regression to earlier behaviors may help them until they incorporate the loss. The preschooler's concept of death is more a special sleep or departure.

Diagnostic Evaluation

CBC Decreased RBCs Decreased Hgb and Hct Transfuse at 8 & 28 Other tests for particular type of anemia Routine Screening for pediatric patients across childhood (U.S. recommendations) Know these number 8 and 28 is when a transfusion is needed MCV-size MCH-color

Ataxic CP

Characterized by wide-based gait; rapid, repetitive movements performed poorly; disintegration of movements of the upper extremities when the child reaches for objects

If a staff member is unable to reach Madison's parent, what guidelines will determine the staff's ability to provide needed care?

Emergency care may be provided after a reasonable attempt to reach the parents has been made.

What causes the facial deformities in children with untreated or inadequately treated beta-thalassemia? Profound tissue hypoxia Abnormal clot formation Extramedullary marrow expansion Decreased red blood cell production

Extramedullary marrow expansion The facial deformities develop from extramedullary marrow expansion, a consequence of the marrow's effort to keep up with the demand for red blood cells as a result of anemia.

The function of the RBC is to carry oxygen to the tissues. Which factor is essential for the RBC to complete this task? Glycogen Leukocytes Hemoglobin Erythropoietin

Hemoglobin Hemoglobin is essential to ensure oxygen is carried to the body tissues by red blood cells. Oxygen binds to the iron found within the hemoglobin, which helps to facilitate the delivery of oxygen.

What is the issue with physeal damage (physis, growth plate)?

May result in unequal lengths of the bones, esp legs or angular deformity

The nurse is preparing the long-term care plan for a child with CP. Which of the following is included in the plan? a. No delay in gross motor development is expected b. The illness is not progressively degenerative c. There will be no persistence of primitive infantile reflexes d. All children will need genetic counseling as they get older before planning for a family

b. The illness is not progressively degenerative

Spinal cord injuries are classified as either complete or incomplete. In a complete spinal cord injury: a. there is loss of sensation, pain, and propioception with normal cord function, including motor function b. there is transient loss of neural function below the level of the acute spinal cord lesion, resulting in flaccid paralysis and loss of tendon autonomic and cutaneous reflex activity c. there is no motor or sensory function more than three segments below the neurologic level of injury d. there is tetraplegia, commonly with sacral sparing, with patients gaining some motor recovery

c. there is no motor or sensory function more than three segments below the neurologic level of injury

Management of the genitourinary function in the patient with myelomeningocele includes clean intermittent catheterization and anticolinergic medication. Which of the following statements about their use is correct? i. CIC is used to prevent spontaneous voiding ii. Parents are taught to catheterize the infant every 4 hours during the day and once at night iii. Anticholinergic medications enhance sphincter competence iv. Anticholinergic medications reduce detrusor muscle tone and reduce bladder pressure a. i and iii b. i and iv c. ii and iii d. ii and iv

d. ii and iv

DI often accompanies.....

intracranial trauma, increased urinary volume danger of dehydration, hypernatremia, hyperosmality

Slipped capital femoral epiphysis

refers to the spontaneous displacement of the proximal femoral epiphysis in a posterior and inferior direction. The treatment goals are to (1) prevent further slipping until physeal closure, (2) avoid further complications such as avascular necrosis, and (3) maintain adequate hip function

A subdural hemorrhage

s bleeding between the dura and the arachnoid membrane, which overlies the brain and the subarachnoid space. The hemorrhage may be from two sources (1) tearing of the veins that bridge the subdural space and (2) hemorrhage from the cortex of the brain caused by direct brain trauma. Subdural hematomas are much more common than epidural hematomas and occur most often in infancy, with a peak incidence between 0 to 4 months of age.

longitudinal fissure

separates the upper part of the two large cerebral hemispheres that occupy that anterior and medial fossae of the skull

Families' reactions to disability or chronic illness are manifested in the following stages:

shock and denial, adjustment, reintegration, and acknowledgment

overuse injuries

stress fractures bursitis tendonitis apophysitis injuries of the joint surface

nursing care of legg calve perthes

teaching the family of required care and management coping with well but must stay inactive child

von Willebrand assessment

1. Epistaxis 2. Gum bleeding 3. Easy bruising 4. excessive menstrual bleeding

What additional assessment should the nurse perform?

toe movement

why do we provide these kids with speech therapy?

again in order to make them as normal as possible muscles in mouth/tongue can be affected making speech difficult to understand problems with feeding Muscle development and motor abilities affect all muscles, including those of the mouth/face. SLP can help with feeding problems and adaptive communication techniques.

ITP clinical manifestations

asy bruising - petechiae, ecchymoses Epitaxis, bleeding gums Internal hemorrhage, hematuria, hematemesis, melena, hemarthorsis, menorrhagia, hematoma

MCHC

average concentration of hemoglobin in the RBC

SMO (supramalleolar orthosis)

bracing for the foot

diagnostic evaluation of CP

neurologic examination neuroimaging (MRI) general movements assessment metabolic and genetic testing early recognition is difficult due to lack of signs, but monitor if known etiologic factors cross extensor reflex (stimulus to the sole) and obligatory response ( head on one side) evaluate muscle spasticity, self-initiated movements

migraines in children

paroxysmal -nausea, vomiting, abdominal pain, episodic pallor, decreased activity, bifrontal, temporal or unilateral pain shorter than in adults Tx - keep diary, ibuprofen riboflavin and magnesium might help topiramate for prevention triptans (serotonin agonists) - almotriptan, zolmitritan, sumatriptan for 12-17 - rizatriptan for 6 and older

major causes of neonatal meningitis

premature rapture of fetal membranes maternal infection during the last week of pregnancy

anger directed inward

punitive behavior - neglecting self's health, verbally disregarding self

Choreiform movements

quick, jerky, grossly uncoordinated movements that may disappear on relaxation

choreiform movements

quick, jerky, grossly uncoordinated movements that may disappear on relaxation

The American Association on Intellectual and Developmental Disabilities defines intellectual disability as

significantly subaverage general intellectual functioning, existing concurrently with deficits in adaptive behavior and manifested during the developmental period.

nursing care of traction

skin observation pin sites checked and cleaned observe for infection or pin instability using return demonstration method analgesics and muscle relaxants The pin sites need to be assessed and cleaned to prevent infection. After the first 48-72 hours, cleaning can be done 1x/day or 1x/weeK

spiral fracture

slanting and circular, twisting around the bone shaft

types of fractures in children

plastic deformation buckle or torus fracture greenstick fracture complete fracture

Nursing alert re: casts

- Immediately report any of the 5 P's of ischemia: Pain, esp with ROM Pallor Pulselessness (late sign) Paresthesia Paralysis

Why are infants and toddlers predisposed to head injuries?

Because their heads are large - Also, their spleen, liver, and broad costal arch are large relative to their small bodies and so these areas are prone to injury - Their small size make them easily thrown around in a moving vehicle and their natural curiosity gets them into trouble natural curiosity

heat cramps

Heat cramps R/T sodium depletion > need fluid and electrolytes due to exercise in hot weather

diagnostic test for cerebral palsy

NEURO ASSESSMENT MRI The neurological exam and history are the primary modalities for diagnosis

5 clinical manifestations of compartment syndrome:

Pain Pulselessness Pallor Paresthesia Paralysis

MCH

average weigt of hemoglobing in each RBC

Upon delivery of an infant with myelomeningocele, which one of the following nursing actions is contraindicated? a. Examination of the membranous cyst for intactness b. Diapering the infant c. Keeping moist, sterile normal saline dressings on the defect d. Keeping the infant in the prone position

b. Diapering the infant

cerebral palsy causes

hypoxia during birth birth injury neuro / muscular disorder caused by trauma in utero or during birth some kind of trauma during birth or in utero right before birth Disorder of posture and movement from static brain injury perinatally or postnatally, which limits activity. The condition often involves disturbances of sensation, perception, communication, cognition, and behavior, secondary musculoskeletal problems and epilepsy. Most common permanent physical disability of childhood Some type of trauma at birth or in utero right before birth. There is no one specific cause

pia mater

innermost covering of the brain; delicate , transparent membrane that, unlike other coverings, adhere closely to the outer surface of the brain, conforming to the folds (gyri) and furrows (sulci)

which assessment technique will determine if the infant has a cleft palate

insert gloved finder and palpate

How is the presence of crepitus related to this femur fracture determined?

Listen for a grating sound when the affected area is moved

True/False Surgical intervention is used in the child with CP to correct contractures or spastic deformities and provide stability

True

A patient develops hives and begins wheezing immediately after being stung by a bee. Which white blood cells are active at this time? Basophils Eosinophils Neutrophils Macrophages

Eosinophils Eosinophils are most active through an allergic hypersensitivity reaction.

A child presents with signs of developmental dysplasia of the hip. Which statement shows the nurse understands the dysplasia? A. Dislocation has a femur that is partially in place but still stable. B. Subluxation has a femur that is fully displaced from the acetabulum. C. Dislocation has a partial displacement of the femur from the acetabulum. D. Subluxation has a partial displacement of the femur from the acetabulum.

Subluxation has a partial displacement of the femur from the acetabulum. Subluxation is a partial displacement of the femur from the acetabulum, whereas dislocation is a complete displacement.

migrane without aura

prodrome - pallor, alteration in personality, change in appetite or thirst - unilateral or bilateral headache couled with nausea and nausea, photophobia, phonophobia pulsating quality moderate or severe pain intensity

factors to consider when choosing protheses

level of amputation age weight activity agility skin condition

diagnostic evaluation of myelomeningocele

made on the basis of clinical manifestations and examining the meningeal sac (MRI, ultrasound, CT) - neurologic evaluation of bladder and bowel involvement

nonspecific measures for bacterial meningitis

maintaining hydration - prime concern - 1. correction of fluid deficits and electrolyre abnormalities 2. fluid restriction until normal serum sodium and no signs of increased ICP shock is managed by restoration of circulating blood volume and maintenance of electrolyte balance seizures controlled with antiepileptic drugs

Developmental dysplasia of the hip

spectrum of disorders related to abnormal development of the hip that may occur any time during fetal life, infancy, childhood -risks breech birth, female gender, fam hx, left hip, first-born child, positive clicking hips with clinical exam, high birth weight, joint laxity predisposing factors: - physiologic (intrauterine positioning - mechanical factor ( breech presentation, multiple fetuses) - genetic ( high incidence in siblings) - Prevention: Swaddle with hips and knees flexed and slightly abducted - Dx: Barlow test and Ortolani test, asymmetry of gluteal and thigh folds, positive trendelenburg sign if child is weight bearing - Tx immediately, do not delay - Newborn - 6 months: Pavlik harness - 6-24 months: Surgical closed reduction followed by spica cast for 12 w - Older child: Operative reduction radiologic examination is not reliable, only physical examination

clinical manifestations of bacterial meningitis/ children and adolescents

usually abrupt onset fever chills headache vomiting alterations in sensorium seizures (often in initial sign) irritability agitation may develop - photophobia, delirium, hallucinations, aggressive behavior, drowsiness, stupor, coma nuchal rigidity positive kernig and brudzinski signs hyperactivity but variable reflex responses signs and symptoms peculiar to individual organisms - petechial or purpuric rashes ( meningococcal infection) - joint involvement ( meningococcal or flu) - chronically draining ear ( pneumococcal meningitis)

Which clinical manifestations suggest increased intracranial pressure in an infant? Select all that apply. Nausea Drowsiness Poor feeding Tense, bulging fontanel Irritability and restlessness

Drowsiness Poor feeding Tense, bulging fontanel Irritability and restlessness Rationale Poor feeding; tense, bulging fontanel; drowsiness; and irritability and restlessness are all clinical manifestations of increased intracranial pressure in infants. Nausea is a sign of increased intracranial pressure in children, not infants. Infants cannot usually verbalize nausea, whereas an older child can.

Which symptoms, observed with a high-pitched cry and poor feeding, are early signs of increased intracranial pressure? Blurred vision, lethargy, increased sleeping, drowsiness Forceful vomiting, headache, drowsiness, increased sleeping Changes in pupil size and reactivity, decreased consciousness, bradycardia Drowsiness, increased sleeping, Macewen sign, tense and bulging fontanel

Drowsiness, increased sleeping, Macewen sign, tense and bulging fontanel Rationale Clinical manifestations of increased intracranial pressure in infants are drowsiness, increased sleeping, Macewen sign (cracked-pot sound on percussion of the skull), tense and bulging fontanel, high-pitched cry, and poor feeding. Blurred vision, lethargy, forceful vomiting, and headache are early symptoms observed in children with increased intracranial pressure. Infants are not able to verbalize that they have blurred vision or headaches. Changes in pupil size and reactivity, a decreased level of consciousness, and bradycardia are later signs of increased intracranial pressure in infants and children.

Which clinical manifestations are appropriate for Down syndrome? Select all that apply. Flat nasal bridge Early-onset dementia Hyperplastic mandible Separated sagittal suture High, arched, narrow palate

Flat nasal bridge Separated sagittal suture High, arched, narrow palate Rationale The most common clinical manifestations of Down syndrome include a separated sagittal suture; small nose; high, arched, narrow palate; wide space between the big and second toes; plantar crease between the big and second toes; hyperflexibility and muscle weakness; neck skin excess and laxity; depressed nasal bridge; and oblique palpebral fissures. Early-onset dementia occurs in one third of those with Down syndrome. Hypoplastic mandible, rather than hyperplastic mandible, is a clinical manifestation of Down syndrome.

Which specific signs are appropriate in the diagnosis of bacterial meningitis in neonates? Select all that apply. Weak cry Poor tone Nuchal rigidity Normal feedings Full, tense, bulging fontanel

Rationale Poor tone, weak cry, and a full, tense, and bulging fontanel are specific signs of bacterial meningitis in neonates. The neck is usually supple, not rigid, in neonates. Feedings are refused, not taken normally.

Which is an appropriate reason why a patient with an alarming rise in intracranial pressure would be induced into a coma using barbiturates? Select all that apply. To increase cerebral metabolic rate To maintain cardiac system functioning To maintain respiratory system functioning To reduce the intracranial pressure of the brain To protect the brain from the effects of low brain perfusion

To reduce the intracranial pressure of the brain To protect the brain from the effects of low brain perfusion Rationale Barbiturates reduce intracranial pressure when all other measures have failed. Barbiturates protect the brain during times of reduced cerebral perfusion pressure. Barbiturates decrease the cerebral metabolic rate for oxygen. Barbiturates do not maintain cardiac function; they decrease it, which requires intensive cardiac monitoring. Barbiturates do not maintain respiratory function; they reduce it, which requires intensive monitoring.

Phocomelia

congenital condition in which the proximal portions of the limbs are poorly developed or absent

A 2-year-old patient diagnosed with beta-thalassemia requires chelation therapy. The child's parents state, "So we bring her here for the needle to be placed for each treatment." Which response by the nurse is most appropriate? "Chelation therapy will be a one-time occurrence." "Your child will be given a tablet four times each day." "You will be taught how to prepare medications for chelation therapy." "Chelation therapy can only be performed during an inpatient hospital visit."

"You will be taught how to prepare medications for chelation therapy." The nurse should inform the parents that they will be properly educated on administration of home chelation therapy, so frequent visits to the provider's office will not be necessary.

Iron Deficiency Anemia Interventions

1. Increase oral intake of iron; iron fortified formula is needed for an infant 2. Instruct the child and parents in food choices that are high in iron. 3. Administer iron supplements as prescribed. 4. Intramuscular injections of iron (using z-track method) or IV administration of iron may be prescribed in severe cases of anemia. 5. Teach parents how to administer the iron supplements. a.) give between meals for maximum absorption b.) Give with a multivitamin or fruit juice because Vitamin C increases absorption. 6. Instruct the child and parents about the side effects of iron supplements (black stools, constipation, and foul aftertaste)

After spinal fusion surgery the nurse should check for signs of what? a. Seizure activity b. Increased intracranial pressure c. Impaired color, sensitivity, and movement to the lower extremities d. Impaired pupillary response during neurologic checks

ANS: C In addition to the usual postoperative assessments of wound, circulation, and vital signs, the neurologic status of the patient's extremities requires special attention. Prompt recognition of any neurologic impairment is imperative because delayed paralysis may develop that requires surgical intervention.

The nurse is caring for a child with myasthenia gravis (MG). What health care prescription should the nurse verify before administering? a. Ceftizoxime (Cefizox) b. Cefotaxime (Claforan) c. Ceftriaxone (Rocephin) d. Garamycin (gentamicin)

ANS: D Avoid aminoglycoside antibiotics such as gentamicin because they potentiate MG symptoms. Cefizox, Claforan, and Rocephin are cephalosporin antibiotics.

When a child develops latex allergy, which food may also cause an allergic reaction? a. Yeast b. Wheat c. Peanuts d. Bananas

ANS: D There are cross-reactions between allergies to latex and to a number of foods such as bananas, avocados, kiwi, and chestnuts. Although yeast, wheat, and peanuts are potential allergens, currently they are not known to cross-react with latex allergy.

A preteen with no previous issues is noted to have uneven shoulders, and when performing the Adam's forward bend test the nurse observes what appears to be a rib hump. On which condition does the nurse anticipate educating the patient? A. Scheuermann's Kyphosis B. Early onset scoliosis (EOS) C. Adolescent idiopathic scoliosis D. Previously undiagnosed congenital scoliosis

Adolescent idiopathic scoliosis Adolescent, idiopathic scoliosis would have onset at 10 years or older. A lack of previous history suggests this is idiopathic onset.

A patient presents to the clinic with significant swelling, bruising, and pain around the ankle following a fall during a track meet 3 hours earlier. The athletic trainer provided initial treatment at the event. Which action by the trainer indicates a need for correction by the nurse? Kept the ankle elevated Application of a heat pack Encouraged rest until parents arrived Placement of a compression bandage

Application of a heat pack In the initial stages of an injury, ice should be applied to reduce swelling, not heat. This action indicates need for correction by the nurse.

When would children with cognitive impairment be referred for stimulation and educational programs? As young as possible. As soon as they have the ability to communicate in some way. At age 3 years, when schools are required to provide services. At age 5 or 6 years, when schools are required to provide services.

As young as possible. Correct The child's education should begin as soon as possible. Considerable evidence exists that early intervention programs for children with disabilities are valuable for cognitively impaired children. The early intervention may facilitate the child's development of communication skills. States are encouraged to provide early intervention programs from birth under Public Law 101-476, the Individuals with Disabilities Education Act.

A newborn infant presents to the emergency department with papilledema. Which assessment does the nurse perform first? Examine gross motor skills. Assess for patent fontanels. Measure head circumference. Evaluate the primitive reflexes.

Assess for patent fontanels. Infants typically do not experience papilledema because the patent fontanels allow for increased intracranial pressure; whereas, an older child or adult has no fontanels and can exhibit papilledema in reaction to increased intracranial pressure (ICP). Papilledema in infants can be a sign of immature closure of the fontanelles.

protruding tongue in down syndrome

interferes with feeding tongue thrush is not refusal but a physiological response use small but long spoons to push foot toward back and side of the mouth if food is thrush out, it should be refed

A 3-year-old has just returned from surgery in a hip spica cast. Which is the priority nursing intervention? Offering sips of water Elevating the head of the bed Checking circulation, sensation, and motion of toes Turning the child to the right side and then the left side every 4 hours

Checking circulation, sensation, and motion of toes Rationale The chief concern is that the extremity may continue to swell. The circulation, sensation, and motion of the toes must be assessed to ensure that the cast does not become a tourniquet and cause complications. Elevating the head of the bed might help with comfort, but it is not a priority. The nurse must be alert to the risk of increased swelling in the extremities. Offering sips of water is acceptable once assessment of the extremities has been completed. The child's position should be changed every 2 hours. Correct positioning of a child with a spica cast is important to prevent injury.

sports injuries

Children and adolescents involved in sports may experience a number of injuries as a result of collision with another individual or an immobile object or inadequate muscle preparation beforehand. Injuries include contusions, dislocations, sprains and strains, and fractures. Basic principles for managing sprains and other soft tissue injuries are summarized in the mnemonic RICES: rest, ice, compression, elevation, and support.

How about juvenile idiopathic arthritis?

Chronic autoimmune disease. Causes inflammation in the synovium, joints, and surrounding tissue. Infectious agents activate autoimmune inflammatory process. Eventually causes joint effusion, erosion, destruction, and fibrosis of articular cartilage.

Mixed type CP

Combination of spastic CP and dyskinetic CP

The nurse is performing a neurologic assessment on a patient with cerebral palsy (CP). The nurse notes bilateral arm spasticity and the child is unable to grip the nurse's fingers. What action should the nurse perform? Notify health care provider Assess child's gait for ataxia Complete neurologic assessment Inquire about learning difficulties

Complete neurologic assessment Spasticity is an expected finding in a child with cerebral palsy. The nurse should continue the assessment.

An infant is brought to the emergency department with retinal hemorrhages, increased irritability, and a burn mark on the arm. Once stabilized, what is the nurse's priority intervention for this patient? Consult with child protective services. Ask the case manager to arrange home health care. Provide stress management teaching to the parents. Gather a timeline of events based on the parents' reports.

Consult with child protective services. Abusive head trauma or "shaken baby" syndrome is the most likely condition in an infant with retinal hemorrhages. The child should not go home with the parents until the cause of the traumatic brain injury is determined. If parents are not charged with abuse, someone must still make sure safeties are in place so consultation with child protective services is a priority intervention.

Identify the lab test that would best indicate muscle damage. C-reactive protein (CRP) Rheumatoid factor (RF) Alkaline phosphatase (ALP) Creatine phosphokinase (CPK)

Creatine phosphokinase (CPK) CPK is found in heart and skeletal muscle. Specific forms of CPK can be tested to determine what muscle type is damaged.

Which finding in the cerebrospinal fluid (CSF) report is suggestive of bacterial meningitis? Select all that apply. Normal protein Decreased glucose Cloudy appearance Increased neutrophils Increased lymphocytes

Decreased glucose Cloudy appearance Increased neutrophils Rationale In bacterial meningitis, the glucose levels in cerebrospinal fluid decrease, the appearance of CSF is cloudy, and neutrophils in CSF increase. The CSF proteins are elevated in bacterial meningitis, and proteins will be normal or slightly elevated in viral meningitis. An increased number of lymphocytes in CSF is suggestive of viral meningitis rather than bacterial meningitis.

Which finding is appropriate when identifying cognitive impairment in children? Fine motor delays Normal eye contact Delayed developmental milestones Increased alertness to voice or movement

Delayed developmental milestones Rationale Delayed achievement of developmental milestones is a major clue to the presence of cognitive impairment in children. Children with Down syndrome or fragile X syndrome have dysmorphic features that make cognitive impairment easier to recognize. Abnormal eye contact, gross motor delay, decreased alertness to voice or movement, language difficulties, and feeding difficulties are all early behavioral signs of cognitive impairment.

The nurse is assessing a patient with cystic fibrosis (CF). To determine if enzyme replacements are adequate, which assessment detail should the nurse look for? Select all that apply. Urine output Color of stool Dietary intake Regularity of stool Constancy of stool

Dietary intake Dietary intake will be assessed in patients taking enzyme replacements. Regularity of stool The regularity of stool will be assessed by the nurse in a patient taking enzyme replacements. Constancy of stool The constancy of stool will be addressed in a patient taking enzyme replacements.

Ponseti method

Involves the following steps: • Manipulation of foot and serial casting weekly in doctor's office for 6-8 weeks • Small surgical procedure under local anesthesia in doctor's office to lengthen heel cord • Special orthopedic device with a bar separating feet (Dennis Brown Brace) is worn for 2 months, and then only at night time for 2 years

The nurse is caring for a patient with disseminated intravascular coagulation (DIC) after surgery. The nurse has given platelets and blood transfusions to the patient. Which physical assessment finding would be most concerning? Purpuric rash Dilated, fixed pupil Abdominal distention Bruising around the intravenous (IV) site

Dilated, fixed pupil A dilated, fixe pupil indicates increased intracranial pressure, which may be secondary to a possible intracranial bleed in this patient with DIC. It would be the most concerning finding.

dyskinetic CP/

Dyskinetic Cerebral Palsy problems controlling the movement of their hands, arms, feet, and legs, making it difficult to sit and walk. Muscle tone might change not only from day to day, but even during a single day Athetoid—Slow, wormlike movements of extremities, trunk, face, tongue Lead to chorea—jerky involuntary movements worsen w/ emotional stress dystonic - slow twisting movements of the trunk or extremities, abnormal posture; Drooling, speech problems

Diagnostic Tools

EMG (measures potential generated in muscles) Nerve conduction velocity Muscle biopsy (vastus lateralis) Serum enzyme measurement/CPK will be high Needle in the muscle in EMG Nerve impulse issue CPK blood measurement

Which methods are appropriate when planning to perform a caloric test on a patient? Select all that apply. Using 10 mL of ice water for irrigation Irrigating internal auditory canal only Using water at room temperature Elevating head of bed to 30 degrees Irrigating the auditory canal for 20 seconds

Elevating head of bed to 30 degrees Irrigating the auditory canal for 20 seconds Using 10 mL of ice water for irrigation Rationale To perform a caloric test, the nurse uses 10 mL of ice water. The head of the patient is elevated to 30 degrees. The external auditory canal is irrigated for 20 seconds. The internal auditory canal is not irrigated in a caloric test. Warm water is not used.

Match the term with its definition.

Erythrocyte red blood cell Erythropoietin hormone that stimulates red blood cell production and maturation Polycythemia increased production of red blood cells Hematopoiesis red blood cell production

What assessment finding would suggest to the nurse a possible visual impairment in a child? Excessive rubbing of the eyes Delay in speech development Rapid lateral movement of the eyes Lack of interest in casual conversation with peers

Excessive rubbing of the eyes Correct Excessive rubbing of the eyes is a clinical manifestation of visual impairment. A delay in speech development, rapid lateral movement of the eyes, and lack of interest in casual conversation with peers are not associated with visual impairment.

Which blood coagulation factor abnormality can be determined using only the prothrombin time (PT) rather than using both the PT and the partial thromboplastin time (PTT)? Factor II Factor V Factor VII Factor X

Factor VII Rationale Abnormalities of factor VII can be measured by determining the prothrombin time (PT) rather than the partial thromboplastin time (PTT). The PT measures the activity of prothrombin and the factors necessary for its conversion to thrombin and fibrinogen. It measures abnormalities of the extrinsic pathway. Abnormalities of factor VII cannot be determined by the PTT, which measures abnormalities of the intrinsic pathway. Abnormalities of factors II, V, and X can be determined by both PT and PTT, because these factors are involved both in the intrinsic and extrinsic pathways.

Which emotional characteristics might the nurse observe in a patient with fragile X syndrome? Greeting everyone who passes by Blunted affect with social interaction Failure to make eye contact with the nurse Repeating the sound "Ba baba" repetitively

Failure to make eye contact with the nurse Gaze aversion is considered a social and emotional relatedness characteristic observed in fragile X syndrome.

An adolescent has sustained a spinal cord injury. What are the characteristics of the first stage, known as spinal shock syndrome? Increasing spasticity Spinal reflex activity Symptoms of hypertension Flaccid paralysis below level of damage

Flaccid paralysis below level of damage Correct Reflexes are absent at or below the cord lesion. There is flaccidity or limpness of the involved muscles. Spinal reflex activity occurs in the second stage. Symptoms of hypotension occur.

Which drug is appropriate for the infant with Lennox-Gastaut Syndrome (LGS) that is not responding to front-line therapy? Clobazam Felbamate Gabapentin Lamotrigine

Gabapentin Rationale Gabapentin is the drug of choice to treat the infant because it is an antiepileptic drug that is used when a patient is not responding to front-line therapy. Clobazam, felbamate, and lamotrigine are also antiepileptic drugs used to treat Lennox-Gastaut Syndrome, but these are used as front-line therapy.

Which nursing instruction is appropriate for the 4-year-old child that is prescribed liquid iron for iron-deficiency anemia? Select all that apply. Give the iron using a straw. Give the iron using a tablespoon. Brush the child's teeth after administration of iron. Brush the child's teeth before administration of iron. Give iron along with milk or milk products.

Give the iron using a straw. Brush the child's teeth after administration of iron. Rationale It is important for a nurse to inform the parents that giving iron in liquid form can stain the teeth. Therefore it would be administered using a straw. Brushing the teeth after administration of iron can prevent discoloration of teeth. Iron would not be given using a tablespoon, because it can stain the teeth. Brushing the teeth before liquid iron administration has no effect on the staining of the teeth. Cow's milk contains substances that interfere with iron absorption. Therefore, iron would never be given along with milk or milk products.

Which clinical manifestations are appropriate with the diagnosis of hemophilia? Select all that apply. Fever Excessive bruising Nausea and vomiting Hemorrhage from any trauma Prolonged bleeding from or in the body

Hemorrhage from any trauma Prolonged bleeding from or in the body Excessive bruising Rationale The most common clinical manifestations of hemophilia are prolonged bleeding anywhere from or in the body, hemorrhage from any trauma, and excessive bruising. Fever, nausea, and vomiting are not common clinical manifestations of hemophilia.

How do lower motor neuron lesions affect the muscular function differently than upper motor neuron lesions? Interrupting the reflex arc Increasing deep tendon reflexes Producing abnormal superficial reflexes Producing weakness associated with spasticity

Interrupting the reflex arc Rationale Lower motor neuron lesions interrupt the reflex arc and cause weakness and atrophy of skeletal muscles. Deep tendon reflexes are diminished or absent in lower motor neuron lesions, whereas in upper motor neuron lesions, deep tendon reflexes are increased. Upper motor neuron lesions produce abnormal superficial reflexes and weakness associated with spasticity.

A patient diagnosed with Legg-Calvé-Perthes (LCP) disease is informed that the disease has progressed to a point where necrosis is occurring in the joint. Which statement demonstrates the nurse's knowledge of LCP progression? A. Necrosis occurs after bone is reabsorbed by the body during Stage 4. B. Necrosis occurs after the fragmentation stage and is Stage 3 of the disease. C. LCP is caused by ischemia of the head of the femur. This leads to extensive reconstitution before the onset of necrosis, stage 5. D. LCP is caused by limited blood supply to the ball of the femur. As blood flow decreases, the tissue begins to die, entering necrosis or Stage 2 of the disease.

LCP is caused by limited blood supply to the ball of the femur. As blood flow decreases, the tissue begins to die, entering necrosis or Stage 2 of the disease. LCP is caused by ischemia of the head of the femur. As tissues receive less blood, the cells die, which is necrosis and Stage 2 of LCP.

The nurse is caring for a 5-year-old patient who was struck by a car and has been admitted for observation. On assessment, the nurse notes a firm, distended abdomen; bleeding from the intravenous (IV) site and oozing from a head laceration. Which action should the nurse take first? Provide an oral analgesic. Administer an IV fluid bolus. Administer clotting factors. Notify the health care provider.

Notify the health care provider. Bleeding from the IV site and a distended abdomen are signs of DIC. The provider should be notified immediately to obtain orders

A 5-year-old patient presents with a rib hump and visible curvature of the spine measuring 20 degrees. Which is the preferred treatment that the nurse can anticipate for this patient? A. Brace B. Surgery C. Exercise D. Observation

Observation Observation is recommended for a curve less than 25 degrees. Patients closer to 25 degrees who are skeletally immature will require more frequent radiographic observation than a patient with a smaller curvature who is skeletally mature.

What is the most widely used therapy to treat the 1-week-old infant who has developmental dysplasia of the hip (DDH)? Traction Pavlik harness Surgical closed reduction Proximal femoral osteotomy

Pavlik harness Rationale The Pavlik harness is most widely used to manage DDH in a newborn. When adduction contracture is present at 6 to 10 weeks of age, other devices such as skin traction can be used to manage DDH. In 6 to 24 months of age, if DDH is detected, it is treated with surgical closed reduction and then application of spica cast. In an older child, proximal femoral osteotomy is performed to manage DDH.

What is the most comprehensive therapeutic management for juvenile idiopathic arthritis? Pain control, physical and occupational therapy, splints, and ice packs Pain control, physical and occupational therapy, splints, and acetaminophen to reduce inflammation Pain control, physical and occupational therapy, splints, and nonsteroidal antiinflammatory drugs for inflammation Pain control, physical and occupational therapy, splints, and range-of-motion exercises during periods of inflammation

Pain control, physical and occupational therapy, splints, and nonsteroidal antiinflammatory drugs for inflammation Rationale Pain control, physical and occupational therapy, splints, and nonsteroidal antiinflammatory drugs for inflammation comprise the most comprehensive therapeutic management for juvenile idiopathic arthritis. Acetaminophen does not reduce inflammation, and warm, moist heat is better than ice for relieving stiffness and pain. Range-of-motion exercises should not be performed during periods of inflammation.

What is the best diagnostic evaluation tool to diagnose juvenile idiopathic arthritis (JIA)? Physical manifestations Positive rheumatoid factor An elevated C-reactive protein Presence of antinuclear antibody

Physical manifestations Rationale There are no definitive tests to diagnose JIA; physical manifestations are used to diagnose JIA. Laboratory test results may provide supporting evidence of disease. Rheumatoid factors are usually negative in JIA cases. An elevated C-reactive protein may or may not be present. The presence of antinuclear antibodies is common in JIA, but it is not a confirmatory test for JIA.

Which finding regarding a female with Down syndrome is correct? Rapid weight gain by 3 years old Rapid increase in height by 18 months Menstruation occurs at a delayed age Infertility happens in postpubertal women

Rapid weight gain by 3 years old Rationale A patient with Down syndrome would have rapid weight gain. Weight gain is more rapid compared to growth in stature, which often results in the patients being overweight. The height of the patient with Down syndrome would be reduced. In females with Down syndrome, menstruation occurs at the average age. Postpubertal women with Down syndrome can be fertile, and a small number of women with Down syndrome have had offspring.

Which neurologic assessment is appropriate for the child whose level of consciousness has been variable since she sustained a cervical neck injury 12 hours ago? Reactivity of pupils Doll's head maneuver Oculovestibular response Funduscopic examination to identify papilledema

Reactivity of pupils Rationale Pupil reactivity is an important indication of neurologic health. The pupils would be assessed for no reaction, unilateral reaction, and rate of reactivity. The doll's head maneuver would not be performed if there is a cervical spine injury. Assessment for an oculovestibular response is a painful test that would not be done in a child who is displaying a variable level of consciousness. Papilledema does not develop for 24 to 48 hours in the course of unconsciousness.

A child in traction is frustrated because of feeling trapped. The child is experiencing constipation and says breathing feels hard at times. Which nursing interventions would benefit this patient? Select all that apply. A. Decrease fluid intake. B. Reposition the body. C. Provide a high fiber diet. D. Move the bed to play area. E. Play with bubbles or an incentive spirometer. F. Encourage more rest to decrease movement.

Reposition the body. Repositioning the body is useful to prevent skin damage, promote defecation, and facilitate breathing. Correct Provide a high fiber diet. A high fiber diet should decrease constipation. Correct Move the bed to play area. Moving the bed to the play area provides a new environment for the child. Correct Play with bubbles or an incentive spirometer. Playing with bubbles or an incentive spirometer will encourage deep breaths and better lung function.

A child presents to the emergency department with sudden bilateral ascending weakness and is diagnosed with Guillain-Barré syndrome. What should the nurse most closely monitor? Capillary refill Respiratory status Heart rate and rhythm Level of consciousness

Respiratory status Along with achieving optimal neurologic function, the nurse should prioritize monitoring the respiratory rate in a child with bilateral ascending weakness.

Distortion of sound and problems in discrimination are characteristic of what type of hearing loss? Conductive Sensorineural Central auditory imperceptive Mixed conductive-sensorineural

Sensorineural Correct Sensorineural hearing loss, also known as perceptive or nerve deafness, involves damage to the inner ear structures or the auditory nerve. It results in distortion of sounds and problems in discrimination. Conductive hearing loss involves mainly interference with loudness of sound. Central auditory imperceptive hearing loss manifests as a combination of both sensorineural and conductive loss. The mixed conductive-sensorineural category includes all hearing losses that do not demonstrate defects in the conduction or sensory structures.

The nurse is caring for a child with urinary retention and bladder infection. The child has not passed urine for the last 4 to 5 hours. What intervention does the nurse perform first to relieve the child of urinary retention? Administer diuretics as prescribed. Catheterize the child immediately. Stimulate the bladder using warm water. Keep fluid intake to a minimum.

Stimulate the bladder using warm water. Rationale Urinary retention can be relieved by pouring warm water on the genital organs. This helps to stimulate the bladder for contraction and voiding. Diuretics are helpful in increasing urinary output, but they do not relieve urinary retention. They can worsen the retention by increasing the formation of urine. Catheterization is done only in severe urinary retention. The patient is encouraged to take adequate fluids, and fluid intake is not restricted.

Which nursing action is appropriate for the parents of a child with cognitive impairment finding it difficult to teach self-care skills to their child? Say that it requires patience. Inform about effective coping strategies. Teach developmental sequences of learning. Ask whether proper verbal instructions are given.

Teach developmental sequences of learning. Rationale Parents need to know about the developmental sequences of learning. This helps them assess the child's readiness for learning the skill before teaching it. The parents need to be taught about coping strategies when they are stressed or are unable to cope while providing care for their child. Telling the parent that teaching requires patience is not as effective as teaching them how to solve the problem. The nurse suggests demonstration rather than verbal instructions while teaching, because it is easier to comprehend.

Given the available medical personnel, what staff assignment is best? for the client with open facture after removal of cast due to compartment syndrome

ongoing monitoring of Madison's foot by the LPN, while the UAP assists the postoperative client with crutches and the nurse determines if the new client has any immediate problems

Which type of seizure is occurring when the child's arms are flexed, eyes are rolled upwards, and legs, head, and neck are extended with increased salivation and loss of swallowing reflex, followed by violent jerking movements as the trunk and extremities undergo rhythmic contraction and relaxation? Status epilepticus Myoclonic seizure Tonic-clonic seizure Psychomotor seizure

Tonic-clonic seizure Rationale The child has experienced a tonic-clonic seizure. During the tonic phase of the seizure, the child's arms are flexed, eyes are rolled upwards, and legs, head, and neck are extended with increased salivation and loss of swallowing reflex. This is followed by the clonic phase with violent jerking movements as the trunk and extremities undergo rhythmic contraction and relaxation. Status epilepticus is a series of seizures with very brief intervals to allow the child to regain consciousness and can lead to exhaustion, respiratory failure, and death. Myoclonic seizures are characterized by sudden, brief contractures of muscles and can occur either once or repetitively. Psychomotor seizures are manifested by a period of altered behavior, amnesia for the duration of the event, inability to respond to environment, impaired consciousness during the event, drowsiness or sleep following the seizure, prolonged confusion, and complex sensory phenomenon.

True/False Ankle foot orthoses are molded to fit the feet and worn inside shoes to reduce deformity

True

manual traction

applied to body part by the hand placed distally to fracture site used when a closed reduction is performed

Osteomyelitis

inflammation of bone and bone marrow caused staph aureus

epidural space

separates the outer meningeal layer and the inner periosteum of the cranial bones

The primary therapy for hemophilia

s replacement of the missing clotting factor. The products currently available are factor VIII concentrates, either produced through genetic engineering (recombinant form) or derived from pooled plasma, which are reconstituted with sterile water immediately before use.

Axillary swing-through crutches

Used most frequently for temporary assistance:

health education

promotes coping

oblique fracture

shunting but straight, between a horizontal and a perpendicular direction

Which activity is the best choice for Madison - immobilized child?

Pretend beauty parlor

True/False Education requirements of all children with CP are determined by the child's needs and potential

True

leading cause of nonfatal injury 0-14

falls

sublaxation ddh

incomplete dislocation

Charleston Brace

more flexible, can wear at night

Which prenatal screening could have detected cleft lip?

ultrasound

A parent brings their 6-year-old child with Duchenne muscular dystrophy (DMD) to the primary care clinic for a well child visit. Upon assessing the patient, the nurse notes that the child displays signs of muscle weakness and difficulty walking. The nurse should ensure that the patient is using which assistive device correctly? Select all that apply. Hearing aids A back brace Soft, flexible soles Corrective lenses Compression socks

A back brace Lordosis, excessive inward curvature of the back, would be present in a patient who has DMD. A back brace would provide the necessary support to the patient and therefore this may be seen on assessment. Correct Soft, flexible soles Foot deformities would be present in DMD secondary to muscle wasting. Soft, flexible shoes would provide adequate space for the child's foot and therefore these may be seen during the assessment.

The nurse is caring for a child with beta-thalassemia, who had a splenectomy 3 weeks ago. The patient reports malaise and has a runny nose and a cough. The nurse notes a temperature of 101.2° F, heart rate of 116, blood pressure of 106/88, and respiratory rate of 26. In which order should the nurse implement interventions?

Assess lung sounds Obtain complete blood count Administer an antibiotic Administer antipyretic The first step of the nursing process is assessment. The nurse should assess lung sounds for potential cause of symptoms. Then the nurse would obtain a complete blood count to determine whether the white blood cell (WBC) count reveals the presence of an infection. If the WBC count is elevated, the nurse would administer an antibiotic, then give an antipyretic to treat the fever.

DIC care

Diagnostic evaluation Physical exam findings Prolonged coagulation results on CBC (increased PT and PTT) Therapeutic management Treat underlying cause Platelet and FFP infusion IV admin heparin Nursing consideration Epistaxsis Patient and family education Support

Posturing

Flexion Posturing Also known as decorticate Occurs with severe dysfunction of cerebral cortex or with lesions to corticospinal tracts above the brainstem Extension Posturing Also known as decerebrate Sign of dysfunction at midbrain or lesions to the brainstem

which therapeutic response is appropriate for nicu nurse to give Jose?

I understand this can be upsetting. this deformity occurs early in pregnancy often before a woman knows she is pregnant

bone healing stage 1/ hematoma formation

IMPACT - facture occures; - injury to soft tissue - tissue tears - vessels rapture 3-5 MIN - bleeding occurs from bone and tissue into area between and around bone fragments FIRST 24 HRS - hematoma forms and clots; fibrin assists in clotting - clot provides fibrin network for cellular invasion - granulation tissue forms by fibroblasts and new capillaries - osteoblastic activity stimulated

stress fx

Occurs as a result of repeated muscle contraction from repetitive weight-bearing sports

Nursemaid's elbow

Partial dislocation of the radial head in elbow

Cerebral Blood Flow (CBF)

The amount of blood in milliliters passing through 100 g of brain tissue in 1 minute autoregulation

dexamethasone

first dose for bacterial meningitis before antibiotics start to avoid hearing impairment antibiotic treatment with cephalosporins - sterilizing the CSF and reducing the incidence of severe hearing impairment

long term care

forming parent-professional partnerships that can support a family's adaptation across the trajectory of the illness

Which nursing action is appropriate during a blood transfusion, when a child reports mild precordial pain? Administer antihistamine. Transfuse the blood slower. Increase the rate of transfusion. Administer epinephrine immediately.

Transfuse the blood slower. Rationale During a blood transfusion, it is the responsibility of a nurse to monitor the patient regularly to look for any adverse effects. Precordial pain indicates that the child has circulatory overload, and therefore the nurse would slow down the rate of transfusion. If this does not relieve the symptoms, the nurse should stop the transfusion. Antihistamines are administered prophylactically to a patient who tends to have allergic reactions. If a patient has asthmatic wheezing during a blood transfusion, epinephrine should be administered.

The parents of a child newly diagnosed with a chronic illness ask the nurse how long the illness will last. How should the nurse respond? Select all that apply. "A chronic illness may have residual aspects that can last a lifetime." "A chronic illness is incurable, so the child will always struggle with the illness." "A chronic illness usually affects activities of daily living (ADLs) and lasts a long time." "A chronic illness can occur during a wide range of times and often spontaneously resolves." "A chronic illness lasts at least three months and up to a lifetime, depending on the illness."

"A chronic illness may have residual aspects that can last a lifetime." A chronic illness can have effects that last a lifetime, even if the active symptoms of the illness have resolved. "A chronic illness lasts at least three months and up to a lifetime, depending on the illness." Lasting at least three months is part of the definition of a chronic condition. Many chronic illnesses are incurable and therefore can last a lifetime.

Traumatic amputation nursing alert

1. Rinse limb gently with NS 2. Loosely wrap it in sterile gauze 3. Place it in a watertight bag 4. Chill bag in ice water but do not pack in ice or freeze 5. Label with child's name, date, and time > transport to hospital

anticipated parental stress points

1. diagnosis of the condition - parents require considerable education while dealing with an emotional response 2. developmental milestones - times that children normally achieve waling, talking and self-care are delayed or impossible for the child 3.start of schooling - particularly stressful are situations in which appropriate schooling will not be in a regular class placement 4. reaching the ultimate attainment - parents must handle situations such as realizing that ambulation will be impossible or that the child will not learn to read 5. adolescence - issues such as sexuality and independence become prominent 6. future placement - decisions about placement must be made when the child becomes an adult or when the parents can no longer care for the child 7 death of a child

characteristics of parental overprotection

1. sacrifices self and rest of the family for the child 2. continually helps the child even when the child is capable 3. is inconsistent with regard to discipline or uses no discipline; applies different rules to the siblings 4. making decisions without considering the child's wishes 5. hovers and offers suggestions; calls attention to every activity; overdoes praise 6. protects the child from every possible discomfort 7. restricts play, often because of fear that the child will be injuries 8 denies the child opportunities for growing up and assuming responsibility, such as learning to take medications 9. does not understands the child's capabilities, and sets goals too high or too low 10. monopolizes the child's time, such as sleeping with the child, permitting few friends, refusing participation in activities

A patient with frequent epistaxis presents to the clinic. The nurse notes that the enzyme-linked immunosorbent assay (ELISA) reveals a ristocetin cofactor (RCoF) level of 30 IU/dL. Which order can the nurse expect after the patient has been seen by the provider? Obtain hemoglobin level. Assess level of consciousness. Provide intravenous (IV) fluid bolus. Administer DDAVP intravenously.

Administer DDAVP intravenously. DDAVP is the treatment of choice for patients whose ELISA reveals a ristocetin cofactor (RCoF) level of 30 IU/dL because it stimulates the release of Von Willebrand factor.

The parents of a 3-year-old child with Down syndrome tell the nurse that their child's lips and fingertips turn dusky purple when the child runs around the house for more than a few minutes. They also report a persistent cough over the past 4 months. The parents state that their child's last physical exam was 2 years ago. Which action is most appropriate for the nurse to take? Perform a focused respiratory system assessment. Assess the child for digital clubbing and central cyanosis. Alert the health care provider to the symptoms described by the parents. Review the physical assessment findings from the child's one year of age visit.

Alert the health care provider to the symptoms described by the parents. The symptoms reported by the parents are associated with serious cardiac anomalies. The health care provider must be informed immediately on these findings so that diagnostic testing can be done.

The home health nurse is caring for a child who requires complex care. The family expresses frustration related to obtaining accurate information about their child's illness and its management. What is the best action for the nurse? Determine why the family is easily frustrated. Refer the family to the child's primary care practitioner. Provide the information the family requests, being confident that every question can be answered by the nurse. Answer questions in a straightforward manner and get professional assistance when an answer is unknown.

Answer questions in a straightforward manner and get professional assistance when an answer is unknown. Correct The philosophic basis for family-centered practice is the recognition that the family is the constant in the child's life. It is essential that the family have complete and accurate information about their child's illness and management. The nurse may first have to clarify what information the family believes has not been communicated. The family's frustration arises from their perception that they are not receiving information pertinent to their child's care. It does not help the family to refer the family to the child's primary care practitioner. The home health nurse should have access to the necessary information. Questions about what they need and want to know concerning their child's care should be addressed, but the nurse may not be able to answer every question.

Which advantages are appropriate in the use of ankle-foot orthoses for a child with cerebral palsy? Have custom seats for dependent mobilization Are useful for independent mobility Are used to prevent deformity and increase energy efficiency of gait Provide sitting balance

Are used to prevent deformity and increase energy efficiency of gait Rationale Ankle-foot orthoses (or braces) are used to prevent deformity, increase the energy efficiency of gait, and control alignment. Ankle-foot orthoses do not have custom seats for dependent mobilization. They are supportive devices for the ankle or a part of the foot. Custom seats cannot be adjusted within this device. They also do not allow independent mobility or provide sitting balance. They control the ankle position and compensate for the muscle weakness.

Which cerebral palsy is appropriate in the child who suffered birth asphyxia? Ataxic Athetoid Hemiplegic Spastic diplegic

Athetoid Rationale Athetoid cerebral palsy is most likely associated with birth asphyxia. Ataxic cerebral palsy is usually associated with cerebral hypoplasia and severe neonatal hypoglycemia. Hemiplegic cerebral palsy is often associated with a focal cerebral infarction secondary to an intrauterine or perinatal thromboembolism. Spastic diplegic cerebral palsy is associated with preterm birth caused by hypoxic infarction with periventricular leukomalacia in the area adjacent to the lateral ventricles.

Just take a look at Legg-Calve'- Perthes disease (1282) What it is...what area of the body does it affect?

Avascular necrosis of the femoral head. Temporary disturbance of circulation to the femoral epiphysis = ischemic aseptic necrosis= limp on affected side. They can dislocate their hip. They can fracture. It's usually pretty benign, but I'ts picked up because the child starts walking funny and complains of pain in the hip with no known injury.

Emergency treatment for the child with a fraction includes: a. moving the child to allow removal of clothing from the area of injury. b. immobilization of the limb, usually including joints above and below the injury site. c. pushing the protruding bone under the skin. d. keeping the area of injury in a dependent position.

B. Immobilization of the limb, usually including joints above and below the injury site

How should the nurse communicate with the family to educate them on a child's clubfoot condition? Be very descriptive but use lay terms. Be slightly vague to avoid causing distress. Give a handout and encourage self-directed reading. Use scientific verbiage to provide an in-depth explanation of the condition.

Be very descriptive but use lay terms. When talking with families, use descriptive lay language rather than scientific terms.

A child, age 10 years, sustained a fracture in the epiphyseal plate of her right fibula when she fell out of a tree. What does the nurse consider when discussing this injury with her parents? This type of fracture is inconsistent with a fall. Bone growth can be affected by this type of fracture. This is an unusual fracture site in young children. Healing is usually delayed in this type of fracture.

Bone growth can be affected by this type of fracture. Correct Detection of epiphyseal injuries is sometimes difficult, but fractures involving the epiphysis or epiphyseal plate present special problems in determining whether bone growth will be affected. The epiphysis is the weakest point of the long bones. This is a frequent site of damage during trauma. Healing of epiphyseal injuries is usually prompt.

An infant was brought to the emergency department (ED) after falling from a high chair, sustaining a basilar skull fracture. Which concerning assessment findings does the nurse expect? Clear drainage from the ear Pupils are unequal and sluggish Bleeding from the fractured area Intermittent confusion and lethargy

Clear drainage from the ear A basilar skull fracture is characterized with a patient who exhibits raccoon eyes, otorrhea, and hemotympanum.

An eleven-year-old patient with a history of allergic rhinitis was brought to the emergency department with headache and nuchal rigidity. In addition to a lumbar puncture, for what other testing does the nurse prepare the patient? Lateral chest x-ray Urine culture for Escherichia coli Rectal swab for group B streptococci (GBS) Computed tomography (CT) scan of sinus cavities

Computed tomography (CT) scan of sinus cavities Sinusitis is often a precipitating infection in meningitis in children because olfactory nerves provide unimpeded access into central nervous system (CNS).

Which structure is most likely to be disrupted if the left cerebral hemisphere is pushed across the cranium toward the right cerebral hemisphere? Fontanel Tentorium Cerebellum Corpus callosum

Corpus callosum The corpus callosum is a band of commissural fibers that connects the right and left cerebral hemispheres and is the most likely to be damaged when the left cerebral hemisphere is pushed toward the right cerebral hemisphere (herniation).

A nurse is assessing a young child who, the parents state, is walking with a wide gait. Which laboratory test does the nurse anticipate that the physician will order? Sodium Creatine kinase Leukocyte esterase Complete blood count (CBC)

Creatine kinase During the early stages of the onset of MD, the muscle fibers begin to leak the protein creatine kinase (CK).

What effects does immobilization have on the respiratory system? Decreased need for oxygen, increased vital capacity, and increased chest expansion Decreased need for oxygen, loss of respiratory muscle strength, and increased vital capacity Decreased need for oxygen, loss of respiratory muscle strength, and decreased vital capacity Decreased need for oxygen, loss of respiratory muscle strength, and increased chest expansion

Decreased need for oxygen, loss of respiratory muscle strength, and decreased vital capacity Rationale Effects of immobilization on the respiratory system include a decreased need for oxygen, loss of respiratory muscle strength, and decreased vital capacity. Chest expansion is reduced, rather than increased, during immobilization.

How about Iron deficiency anemia?

Dietary deficiency of iron = low HGB= low O2 to tissues. Basically we don't have enough hemoglobin and we need iron to make HGB. What happens if we don't have enough HGB? Oxygen fatigue, can't carry the O2. Iron supplements. What about cow's milk? No cow's milk. Iron supplements a couple times a day on an empty stomach. Anything we want to teach the parents about iron supplements? Can constipate, if they are vomiting you CAN give it with meals. Put lots of fiber in their diet.

True/False Baclofen and diazepam are effective in improving muscle coordination in children with CP and in decreasing overall spasticity

False

True/False Children who have CP and sucking and swallowing difficulty should be held in a semireclining posture during feeding to make use of gravity flow

False

Glasgow Coma Scale Pediatric Categories

Eye Opening 1 = none 2 = to pain 3 = to speech 4 = spontaneously Best Response to Auditory/Visual 1 = none 2 = incomprehensible speech (infant restless, moaning) 3 = inappropriate words (infant, cries to pain) 4 = confused conversation but able to answer questions (infant cries but is consolable) 5 = alert, normal vocalization (infant babbles/coos) Best Motor Response 1 = none 2 = abnormal extension or decerebrate 3 = abnormal flexion or decorticate 4 = withdrawals from pain 5 = localizes pain 6 = spontaneous, purposeful

Which nursing instruction is appropriate for the child diagnosed with iron-deficiency anemia and is being treated with iron supplements that cause vomiting after ingestion? Select all that apply. Increase the dosage of iron. Give iron supplements with meals. Give iron supplements with milk. Avoid giving iron supplements with meals or with milk. Decrease dose and then increase to ordered dose as tolerated.

Give iron supplements with meals. Decrease dose and then increase to ordered dose as tolerated. Rationale Vomiting and diarrhea sometimes occur when a child is given iron supplements. In such cases the nurse should advise the parents to give iron supplements with meals. The dose of iron supplements should also be decreased and then gradually increased until more tolerable. Milk reduces the absorption of iron; therefore, iron should not be given with milk or milk products.

Effects of Anemia on the Circulatory System

Hemodilution Viscosity of blood depends on # of RBC Decreased peripheral resistance Increased cardiac circulation and turbulence May have murmur Cardiac workload is increased May lead to cardiac failure Cyanosis May be present in children despite low levels of Hgb Growth retardation & delayed sexual development

A preteen has been diagnosed with scoliosis and the nurse suspects it is an idiopathic scoliosis. How does the nurse come to this conclusion? Congenital scoliosis is present at birth and idiopathic scoliosis occurs after age 10 years. Congenital scoliosis typically involves a curvature 20-45 degrees. Idiopathic scoliosis involves a curvature > 45 degrees. Congenital scoliosis is present at birth and then resolves shortly after birth. Idiopathic scoliosis appears later in adolescence. Idiopathic scoliosis can be diagnosed for the first time in older children, whereas congenital scoliosis is present at birth.

Idiopathic scoliosis can be diagnosed for the first time in older children, whereas congenital scoliosis is present at birth. Congenital scoliosis is present at birth and idiopathic scoliosis can occur at any point in a child's growth and is classified by age at presentation.

A school-age child is admitted for treatment of osteomyelitis. Which complication would the nurse expect to see in this patient? A. Depression B. Social isolation C. Impaired mobility D. Fluid volume insufficiency

Impaired mobility The nurse would expect the child to have difficulty with mobility secondary to the infection of the bone in the right leg. It is most often accompanied by pain.

What is kyphosis?

Lateral convex angulation in the curvature of the thoracic spine. Children are prone to postural (transient) round back kyphosis during times when skeletal growth outpaces muscle growth.

Spina bifida occulta

Not visible externally Usually lumbosacral, L5-S1 Skin indicators (absent, singly or combination): Sacral dimple Sacral angioma or port wine nevus Sacral tufts of dark hair Sacral lipoma- benign cyst, curved like a finger

dislocation

Occurs when the force of stress on a ligament is so great that it displaces the normal position of the opposing bone ends or the bone end in relation to its socket

For case management to be most effective, who is the most appropriate case manager? One nurse A panel of experts The insurance company A multidisciplinary team

One nurse Correct Nursing case managers are ideally suited to provide the care coordination necessary. Care coordination is most effective if a single person works with the family to accomplish the many tasks and responsibilities that are necessary. The family retains the role as primary decision maker. It is preferable that the family have only one individual to work with. Most likely, the insurance company will have a case manager focusing on the financial aspects of care. This does not include coordination of care to assist the family.

An adolescent presents to the urgent care clinic with a suspected fracture in the lower extremity. What are the priority assessments? Select all that apply. Pain assessment Ability to properly use crutches for ambulation Ability to bear weight on the affected extremity Sensation, color and pulse of the affected extremity Thorough review of pain medication that was taken prior to arriving to the clinic

Pain assessment Pain should be addressed in the immediate assessment of the patient's suspected fracture. The presence of pain is part of the neurovascular assessment. Sensation, color and pulse of the affected extremity Sensation, color and pulse are part of the 5Ps that should be immediately addressed with suspected fractures in order to assess the neurovascular status.

The eleven-year-old patient is admitted with an incomplete spinal cord injury at C4. Which findings cause the nurse the most concern? Select all that apply. Patient reports difficulty taking a deep breath. Patient is unable to lift arms or grasp eating utensils. Patient's blood pressure increases to 150/92 mm Hg. Patient has "goose bumps," small raised bumps on the skin. Patient's whole blood glucose level increases to 190 mmol/L.

Patient reports difficulty taking a deep breath. A cervical injury affects the ability to control abdominal muscles and other muscles in the thorax, such as the diaphragm which helps to control depth of purposeful inspiration. This could be a sign that the spinal cord injury is actually complete, has progressed, or that there is additional damage to the spinal cord. The patient may require intubation as progression continues. Patient's blood pressure increases to 150/92 mm Hg. Autonomic dysreflexia is characterized by a sudden rise in blood pressure and will need to be a priority in caring for this patient to prevent intracranial hemorrhage, seizures, and heart attack. Patient has "goose bumps," small raised bumps on the skin. Typically associated with being cold, these raised papillae could indicate that the patient is experiencing hypothermia or autonomic dysreflexia and intervention is needed quickly.

The nurse is caring for an eight-year-old child undergoing a lumbar puncture. What actions should the nurse take? Select all that apply. Allow the parents to the hold the child. Position the child to control movement. Encourage the child to take deep breaths. Tell the child that it "won't last much longer." Explain to the child what to expect during the procedure.

Position the child to control movement. It is essential that the child remain in a controlled position to allow correct spinal needle insertion and minimize side effects and adverse events. Encourage the child to take deep breaths. Taking deep breaths is a well-known, easy to implement pain and anxiety-reduction strategy that also prevents the child from holding his or her breath while experiencing pain. Explain to the child what to expect during the procedure. Children cooperate better when they know what to expect and understand what is happening to them.

arachnoid membrane

middle meningeal layer and the inner periosteum of the cranial bones

A 10-year-old patient with scoliosis comes to the clinic and asks the nurse a question about an uncomfortable medical procedure. Which is the nurse's most appropriate response? "The procedure isn't too bad; you'll be fine." "The procedure may be uncomfortable, but we all feel things differently." "The procedure is somewhat uncomfortable, but you have nothing to worry about." "The procedure can be uncomfortable; let's talk about things you can do to help you through it."

"The procedure can be uncomfortable; let's talk about things you can do to help you through it." An honest answer allows the patient to trust the nurse and staff and therefore would be the most appropriate response for the nurse to make. Additionally, it is important to involve the patient as much as possible.

Kyphosis

- Looking at the person laterally, their thoracic spine is convex, hunching forward - Some girls develop this from poor posture as they try to hide from their developing breasts and height - Tx: PT, brace, surgery

Growth plate injuries

- The epiphyseal growth plate, aka physis, is the weakest point of a long bone and is a frequent site of injury

sibling suppor

1) may respond w/ negativity, anger is expressed in different forms. 2) embarrasment is also common 3) encourage parents to talk to siblings 4) support groups for siblings

Helping siblings cope

1) provide active listening to siblings 2) schedule special times w/ siblings 3) praise siblings when they demonstrate patience. 4. provide age-appropriate information about the child's condition 5. recognize special stress for siblings and use services of professionals when needed

Iron Deficiency Anemia Description:

1. Iron stores are depleted, resulting in a decreased supply of iron for the manufacture of hemoglobin in red blood cells. 2. Commonly results from blood loss, increased metabolic demands, syndromes of gastrointestinal malabsorption, and dietary inadequacy.

Aplastic Anemia Assessment

1. Pancytopenia (deficiency of erythrocytes, leukocytes, and thrombocytes) 2. Petechieae, purpura, bleeding, pallor, weakness, tachycardia, and fatigue

factors in determening the reduction method for fracture

1. age of child 2. degree of displacement 3. amount of overriding 4. degree of edema 5. condition of skin and soft tissue 6. sensation and circulation distal to fracture

Cerebral palsy Description

1. disorder characterized by impaired movement and posture resulting from an abnormality in the extrapyramidal or pyramidal motor system 2. the most common clinical type is spastic cerebral palsy which represents an upper motor neuron type of muscle weakness 3. less common types of cerebral palsy are athetoid. ataxic, and mixed

head injury description

1. pathological result of any mechanical force to skull, scalp, meninges, or brain a. open head injury occurs when there is a fracture of the skull or penetration by an object b. closed head injury is the result of blunt trauma (this is more serious than an open head injury because of the increased chance of increased ICP in a closed vault); this type of injury can also be caused by shaken baby syndrome. 2. manifestations depend on the type of injury and the subsequent amount of increased ICP

What do we do for it? Bracing and observation skoliosis

25< 45 degrees. Spinal Fusion in some cases > 45 degrees. Imagine a teenager with spinal fusion. Oh man.

The nurse is caring for a four-year-old patient diagnosed with fungal meningitis. What is the most likely etiology in this child? Parents refuse to vaccinate the child Attending a child care center during the week Child has been recently diagnosed with type II diabetes A diagnosis of acquired immunodeficiency syndrome (AIDS)

A diagnosis of acquired immunodeficiency syndrome (AIDS) Infection with the virus that causes AIDS is the most likely concomitant infection in a child who has been diagnosed with fungal meningitis. Fungal meningitis occurs in those who are immunocompromised, not in children who are otherwise healthy.

Nursing interventions aimed at preventing problems associated with immobility include which one of the following? A. Encouraging self-care B. Restricting fluids with strict I&O C. Limiting active range-of-motion exercises to once per day D. Decreasing sensory stimulation to allow adequate rest

A. Encouraging self care

One of the supervisors for a home health agency asks the nurse to give a family of a child with a chronic illness a survey evaluating the nurses and other service providers. How should the nurse recognize this request? a. Appropriate to improve quality of care b. Improper because it is an invasion of privacy c. Inappropriate unless nurses and other providers agree to participate d. Not acceptable because the family lacks remembering necessary to evaluate professionals

ANS: A Quality assessment and improvement activities are essential for virtually all organizations. Family involvement in evaluating a home care plan can occur on several levels. The nurse can ask the family open-ended questions at regular intervals to assess their opinion of the effectiveness of care. Families should also be given an opportunity to evaluate the individual home care nurses, the home care agency, and other service providers periodically. Evaluation of the provision of care to the patient and family requires evaluation of the care provider, that is, the nurse. Quality-monitoring activities are required by virtually all health care agencies. During the evaluation process, the family is asked to provide their perceptions of care.

The nurse outlines short- and long-term goals for a 10-year-old child with many complex health problems. Who should agree on these goals? a. Family and nurse b. Child, family, and nurse c. All professionals involved d. Child, family, and all professionals involved

ANS: D In the home, the family is a partner in each step of the nursing process. The family priorities should guide the planning process. Both short- and long-term goals should be outlined and agreed on by the child, family, and professionals involved. Elimination of any one of these groups can potentially create a care plan that does not meet the needs of the child and family.

Which nursing explanation is appropriate for the parent of a child receiving an iron preparation whose stools are a tarry black color? A symptom of iron deficiency anemia An adverse effect of the iron preparation An indicator of an iron preparation overdose An expected change caused by the iron preparation

An expected change caused by the iron preparation Rationale An adequate dosage of iron turns the stools a tarry black color. Tarry black stools are not a sign of iron-deficiency anemia. Tarry black stools are not an adverse effect of the iron preparation but an expected effect. Tarry black stools are not an indicator of iron preparation overdose

LATEX ALLERGY

Anaphylaxis -put green band on child Spina Bifida patient High risk due to repeated latex exposure Multiple surgeries Repeated urinary catheterizations Range from urticaria, wheezing, rash, to anaphylaxis Reactions increase in severity when latex comes in contact with mucous membranes, wet skin, bloodstream, or airway Cross-reactions with foods: bananas, kiwis, avocados, chestnuts Patients with: urogenital anomalies multiple surgeries Health care workers Reduce exposure to latex for patients at risk of developing allergy Create a latex-safe environment for allergic individuals Get: LATEX ALLERGY BRACELET LATEX CART LATEX ALLERGY STICKER ON CHART

The nurse is suspicious of child abuse (nonaccidental injury) when: I. There is a delay in seeking medical assistance for the injury II. The parent's history of the injury in not congruent with the actual injury III. X-ray studies demonstrate previous fractures in different stages of healing IV. the child is crying and fearful of separation from the parent A. I, II, III, IV B. I, II, III C. II, III D. II, III, IV

B. I, II, III I. There is a delay in seeking medical assistance for the injury II. The parent's history of the injury in not congruent with the actual injury III. X-ray studies demonstrate previous fractures in different stages of healing

Assessment of Hematologic Function

Complete blood count History and assessment findings Child's energy and activity level Growth patterns White blood cells on top CBC look at HNH

Volkmann's contracture

Condition in which the muscles in the palm side of the forearm shorten, causing the fingers to form a fist and the wrist to bend

What findings would indicate to the nurse further assessment and treatment is needed for a child with mild head injury? Vomiting Sleepiness Headache, even if slight Confusion or abnormal behavior

Confusion or abnormal behavior Correct Altered mental status is a clinical manifestation that the damage from the head injury is progressing. Medical evaluation is necessary. Vomiting may occur after a minor head injury. Observation is required for changes in behavior or vital signs that indicate progression. Sleepiness may occur after a minor head injury. Observation is required to ensure that the child is arousable. Headache is common after a head injury and does not require medical evaluation unless accompanied by other signs of progression.

Thoracolumbosacral orthosis (TLSO)

Custom molded and fit snugly around the trunk of the body to exert pressure on the ribs and back to support the spine in a straight position:

Parents of a child with cognitive impairment ask the nurse for guidance in toilet training. They have older children who were successfully toilet trained but do not know how to do this with the impaired child. What would be included in the nurses recommendation? Accidents cannot be ignored if toilet training is to be successful. Determine the child's readiness to begin toilet training. Toilet training should not be initiated until the child has sufficient speech. A more punitive approach is needed because of the child's cognitive impairment.

Determine the child's readiness to begin toilet training. Correct The child's capabilities and readiness are essential to success. Parents must also be able to devote the time to a positive, consistent, individualized, nonpressured style of teaching. Successful training occurs by focusing on the achievement of small goals. Feelings of comfort and adequacy are necessary. Calling undue attention to "accidents" impedes the program's success. Speech fluency is an independent skill. Positive reinforcement, not negative feedback, is needed.

A nurse is observing children in the unit's playroom. She notices a child placing his/her hands on the knees and moving the hands up the legs in order to stand up from a seated position on the floor. The nurse suggests further assessment for what condition? Becker Muscular Dystrophy Steinert Muscular Dystrophy Duchenne Muscular Dystrophy Congenital Myotonic Muscular Dystrophy

Duchenne Muscular Dystrophy This maneuver is called Gower's maneuver and is a classic sign of Duchenne Muscular Dystrophy.

Analysis of cerebrospinal fluid (CSF) obtained from lumbar puncture shows bacterial meningitis. The nurse performs what action for the nine-year-old pediatric patient? Ensure patient is receiving antibiotics. Place patient on airborne precautions. Administer the meningococcal vaccine. Provide family with home care instructions.

Ensure patient is receiving antibiotics. Patient should have been on antibiotics shortly after symptoms began, but the nurse must ensure that antibiotics are being administered as ordered based on the results of CSF findings. A delay in antibiotic administration can be fatal.

A 14-year-old female patient with a newly diagnosed chronic illness presents to the nurse with feelings of worthlessness and showing signs of decreased self-esteem. What action by the nurse should help to strengthen this adolescent's feeling of autonomy as the patient learns to cope with a chronic illness? Correct Ensure that the teen is involved in care decisions and meetings with the health care team Give the adolescent opportunities to discuss needs and fears related to coping with the illness Encourage the patient to do as many of the usual activities with friends as the teen's condition allows Allow the adolescent to ask questions and give honest answers regarding the illness to increase understanding of the condition Stress that adherence is important and that making decisions is a privilege that can be lost if the treatment regimen is not followed

Ensure that the teen is involved in care decisions and meetings with the health care team Being involved in care decisions and care planning can increase the patient's sense of control and self-esteem. Correct Give the adolescent opportunities to discuss needs and fears related to coping with the illness Some adolescents, especially younger teens, might have feelings of guilt related to the illness. Expression of these feelings can decrease sadness and help the teen gain back some control in life. Correct Encourage the patient to do as many of the usual activities with friends as the teen's condition allows Assisting the teen to maintain contact with peer group and maintain usual activities can increase coping and age-appropriate development. Correct Allow the adolescent to ask questions and give honest answers regarding the illness to increase understanding of the condition The adolescent should be an integral decision maker, which cannot be possible without an adequate understanding of the illness.

Goals of Therapy for CP

Establish locomotion, communication, and self-help Gain optimum integration of motor functions Correct associated defects as early and effectively as possible Provide educational opportunities Promote socialization experiences

What is included in the assessment for the Glasgow Coma Scale? Pupil reactivity and motor response Level of consciousness and verbal response Eye opening and verbal and motor response Intracranial pressure and level of consciousness

Eye opening and verbal and motor response Correct The scale is a three-part assessment that includes eye opening, verbal response, and motor response. It is an observational tool to detect a life-threatening complication such as cerebral edema. Pupil reactivity, level of consciousness, and intracranial pressure are not included in the scale.

The nurse is assessing a newborn patient. Which musculoskeletal finding is most concerning? Dislocated left hip Cartilaginous hands Fused cranial sutures Crepitus at the clavicle

Fused cranial sutures Infants are born with open sutures that allow for growth of the brain; fused cranial sutures at birth would be the most concerning finding in a newborn.

Which is a sign of increased intracranial pressure in infants? Irritability Photophobia Vomiting and diarrhea Pulsating anterior fontanel

Irritability Rationale Irritability is one of the changes that may indicate increased intracranial pressure. Photophobia is not indicative of increased intracranial pressure. Vomiting and diarrhea suggest a gastrointestinal problem rather than increased intracranial pressure. A pulsating anterior fontanel is a normal variation observed in infants and is not a common sign of increased intracranial pressure.

How about lordosis?

Lateral convex/inward curvature of the cervical and lumbar spine. Butt and belly stick out because of inward curvature of lumbar area.

In a child with a complete spinal cord injury at T6, which interventions should the nurse implement to prevent complications? Administer saline nose spray for nasal congestion. Maintain patient's bed position at 45-degree angle. Notify health care provider for BP of 162/89 mm Hg. Cover with blanket when goose bumps are observed. Hold steroid medications for heart rate less than 50 bpm.

Maintain patient's bed position at 45-degree angle. Angling the bed at 45 degrees promotes venous blood return, decreased intracranial pressure (ICP), and keeping child's head midline. Correct Notify health care provider for BP of 162/89 mm Hg. The onset of autonomic dysreflexia in child with a complete spinal cord injury at T6 includes sudden increase in systolic blood pressure. Urgent intervention will be needed.

Acetabular dysplasia (preluxation) DDH

Mildest form of DDH; osseous hypoplasia of acetabular roof

Which statement concerning osteogenesis imperfecta (OI) is true? OI is easily treated. OI is an inherited disorder. Braces and exercises are of no therapeutic value. With a later onset, the disease usually runs a more difficult course.

OI is an inherited disorder. Rationale Osteogenesis imperfecta (OI) is an autosomal dominant inherited disorder, a lifelong problem caused by defective bone mineralization, abnormal bone architecture, and increased susceptibility to fracture. OI has a predictable course that is determined by the pathophysiologic processes, not the time of onset. Lightweight braces and splints can help support limbs and fractures.

what do we do for them anencephaly

Provide comfort with no effort at resuscitation. Very strong support of parents and counseling to cope with the birth and loss of infant. Parents need a lot of comfort and support.

Which statement explains how cerebral spinal fluid (CSF) maintains homeostasis? Drains into choroid plexus Produces white blood cells Flows through brain tissue Removes wastes from the brain

Removes wastes from the brain The CSF functions to maintain homeostasis by removing wastes from the brain because as it flows through the cranial vault and vertebral column it collects wastes and returns them to the venous system.

Motor Function

Spontaneous vs. response to pain Symmetrical vs. asymmetrical

myelomeningocele before and after surgery

They stay NPO until they have the surgery. So, they aren't breastfed a whole lot before surgery. They do try as much as possible, but again, if their in the first 24 hours, they're going to be NPO. After surgery what is one of the things, one of the assessment things we want to do? Measure head circumference. These kids are prone to hydrocephalus. So, we measure the head circumference. The cranial suture lines are not closed, so if anything gets blocked the brain swells. You measure same as abdominal circumference. Same place all the time

Forearm crutches

Usual selection for children who anticipate permanent use; used for paraplegic children who are unable to use braces:

Spina Bifida Occulta Diagnosis

X-ray MRI CT Ultrasound

Amelia

absence of limb

meningitis is caused

by pneumococcal and meningococcal infection mostly late winter and early spring

spondylolysis

fracture of pars interarticularis

causes of cognitive impairment

genetic biochemical infectious

postoperative complications of amputation

hemorrhage infection

Cognitive impairment (CI)

is a general term that encompasses any type of mental difficulty or deficiency. In this chapter, the term is used synonymously with intellectual disability (cognitive impairment ). The definition of CI (intellectual disability) emphasizes abilities, environments, supports, and empowerment. The intensity of support required is classified as intermittent, limited, extensive, or pervasive. The underlying assumption is that when appropriate supports are given over a prolonged period, the ability of the person with CI to function each day will generally improve.

do kids with CP have normal intelligence?

many have normal intellect

sporadic hemiplegic migrane

migrane with aura motor weakness no first or second degree relative who also has migrane with aura and motor weakness

exercises for reduceable contractures

passive range of motions exercises stretching active exercises

A nurse is providing education to parents who have a child recently diagnosed with a chronic illness. Which statement made by the nurse should be helpful for this family trying to cope with the changes the family is experiencing? "Coping with a chronic illness is not easy, but can be done if the family works to achieve cohesiveness." "Coping with a chronic illness can be very difficult and unfortunately leads to a weakened family unit." "A chronic illness will impact the daily life of your family and will require a lot of change for everyone involved." "Change impedes the process of normalization. Achieving normalcy will not be possible while dealing with a chronic condition.

"Coping with a chronic illness is not easy, but can be done if the family works to achieve cohesiveness." Cohesiveness is an important part of obtaining resiliency. The nurse will need to provide specific ways to obtain cohesiveness, but this answer provides hope and is a starting point for a family with a child newly diagnosed with a chronic illness.

The parents of an infant diagnosed with cystic fibrosis ask the nurse to define cystic fibrosis for them. Which response by the nurse is accurate? "Cystic fibrosis is an autosomal dominant disorder that affects the endocrine glands of the infant." "Cystic fibrosis is an autosomal recessive disorder that affects the infant's lungs, pancreas, and intestines." "Cystic fibrosis is an autosomal recessive disorder that affects the infant's ability to produce digestive enzymes." "Cystic fibrosis is an autosomal dominate disorder that mainly affects the infant's ability to exchange oxygen and carbon dioxide effectively."

"Cystic fibrosis is an autosomal recessive disorder that affects the infant's lungs, pancreas, and intestines." This response is appropriate since cystic fibrosis is an autosomal recessive disorder that affects the exocrine glands of the infant, including the lungs, pancreas, and intestines.

The parents of a toddler newly diagnosed with a chronic illness ask, "Will our little girl develop just like her peers?" How should the nurse respond? "Children with chronic illnesses often lag behind their peers developmentally." "Because your child was diagnosed with a chronic illness at such a young age, the outlook for the child is poor." "Development is unique to each child, but can be supported through consistency at home and community resources to help minimize any developmental delays." "A chronic illness does not usually impact the social or behavioral development, so your child might only struggle with physical development because of the chronic illness."

"Development is unique to each child, but can be supported through consistency at home and community resources to help minimize any developmental delays." This information provides accurate and reasonable expectations and gives the parents some control over how they can best assist the development of their child.

A school-aged child with a seizure disorder asks the nurse, "Will I have this for a long time?" What is the nurse's best response? "All chronic illnesses are lifelong conditions." "It just depends. Hopefully your disorder will go away as you get older." "A seizure disorder is a chronic illness, which means it will require long-term care." "Seizures are sometimes just a one-time occurrence. You will likely not have to deal with seizures your entire life."

"A seizure disorder is a chronic illness, which means it will require long-term care." Any chronic illness will require long-term care, although the severity of the illness can vary over time.

In teaching the mother, what response should the nurse suggest she use with Madison when Madison throws toys at her mother

"I am sad that you are throwing things at me."

Which statement by the nurse can explain the normal function of joints in the pediatric patient? "Joints are converted to bone." "Joints help your bones grow." "Joints help your bones move." "Joints firmly bind bones together."

"Joints help your bones move." Many joints function to allow movement of bones.

Which nursing response is appropriate for the parents of a child admitted to the pediatric intensive care unit for a submersion injury that feel guilty over the injury? "Tell me more about your feelings." "Your child will be fine, so don't worry." "Have you considered building a fence?" "You'll need to watch your son more closely in the future."

"Tell me more about your feelings." Rationale The nurse needs to be nonjudgmental and provide the parents an opportunity to express their feelings. Saying that the child will be fine may not be true. "Have you considered building a fence?" and "You'll need to watch your child more closely in the future," are judgmental responses.

Internal fixation

- Requires surgery to place screws, rods, staples and plates for fixation - Sometimes additional casting and bracing is needed - Hardware is usually removed after healing - Most common complication is infection

Distraction

- The process of separating opposing bone to encourage generation of new bone in the created space, i.e.: unequal bone lengths

Hydrocephalus Post operative management

-Routine post op care and observation -Positioning -Pain management -Monitor for signs of increased ICP, and infection -Family support and education

risk factors negative psychological effects

-responsibility for caregiving -differential treatment by parents - limitations in family reasources and recreational time

Coping mechanisms: Avoidance behaviors

1) result in movement away from adjustment and represent maladaptation to the crisis.

Aplastic Anemia Interventions

1. Prepare the child for bone marrow transplantation if planned. 2. Administer immunosuppressive medications as prescribed; anti-lymphocyte globulin or antithymocyte globulin may be prescribed to suppress the autoimmune response 3. Colony Stimulating factors may be prescribed to enhance bone marrow production 4. Corticosteroids and cyclosporine may be prescribed 5. Administer blood transfusions if prescribed and monitor for transfusion reactions.

overuse injuries

1.frictional or rubbing of one structure against another 2. tractional, or repeated pull on a ligament or tendon 3. cyclic, or repetitive loading of impact forces treatment - ice, rest NSAID

bone healing/stage 4/ ossification

3-10 weeks callus forms into bone, which grows beneath periosteum of fragments; fuses(knits together) fracture defect also called union stage

Disseminated Intravascular Coagulation (DIC)

A disorder of coagulation that occurs as a complication of numerous pathologic processes - it is diffuse fibrin deposition, consumption of anticoagulation factors and generation of thrombin and plamin complication of Hypoxia Acidosis Shock Endothelial damage CLOT AND BLEEDING AT THE SAME TIME

A lumbar puncture (LP) is being done on an infant with suspected meningitis. The nurse expects which results for the cerebrospinal fluid that can confirm the diagnosis of meningitis? a. WBCs; ¯glucose b. RBCs; normal WBCs c. glucose; normal RBCs d. Normal RBCs; normal glucose

ANS: A A lumbar puncture is the definitive diagnostic test. The fluid pressure is measured and samples are obtained for culture, Gram stain, blood cell count, and determination of glucose and protein content. The findings are usually diagnostic. The patient generally has an elevated white blood cell count, often predominantly polymorphonuclear leukocytes. The glucose level is reduced, generally in proportion to the duration and severity of the infection.

When is an autopsy required? a. In the case of a suspected suicide b. When a person has a known terminal illness c. With a hospice patient who dies at home d. With the victim of a motor vehicle collision

ANS: A Autopsy is usually required in cases of unexplained death, violent death, or suspected suicide. In other instances it may be optional, and parents should be informed. The cause of death is not unknown in a person with a known terminal illness, a hospice patient at home, or a victim of a motor vehicle collision. Autopsy can be requested by family, but it is not required.

A child has been admitted with status epilepticus. An emergency medication has been ordered. What medication should the nurse expect to be prescribed? a. Lorazepam (Ativan) b. Phenytoin (Dilantin) c. Topiramate (Topamax) d. Ethosuximide (Zarontin)

ANS: A For in-hospital management of status epilepticus, intravenous diazepam or lorazepam (Ativan) is the first-line drug of choice. Lorazepam is the preferred agent because of its rapid onset (2-5 minutes) and long half-life (12-24 hours) with few side effects

What is a major goal for the therapeutic management of juvenile idiopathic arthritis (JIA)? a. Control pain and preserve joint function. b. Minimize use of joint and achieve cure. c. Prevent skin breakdown and relieve symptoms. d. Reduce joint discomfort and regain proper alignment.

ANS: A The goals of therapy are to control pain, preserve joint range of motion and function, minimize the effects of inflammation, and promote normal growth and development. There is no cure for JIA at this time. Skin breakdown is not an issue for most children with JIA. Symptom relief and reduction in discomfort are important. When the joints are damaged, it is often irreversible

The nurse is preparing to admit an adolescent with bacterial meningitis. What clinical manifestations should the nurse expect to observe? (Select all that apply.) a. Fever b. Chills c. Headache d. Poor tone e. Drowsiness

ANS: A, B, C, E Clinical manifestations of bacterial meningitis in an adolescent include, fever, chills, headache, and drowsiness. Hyperactivity is present, not poor tone.

A young child with human immunodeficiency virus (HIV) is receiving several antiretroviral drugs. What is the purpose of these drugs? a. Cure the disease. b. Delay disease progression. c. Prevent spread of infection. d. Treat Pneumocystis carinii pneumonia.

ANS: B Although not a cure, these antiretroviral drugs can suppress viral replication, preventing further deterioration of the immune system, and delay disease progression. At this time, cure is not possible. Antiretroviral drugs do not prevent the spread of the disease. P. carinii prophylaxis is accomplished with antibiotics.

What is the single most prevalent cause of disability in children and responsible for the recent increase in childhood disability? a. Cancer b. Asthma c. Seizures d. Heart disease

ANS: B Asthma is the single most prevalent cause of disability in children and has been largely responsible for much of the recent increase in childhood disability.

The regulation of red blood cell (RBC) production is thought to be controlled by which physiologic factor? a. Hemoglobin b. Tissue hypoxia c. Reticulocyte count d. Number of RBCs

ANS: B Hemoglobin does not directly control RBC production. If there is insufficient hemoglobin to adequately oxygenate the tissue, then erythropoietin may be released. When tissue hypoxia occurs, the kidneys release erythropoietin into the bloodstream. This stimulates the marrow to produce new RBCs. Reticulocytes are immature RBCs. The "retic" count can be used to monitor hematopoiesis. The number of RBCs does not directly control production. In congenital cardiac disorders with mixed blood flow or decreased pulmonary blood flow, RBC production continues secondary to tissue hypoxia.

At which age do most children have an adult concept of death as being inevitable, universal, and irreversible? a. 4 to 5 years b. 6 to 8 years c. 9 to 11 years d. 12 to 16 years

ANS: C By age 9 or 10 years, children have an adult concept of death. They realize that it is inevitable, universal, and irreversible. Preschoolers and young school-age children are too young to have an adult concept of death. Adolescents have a mature understanding of death.

What medication is classified as an antiretroviral? a. Dapsone (Aczone) b. Pentamidine (Pentam) c. Didanosine (Videx) d. Trimethoprim-sulfamethoxazole (Bactrim)

ANS: C Classes of antiretroviral agents include nucleoside reverse transcriptase inhibitors (e.g., zidovudine, didanosine, stavudine, lamivudine, abacavir), nonnucleoside reverse transcriptase inhibitors (e.g., nevirapine, delavirdine, efavirenz), and protease inhibitors (e.g., indinavir, saquinavir, ritonavir, nelfinavir, amprenavir, lopinavir, ritonavir). Dapsone, pentamidine, and Bactrim are anti-infectives

When does idiopathic scoliosis become most noticeable? a. In the newborn period b. When the child starts to walk c. During the preadolescent growth spurt d. During adolescence

ANS: C Idiopathic scoliosis is most noticeable during the preadolescent growth spurt. It is seldom apparent before age 10 years.

A child with juvenile idiopathic arthritis (JIA) is started on a nonsteroidal antiinflammatory drug (NSAID). What nursing consideration should be included? a. Monitor heart rate. b. Administer NSAIDs between meals. c. Check for abdominal pain and bloody stools. d. Expect inflammation to be gone in 3 or 4 days.

ANS: C NSAIDs are the first-line drugs used in JIA. Potential side effects include gastrointestinal (GI), renal, and hepatic side effects. The child is at risk for GI bleeding and elevated blood pressure. The heart rate is not affected by this drug class. NSAIDs should be given with meals to minimize gastrointestinal problems. The antiinflammatory response usually takes 3 weeks before effectiveness can be evaluated.

What intervention is most appropriate for fostering the development of a school-age child with disabilities associated with cerebral palsy? a. Provide sensory experiences. b. Help develop abstract thinking. c. Encourage socialization with peers. d. Give choices to allow for feeling of control.

ANS: C Peer interaction is especially important in relation to cognitive development, social development, and maturation. Cognitive development is facilitated by interaction with peers, parents, and teachers. The identification with those outside the family helps the child fulfill the striving for independence. Sensory experiences are beneficial, especially for younger children. School-age children are too young for abstract thinking. Giving school-age children choices is always an important intervention. Providing structured choices allows for a feeling of control.

The nurse is counseling a pregnant 35-year-old woman about estimated risk of Down syndrome. What is the estimated risk for a woman who is 35 years of age? a. One in 1200 b. One in 900 c. One in 350 d. One in 100

ANS: C The estimated risk of Down syndrome for a 35-year-old woman is one in 350. One in 1200 is the risk for a 25-year-old woman, one in 900 is the risk for a 30-year-old woman, and one in 100 is the risk for a 40-year-old woman.

The nurse is teaching a parent of an infant to limit the amount of formula to encourage the intake of iron-rich food. What amount should the nurse teach to the parent? a. 500 ml b. 750 ml c. 1000 ml d. 1250 ml

ANS: C The nurse should teach the parent to limit the amount of formula to no more than 1 1/day to encourage intake of iron-rich solid foods.

What is the antiepileptic medication that requires monitoring of vitamin D and folic acid? a. Topiramate (Topamax) b. Valproic acid (Depakene) c. Gabapentin (Neurontin) d. Phenobarbital (Luminal)

ANS: D Children taking phenobarbital or phenytoin should receive adequate vitamin D and folic acid because deficiencies of both have been associated with these drugs.

The camp nurse is choosing a toy for a child with cognitive impairment to play with during swimming time. What toy should the nurse choose to encourage improvement of developmental skills? a. Dive rings b. An inner tube c. Floating ducks d. A large beach ball

ANS: D Toys are selected for their recreational and educational value. For example, a large inflatable beach ball is a good water toy; encourages interactive play; and can be used to learn motor skills such as balance, rocking, kicking, and throwing. Dive rings, an inner tube, and floating ducks are not interactive toys.

Which is the most common type of head injury in children? Deceleration injuries Deformation injuries Acceleration injuries Acceleration-deceleration injuries

Acceleration-deceleration injuries Rationale The most common type of head injury in infants is the acceleration-deceleration injury, not deceleration injury, deformation injury, or acceleration injury.

Which hematologic disorder is common in infancy and childhood? Anemia Leukemia Immune thrombocytopenia Disseminated intravascular coagulation

Anemia Rationale Anemia is the most common hematologic disorder of infancy and childhood. Leukemia is a neoplastic disorder. Immune thrombocytopenia and disseminated intravascular coagulation are hematologic disorders that are less common than anemia.

A nurse is caring for a young girl who fell and injured her knee. The young girl is very upset because she doesn't want to have a swollen leg. The nurse should prioritize which nursing intervention to help reduce swelling? Applying ice Taking analgesics Stretch the leg joints Recommending bed rest

Applying ice Applying ice can help reduce swelling and inflammation.

Platelets survive in the bloodstream for approximately 10 days. Identify causes of a platelet's shorter life span. Select all that apply. Antibiotic therapy Bacterial pneumonia Stage 2 hepatic cancer Daily vitamin supplements Temperature 104 degrees Fahrenheit

Bacterial pneumonia Infection can shorten a platelet's lifespan since the bacteria induce platelet activation. Once a platelet is activated, it becomes the target of phagocytes that begin to break down the cell. Bacteria also have cytotoxic compounds that induce apoptosis of the platelet. Stage 2 hepatic cancer Disease can shorten a platelet's lifespan since it affects the health of the liver, which is responsible for clotting factor production, and can affect overall health of the body and production of healthy megakaryocytes. Temperature 104 degrees Fahrenheit Fever can shorten a platelet's lifespan since the increased temperature creates an environment that is not favorable to the platelet's survival.

A 6-month-old infant does not smile, has poor head control, has a persistent Moro reflex, and often gags and chokes while eating. What are these findings are most suggestive of in this infant? Hypotonia Cerebral palsy Spinal cord injury Neonatal myasthenia gravis

Cerebral palsy Correct Poor head control, a persistent Moro reflex, and feeding difficulties in a 6-month-old infant are suggestive of cerebral palsy. Not smiling, poor head control, a persistent Moro reflect, and gagging and choking while eating are not consistent with hypotonia, spinal cord injury, or neonatal myasthenia gravis.

A child comes to a hospital with an amputated thumb. What is the appropriate method of handling the amputated thumb so that it can be reattached? Keep the thumb in ice and label the container. Cover the part in wet cloth and put it in an ice bag. Cover in gauze, seal in plastic bag, and keep the bag in ice water. Wash the part with water, wrap in plastic bag, and keep the plastic bag on ice

Cover in gauze, seal in plastic bag, and keep the bag in ice water. Rationale Any amputated body part such as a thumb can be reattached if the detached part is stored well. The thumb should be rinsed with normal saline. It is then wrapped in sterile gauze and sealed in an airtight bag. This bag is then placed in ice water, and the container is properly labeled. Placing the part directly on ice may damage the cells. Covering the thumb with a wet cloth and putting it in an ice bag may damage the thumb due to contact with ice. Washing in plain water would contaminate it and should be avoided.

The mother of a 4-year-old child with a heart defect is feeling overwhelmed with explaining an upcoming surgery to the child. What should the nurse do to help the parent's communication in this circumstance? Encourage the parent to show the child a picture of the heart Describe the operation in appropriate terms using dolls and a model of the heart Provide a pamphlet to the parents on congenital heart defects to review with the child Advise the parent to read a book about being in the hospital to the child and answer any questions

Describe the operation in appropriate terms using dolls and a model of the heart Describing the operation using age-appropriate terms and props provides the information necessary for the child to understand what is going to happen.

what is the setting sun sign

Downward deviation of gaze from ↑ICP (CNs III, IV, and VI)

The school nurse is caring for a child with hemophilia who fell on his arm during recess. What supportive measures would the nurse implement first? Apply warm, moist compresses. Apply a tourniquet for at least 5 minutes. Elevate the arm above the level of the heart. Begin passive range of motion unless pain is severe

Elevate the arm above the level of the heart. Correct The initial response should include elevation. Cold should be applied to the arm. This will aid in vasoconstriction. Pressure is effective in small areas but would not work for an extremity. Passive range of motion is not recommended. The child can perform active range of motion after the bleeding episode has resolved.

Incomplete fracture

Fracture in which fracture fragments remain attached:

Spiral

Fracture that is slanting and circular, twisting around the bone shaft:

An emergency department nurse prepares to treat a child with disseminated intravascular coagulation (DIC) experiencing increased clotting. The nurse reviews the health care provider's prescriptions and prepares to administer which medication? Heparin Succimer Vitamin K Protamine sulfate

Heparin Heparin is an anticoagulant that is used to treat clots.

Which is the best corrective measure to treat a child who has a shorter lower limb that causes asymmetrical weight bearing? Crutches Manual traction Prosthetic device Ilizarov external fixator

Ilizarov external fixator Rationale An Ilizarov external fixator is the common external fixation device used to lengthen bones. The child should be put on the device as soon as the child starts bearing weight. While on the external fixation device, partial weight bearing is allowed. The child should learn to use crutches to support walking. Manual traction is used to realign a bone. It does not promote bone growth. A prosthetic device is used in a limb deficiency.

Which treatment is appropriate to ease spasticity in a child with cerebral palsy? Exercise Diuretic medication Anticonvulsant medication Implanted medication pump

Implanted medication pump Rationale Implantation of a pump to deliver medication into the intrathecal space can help ease spasticity in a child with cerebral palsy. Exercises, though beneficial, will not be enough to ease the spasticity found in a child with cerebral palsy. Anticonvulsant medications are used when seizures occur in children with cerebral palsy. Diuretics do not help ease spasticity.

Plastic deformation

Occurs when bone is bent but not broken:

The nurse is caring for a pediatric patient with immune thrombocytopenic purpura (ITP) who is returning to the unit after a tonsillectomy. Which action by the nurse demonstrates appropriate understanding of patient safety? Pad the bed rails. Administer aspirin for pain. Start patient on a regular diet. Give the patient red cherry popsicles for comfort.

Pad the bed rails. The nurse would pad the side rails to protect the patient from injury that can result in increased bleeding.

Which white blood cell processes assist with immunity? Select all that apply. Phagocytosis Oxygen transport Antibody production Histamine degradation Hemoglobin production

Phagocytosis Monocytes and macrophages are responsible for phagocytosis of large cells. They help to rid the body of foreign organisms. Antibody production White blood cells contribute to antibody production during infection.

Match the meningeal layers and structures with the appropriate description.

Pia Vascular, transparent membrane Dura Composed of two meningeal layers Arachnoid Avascular, serous membrane Tentorium Separates Cerebrum from cerebellum

Which clinical manifestation is found in a child with spastic cerebral palsy? Hypotonicity Wide-based gait Dystonic movements Positive Babinski sign

Positive Babinski sign Rationale The Babinski sign, primitive reflexes, ankle clonus, exaggerated stretch reflexes, and contractures are clinical manifestations in children with spastic cerebral palsy. Hypertonicity, rather than hypotonicity, is present. A wide-based gait is found in ataxic cerebral palsy. Dystonic movements are more common in dyskinetic cerebral palsy.

prevention of complications of anemia

Prevent infection, good hygiene, adequate nutrition Anticipatory guidance and education Iron-rich foods Support family Iron give with food Give with orange juice or vitamin C to absorb faster Stain teeth use a straw GI can constipate them Lab test to see if they work

Dystonic

Slow, twisting movement of the trunk or extremities; abnormal posture

Neural Tube Defects. What do I mean when I say NTD?

Spina bifida, anencephaly, myelodysplasia, myelomeningocele (pages 1310-1320) any malformation of the spinal canal and/or cord. Somewhere in utero something didn't close/complete.

Match the white blood cell with its function.

T-cell cell-mediated immunity B-cell humoral immunity (antibody production) Basophil activate inflammatory response Monocyte fight chronic infection

Match the cerebral palsy (CP) symptoms to the nursing intervention for the hospitalized pediatric patient.

Tense muscles Continue to monitor Visible tremors Provide guided imagery Loss of coordination Place bed in lowest position Involuntary movement Administer a benzodiazepine

What anatomical features of pediatric bone reduce the risk of fracture compared to adults? Select all that apply. Thick periosteum Articular cartilage Epiphyseal (growth) plate Presence of calcium salts Increased cartilage to bone ratio

Thick periosteum Children have an increased thickness of the periosteum compared to adults. This provides additional protection against fracture. Epiphyseal (growth) plate The epiphyseal (growth) plate is not sealed in the child. It still contains hyaline cartilage, which acts as a shock absorber. Increased cartilage to bone ratio There is an increased cartilage to bone ratio in pediatric bone. The bone is more flexible and therefore less likely to fracture.

Which mechanism immediately follows tissue injury? Vessel spasm Fibrin mesh forms Release of clotting factor Increased platelet formation

Vessel spasm The immediate response of a blood vessel to injury is vasoconstriction or vascular spasm.

Which vitamin can most affect the clotting process? Calcium Potassium Vitamin C Vitamin K

Vitamin K Vitamin K is essential since it affects the clotting process—it is essential for the production of four distinct clotting factors: prothrombin, and factors VII, IX, and X.

which nursing diagnosis for patient with cleft palate should be addressed first

altered nutrition

skeletal traction

applied directly to the skeletal structure by a pin, wire, or tongs inserted into or through the diameter of the bone distal to the fracture

Which nursing action has the highest priority?

assess the appearance of Madison's foot

Major goals in the therapeutic management of children with muscular dystrophy include which of the following? a. Promoting strenuous activity and exercise b. Promoting large caloric intake c. Promoting optimum function in all muscles for as long as possible d. Preventing cognitive impairment

c. Promoting optimum function in all muscles for as long as possible

decreased muscle tone in down syndrome

compromises respiratory expansion

Which one of the following is not considered a standard treatment for GBS? a. Plasmapheresis b. Corticosteroids c. Administration of IVIG d. Administration of neostigmine

d. Administration of neostigmine

encouraging expression of emotion

describe the behavior - you seem angry at everypne give evidence of understanding - being angry is only natural give evidencedence of caring - it must be difficult to endure so many painful procedures help focus on feelings - maybe you wonder why this happened to you

dura mater

double-layered membrane that serves as the outer meningeal layer and the inner periosteum of the cranial bones

ephedra

increased alertness but may lead †o exceed †he limits in use wi†h caffeine burns fat makes person more agressive

Which change in serum lab values would most likely indicate the onset of osteomyelitis?

increased erythrocyte sedimentation rate (ESR)

care of myelomeningocele sac

infant place in intubator without clothing wet dressing is placed on sac monitor 2-4 hrs that dressing is not dry check sac for leaks, abrasion, irritation, signs of infection prono position diapering contraindicated until repair

therapeutic management of down syndrom

no cure - surgery to correct congenital abnormalities - evaluation of sight and hearing and treatment of otitis media - periodic testing of thyroid function

neurological examination of reflesxes

normal vs persistent (know when they should be gone) moro: expected until 4 months tonic neck: asymmetrical type of response - turn head, one leg extends, another flexes (way head turns = extended leg), expected until 4 months, after that severe brain damage babinski until 1 yo, after that pyramidal sign, often in seizures

anger outward

open argument withdrawal from communication

Based on newborn assessment, Which areas need to be addressed?

sucking reflex, palate intact, nares patent

Special needs of the family facing the unexpected death of a child include

support while awaiting news of the child's status; a sensitive pronouncement of death; acknowledgment of feelings of denial, guilt, and anger; an opportunity to view the body; closure; and referrals for support.

leading cause of death children 1-19

unintentional injuries (mostly motor vehicles)

A parent of a six- and four-year-old, the youngest one with juvenile arthritis, states, "I feel terrible when I have to discipline the youngest one." What is the nurse's best response? "A child with a chronic illness should not be disciplined because it causes unnecessary stress." "Permissive parenting of children with a chronic illness can cause long-term damage to the child." "Although disciplining a child with a chronic illness can be hard, it is important for the development of the child." "It is important that your older child sees that you still discipline the four-year-old so that there is not any resentment."

"Although disciplining a child with a chronic illness can be hard, it is important for the development of the child." This is the best response since it is important for children to have discipline in order to maintain age-appropriate expectations and development.

The parents of an infant suspected to have fragile X syndrome ask the nurse which testing confirms the suspicions. Which response by the nurse is appropriate? "DNA testing is the definitive method for diagnosing fragile X syndrome." "Karyotyping identifies the defective chromosome needed to diagnose fragile X syndrome." "An abnormal triple maternal serum screen value confirms the suspicion of fragile X syndrome." "A genogram confirms the presence of the genetic disorder and is the definitive method for diagnosing fragile X syndrome."

"DNA testing is the definitive method for diagnosing fragile X syndrome." DNA testing is the definitive method for diagnosing fragile X syndrome. Excessive repetition of the cytosine-guanine-guanine sequence is identified in affected individuals.

The nurse is caring for a newborn with Down syndrome. The parents state, "The health care provider just informed us that our daughter has Down syndrome. How long will my child be sick?" How should the nurse respond? "All chronic conditions are lifelong conditions." "I am very sorry that your daughter was born with this lifelong illness." "Down syndrome is a chronic condition, so the child will always have this condition." "Your child is not sick but has a developmental disability that will require her to need care her entire life."

"Down syndrome is a chronic condition, so the child will always have this condition." The nurse knows that many chronic illnesses are lifelong and that Down syndrome is a type of chronic illness that lasts a lifetime.

Which statement is appropriate when explaining the diagnosis of dystonic cerebral palsy (CP) to the parents of a 2-month-old infant that is showing slow, twisting movements of the trunk and abnormal posture? "The child has CP, in which upper motor neuron muscular weakness occurs." "The child has CP, in which altered skeletal muscle performance occurs." "Your child shows disintegration of movements of the upper extremities." "Your child has CP, in which combination of spastic and dyskinetic CP symptoms occur."

"The child has CP, in which upper motor neuron muscular weakness occurs." Rationale There are different types of CP. The dystonic CP is a condition in which involuntary muscle contractures and abnormal posture occurs in the child. This spastic CP represents an upper motor neuron muscular weakness. Athetoid and dystonic CP are nonspastic CP. Therefore the nurse should not describe the condition as one in which altered skeletal muscle performance occurs. Disintegration of movements of the upper extremities is noticed during ataxic CP. In such condition the common symptoms are wide-based gait and rapid, repetitive movement. Again, the child does not have mixed CP. In mixed CP, symptoms of spastic and dyskinetic CP are observed.

Immobility effects on the respiratory system

- Atelectasis and pneumonia R/T shallow respirations, supine position, decreased movement of secretions, stasis of secretions, airway obstruction with mucus

Immobility effects on metabolism

- Bone break down causes hypercalcemia > nausea, vomiting, polydipsia, polyuria, lethargy appear 4-8 weeks after immobilization - Hypercalcemia depresses the CNS and the PNS > muscle fatigue, diminished reflexes, atony of the GI hypercalcemia symptoms - nausea, vomiting, polidipsia, polyuria, lethargy

nursing care management osteomyelitis

-Immobility of limb in acute phase -Pain management -Long-term IV access (for antibiotic administration) -Psychosocial needs check vital especially temp wound care i/o, wound drainage monitor

The nurse understand that which signs and symptoms are indicative of the osteomyelitis complication? (select all that apply)

-pain that increases with movement -edema -irritability

adolescence

1) Developing independence and autonomy 2) A sense of feeling different from peers can lead to loneliness, isolation, and depression major task - planning for the future, -> physical independence, responsibility for behavior, make decision about life, maintain supportive social relationships needs teaching to distinguish between changes due tp age and due to disease

educating the family about illness

1) Educating the family about the disorder is an extension of revealing the diagnosis. 2) Parents also need guidance in how the condition may interfere with or alter activities of daily living, such as eating, dressing, sleeping, and toileting 3) Nutrition can often be affected-overnutrition and undernutrition. 4. modification of safe transportation ( like different restraints, wheelchairs for cars) 5. attention to injury prevention, immunization. dental health and regular physical examination 6. specifics of the medical emergency

4 types of parental reactions during adjustment period

1) Overprotection: parents fear letting the child achieve any new skill, avoid all discipline, cater to every desire to prevent frustration. 2) Rejection: parents detach themselves emotionally, but provide adequate physical care or constantly nag and scold the child 3) Denial: acts as if disorder does not exist or attempt to have the child overcompensate for it 4) Gradual acceptance: necessary and realistic restrictions on the child, encourage self-care activities, promote reasonable physical and social abilities

Concurrent stresses within the family

1) The ability to deal with the overwhelming stress of a chronic illness is challenged further when additional stresses are present. (marital issues, sibling needs, social isolation, homelessness) managing home, travelling for treatments

Aplastic Anemia Description

1. Aplastic anemia is a deficiency of circulating erythrocytes and all other formed elements of blood, resulting from the arrested development of cells within the bone marrow. 2. It can be primary (present at birth) or secondary (acquired) 3. Several possible causes exist, including chronic exposure to myelotoxic agents, viruses, infection, autoimmune disorders, and allergic states. 4. The definitive diagnosis is determined by bone marrow aspiration (shows conversion of red bone marrow to fatty bone marrow) 5. Therapeutic management focuses on restoring function to the bone marrow and involves immunosuppressive therapy and bone marrow transplantation (treatment of choice if a suitable donor exists.) 6. If the cause is a myelotoxic medication that is being administered for another purpose, the medication may be discontinued to improve bone marrow function.

Hemophilia Interventions

1. Monitor for bleeding and maintain bleeding precautions 2. Prepare to administer factor VIII concentrates, either produced through genetic engineering (recombinant) or derived from pooled plasma, as prescribed 3. DDAVP (1-deamino-8-D-arginine vasopressin) a synthetic form of vasopressin, increases plasma factor VIII and may be prescribed to treat mild hemophilia 4. Monitor for joint pain; immobilize the affected extremity if joint pain occurs 5. Assess neurological status (child is at risk for intracranial hemorrhage) 6. monitor urine for hematuria 7. Control joint bleeding by immobilization, elevation, and application of ice; apply pressure (15 minutes) for superficial bleeding. 8. Instruct parents and child about the signs of internal bleeding. 9. Instruct parents in how to control the bleeding. 10. Instruct parents regarding activities for the child, emphasizing the avoidance of contact sports and the need for protective devices while learning to walk;; assist in developing an appropriate exercise. 11. Instruct the child to wear protective devices such as helmets and knee and elbow pads when participating in sports such as bicycling and skating.

ADAPTIVE TASKS of parents having children with chronic conditions

1. accept the child's condition 2. manage the child's condition on a day-to day basis 3. meet the child's normal developmental needs 4. meet the develpmental needs of other family members 5. cope with ongoing stress and periodic crises 6. assist family members to manage their feelings 7. educate others about the child's condition 8. establish a support system

Features of Fractures in Children

1. damage to the growth plate can cause shortening and often a progressive angular deformity 2. periosteum of a child's bone is thicker and stronger and has more ostogenic potantial 3. bone are more porous, which allows bend, buckle and break in a greenstick manner 4. healing is more rapid 5. stiffness is unusual even after long time immobilization 6. children complain only when something is wrong

head injury assessment: increased ICP

1. early signs a. slight change in vital signs b. slight change in level of consciousness c. infant: irritability, high pitched cry, bulging fontanel, increased head circumference, dilated scalp veins, Macewens sign (cracked pot sound on percussion of head), setting sun sign (Sclera visible above iris) d. child: headache, nausea, vomiting, visual disturbances (diplopia), seizures 2. late signs a. significant decrease in level of consciousness b. brachycardia c. decreased motor and sensory responses d. alteration in pupil size and reactivity e. decorticate (flexion) posturing: adduction of the arms at the shoulders; arms are flexed on the chest with the wrists flexed and the hands fisted, and the lower extrememities are extended and adducted; seen with severe dysfunction of cerebral cortex. f. decerebrate (Extension) posturing: rigid etension and pronation of the arms and legs; sign of dysfunction at the level of the midbrain g. cheyne-stokes respirations h. coma

hydrocephalus postoperative interventions

1. monitor vital signs and neurological signs 2. position the child on the unoperated side to prevent pressure on the shunt valve 3. keep the child flat as prescribed to avoid rapid reduction of intracranial fluid 4. observe for increased ICP; if increased ICP occurs, elevate the head of the bed to 15 to 30 degrees to enhance gravity flow through the shunt. 5. measure head circumference 6. monitor for signs of infection and assess dressings for drainage 7. monitor intake and output 8. provide comfort measures and administer medications as prescribed 9. instruct parents on how to recognize shunt infection or malfunction 10. in an infant, irritability; a high, shrill cry, lethargy, and feeding poorly may indicate shunt malfunction or infection. 11. in a toddler, headache and a lack of appetite are the earliest common signs of shunt malfunction 12. in older children, an indicator of shunt malfunction is an alteration in the child's level of consciousness.

nursing care management anemia

1. nutrition, especially if lactose intolerant 2. past history of chronic, recurrent infection 3. eating habits, particularly pica 4. bowel habits and presence of frank stool or black tarry stool 5. familial history of hereditary diseases Prepare child and family for laboratory tests Topical application of EMLA before needle punctures can eliminate pain Explain the significance of each test (why all tests are not done at once) Encouraging parents or another supportive person to be with child during procedure Allowing child to play with equipment on a doll or participate in the actual procedure Decrease oxygen demands (rest) Assess tolerance of ADL's - activities throughout the dsy with periods of rest, if cannot walk to bathroom, bedside comode, avoid stress like from separating from parent - crying requires energy

child support

1. observe responses to disorder, ability to function, adaptive behavior 2. explores the child's own understanding of his/her illness 3. support while child learns to cope 4. encourage to express conserns 5. alleviate feelingsof being different 6. assist in child's routine, normal aspects of appearance and capabilities

traction for reducton of fractures

1. to fatigue the involved muscle and reduce muscle spasm so that the bones can be realligned 2. to position the distal and proximal bone ends in the desired realignment to promote satisfactory bone healing 3. to immobilize the fracture site until realignment has been achieved and sufficient healing has taken place to permit casting or splinting

The nurse is caring for a child with prolonged bleeding after losing a tooth. The nurse notes bruising around the gum line. Which laboratory values would be concerning to the nurse? Select all that apply. Hemoglobin 11 Fibrinogen level 467 Platelet count 162,000 2% factor VIII activity Prothrombin time (PT) 20 seconds

2% factor VIII activity Factor VIII activity of 2% indicates moderate hemophilia and would be concerning for the nurse. Correct Prothrombin time (PT) 20 seconds A PT of 20 seconds is considered prolonged bleeding and would be a concerning finding.

Which percentage is appropriate when describing the chances that a hemophilic male and a female who carries the gene for hemophilia will have a normal male offspring? 0% 25% 50% 100%

25% Rationale Hemophilia is an X-linked recessive disorder. The chances are equal (1 in 4) that the offspring of an affected male and carrier female will be a normal son, an affected son, an affected daughter, or a carrier daughter. The chance of having a normal son is 25%. Hence, the options 0%, 50%, and 100% are incorrect.

immobility effects on muscular system

3% a day mass loss disuse leads to atrophy - tissue breakdown effect cardiovascular by decreasing venous return and cardiac output decreased strength and endurance in the absence of natural stretching, colagen fibers become fibrotic and limit movement. - shortening of the muscles and contracture of the joint cyclic - contracture -> pain -> child protects site-> more fibrosis

A child with beta-thalassemia is receiving long-term blood transfusion therapy for the treatment of the disorder. Chelation therapy is prescribed as a result of too much iron from the transfusions. Which medication should the nurse anticipate to be prescribed? 1. Fragmin 2. Meropenem (Merrem) 3. Metoprolol (Toprol-XL) 4. Deferoxamine (Desferal)

4 Beta-thalassemia is an autosomal recessive disorder characterized by the reduced production of one of the globin chains in the synthesis of hemoglobin (both parents must be carriers to produce a child with beta-thalassemia major). The major complication of long-term transfusion therapy is hemosiderosis. to prevent organ damage from too much iron, chelation therapy with either Exhade of deferoxamine (Desferal) may be prescribed. Deferoxamine is classified as an antidote for acute iron toxicity. Fragmin is an anticoagulant used as prophylaxis for postop DV. Meropenem is an antibiotic. Metoprolol is a beta blocker used to treat hypertension

he clinic nurse instructs parents of a child with sickle cell anemia about the precipitating factors related to sickle cell crisis. Which, if identified by the parents as a precipitating factor, indicates the need for further instructions? 1. Stress 2. Trauma 3. Infection 4. Fluid overload

4 Sickle cell crises are acute exacerbations of the disease, which vary considerably in severity and frequency; these include vaso-occlusive crisis, splenic sequestration, hyperhemolytic crisis, and aplastic crisis. Sickle cell crisis may be precipitated by infection, dehydration, hypoxia, trauma, or physical/emotional stress. The mother of a child with sickle cell disease should encourage fluid intake of 1 1/2 to 2 times the daily requirement to prevent dehydration.

A patient is born with spina bifida, a neural tube defect. The nurse understands that this defect occurred during what week of gestation? 4th week of gestation 15th week of gestation 20th week of gestation 30th week of gestation

4th week of gestation The 4th week of gestation is the stage in which the neural tube closes. Lack of maternal folic acid during first 28 days of gestation is considered a teratogenic cause of spinal cord malformations.

Which setting is appropriate when adjusting the flow rate to 0.5mL/minute during a blood transfusion? 5 to 7 drops/minute 12 to 15 drops/minute 20 to 25 drops/minute 30 to 40 drops/minute

5 to 7 drops/minute Rationale In children, the infusion pumps of blood administration sets use regular drops rather than microdrops. One milliliter consists of 10 or 15 regular drops. Thus, a flow rate of 0.5 ml/minute will be 5 to 7 drops/minute. Therefore 12 to 15, 20 to 25, and 30 to 40 drops/minute are not correct.

Nutritional needs for sports activities

50% of diet from carbs protein 1.2-1.7 g /kg/day 25-30% from fat - More calories, esp carbs, iron, calcium, water, electrolytes (Na and Cl) - For adolescents 14-18 years: Calcium: 1300 mg/day (orange juice, calcium, pudding) Vitamin D 400-1000 IU/day

bone healing/stage 3/ callus formation

6-10 DAYS - fibroblasts form in granulation tissue; form bone in areas adjacent to surface of bone shaft; form cartilage at surfaces more distal to blood supply - provisional callus develops, bringing fracture ends; holds bone together but will not support body weight 14-21 DAYS - true callus develops, seen on radiograph, more than needed is formed, but with remodelling, excess callus is resorbed - cartilage defferentiates to bone tissue

90-90 traction

90 degree angle at knee and hip, facilitates position changes and toileting the lower leg is supported by a boot cast or a calf sling and a skeletal wire is placed

The clinic nurse wants to develop a teaching program for parents of patients at risk for developing iron deficiency anemia (IDA). Which patient does the nurse correctly identify as being at greatest risk for developing IDA? A 4-year-old child who will only eat cereal A 5-year-old child who lives in a home built in 1942 A 4-year-old child whose father has sickle cell anemia A 6-year-old child whose mother had gestational diabetes

A 5-year-old child who lives in a home built in 1942 Older homes are more likely to have lead paint and pipes. Lead exposure is a risk factor for iron deficiency and IDA.

Why is it important to identify the special needs for children with chronic and terminal illness? The definition helps to define the illness trajectory that a chronic illness might take. The definition describes the specific types of support and interventions that a child and family will need. A specific definition of chronic and terminal illness allows for easier billing and obtaining insurance coverage for treatments. A clear definition can help families and caregivers obtain the support necessary to adequately care for children with special health care needs.

A clear definition can help families and caregivers obtain the support necessary to adequately care for children with special health care needs. A clear definition of special health needs allows for enhanced advocacy and planning care specific to this category of chronic conditions.

A terminally ill child is receiving morphine sulfate (Morphine) and is experiencing respiratory depression. A health care provider prescribes naloxone (Narcan), 0.5 mcg/kg IV in 2 minute increments until breathing improves. The medication label states: "Naloxone 400 mcg/1 ml." The child weighs 60 kg. The nurse prepares to administer one dose. How many milliliters will the nurse prepare to administer one dose? Fill in the blank. Record your answer using two decimal places.

ANS: 0.08 ml Follow the formula for dosage calculation. Multiply 0.5 mcg ´ 60 kg to get the dose = 30 mcg Desired ———- ´ Volume = ml per dose Available 30 mcg ———- ´ 1 ml = 0.075 ml rounded to 0.08 ml 400 mcg

What type of seizure may be difficult to detect? a. Absence b. Generalized c. Simple partial d. Complex partial

ANS: A Absence seizures may go unrecognized because little change occurs in the child's behavior during the seizure. Generalized, simple partial, and complex partial all have clinical manifestations that are observable.

What pain medication is contraindicated in children with sickle cell disease (SCD)? a. Meperidine (Demerol) b. Hydrocodone (Vicodin) c. Morphine sulfate d. Ketorolac (Toradol)

ANS: A Meperidine (pethidine [Demerol]) is not recommended. Normeperidine, a metabolite of meperidine, is a central nervous system stimulant that produces anxiety, tremors, myoclonus, and generalized seizures when it accumulates with repetitive dosing. Patients with SCD are particularly at risk for normeperidine-induced seizures.

The nurse understands that which gestational disorders can cause a cognitive impairment in the newborn? (Select all that apply.) a. Prematurity b. Postmaturity c. Low birth weight d. Physiological jaundice e. Large for gestational age

ANS: A, B, C Prematurity, postmaturity, and low birth weight can be causes of cognitive impairment in newborns. Physiological jaundice and large for gestational age are not associated causes of cognitive impairment in newborns.

The nurse is teaching coping strategies to parents of a child with a chronic illness. What coping strategies should the nurse include? (Select all that apply.) a. Listen to the child. b. Accept the child's illness. c. Establish a support system. d. Learn to care for the child's illness one day at a time. e. Do not share information with the child about the illness.

ANS: A, B, C, D Coping strategies for parents caring for a child with a chronic illness include listening to the child, accepting the child's illness, establishing a support system, and learning to care for the child's illness one day at a time. Information should be shared with the child about the illness.

What are signs and symptoms of anemia? (Select all that apply.) a. Pallor b. Fatigue c. Dilute urine d. Bradycardia e. Muscle weakness

ANS: A, B, E Signs and symptoms of anemia include, pallor, fatigue, and muscle weakness. Tachycardia, not bradycardia, and dark urine, not dilute, are signs and symptoms of anemia.

The nurse is preparing to admit a 5-year-old child with a lower motor neuron syndrome. What clinical manifestations of a lower motor neuron syndrome should the nurse expect to observe? (Select all that apply.) a. Loss of hair b. Babinski reflex present c. Skin and tissue changes d. Marked atrophy of atonic muscle e. Hyperreflexia with tendon reflexes exaggerated

ANS: A, C, D Clinical manifestations of a lower motor neuron syndrome include loss of hair, skin and tissue changes, and marked atrophy of atonic muscle. Babinski reflex present and hyperreflexia with tendon reflexes exaggerated are manifestations of an upper motor neuron syndrome.

What are supportive interventions that can assist a school-age child with a chronic illness to meet developmental milestones? (Select all that apply.) a. Encourage socialization. b. Discourage sports activities. c. Encourage school attendance. d. Provide instructions on assertiveness. e. Educate teachers and classmates about the child's condition.

ANS: A, C, E To develop a sense of accomplishment, school attendance should be encouraged, and teachers and classmates should be educated about the child's condition. To form peer relationships, socialization should be encouraged. Sports activities should be encouraged (e.g., Special Olympics), not discouraged. Providing instructions on assertiveness is appropriate for adolescence.

The school nurse recognizes that the adverse effects of performance-enhancing substances can include what? (Select all that apply.) a. Depression b. Dehydration c. Hypotension d. Aggressiveness e. Changes in libido

ANS: A, D, E Mood changes have been observed as adverse effects of using performance-enhancing substances, including aggressiveness, changes in libido, depression, anxiety, and psychosis. Fluid retention, not dehydration, and hypertension, not hypotension, are adverse effects of performance-enhancing substances.

Gingivitis is a common problem in children with cerebral palsy (CP). What preventive measure should be included in the plan of care? a. High-carbohydrate diet b. Meticulous dental hygiene c. Minimum use of fluoride d. Avoidance of medications that contribute to gingivitis

ANS: B Meticulous oral hygiene is essential. Many children with CP have congenital enamel defects, high-carbohydrate diets, poor nutritional intake, and difficulty closing their mouths. These, coupled with the child's spasticity or clonic movements, make oral hygiene difficult. Children with CP have high carbohydrate intake and retention, which contribute to dental caries. Use of fluoride should be encouraged through fluoridated water or supplements and toothpaste. Certain medications such as phenytoin do contribute to gingival hyperplasia. If that is the drug of choice, then meticulous oral hygiene must be used.

What term is used to describe an abnormally increased convex angulation in the curvature of the thoracic spine? a. Scoliosis b. Lordosis c. Kyphosis d. Ankylosis

ANS: C Kyphosis is an abnormally increased convex angulation in the curvature of the thoracic spine. Scoliosis is a complex spinal deformity usually involving lateral curvature, spinal rotation causing rib asymmetry, and thoracic hypokyphosis. Lordosis is an accentuation of the cervical or lumbar curvature beyond physiologic limits. Ankylosis is the immobility of a joint.

The clinical manifestations of sickle cell anemia (SCA) are primarily the result of which physiologic alteration? a. Decreased blood viscosity b. Deficiency in coagulation c. Increased red blood cell (RBC) destruction d. Greater affinity for oxygen

ANS: C The clinical features of SCA are primarily the result of increased RBC destruction and obstruction caused by the sickle-shaped RBCs. When the sickle cells change shape, they increase the viscosity in the area where they are involved in the microcirculation. SCA does not have a coagulation deficit. Sickled red cells have decreased oxygen-carrying capacity and transform into the sickle shape in conditions of low oxygen tension.

What condition precipitates polycythemia? a. Dehydration b. Severe infections c. Immunosuppression d. Prolonged tissue hypoxia

ANS: D Oxygen transport depends on both the number of circulating RBCs and the amount of normal hemoglobin in the cell. This explains why polycythemia (increase in the number of erythrocytes) occurs in conditions characterized by prolonged tissue hypoxia, such as cyanotic heart defects. Dehydration, severe infections, or immunosuppression will not precipitate polycythemia.

How about scoliosis? What is scoliosis?

Abnormal lateral curvature of the spine. When do we usually pick up on scoliosis? Adolescence. How do you detect it? What do you look for? How do you check for it? Page 1276. Wearing only shorts or briefs viewed from behind, exhibit asymmetry of shoulder height, scapular or flank shape, hip height, or pelvic obliquity. When the child bends forward at the waist, asymmetry of ribs and flanks. Scoliometer used in initial screening. Definitive Dx = radiography and Cobb measurement.

A 4-year-old child with cystic fibrosis presents to the emergency department with a 1-day history of a productive cough, wheezing, fever, and poor appetite. The child's oxygen saturation level is 85% on room air, respiratory rate of 22 and shallow, and heart rate of 120. Which action is priority for the nurse to take? Start bronchodilator nebulization. Administer oxygen via nasal cannula. Obtain a temperature to assess for fever. Perform chest physiotherapy twice a day

Administer oxygen via nasal cannula. Administering oxygen at 2 LPM is a priority action for a child with cystic fibrosis whose oxygen saturation is 85%. Oxygen inhalation delivers sufficient oxygen to the lungs and facilitates gas exchange.

Reciprocal gait orthosis (RGO)

Allows children with paraplegia to walk on a flat surface; used in children with spinal cord injury, sacral agenesis, and spina bifida:

etiologic risk factors for cerebral palsy

Any perinatal or neonatal brain lesion or brain maldevelopment, regardless of the cause, may be linked to as many as 80% of the total cases of CP Prenatal/postnatal infection Prenatal/postnatal hypoxia/asphyxia Often no identifiable immediate cause Know this Delay in oxygen Trauma can lead to CP Physical delay Preterm birth of ELBW and VLBW is single most important determinant of CP Anoxia—most common cause of brain damage whenever it occurs Anoxia- in utera, birth or shortly after perinatal ischemic stroke postnatally factors - bacterial meningitis, multiple births, viral encephalitis, motor vehicle accidents, child abuse, shaken baby syndrme

A child with sickle cell disease expresses a desire to spend more time with a sibling who has not been seen because the family lives far away from the hospital. Which actions by the nurse should be most appropriate? Ask the health provider for a day pass so the ill child can visit the sibling. Ask the parents for a picture of the sibling to keep at the ill child's bedside. Ask the parents to bring the sibling to visit in the next few days when possible. Provide the child with books and movies to distract the ill child from thinking about the sibling.

Ask the parents to bring the sibling to visit in the next few days when possible. It is important to attempt to maintain the sibling relationships. Bringing the sibling to visit allows them to reconnect and maintain their sibling relationship.

Question 1 of 3 A 3-month-old infant is seen in the emergency department and an x-ray shows a femur fracture. What is the primary nursing assessment? Assess pedal pulse in affected limb Assess parent knowledge on infant care Assess family home and financial situation Assess feeding schedule and weight gain of infant

Assess pedal pulse in affected limb Circulation of the injured limb is a priority assessment to detect potential serious complications.

The nurse is caring for a patient in a Pavlik harness. Which critical task specific to this harness should be performed by the nurse? Ensure immobility Assess the skin under the harness Assess the anxiety level of the parents Ensure the harness is tight against the chest

Assess the skin under the harness Monitoring the skin integrity of child in a Pavlik harness or spica cast is important because extended wear can lead to impaired skin integrity.

A child is in skeletal traction. What nursing interventions will decrease the risk of infection? Select all that apply. A. Check pin sites for signs of redness and inflammation. B. Use soap and water to cleanse the area around the pins. C. Leave drainage in place to keep from disrupting the area where the pins are arranged on the skin. D. Careful examination of the pin entry sites to view any signs of loose pins. E. Remove the patient from traction for a short period of time to improve circulation.

Check pin sites for signs of redness and inflammation. Check the pin entry sites for redness and inflammation that may indicate infection. Correct Use soap and water to cleanse the area around the pins. Cleanse the area around the pin entry sites with soap and water. Other approved cleanser may also be used. Careful examination of the pin entry sites to view any signs of loose pins. To decrease the likelihood of an infection in a patient with pins from traction, meticulous assessment of entry sites for signs of loose pins is needed.

Which procedure is appropriate when identifying the means of eliminating excess iron in a child with thalassemia major? Antiemetics Splenectomy Chelation therapy Blood transfusions

Chelation therapy Rationale Chelation therapy minimizes the development of hemosiderosis (iron overload), a complication of blood transfusions. Antiemetics help ease nausea and vomiting. Splenectomy is necessary when severe splenomegaly develops. Blood transfusions are the primary medical management.

How is complex partial seizure similar to absence seizure? Postictal impairment is absent. Mental disorientation is common. Clonic movements may occur occasionally. Seizure usually lasts more than 60 seconds.

Clonic movements may occur occasionally. Rationale Clonic movements may occur occasionally in both complex partial seizures and absence seizures. Postictal impairment is absent in absence seizures but frequently seen in complex partial seizures. Mental disorientation is common in complex partial seizures and unusual in absence seizure. The seizure usually lasts more than 60 seconds in complex partial seizures. However, it usually lasts less than 10 seconds in absence seizures.

Which statement is appropriate when describing pseudohypertrophic (Duchenne) muscular dystrophy (DMD)? DMD is inherited as an autosomal dominant disorder. The onset of DMD occurs in later childhood and adolescence. DMD is characterized by muscle weakness, usually beginning around the age of 3 years. DMD is characterized by weakness of the proximal muscles of both the pelvic and shoulder girdles.

DMD is characterized by muscle weakness, usually beginning around the age of 3 years. Rationale Usually children with DMD reach the early developmental milestones, but the muscle weakness is initially observed in the third year of life. DMD is inherited as an X-linked recessive disorder. Weakness in a child with DMD is usually first noted in walking. Progressive muscle weakness in other muscle groups then follows.

Which symptoms are appropriate in a child admitted for anemia related to increased blood cell destruction? Select all that apply. Pallor Jaundice Cool skin Dark urine Splenomegaly

Dark urine Splenomegaly Jaundice Rationale Symptoms the nurse would expect in a child admitted for anemia related to increased blood cell destruction include jaundice, dark urine, and splenomegaly. Pallor and cool skin is most related to anemia related to increased red blood cell loss.

Diagnosis Duchenne Muscular Dystrophy (DMD)

Diagnosis Prenatal diagnostics - muscle biopsy (degeneration of muscle fibers, replacement with fat), DNA analysis from peripheral blood to detect deficient dystrophin Blood PCR - blood polymerase chain reaction for dystrophin egene mutation serum enzymes, muscle biopsy ( if usual characteristics are present, positivefamily history and PCR is positive), EMG ↑ serum creatine kinase level (very highfirst two years of life)

Which interventions are helpful in placing a 3-year-old child in a cast for an upper extremity fracture? Select all that apply. Explain that the cast will heal the injured extremity. Use colorful, lightweight, and water-resistant materials. Explain that the synthetic cast material may cause a burn. Discuss favorite activities at school during the cast application. Put a cast on a plastic doll to explain the procedure beforehand.

Discuss favorite activities at school during the cast application. Put a cast on a plastic doll to explain the procedure beforehand. Use colorful, lightweight, and water-resistant materials. Rationale The synthetic material is lightweight, water-resistant, and comes in a variety of colors. Distractions such as discussing favorite school activities are used during the cast application so that the child remains calm and less anxious during the procedure. The procedure is explained beforehand using various play methods, such as casting a plastic doll the same way the child is going to be cast. A 3-year-old child has a limited concept of cause and effect so it may not help to explain that the cast will make the extremity heal. The synthetic material may become warm, but it will not burn.

What is a common clinical manifestation of autism? Deafness Tendency to overeat Early abnormal eye contact Intense desire to be held and cuddled

Early abnormal eye contact Correct The inability to maintain eye contact with another individual is a hallmark of autism. The child with autism has spoken language delays, not deafness. Children with autism may be picky eaters or willfully starve themselves or gag to prevent eating. They may also eat any available edible or inedible object. A goal of nursing care is to decrease stimulation. Physical contact can be upsetting to children with autism.

Which nursing intervention is appropriate to include when working with the parents of a 2-year-old child with Down syndrome to teach the child feeding skills? Holding the spoon for the child Encouraging the use of adaptive utensils Including new feeding skills when on vacation Offering pureed foods to the child to make chewing easier

Encouraging the use of adaptive utensils Rationale Children with Down syndrome may need extra assistance in learning how to self-feed. They may have difficulty holding utensils; therefore, the nurse can suggest specially adapted utensils. The parent would encourage self-feeding; holding the spoon for the child will not promote independence. New feeding skills would not be introduced during stressful or new experiences, such as going on vacation or having company over to visit. The parents would not offer pureed foods to the child. Food would be provided in separate servings.

True/False Physical therapy consisting of stretching, passive, active, and resistive movements applied to specific muscle groups is able to achieve spectacular changes in the ultimate outcome of the CP child.

False

Which focused assessment should the nurse use to evaluate a child's motor coordination? Select all that apply. Gait analysis Vibration sense Manual dexterity Muscle development Deep tendon reflexes

Gait analysis The analysis of a child's gait should be used to evaluate motor skills because the ability to walk is a complex skill and involves the motor system. Manual dexterity Assessing manual dexterity of a child such as ability to use a pencil can be used to evaluate motor skills because it evaluates fine motor skills and is a measure of myelinization. Muscle development The muscular development of a child is part of the focused assessment used to evaluate motor skills because poor muscle development will impede child's motor skill development

The parents of a child with cerebral palsy ask the nurse if any drugs can decrease their child's spasticity. What is the basis for the nurse's response? Anticonvulsant medications are sometimes useful for controlling spasticity. Medications that would be useful in reducing spasticity are too toxic for use with children. Many different medications can be highly effective in controlling spasticity. Implantation of a pump to deliver medication into the intrathecal space decreases spasticity

Implantation of a pump to deliver medication into the intrathecal space decreases spasticity. Correct Baclofen, given intrathecally, is best suited for children with severe spasticity that interferes with activities of daily living and ambulation. Anticonvulsant medications are used when seizures occur in children with cerebral palsy. The intrathecal route decreases the side effects of the drugs that reduce spasticity. Few medications are currently available for the control of spasticity.

A child was just diagnosed with benign postural kyphosis. What should the nurse emphasize to facilitate correction of this type of kyphosis? A. Importance of wearing a brace every day B. Importance of standing rather than sitting C. Importance of following and maintaining a core muscle strengthening program D. Importance of scheduling surgery as soon as possible to prevent further injury

Importance of following and maintaining a core muscle strengthening program By strengthening the core muscles, proper postural techniques will improve and the postural kyphosis will resolve.

What is the similarity seen between sprains and strains? In general, both sprains and strains occur rapidly. In both cases, joint or ligamentous laxity is common. In both cases, the area is painful to the touch and is swollen. In general, both happen at the level of the musculotendinous unit.

In both cases, the area is painful to the touch and is swollen. Rationale Sprains and strains share some common features. The area is painful to the touch and is swollen in both cases. Sprains occur rapidly, resulting from a severe trauma. In general, strains happen over time. In case of a severe injury, strains may develop rapidly. Strains do not involve ligaments. Joint laxity is not common in strains but is in sprains. Strains involve the muscles and tendons. Sprains happen at the level of muscles and ligaments.

therapeutic management bacterial meningitis

Isolation precautions Antimicrobial therapy Maintenance of hydration Maintenance of systemic shock Maintenance of ventilation Control of seizures and temperature treatment of complicastions IV - antimicrobial agents, fluids, antiepileptic drugs, blood respiratory isolation

Which nursing intervention is appropriate for a school-age child with bacterial meningitis? Keeping environmental stimuli to a minimum during hospital stay Avoiding giving pain medications that could dull the sensorium Having the child move the head from side to side at least every 2 hours Measuring the head circumference to facilitate detection of developing complications

Keeping environmental stimuli to a minimum during hospital stay Rationale Children with meningitis are sensitive to noise, bright lights, and other external stimuli because of the irritation on the meningeal nerves. The nurse would keep the room as quiet as possible with a minimum of external stimuli, including lighting. After consultation with the practitioner, pain medications may be used on an as-needed basis. A school-age child will have closed sutures; therefore the head circumference cannot change and is not relevant. The child is placed in a side-lying position with the head of the bed slightly elevated. The nurse would avoid measures such as lifting the child's head that increase discomfort and put tension on the neck.

Which condition is appropriate for the child with impaired motor activity, reduced range of abduction movement in the upper limb, and normal muscle strength in the lower limb? Duchenne muscular dystrophy Congenital muscular dystrophy Kugelberg-Welander muscular disease Landouzy-Dejerine muscular dystrophy

Landouzy-Dejerine muscular dystrophy Rationale Landouzy-Dejerine muscular dystrophy is a genetic disorder most common in early adolescents. The disorder affects the skeletal muscles of the face, scapula, and shoulder girdle. Patients with Landouzy-Dejerine muscular dystrophy find it difficult to lift their arms over the head and have reduced facial movements. Duchenne muscular dystrophy, congenital muscular dystrophy, and Kugelberg-Welander muscular disease may not affect abduction movement in patients. Duchenne muscular dystrophy results in weakening of the pelvic muscles. Congenital muscular dystrophy results in the weakening of all skeletal muscles and joint deformities. Kugelberg-Welander muscular disease causes the weakening of all skeletal muscles due to altered motor nerve functioning.

Muscular Dystrophies (MDs)

Largest group of muscle diseases in children genetic origin No Cure , supportive measure ( physical therapy, orthopedic procedures to minimize deformity, ventilator support, airway clearance, assisting in meeting demands of dily living Gradual degeneration of muscle fibers Progressive weakness and wasting of symmetric groups of skeletal muscles, loss of strength Increasing deformity & disability

A child is rushed to the emergency department following a collision on the school yard impacting the left side of the head. The nurse expects which physical finding associated with this injury? Raccoon eyes Hemotympanum Left arm weakness Right leg numbness

Left arm weakness In a contrecoup injury, the weakness will occur ipsilateral to the side of injury because of the rebound of the brain within the skull. Normally an injury to the left side of the head would impact function on the right side of the body.

A nurse is preparing discharge instructions for a school-aged child newly diagnosed with an autoimmune disease. What information should the nurse provide to both the child and the parents to prevent behavioral problems at home? Be lenient, which will be helpful, while your child learns to cope with the autoimmune disorder. Remain firm in the expectations of your child and refrain from any modifications of those expectations. Maintain consistent discipline, which is important to help your child continue to develop socially and emotionally. . Minimize the expectation that the child performs chores and understand that the child might struggle in school so allowing a decrease in grades is acceptable.

Maintain consistent discipline, which is important to help your child continue to develop socially and emotionally. Consistency is important to the development of a school-aged child. The child should have similar expectations about behavior after a chronic illness diagnosis.

hemophilia

Minimal or absent clotting factors. They're basically missing one of the clotting factors, factor 7, factor 8. How is it treated? Replacement of specific factors. They get an infusion of whatever factor they're missing. We had a 7-year-old boy- he taught me how to give it. They used to not let them do it at home. Nowadays they let them do it at home. Most of the time they are missing factor 8. Remember- a lot of times these kids get joint injuries. Best treatment is RICE like for everybody, unless major bleed and they usually can tell you. They need factor injection if its more severe. No PROM - nurse can rupture a joint and cause massive hemorrhage. No contact sports. Remember your key words! Except, Priority...keep an eye peeled for those.

What is the appropriate description of a young child who has an intelligence quotient (IQ) of 45? Mildly cognitively impaired but educable Moderately cognitively impaired but trainable Within the lower limits of the range of normal intelligence Severely cognitively impaired and completely dependent on others for care

Moderately cognitively impaired but trainable Correct Children are considered trainable and moderately cognitively impaired if their IQ falls within range of 36 to 49. Individuals with IQs of 50 to 75 are considered mildly cognitively but educable. The lower limit of normal intelligence is approximately 70. An IQ of 20 to 35 results in severe cognitive impairment.

A patient with immune thrombocytopenic purpura (ITP) is admitted to the hospital for medication therapy. The patient begins receiving intravenous (IV) steroids. What should the nurse plan to do to safely administer the medication? Select all that apply. Monitor for bleeding. Administer ibuprofen for discomfort. Monitor the transition from IV to oral steroids. Increase the dose of the medication if improvements are not seen. Abruptly stop the medication once improvements have been noted.

Monitor for bleeding. Patients with ITP are at risk for bleeding; the nurse should monitor the patient for bleeding during treatment to assess for effectiveness. Monitor the transition from IV to oral steroids. The nurse would need to monitor the transition from IV to oral steroids.

sprain

Occurs when trauma to a joint is so severe that is either stretched or partially or completely torn by the force created as a joint is twisted or wrenched:

A child has just been fitted with a cast to heal a fracture of the arm. The nurse is vigilant for which early signs of compartment syndrome? Select all that apply. Pain Pallor Paralysis Paresthesia Pulselessness

Pain Pallor Paresthesia Pulselessness Rationale Compartment syndrome is a serious complication that results from compression of nerves, blood vessels, and muscle inside a closed space. It can occur after a cast is applied. Early clinical signs of compartment syndrome include pain, pallor, pulselessness, paresthesia, and pressure. If left untreated, paralysis and deformity may occur. Therefore paralysis is a late sign of compartment syndrome.

Which presentation is a nurse most likely to see in a patient with chronic immune thrombocytopenic purpura (ITP)? Confusion, tachycardia Petechial rash, bleeding gums Pallor, low oxygen saturation Distended abdomen, joint pain

Petechial rash, bleeding gums Patients with ITP often have petechial rashes and bleeding gums. The nurse would also expect other signs of bleeding.

Which laboratory finding is appropriate when distinguishing bacterial meningitis from viral meningitis? Clear cerebrospinal fluid Positive Gram stain Normal glucose content Normal protein content

Positive Gram stain Rationale The CSF of a child with bacterial meningitis is positive for Gram stain, whereas viral meningitis is negative. The CSF is cloudy, with decreased glucose content and elevated protein content. The CSF in viral meningitis is clear, with normal glucose content, and normal or slightly increased protein content.

Which focus is appropriate when teaching the parents of a child whose laboratory results show an absolute neutrophil count of less than 500/mm 3? Administering iron supplements Administering folate supplements Preventing infection Preparing the child for blood transfusion

Preventing infection Rationale An absolute neutrophil count less than 500/mm 3 makes the child susceptible to infection. In this condition, all efforts are made to prevent the child from contracting an infection. Iron and folate supplements are required only if the child is suffering from iron deficiency anemia. This condition does not commonly require a blood transfusion.

A child is observed limping and walking on his toes. An x-ray reveals subluxation of the femoral head. Which nursing care plan is indicated for this patient? A. Have the patient walk for 30 minutes to monitor gait. B. Provide ways to alleviate the anxiety of the diagnosis. C. Monitor neurovascular status of the affected extremity. D. Teach the patient how to walk straight without limping.

Provide ways to alleviate the anxiety of the diagnosis. After diagnosis, nursing interventions focus on the parents' anxiety and coping abilities.

The school nurse is discussing prevention of acquired immunodeficiency syndrome with some adolescents. What is appropriate to include? The virus is easily transmitted. It is only transmitted through blood. Condoms should be used if adolescents are homosexual. Recreational drug users should not share needles or other equipmen

Recreational drug users should not share needles or other equipment. Correct Human immunodeficiency virus is spread through blood and body fluids. Intravenous needles that have been used should not be shared. They may be contaminated with the virus. The virus is not easily transmitted. It requires direct contact with blood or body fluids on a nonintact skin surface. Body fluids may also transmit the virus. Condoms should be used for both heterosexual and homosexual sex.

The structure of red blood cells supports ease of flow through the blood vessels. Which patient condition is most likely to inhibit blood flow? Diabetes Leukemia Pneumonia Sickle cell anemia

Sickle cell anemia The altered shape of the RBC in patients with sickle cell anemia can lead to impaired flow through the capillaries, which is why sickle cell patients can present with intense pain or "sickle cell crisis."

An infant is being evaluated for bacterial meningitis. The nurse holds the patient in which position for the sampling of cerebrospinal fluid (CSF)? Prone Sitting Standing Squatting Side-lying

Sitting Having the child in a sitting position will provide adequate flexion of the lumbar spine for performing a lumbar puncture. Side-lying The side-lying position provides adequate flexion of the lumbar spine for performing a lumbar puncture.

Jewett-Taylor brace

Sometimes used to support the spine and trunk during ambulation to prevent compression after fracture of the spinal column:

Pathophysiology hemophilia A

The basic defect is a deficiency of factor VII (AHF anti hemophilic factor) AHF is produced by the liver and is necessary for the formation of thromboplastin in phase I of blood coagulation The less AHF found in the blood, the more severe the disease Individuals with hemophilia have two of the three factors required for coagulation (vascular influence and platelets) They may bleed for longer periods but not at a faster rate

A child who plays soccer is brought to the clinic by the mom who suggests her child is not acting right. Which associated finding does the nurse evaluate further? The child cannot recall yesterday's events. The child requests a drink of water and a popsicle. The mother is pacing in the triage room continuously. The mother states that the child did not eat after the game.

The child cannot recall yesterday's events. Memory loss can be associated with postconcussion syndrome, and therefore the patient may have difficulty remembering yesterday's events. This information helps the nurse direct further care.

The nurse performs a developmental assessment of a nine-month-old infant. Which finding causes the nurse to be concerned that myelinization of the neurons is impaired? The child is not yet making babbling sounds such as "mama" or "baba." The child is unable to transfer an object from one hand to the other hand. The child does not look for a toy that the nurse has shown and then hidden. The child is not yet able pull to stand while holding onto a hand or furniture.

The child is unable to transfer an object from one hand to the other hand. The myelinization of the neurons allows for effective transmission of neural signals required for proper coordination of movements. The ability to coordinate movement of the hands to move an object is a 9-month developmental milestone.

A twelve-year-old child's spina bifida lesion affects the upper lumbar vertebrae. The nurse evaluates that the child is meeting therapeutic goals when the child demonstrates which behaviors? The child participates in exercise activities daily. The child walks without using leg braces prescribed. The child has successful attempts at bladder emptying. The child chooses to play alone at school during recess. The child bathes, dresses, and puts on shoes without help.

The child participates in exercise activities daily. Increasing the child's mobility is a goal in the care of a child with spina bifida, and therefore the child's continued ability to participate in daily exercise indicates the goals of care are being met. The child has successful attempts at bladder emptying. A child with spina bifida must be placed in a bladder-emptying program, and therefore successful attempts at emptying the bladder is an indicator that goals are being achieved. The child bathes, dresses, and puts on shoes without help. This is an appropriate act of independence and demonstrates the child is maintaining mobility and is actively working toward therapeutic goals.

Which nursing information is appropriate when teaching pregnant women about the preventative benefits of food rich in folic acid? Onset of cerebral palsy The development of neural tube defects Guillain-Barré syndrome Werdnig-Hoffmann disease

The development of neural tube defects Rationale Neural tube defects are the major group of congenital anomalies seen in infants. They are characterized by a defective closure of the neural tube. Eating foods enriched with folic acid prevents the occurrence of neural tube defects during the gestation period. Folic acid supplementation has not been shown to prevent disorders like cerebral palsy, Guillain-Barré syndrome, and Werdnig-Hoffmann disease in infants.

Which similarity is appropriate between the manual jaw control provided from the front and the manual jaw control provided from the side? The thumb of the nonfeeding hand is placed below the bottom lip. The index finger of the nonfeeding hand is placed parallel to the mandible. The ring finger of the nonfeeding hand is placed obliquely across the cheek. The middle finger of the nonfeeding hand is placed posterior to the bony portion of the chin.

The middle finger of the nonfeeding hand is placed posterior to the bony portion of the chin. Rationale The middle finger of the nonfeeding hand is placed posterior to the bony portion of chin, whether the child's jaw is controlled from the front or from the side. When the child's jaw is controlled from the front, the thumb of the nonfeeding hand is placed below the bottom lip and index finger of the nonfeeding hand is placed parallel to the mandible. When the child's jaw is controlled from the side, the thumb of the nonfeeding hand is placed obliquely across the cheek and the index finger of the nonfeeding hand is placed on the chin below the lower lip.

Anemia

The most common hematologic disorder of childhood Decrease in number of RBCs and/or hemoglobin concentration below normal Decreased oxygen-carrying capacity of blood Know this Fatigue huge sign of concern Check hydration too but anemia is a secondary thing Lead to exposed to toxic Too much milk Lead to anemia causes of anemia: 1. inadequate production of RBC 2. increased destruction of RBC 3. excessive loss of RBC through hemorrhage

traction

The primary purposes of traction are to fatigue involved muscles and reduce muscle spasm, position bone ends in desired realignment, and immobilize the fracture site until realignment has been achieved to permit casting or splinting. The two main types of traction are skin traction and skeletal traction. Potential complications of traction and casts include circulatory impairment, nerve compression, nonunion of bones, and skin breakdown.

A child is brought to the emergency department after experiencing a seizure at school. He has no history of seizures. The father tells the nurse that he cannot believe the child has epilepsy. What is the best response by the nurse? "Epilepsy is easily treated." "Very few children have actual epilepsy." "The seizure may or may not mean that your child has epilepsy." "Your child has had only one convulsion; it probably won't happen again."

The seizure may or may not mean that your child has epilepsy." Correct A single seizure event is not classified as epilepsy and is generally not treated with long-term antiepileptic drugs. It can be the result of an acute medical or neurologic disease. True epilepsy is not easily treated, so saying that it is easily treated minimizes the father's concern. The statistics on epilepsy do not address the father's issues about his child. The seizure may or may not mean that a child has epilepsy, so it may not happen again. The nurse needs to provide the information to the parent that the diagnosis is not based on one seizure episode.

Why is the size and structure of the red blood cell important? The size affects iron binding and hemoglobin production. The ovoid shape and size affect the ability to carry oxygen. The oblong shape is required for hemoglobin to bind correctly. The shape and flexibility allows for passage through the blood vessels.

The shape and flexibility allows for passage through the blood vessels. Red blood cells are biconcave, flexible discs, which allows for easy passage through small capillaries.

The nurse understands that weight loss occurs in patients with cystic fibrosis due to which pathophysiologic occurrences? The pancreas becomes fibrotic causing diabetes. The thick mucus prevents the intestinal villi from absorbing nutrients in the small intestine. The thick mucus prevents digestive enzymes from being released that are needed to absorb nutrients. The metabolic rate of the child is elevated because the caloric intake does not meet the energy needs of the child.

The thick mucus prevents digestive enzymes from being released that are needed to absorb nutrients. Trypsin, amylase, and lipase are unable to be excreted by the pancreas into the small intestine. Without the digestive enzymes, proteins, carbohydrates and fats are poorly absorbed causing weight loss.

True/False CP patients are screened before Baclofen pump placement by the infusions of a test dose of intrathecal baclofen delivered via a lumbar puncture

True

True/False Children receiving phenytoin are at risk for developing gum hyperplasia

True

True/False Feeding techniques such as forcing the child to use the lips and tongue in eating can facilitate speech.

True

True/False Prime candidates for botulinum toxin type A (Botox) injections muscle coordination are children with spasticity confined to the lower extremities.

True

Which specific signs are appropriate in the diagnosis of bacterial meningitis in neonates? Select all that apply. Weak cry Poor tone Nuchal rigidity Normal feedings Full, tense, bulging fontanel

Weak cry Poor tone Full, tense, bulging fontanel Rationale Poor tone, weak cry, and a full, tense, and bulging fontanel are specific signs of bacterial meningitis in neonates. The neck is usually supple, not rigid, in neonates. Feedings are refused, not taken normally.

How is the treatment environment related to the anxiety levels of an infant going through casting for clubfoot? A. Making the parents leave the room helps to decrease the anxiety of the infant. B. A treatment environment with a loud noise level decreases the anxiety of the infant. C. A cold environment helps to calm the infant, which will decrease the anxiety level. D. When parental involvement is incorporated, the anxiety level of the infant is decreased.

When parental involvement is incorporated, the anxiety level of the infant is decreased. Parental involvement is comforting to infants and helps them relax, or even sleep during the session.

The term hemophilia refers to

a group of bleeding disorders resulting from congenital deficiency of specific coagulation proteins. Although the symptomatology is similar regardless of which clotting factor is deficient, the identification of specific factor deficiencies has allowed definitive treatment with replacement agents. Hemophilia A involves a deficiency of factor VIII (antihemophilic factor). Factor VIII is produced by the liver and is necessary for the formation of thromboplastin in phase I of blood coagulation. The less factor VIII found in the blood, the more severe the disease.

Recommended diagnostic evaluation for infants at risk for CP include: a. magnetic resonance imaging of the brain b. computed tomographic scan of the brain c. laboratory testing d. metabolic and genetic testing when structural abnormality is identified.

a. magnetic resonance imaging of the brain

support parents

attentive to family responses equality of children to HCP encourage parents to discuss their feelings financial assistance

Diagnostic evaluation for the patient with GBS would include which of the following results? a. Complete blood count elevated b. Cerebrospinal fluid high in protein c. Creatinine phosphokinase elevated d. Sensory nerve conduction time increased

b. Cerebrospinal fluid high in protein

Sixteen year old Ben has been brought to the school nurse's office for heatstroke. He has a temperature of 40 C (104 F) and is awake but disoriented. Which of the following is contraindicated? a. immediate removal of clothing and application of cool water to the skin. b. administration of antipyretics c. use of fans directed at Ben d. activation of EMS for transport to hospital

b. administration of antipyretics

leading cause of nonfatal injury 15-19

being struck by or against a person or object

The nurse is completing a physical examination on 6 month old Brian. Which of the following would be an abnormal finding suggestive of CP? a. Brian is able to hold onto the nurse's hands while being pulled to a sitting position b. Brian has no Moro reflex c. Brian has no tonic neck reflex d. Brian has an obligatory tonic neck reflex

d. Brian has an obligatory tonic neck reflex

In addition to the preventions of fractures, the nursing care of a child with osteogenesis imperfecta should include: a. oral and dental care b. meticulous skin care c. care related to the effects of drug treatment d. all of the above

d. all of the above

Tension-type headache

frontal pressing pain or tightness and nonthrobbing in character typically not accompanied by nausea or vomiting Tx - adequate sleep, appropriate hydration, regular meals and exercise ibyprophen

what would you want to evaluate for neurologic status

general aspects -uncooperative -observations -reflective responses -milestones or reflexes ( delay or milestone or persistent primitive reflexes) - family history esp disabilities - physical examination - size and shape of the head, spontaneous activity and postural reflex activity, sensory responses, facial features, high pitches cry, muscular activity and coordication in children ( ocular, gait) altered states of consciousness -level of consciousness is earliest indicator of change -changes in behavior -coma assessment: glasgow coma scale

Pain management

ibuprofen has been shown to reduce fracture pain in children

Causative factors to avoid to prevent cleft lip/palate

use of retinoids, alcohol consumption, smoking

myelomeningocele clinical manifestations

vary according to the degree of the defect - anatomil level of defect and nerves involved - defective nerve supply to the bladder - dribbling urine or overflow incontinence - lack of bowel control - rectal prolapse - denervation to the muscles of the lower extremities produces joint deformities - contractures, kyphosis, scoliosis, hip dislocation - sensory deficit - flaccid, partial paralysis of lower extremities

care of residual limb

wash with mild nonperfumed soap, rinse and dry thoroughly daily check skin for redness, blisters, sensetive areas, signs of infection

neuromuscular junction disease

when nerve impulses to skeletal muscles are inhibited at the neuromuscular junction botulism, tick paralysis

A 4-year-old child is placed in Buck extension traction for Legg-Calvé-Perthes disease. He is crying with pain as the nurse assesses the skin of his right foot and sees that it is pale with an absence of pulse. What should the nurse do first? a. Reposition the child and notify the practitioner. b. Notify the practitioner of the changes noted. c. Give the child medication to relieve the pain. d. Chart the observations and check the extremity again in 15 minutes.

ANS: B The absence of a pulse and change in color of the foot must be reported immediately for evaluation by the practitioner. This is an emergency condition. Pain medication should be given after the practitioner is notified. The findings should be documented with ongoing assessment.

A 3-year-old child has a femoral shaft fracture. The nurse recognizes that the approximate healing time for this child is how long? a. 2 weeks b. 4 weeks c. 6 weeks d. 8 weeks

ANS: B The approximate healing times for a femoral shaft fracture are as follows: neonatal period, 2 to 3 weeks; early childhood, 4 weeks; later childhood, 6 to 8 weeks; and adolescence, 8 to 12 weeks

A child in the terminal stage of cancer has frequent breakthrough pain. Nonpharmacologic methods are not helpful, and the child is exceeding the maximum safe dose for opiate administration. What approach should the nurse implement? a. Add acetaminophen for the breakthrough pain. b. Titrate the opioid medications to control the child's pain as specified in the protocol. c. Notify the practitioner that immediate hospitalization is indicated for pain management. d. Help the parents and child understand that no additional medication can be given because of the risk of respiratory depression.

ANS: B The child on long-term opioid management can become tolerant to the drugs. Also, increasing amounts of drugs may be necessary for disease progression. It is important to recognize that there is no maximum dosage that can be given to control pain. Acetaminophen will offer little additional pain control; it is useful for mild and moderate pain. Immediate hospitalization is not necessary; increased dosages of pain medications can be administered in the home environment. The principle of double effect allows for a positive intervention—relief of pain—even if there is a foreseeable possibility that death may be hastened.

What test is never performed on a child who is awake? a. Doll's head maneuver b. Oculovestibular response c. Assessment of pyramidal tract lesions d. Funduscopic examination for papilledema

ANS: B The oculovestibular response (caloric test) involves the instillation of ice water into the ear of a comatose child. The caloric test is painful and is never performed on an awake child or one who has a ruptured tympanic membrane. The doll's head maneuver, assessment of pyramidal tract lesions, and funduscopic examination for papilledema are not considered painful and can be performed on awake children.

A recommendation to prevent neural tube defects (NTDs) is the supplementation of what? a. Vitamin A throughout pregnancy b. Folic acid for all women of childbearing age c. Folic acid during the first and second trimesters of pregnancy d. Multivitamin preparations as soon as pregnancy is suspected

ANS: B The widespread use of folic acid among women of childbearing age has decreased the incidence NTDs. In the United States, the rates of NTDs have declined from 1.3 per 1000 births in 1990 to 0.3 per 1000 after the introduction of mandatory folic acid supplementation in food in 1998. Vitamin A is not related to the prevention of NTDs. Folic acid supplementation is recommended for the preconceptual period, as well as during the pregnancy. The NTD is a failure of neural tube closure during early development, the first 3 to 5 weeks.

An 8-year-old girl has been uncooperative and angry since the diagnosis of cancer was made. Her parents tell the nurse that they do not know what to do "because she is always so mad at us." What nursing action is most appropriate at this time? a. Explain to child that anger is not helpful. b. Help the parents deal with her anger constructively. c. Ask the parents to find out what she is angry about. d. Encourage the parents to ignore the anger at this time.

ANS: B To school-age children, chronic illness and dying represent a loss of control. This threat to their sense of security and ego strength can be manifested by verbal uncooperativeness. The child can be viewed as impolite, insolent, and stubborn. The best intervention is to encourage children to talk about feelings and give control where possible. Verbal explanations would not be "heard" by the child. The child may not be cognizant of the anger. Ignoring the anger will not help the child gain some control over the events.

What is an appropriate nursing intervention when caring for a child in traction? a. Removing adhesive traction straps daily to prevent skin breakdown b. Assessing for tightness, weakness, or contractures in uninvolved joints and muscles c. Providing active range of motion exercises to affected extremity three times a day d. Keeping child prone to maintain good alignment

ANS: B Traction places stress on the affected bone, joint, and muscles. The nurse must assess for tightness, weakness, or contractures developing in the uninvolved joints and muscles. The adhesive straps should be released or replaced only when absolutely necessary. Active, passive, or active with resistance exercises should be carried out for the unaffected extremity only. Movement is expected with children. Each time the child moves, the nurse should check to ensure that proper alignment is maintained.

What is the recommended drink for athletes during practice and competition? a. Sports drinks to replace carbohydrates b. Cold water for gastrointestinal tract rapid absorption c. Carbonated beverages to help with acid-base balance d. Enhanced performance carbohydrate-electrolyte drinks

ANS: B Water is recommended for most athletes, who should drink 4 to 8 oz every 15 to 20 minutes. Cold water facilitates rapid gastric emptying and intestinal absorption. Most carbohydrate sports drinks have 6% to 8% carbohydrate, which can cause gastrointestinal upset. Carbonated beverages are discouraged. There is no evidence that these drinks enhance function.

The nurse is preparing to admit a 1-month-old infant with severe congenital neutropenia (Kostmann disease). What clinical features of severe congenital neutropenia should the nurse recognize? (Select all that apply.) a. Anemia is present. b. Neutropenia is present. c. The illness is severe. d. It has a dominant inheritance pattern. e. There are decreased eosinophils in the bone marrow.

ANS: B, C The clinical features of severe congenital neutropenia include anemia and neutropenia, and the illness is severe. It has an autosomal recessive inheritance pattern, and there are increased, not decreased, eosinophils in the bone marrow.

What functional goal should the nurse expect for a child who has a C7 spinal cord injury? (Select all that apply.) a. Able to drive automobile with hand controls b. Complete independence within limitations of a wheelchair c. Can roll over in bed, sit up in bed, and eat independently d. Requires some assistance in transfer and lower extremity dressing e. Ambulation with bilateral long braces using four-point or swing-through crutch gait

ANS: B, C, D A child with a C7 spinal cord injury can expect to be completely independent within the limitations of a wheelchair, can roll over in bed, sit up in bed, and eat independently, and will require some assistance in transfer and lower extremity dressing. The ability to drive an automobile with hand controls is a functional goal for a T1 to T10 spinal cord injury. Ambulation with bilateral long braces using four-point or swing-through crutch gait is a functional goal for a T10 to L2 injury.

What findings should the nurse expect to observe in a 7-month-old infant with Werdnig-Hoffman disease? (Select all that apply.) a. Noticeable scoliosis b. Absent deep tendon reflexes c. Abnormal tongue movements d. Failure to thrive e. Prominent pectus excavatum f. Significant leg involvement

ANS: B, C, D Clinical manifestations of Werdnig-Hoffman disease in an infant include absent deep tendon reflexes, abnormal tongue movements, and failure to thrive. Scoliosis, prominent pectus excavatum, and significant leg involvement are findings observed in a child with intermediate spinal muscular atrophy.

The nurse is planning to use an interpreter with a non-English-speaking family. What should the nurse plan with regard to the use of an interpreter? (Select all that apply.) a. Use a family member. b. The nurse should speak slowly. c. Use an interpreter familiar with the family's culture. d. The nurse should speak only a few sentences at a time. e. The nurse should speak to the interpreter during interactions.

ANS: B, C, D When parents who do not speak English are informed of their child's chronic illness, interpreters familiar with both their culture and language should be used. The nurse should speak slowly and only use a few sentences at a time. Children, family members, and friends of the family should not be used as translators because their presence may prevent parents from openly discussing the issues. The nurse should speak to the family, not the interpreter.

What characterizes a school-aged child's concept of death? (Select all that apply.) a. Have a mature understanding of death b. Can respond to logical explanations of death c. Personify death as the devil or the bogeyman d. Have a deeper understanding of death in a concrete sense e. Fear the mutilation and punishment associated with death

ANS: B, C, D, E A school-aged child's concept of death includes responding to logical explanations of death, personifying death as the devil or bogeyman, having a deeper understanding of death in a concrete sense, and fearing mutilation and punishment associated with death. Adolescents' concept of death is a mature understanding of death.

A child has had a short-arm synthetic cast applied. What should the nurse teach to the child and parents about cast care? (Select all that apply.) a. Relieve itching with heat. b. Elevate the arm when resting. c. Observe the fingers for any evidence of discoloration. d. Do not allow the child to put anything inside the cast. e. Examine the skin at the cast edges for any breakdown.

ANS: B, C, D, E Cast care involves elevating the arm, observing the fingers for evidence of discoloration, not allowing the child to put anything inside the cast, and examining the skin at the edges of the cast for any breakdown. Ice, not heat, should be applied to relieve itching.

What effects of an altered pituitary secretion in a child with meningitis indicates syndrome of inappropriate antidiuretic hormone (SIADH)? (Select all that apply.) a. Hypotension b. Serum sodium is decreased c. Urinary output is decreased d. Evidence of overhydration e. Urine specific gravity is increased

ANS: B, C, D, E The serum sodium is decreased, urinary output is decreased, evidence of overhydration is present, and urine specific gravity is increased in SIADH. Hypertension, not hypotension, occurs.

What risk factors can cause a sensorineural hearing impairment in an infant? (Select all that apply.) a. Cat scratch disease b. Bacterial meningitis c. Childhood case of measles d. Childhood case of chicken pox e. Administration of aminoglycosides for more than 5 days

ANS: B, C, E Risk criteria for sensorineural hearing impairment in infants include bacterial meningitis; a case of measles; and administration of ototoxic medications (e.g., gentamicin, tobramycin, kanamycin, streptomycin), including but not limited to the aminoglycosides, for more than 5 days. Cat scratch disease and a childhood case of chicken pox are not risk factors that can cause a sensorineural hearing impairment.

The parent of a child with a chronic illness tells the nurse, "I feel so hopeless in this situation." The nurse should take which actions to foster hopefulness for the family? (Select all that apply.) a. Avoid topics that are lighthearted. b. Convey a personal interest in the child. c. Be honest when reporting on the child's condition. d. Do not initiate any playful interaction with the child. e. Demonstrate competence and gentleness when delivering care.

ANS: B, C, E To foster hopefulness, the nurse should convey a personal interest in the child, be honest when reporting on a child's condition, and demonstrate competence and gentleness when delivering care. The nurse should introduce conversations on neutral, non-disease-related, or less sensitive topics (discuss the child's favorite sports, tell stories). The nurse should be lighthearted and initiate or respond to teasing or other playful interactions with the child.

What cerebrospinal fluid (CSF) analysis should the nurse expect with viral meningitis? (Select all that apply.) a. Color is turbid. b. Protein count is normal. c. Glucose is decreased. d. Gram stain findings are negative. e. White blood cell (WBC) count is slightly elevated.

ANS: B, D, E The CSF analysis in viral meningitis shows a normal or slightly elevated protein count, negative Gram stain, and a slightly elevated WBC. The color is clear or slightly cloudy, and the glucose level is normal.

The nurse is preparing to admit a 5-year-old with spina bifida cystica that was below the second lumbar vertebra. What clinical manifestations of spina bifida cystica below the second lumbar vertebra should the nurse expect to observe? (Select all that apply.) a. No motor impairment b. Lack of bowel control c. Soft, subcutaneous lipomas d. Flaccid, partial paralysis of lower extremities e. Overflow incontinence with constant dribbling of urine

ANS: B, D, E The clinical manifestations of spina bifida cystica below the second lumbar vertebra include lack of bowel control, flaccid, partial paralysis of lower extremities, and overflow incontinence with constant dribbling of urine. No motor impairment occurs with spina bifida cystica that was below the third lumbar vertebra, and soft, subcutaneous lipomas occur with spina bifida occulta.

The nurse is teaching parents the signs of a hearing impairment in infants. What should the nurse include as signs? (Select all that apply.) a. Lack of a fencing reflex b. Lack of a startle reflex to a loud sound c. Awakened by loud environmental noises d. Failure to localize a sound by 6 months of age e. Response to loud noises as opposed to the voice

ANS: B, D, E The fencing reflex is elicited when the infant is placed on his or her back; it does not indicate a hearing impairment. Awakening by a loud environmental noise is a normal response.

What statement is descriptive of a concussion? a. Petechial hemorrhages cause amnesia. b. Visible bruising and tearing of cerebral tissue occur. c. It is a transient and reversible neuronal dysfunction. d. It is a slight lesion that develops remote from the site of trauma.

ANS: C A concussion is a transient, reversible neuronal dysfunction with instantaneous loss of awareness and responsiveness resulting from trauma to the head. Petechial hemorrhages on the superficial aspects of the brain along the point of impact are a type of contusion but are not necessarily associated with amnesia. A contusion is visible bruising and tearing of cerebral tissue. Contrecoup is a lesion that develops remote from the site of trauma as a result of an acceleration-deceleration injury.

A child has been seizure free for 2 years. A father asks the nurse how much longer the child will need to take the antiseizure medications. How should the nurse respond? a. Medications can be discontinued at this time. b. The child will need to take the drugs for 5 years after the last seizure. c. A step-wise approach will be used to reduce the dosage gradually. d. Seizure disorders are a lifelong problem. Medications cannot be discontinued.

ANS: C A predesigned protocol is used to wean a child gradually off antiseizure medications, usually when the child is seizure free for 2 years. Medications must be gradually reduced to minimize the recurrence of seizures. The risk of recurrence is greatest within 6 months after discontinuation.

What term refers to seizures that involve both hemispheres of the brain? a. Absence b. Acquired c. Generalized d. Complex partial

ANS: C Clinical observations of generalized seizures indicate that the initial involvement is from both hemispheres. Absence seizures have a sudden onset and are characterized by a brief loss of consciousness, a blank stare, and automatisms. Acquired seizure disorder is a result of a brain injury from a variety of factors; it is not a term that labels the type of seizure. Complex partial seizures are the most common seizures. They may begin with an aura and be manifested as repetitive involuntary activities without purpose, carried out in a dreamy state.

The nurse is teaching a child with a cast about cast removal. What should the nurse teach the child about cast removal? a. "The cast cutter will be a quiet machine." b. "You will feel cold as the cast is removed." c. "You will feel a tickly sensation as the cast is removed." d. "The cast cutter cuts through the cast like a circular saw."

ANS: C Cutting the cast to remove it or to relieve tightness is frequently a frightening experience for children. They fear the sound of the cast cutter and are terrified that their flesh, as well as the cast, will be cut. Because it works by vibration, a cast cutter cuts only the hard surface of the cast. The oscillating blade vibrates back and forth very rapidly and will not cut when placed lightly on the skin. Children have described it as producing a "tickly" sensation.

The parents of a child with cognitive impairment ask the nurse for guidance with discipline. What should the nurse's recommendation be based on? a. Discipline is ineffective with cognitively impaired children. b. Cognitively impaired children do not require discipline. c. Behavior modification is an excellent form of discipline. d. Physical punishment is the most appropriate form of discipline.

ANS: C Discipline must begin early. Limit-setting measures must be clear, simple, consistent, and appropriate for the child's mental age. Behavior modification, especially reinforcement of desired behavior and use of time-out procedures, is an appropriate form of behavior control. Aversive strategies should be avoided in disciplining the child.

What nursing intervention is most appropriate when caring for the child with osteomyelitis? a. Encourage frequent ambulation. b. Administer antibiotics with meals. c. Move and turn the child carefully and gently to minimize pain. d. Provide active range of motion exercises for the affected extremity.

ANS: C During the acute phase, any movement of the affected limb causes discomfort to the child. Careful positioning with the affected limb supported is necessary. Weight bearing is not permitted until healing is well under way to avoid pathologic fractures. Intravenous antibiotics are used initially. Food is not necessary with parenteral therapy. Active range of motion would be painful for the child.

A feeding technique the nurse can teach to parents of a child with cerebral palsy to improve use of the lips and the tongue to facilitate speech is which? a. Feeding pureed foods b. Placing food on the tongue c. Placing food at the side of the tongue d. Placing food directly into the mouth with a spoon

ANS: C Feeding techniques such as forcing the child to use the lips and tongue in eating facilitate speech. An example of this technique is placing food at the side of the tongue, first one side and then the other, and making the child use the lips to take food from a spoon rather than placing it directly on the tongue. Feeding pureed foods would not encourage use of the lips and tongue.

The nurse at a summer camp recognizes the signs of heatstroke in an adolescent girl. Her temperature is 40° C (104° F). She is slightly confused but able to drink water. Nursing care while waiting for transport to the hospital should include what intervention? a. Administer antipyretics. b. Administer salt tablets. c. Apply towels wet with cool water. d. Sponge with solution of rubbing alcohol and water.

ANS: C Heatstroke is a failure of normal thermoregulatory mechanisms. The onset is rapid with initial symptoms of headache, weakness, and disorientation. Immediate care is relocation to a cool environment, removal of clothing, and applying of cool water (wet towels or immersion). Antipyretics are not used because they are metabolized by the liver, which is already not functioning. Salt tablets are not indicated and may be harmful by increasing dehydration. Rubbing alcohol is not used.

A pregnant woman asks about prenatal diagnosis of hydrocephalus. The nurse's response should be based on which knowledge? a. It can be diagnosed only after birth. b. It can be diagnosed by chromosome studies. c. It can be diagnosed with fetal ultrasonography. d. It can be diagnosed by measuring the lecithin-to-sphingomyelin ratio.

ANS: C Hydrocephalus can be diagnosed by fetal ultrasonography as early as 14 weeks of gestation. Most incidents of hydrocephalus are not chromosomal in origin. The lecithin-to-sphingomyelin ratio can be used to determine fetal lung maturity.

The nurse is teaching infant care to parents with an infant who has been diagnosed with osteogenesis imperfecta (OI). What should the nurse include in the teaching session? a. "Bisphosphonate therapy is not beneficial for OI." b. "Physical therapy should be avoided as it may cause damage to bones." c. "Lift the infant by the buttocks, not the ankles, when changing diapers." d. "The infant should meet expected gross motor development without assistive devices."

ANS: C Infants and children with this disorder require careful handling to prevent fractures. They must be supported when they are being turned, positioned, moved, and held. Even changing a diaper may cause a fracture in severely affected infants. These children should never be held by the ankles when being diapered but should be gently lifted by the buttocks or supported with pillows. Bisphosphonate and physical therapy are beneficial for OI. Lightweight braces will be used when the child starts to ambulate.

What is the initial clinical manifestation of generalized seizures? a. Confusion b. Feeling frightened c. Loss of consciousness d. Seeing flashing lights

ANS: C Loss of consciousness is a frequent occurrence in generalized seizures and is the initial clinical manifestation. Being confused, feeling frightened, and seeing flashing lights are clinical manifestations of a complex partial seizure.

What nursing intervention is appropriate when caring for an unconscious child? a. Avoid using narcotics or sedatives to provide comfort and pain relief. b. Change the child's position infrequently to minimize the chance of increased intracranial pressure (ICP). c. Monitor fluid intake and output carefully to avoid fluid overload and cerebral edema. d. Give tepid sponge baths to reduce fevers above 38.3° C (101° F) because antipyretics are contraindicated.

ANS: C Often comatose patients cannot cope with the quantity of fluids that they normally tolerate. Overhydration must be avoided to prevent fatal cerebral edema. Narcotics and sedatives should be used as necessary to reduce pain and anxiety, which can increase ICP. The child's position should be changed frequently to avoid complications such as pneumonia and skin breakdown. Antipyretics are the method of choice for fever reduction.

What is the primary method of treating osteomyelitis? a. Joint replacement b. Bracing and casting c. Intravenous antibiotic therapy d. Long-term corticosteroid therapy

ANS: C Osteomyelitis is an infection of the bone, most commonly caused by Staphylococcus aureus infection. The treatment of choice is antibiotics. Joint replacement, bracing and casting, and long-term corticosteroid therapy are not indicated for infectious processes.

The nurse is teaching feeding strategies to a parent of a 12-month-old infant with Down syndrome. What statement made by the parent indicates a need for further teaching? a. "If the food is thrust out, I will reefed it." b. "I will use a small, long, straight-handled spoon." c. "I will place the food on the top of the tongue." d. "I know the tongue thrust doesn't indicate a refusal of the food."

ANS: C Parents of a child with Down syndrome need to know that the tongue thrust does not indicate refusal to feed but is a physiologic response. Parents are advised to use a small but long, straight-handled spoon to push the food toward the back and side of the mouth. If food is thrust out, it should be refed. If the parent indicates placing the food on the tongue, further teaching is needed.

What behavior seen in children should be addressed by the nurse who is providing care to a child with a chronic illness? a. An infant who is uncooperative b. A toddler who expresses loneliness c. A preschooler who refuses to participate in self-care d. An adolescent who is showing independence

ANS: C Preschoolers thrive on being independent and are in the phase of gaining autonomy, so they want to perform as many self-care tasks as possible. If a preschooler is refusing to participate in self-care activities, then the home health nurse should address this. Infants are uncooperative by nature, and toddlers do not understand the concept of loneliness, so these are not observations that would need to be addressed. Adolescents are always striving for independence, so this is a normal observation; if the adolescent were becoming more dependent on family, it might require intervention.

The parents of a child with sickle cell anemia (SCA) are concerned about subsequent children having the disease. What statement most accurately reflects inheritance of SCA? a. SCA is not inherited. b. All siblings will have SCA. c. Each sibling has a 25% chance of having SCA. d. There is a 50% chance of siblings having SCA.

ANS: C SCA is inherited as an autosomal recessive disorder. In this inheritance pattern, each child born to these parents has a 25% chance of having the disorder, a 25% chance of having neither SCA nor the trait, and a 50% chance of being heterozygous for SCA (sickle cell trait). SCA is an inherited hemoglobinopathy.

A neonate is born with mild clubfeet. When the parents ask the nurse how this will be corrected, what should the nurse explain? a. Traction is tried first. b. Surgical intervention is needed. c. Frequent, serial casting is tried first. d. Children outgrow this condition when they learn to walk.

ANS: C Serial casting is begun shortly after birth, before discharge from the nursery. Successive casts allow for gradual stretching of skin and tight structures on the medial side of the foot. Manipulation and casting of the leg are repeated frequently (every week) to accommodate the rapid growth of early infancy. Serial casting is the preferred treatment. Surgical intervention is done only if serial casting is not successful. Children do not improve without intervention

An adolescent with long-term, complex health care needs will soon be discharged from the hospital. The nurse case manager has been assigned to the teen and family. The adolescent's care involves physical therapy, occupational therapy, and speech therapy in addition to medical and nursing care. Who should be the decision maker in the adolescent's care? a. Adolescent b. Nurse case manager c. Adolescent and family d. Multidisciplinary health care team

ANS: C The extent to which children are involved in their own care and decision making depends on many factors, including the child's developmental age, level of interest, physical ability, and parental support. If the adolescent is developmentally age appropriate, then decision making should be the responsibility of child and family. Family needs to be involved because they will be caring for the adolescent in the home. Health care providers have necessary input into the care of the child, but ultimate decision making rests with the adolescent and family.

What is a physiologic effect of immobilization on children? a. Metabolic rate increases. b. Venous return improves because the child is in the supine position. c. Circulatory stasis can lead to thrombus and embolus formation. d. Bone calcium increases, releasing excess calcium into the body (hypercalcemia).

ANS: C The physiologic effects of immobilization, as a result of decreased muscle contraction, include venous stasis. This can lead to pulmonary emboli or thrombi. The metabolic rate decreases with immobilization. With the loss of muscle contraction, there is a decreased venous return to the heart. Calcium leaves the bone during immobilization, leading to bone demineralization and increasing the calcium ion concentration in the blood.

The nurse understands that a school-age child may react to death with what reaction? a. Joking b. Having no reaction c. Fearing the unknown d. Seeing it as a distant event

ANS: C They tend to fear the expectation of the event more than its realization. Their fear of the unknown is greater than that of the known. They would not joke or have no reaction. Adolescents see death as a distant event.

At which developmental period do children have the most difficulty coping with death, particularly if it is their own? a. Toddlerhood b. Preschool c. School age d. Adolescence

ANS: D Adolescents, because of their mature understanding of death, remnants of guilt and shame, and issues with deviations from normal, have the most difficulty coping with death. Toddlers and preschoolers are too young to have difficulty coping with their own death. They fear separation from their parents. School-age children fear the unknown such as the consequences of the illness and the threat to their sense of security.

What should the nurse plan for an immobilized child in cervical traction to prevent deep vein thrombosis (DVT)? a. Elevate the child's legs. b. Place a foot cradle on the bed. c. Place a pillow under the child's knees. d. Assist the child to dorsiflex the feet and rotate the ankles.

ANS: D For a child who is immobilized, circulatory stasis and DVT development are prevented by instructing patients to change positions frequently, dorsiflex their feet and rotate the ankles, sit in a bedside chair periodically, or ambulate several times daily. Elevating the legs or placing a foot cradle on the bed will not prevent DVTs. A pillow under the knee would impair circulation, not improve it.

What should the nurse determine to be the priority intervention for a family with an infant who has a disability? a. Focus on the child's disabilities to understand care needs. b. Institute age-appropriate discipline and limit setting. c. Enforce visiting hours to allow parents to have respite care. d. Foster feelings of competency by helping parents learn the special care needs of the infant.

ANS: D It is important that the parents learn how to care for their infant so they feel competent. The nurse facilitates this by teaching special holding techniques, supporting breastfeeding, and encouraging frequent visiting and rooming in. The focus should be on the infant's capabilities and positive features. Infants do not usually require discipline. As the child gets older, this is necessary, but it is not a priority intervention at this time. The nursing staff negotiates with the family about the need for respite care.

An 8-year-old child is hospitalized with infectious polyneuritis (Guillain-Barré syndrome [GBS]). When explaining this disease process to the parents, what should the nurse consider? a. Paralysis is progressive with little hope for recovery. b. Disease is inherited as an autosomal, sex-linked, recessive gene. c. Disease results from an apparently toxic reaction to certain medications. d. Muscle strength slowly returns, and most children recover.

ANS: D Recovery usually begins within 2 to 3 weeks, and most patients regain full muscle strength. The paralysis is progressive with proximal muscle weakness occurring before distal weakness. The recovery of muscle strength occurs in the reverse order of onset of paralysis. Most individuals regain full muscle strength. Better outcomes are associated with younger ages. GBS is an immune-mediated disease often associated with a number of viral or bacterial infections or the administration of vaccines.

What condition is an inherited immunodeficiency disorder characterized by absence of both humoral and cell-mediated immunity? a. Fanconi syndrome b. Wiskott-Aldrich syndrome c. Acquired immunodeficiency syndrome (AIDS) d. Severe combined immunodeficiency syndrome (SCIDS)

ANS: D SCIDS is a genetic disorder that results in deficits of both humoral and cellular immunity. Fanconi syndrome is a hereditary disorder of red blood cell production. Wiskott-Aldrich syndrome is an X-linked recessive disorder with selected deficiencies of T and B lymphocytes. AIDS is not inherited.

Secondary prevention for cognitive impairment includes what activity? a. Genetic counseling b. Avoidance of prenatal rubella infection c. Preschool education and counseling services d. Newborn screening for treatable inborn errors of metabolism

ANS: D Secondary prevention involves activities that are designed to identify the condition early and initiate treatment to avert cerebral damage. Inborn errors of metabolism such as hypothyroidism, phenylketonuria, and galactosemia can cause cognitive impairment. Genetic counseling and avoidance of prenatal rubella infections are examples of primary prevention strategies to preclude the occurrence of disorders that can cause cognitive impairment. Preschool education and counseling services are examples of tertiary prevention. These are designed to include early identification of conditions and provision of appropriate therapies and rehabilitation services.

Approach behaviors

1) asks for info about Dx 2) seeks help/support from others 3) shares burden w/ others 4) plans realistically for the future 5) expresses feelings 6) realistically perceives childs condition 7. anticipates future problems verbalizes possible child loss

Involve siblings

1) limit caregiving responsibilities and include sibligs in care 2) invite siblings to attend meetings in plans of care 3)Solicit their ideas on treatment and service needs. 4. discuss future plans with them 5 have them visit professionals who work with the child 6. provide opportunities for siblings to advocate for the child

Impact of the Child's Chronic or Complex Illness on the Family

1) parental feelings of inadequacy and failure 2) can be no positive feedback from child 3) Mothers experience a peaks-and-valleys periodic crisis pattern, fathers tend to experience a steady, gradual recovery. 4) The extensive stresses in the family can leave fathers feeling depressed, weak, guilty, powerless, isolated, embarrassed, and angry. 5) siblings have problems w/ peers

examples of avoidance behaviors

1) refuses to agree to treatment 2) failure to recognize seriousness of childs condition 3) angry/hostile to staff, or acoids family members, child, stuff 4) punishes self from guilt/blame 5) suicidal intents 6) use of drugs/alcohol 7. unreaslistic future plans for the child 8. unable to adjust to progression of disease 9. continually looking for new cures 10. uses magical thinking, occult 11. puts complete faith in religion, relinquishing own responsibilities 12 withdrawal from outside world 13makes no changes in lifestyle 14 unable to discuss possible loss of child

supporting siblings of children with special needs

1) value each child individually and avoid comparisons 2) be fair in terms of discipline, attention, resources; require the affected child to do as much as can 3) help child see differences and similarities between them and a child w/ special needs. 4. teach siblings ways to interact with the child 5. intervene only to prevent siblings from hurting each other 6. ligitimize reasonable anger, children behave badly sometimes 7. respect a sibling's reluctance to be with or to include the child with special needs in activities

The nurse is conducting staff in-service training on von Willebrand's disease. Which should the nurse include as characteristics of von Willebrand's disease? Select all that apply. 1. Easy bruising occurs 2. Gum bleeding occurs 3. It is a hereditary bleeding disorder 4. Treatment and care are similar to that for hemophilia 5. It is characterized by extremely high creatinine levels 6. The disorder causes platelets to adhere to damaged endothelium

1, 2, 3, 4, 6 von Willebrand's disease is a hereditary bleeding disorder characterized by a deficiency of or a defect in a protein termed von Willebrand factor. The disorder causes platelets to adhere to damaged endothelium. It is characterized by an increased tendency to bleed from mucous membranes. Assessment findings include epistaxis, gum bleeding, easy bruising, and excessive menstrual bleeding. An elevated creatinine level is not associated with this disorder.

Parents are considering treatment options for their 5-year-old child with Legg-Calvé-Perthes disease. Both surgical and conservative therapies are appropriate. They are able to verbalize the differences between the therapies when they make what statement? a. "All therapies require extended periods of bed rest." b. "Conservative therapy will be required until puberty." c. "Our child cannot attend school during the treatment phase." d. "Surgical correction requires a 3- to 4-month recovery period."

ANS: D Surgical correction involves additional risks of anesthesia, infection, and possibly blood transfusion. The recovery period is only 3 to 4 months rather than the 2 to 4 years of conservative therapies. The use of non-weight-bearing appliances and surgical intervention does not require prolonged bed rest. Conservative therapy is indicated for 2 to 4 years. The child is encouraged to attend school and engage in activities that can be adapted to therapeutic appliances.

A child with hemophilia A will have which abnormal laboratory result? a. PT (ProTime) b. Platelet count c. Fibrinogen level d. PTT (partial thromboplastin time)

ANS: D The basic defect of hemophilia A is a deficiency of factor VIII. The partial thromboplastin time measures abnormalities in the intrinsic pathway (abnormalities in factors I, II, V, VIII, IX, X, XII, HMK, and KAL). The prothrombin time measures abnormalities of the extrinsic pathway (abnormalities in factors I, II, V, VII, and X). Fibrinogen level is not dependent on the intrinsic pathway. Platelets are not affected with hemophilia A.

Coping patterns used by children with special needs

1. develops competence and optimism - find positive aspects of the situation and concentrates more on what he or she has or can do than on what is missing or on what he or she cannot do; is as independent as possible 2. feels different and withdrawals - sees self as being different from other children because of the illness; views it as being negative, less worthy, focuses on things he or she cannot do and sometimes over restricts activities needlessly 3. is irritable, moody, act out - productive and self-initiated coping behaviors, counterproductive, not ego enhancing or socially responsible and do not result in desired outcomes; acts out irritability, may or may not be associated with condition's symtoms 4. complies with treatment - takes necessary medications, treatments, adheres to activity restrictions, uses behaviors that indicate developing independence 5. seeks support - talks with adults, children, physicians, and nurses; develops plans to handle problems as they occur

teach child self-care skills

1. encourage play and exercise 2. provide means of communication 3. establish discipline ( no moral lessons, but evaluation previous mistakes, reinforcement of desired actions, use of time-out) 4. encourage socialization ( need to be exposed to strangers to learn manners, encouraging socially acceptable beahvior like greeting, saying thank you) 5. dressing and grooming 6. provide information on sexuality ( like hygiene and contraception)

nurse interventions in normalization

1. encouraging families to reduce stress through delegation of care and family tasks 2. identify ways to incorporate care into current routines 3. structuring the home environment to encoursage the child's engagement in age appropriate activities 4. ensure families have access to appropriate community support services

effect of immobilization on families

1. financial strain 2. attention focused on affected member and other members/siblings mig†˙ feel neglected 3. family might feel difficulty accepting altered body condition 4. some members migth be unable to express feelings and to cope 5. parent's guilt over situaltion and immobilization, failing to protect

causes of guilt

1. genetics 2. personal failure or wrongdoing 3. cultural or religious beleifs 4. punishment for the past 5. trial by God child may see it as retribution for past misbehavior, deserved punishment

education of children with impaired cognitive function

1. investigate learning abilities and teach skills or prepare for health related procedures 2. they cannot discriminate between stimuli, but they will be able to if the cues are exxagerated 3. concrete ideas are learn much more effectively than abstract ideas, so demonstration is preferrable to explanation learning should be aimed at learning skill and not understanding the schientific principles for the procedure 4. short term memory deficit - need simple, one step directions 5. motivation or use of positive reinforcement

risk factors for children developing meningitis

1. lack of immunization to the specific pathogen 2. recent exposure to someone with invasive Neisseria memingitidis or H influenza type B 3. penetrating head trauma 4. cochlear implant devices 5. anatomic defects such as midline facial defects, inner ear fistulas 6. recent placemrnt of a ventricular shunt

Meningitis description

1. meningitis is an infections process of the central nervous system caused by bacteria or viruses that may be acquired as a primary disease or as a result of complictions of neurosurgery, trauma, infection of the sinuses or ears, or systemic infections. 2. Diagnosis of backterial meningitis is made by testing CSF obtained by lumbar puncture, the fluid is cloudy with increased blood pressure,

hydrocephalus preoperative interventions

1. monitor intake and output; give small, frequent feedings as tolerated until preoperative NPO status is prescribed 2. reposition the head frequently and use special devices such as an egg crate mattress under the head to prevent pressure sores. 3. prepare the child and family for diagnostic procedures and surgery.

Many of the clinical features of Down syndrome present challenges to caregivers. Based on these features, what intervention should be included in the child's care? a. Delay feeding solid foods until the tongue thrust has stopped. b. Modify the diet as necessary to minimize the diarrhea that often occurs. c. Provide calories appropriate to the child's mental age. d. Use a cool-mist vaporizer to keep the mucous membranes moist and secretions liquefied.

ANS: D The constant stuffy nose forces the child to breathe by mouth, drying the mucous membranes and increasing the susceptibility to upper respiratory tract infections. A cool-mist vaporizer will keep the mucous membranes moist and liquefy secretions. Respiratory tract infections combined with cardiac anomalies are the primary cause of death in the first years. The child has a protruding tongue, which makes feeding difficult. The parents must persist with feeding while the child continues the physiologic response of the tongue thrust. The child is predisposed to constipation. Calories should be appropriate to the child's weight and growth needs, not mental age.

What therapeutic intervention is most appropriate for a child with b-thalassemia major? a. Oxygen therapy b. Supplemental iron c. Adequate hydration d. Frequent blood transfusions

ANS: D The goal of medical management is to maintain sufficient hemoglobin (>9.5 g/dl) to prevent bone marrow expansion. This is achieved through a long-term transfusion program. Oxygen therapy and adequate hydration are not beneficial in the overall management of thalassemia. The child does not require supplemental iron. Iron overload is a problem because of frequent blood transfusions, decreased production of hemoglobin, and increased absorption from the gastrointestinal tract.

What is a major goal of therapy for children with cerebral palsy (CP)? a. Cure the underlying defect causing the disorder. b. Reverse the degenerative processes that have occurred. c. Prevent the spread to individuals in close contact with the child. d. Recognize the disorder early and promote optimum development.

ANS: D The goals of therapy include early recognition and promotion of an optimum developmental course to enable affected children to attain their potential within the limits of their dysfunction. The disorder is permanent, and therapy is chiefly symptomatic and preventive. It is not possible at this time to reverse the degenerative processes. CP is not contagious.

17. The nurse is assessing a child who was just admitted to the hospital for observation after a head injury. What clinical manifestation is the most essential part of the nursing assessment to detect early signs of a worsening condition? a. Posturing b. Vital signs c. Focal neurologic signs d. Level of consciousness

ANS: D The most important nursing observation is assessment of the child's level of consciousness. Alterations in consciousness appear earlier in the progression of an injury than do alterations of vital signs or focal neurologic signs. Neurologic posturing is indicative of neurologic damage.

The nurse is teaching parents the proper use of a hip-knee-ankle-foot orthosis (HKAFO) for their 4-year-old child. The parents demonstrate basic essential knowledge by making what statement? a. "Alcohol will be used twice a day to clean the skin around the brace." b. "Weekly visits to the orthotist are scheduled to check screws for tightness." c. "Initially, a burning sensation is expected and the brace should remain in place." d. "Condition of the skin in contact with the brace should be checked every 4 hours."

ANS: D This type of brace has several contact points with the child's skin. To minimize the risk of skin breakdown and facilitate use of the brace, vigilant skin monitoring is necessary. Alcohol should not be used on the skin. It is drying. Parents are capable of checking and tightening the screws when necessary. If a burning sensation occurs, the brace should be removed. If several complaints of burning occur, the orthotist should be contacted.

head injury interventions

1. monitor the airway; administer oxygen as prescribed 2. assess injuries 3. position the client so that the head is maintained midline to avoid jugular vein compression which can increase ICP 4. monitor vital signs and neurological functions (assess level of consciousness closely) 5. notify health care provider if signs of increased ICP occur 6. keep stimuli to a minimum; attempt to minimize crying in an infant 7. withhold sedating medications during the acute phase of the injury so that changes in levels of consciousness can be assessed 8. initiate seizure precautions 9. monitor for decreased responsiveness to pain (a significant sign of altered state of consciousness) 10. maintain NPO status or provide clear liquids, if prescribed, until it is determined that vomiting will not occur 11. monitor prescribed intravenous fluids carefully to avoid increasing any cerebral edema and to minimize the possiblity of overhydration 12. monitor for a fluid or electrolyte alteration (could indicate injury to the hypothalamus or posterior pituitary) 13. assess wounds and dressings for the presence of drainage and monitor for nose or ear drainage, which could indicate leakage of cerebrospinal fluid. 14. administer tepid sponge baths or place child on a hypothermia blanket as prescribed if hyperthermia occurs. 15. avoid suctioning through the nares because of the possibility of the catheter entering the brain through a fracture, which places the child at a high risk for a secondary infection. 16. as prescribed, administer acetaminophen for a headache, anticonvulsants for seizures and antibiotics if a laceration is present; prepare to administer prophylactic tetanus toxoid. 17. a corticosteroid or osmotic diuretic may be prescribed to reduce cerebral edema. 18. monitor for signs of brainstem involvement. 19. monitor for signs of epidural hematoma: asymmetrical pupils (one dilated, nonreactive pupil) may indicate a neurosurgical emergency that requires evacuation of the hematoma.

vulnerability/ health and developmental consequences of chronic and complex disease

1. ongoing functional impairement 2. neurodevelopmental disability 3. dependence on medical technology 4. need for ongoing skilled supportive care from HCP and family 5. risk of behavior and emotional problems

support at the time of diagnosis

1. parents should be encouraged to be together at the time of news 2. in private, comfortable setting free of distraction 3. acceptance of expression 4. arents should receive info they desire ( detailed or not etc, amount) 5. understanding of explanations 6. end conference - potential for development, strength of child 7. assure of the availability of the HCP

impact of the child with special needs

1. the child's need for medical and nursing care 2. the child's fixed deficits 3. the child's age-appropriate dependency in activities of daily living 4. the disruption in the family routine caused by the care 5. the psychologic burden of the prognosis on the family

family resources and ability to cope

1. the family's physical resources 2. the family emotional resources 3. the family educational resources 4. the family's social supports and available help 5. the competing demands for family members' time and energy

A child with a serious chronic illness will soon go home. The case manager requests that the family provide total care for the child for a couple of days while the child is still hospitalized. How should the request be viewed? a. Improper because of legal issues b. Supportive because families are usually eager to get involved c. Unacceptable because the family will have to assume the care soon enough d. Important because it can be beneficial to the transition from hospital to home

ANS: D This type of groundwork is essential for the family. Adequate family training and preparation will assist in the child's transition home. The nursing staff in the hospital is responsible for the child's care. The family will provide the care with assistance as needed. Although parents are eager to be involved, the purpose of this intervention is the development of family competency and confidence that they are capable. Arrangements for respite care are important for the family both during hospitalizations and while the child is at home.

The nurse is caring for an immobilized preschool child. What intervention is helpful during this period of immobilization? a. Encourage wearing pajamas. b. Let the child have few behavioral limitations. c. Keep the child away from other immobilized children if possible. d. Take the child for a "walk" by wagon outside the room.

ANS: D Transporting the child outside of the room by stretcher, wheelchair, or wagon increases environmental stimuli and provides social contact. Street clothes are preferred for hospitalized children. This decreases the sense of illness and disability. The child needs appropriate limits for both adherence to the medical regimen and developmental concerns. It is not necessary to keep the child away from other immobilized children.

A woman who is 6 weeks pregnant tells the nurse that she is worried that, even though she is taking folic acid supplements, the baby might have spina bifida because of a family history. The nurse's response should be based on what? a. Prenatal detection is not possible yet. b. There is no genetic basis for the defect. c. Chromosome studies done on amniotic fluid can diagnose the defect prenatally. d. Open neural tube defects (NTDs) result in elevated concentrations of a-fetoprotein in amniotic fluid.

ANS: D Ultrasound scanning and measurement of a-fetoprotein may indicate the presence of anencephaly or myelomeningocele. The optimum time for performing this analyzing is between 16 and 18 weeks. Prenatal diagnosis is possible through amniocentesis. A multifactorial origin is suspected, including drugs, radiation, maternal malnutrition, chemicals, and possibly a genetic mutation. Chromosome abnormalities are not present in NTDs.

What is a nursing intervention to reduce the risk of increasing intracranial pressure (ICP) in an unconscious child? a. Suction the child frequently. b. Turn the child's head side to side every hour. c. Provide environmental stimulation. d. Avoid activities that cause pain or crying.

ANS: D Unrelieved pain, crying, and emotional stress all contribute to increasing the ICP. Disturbing procedures should be carried out at the same time as therapies that reduce ICP, such as sedation. Suctioning is poorly tolerated by children. When necessary, it is preceded by hyperventilation with 100% oxygen. Turning the head side to side is contraindicated for fear of compressing the jugular vein. This would block the flow of blood from the brain, raising ICP. Nontherapeutic touch and environmental stimulation increase ICP. Minimizing both touch and environmental stimuli noise reduces ICP.

What do we do for it? Legg-calve prthe

Activity restrictions and limited weight bearing helps reduce inflammation of hip. Use of NSAIDS. Physical therapy or ROM exercises. Braces, casting, traction. Do a bone graft, surgery Pelvic or proximal femoral osteotomy may be necessary.

ITP Forms

Acute, self-limiting Often follows URI or other infection Chronic (more than 6 months' duration)

A 12-month-old with a history of otitis media is in the office for her 1-year check-up. She was diagnosed at 9 months of age with immune thrombocytopenic purpura (ITP) and treated with intravenous immunoglobulin (IVIG). Which provider order should the nurse question? Obtain a platelet level. Perform a hearing screening. Assess for psychosocial delays. Administer MMR (measles-mumps-rubella) vaccine.

Administer MMR (measles-mumps-rubella) vaccine. IVIG prevents formation of antibodies. Vaccinations should be delayed for 8 to 10 months after IVIG administration.

The nurse is admitting a young child to the hospital because bacterial meningitis is suspected. What is a priority of nursing care? Initiate isolation precautions as soon as diagnosis is confirmed. Provide environmental stimulation to keep the child awake. Administer antibiotic therapy as soon as it is available. Administer sedatives and analgesics on a preventive schedule to manage pain.

Administer antibiotic therapy as soon as it is available. Correct Administering antibiotic therapy is the priority action. Antibiotics are begun as soon as possible to avoid resultant disabilities and to prevent death. Isolation should be instituted as soon as diagnosis is anticipated. It is important to decrease the external stimuli. The nurse should keep the room as quiet as possible. Antibiotics are the priority function; pain should be managed if it occurs

The nurse is caring for a child with disseminated intravascular coagulation (DIC) after a surgical procedure. The nurse notes continued bleeding from the ears and intravenous (IV) site and a hemoglobin of 6.7. The child has had no urine output for the past 2 hours. After notifying the health care provider, which orders would the nurse anticipate? Select all that apply. Insert Foley catheter. Establish a second IV access site. Administer clotting factor. Administer a normal saline fluid bolus. Transfuse packed red blood cells (PRBCs).

Administer clotting factor. The nurse would administer clotting factor in a child with signs of DIC. Administer a normal saline fluid bolus. The nurse would give a fluid bolus to a patient with low hemoglobin and other signs of DIC. Transfuse packed red blood cells (PRBCs). The nurse would start a transfusion of PRBCs for a child with signs of DIC.

The nurse is caring for a pediatric patient admitted with seizure activity related to cerebral palsy (CP). Which interventions should the nurse perform immediately? Select all that apply. Administer intravenous diazepam (Valium). Institute safety measures such as seizure padding. Assess gait disturbances and muscle coordination. Consult speech therapy for a swallowing evaluation. Evaluate gross motor development and muscle tone.

Administer intravenous diazepam (Valium). Administering a benzodiazepine will decrease spasticity and seizure activity. This is an immediate need. Institute safety measures such as seizure padding. Safety is a primary goal, and the nurse must prevent the child from sustaining unnecessary injuries.

The nursing student is presenting a clinical conference and discusses the cause of β-thalassemia. The nursing student informs the group that a child at greatest risk of developing this disorder is which of these? a. a child of mexican descent b. a child of mediterranean descent c. a child whose intake of iron is extremely poor d.a breast fed child of a mother with chronic anemia

A child of mediterranean descent; β-Thalassemia is an autosomal recessive disorder characterized by the reduced production of 1 of the globin chains in the synthesis of hemoglobin (both parents must be carriers to produce a child with β-thalassemia major). This disorder is found primarily in individuals of Mediterranean descent.

A nurse has been assigned to see four patients who are all recovering from soft tissue injuries in the orthopedic unit. Which patient should she see first? A patient who is 6 days post injury with bruising A patient who is 2 days post injury with swelling at the site A patient who was injured yesterday and has a tingling sensation A patient who is 5 days post injury and beginning to bear weight

A patient who was injured yesterday and has a tingling sensation A tingling sensation can be a sign of a more serious problem and the nurse should see this patient first.

A child who has been in a cast for a long period of time is brought to the hospital for its removal. Which is the most appropriate teaching by the nurse at the time of cast removal? The cast cutter may cut the skin a little bit. The cutting may produce heat, and it may burn the skin. After cast removal, a rigorous scrubbing of the skin is necessary. A prolonged period of casting may result in decreased muscle mass.

A prolonged period of casting may result in decreased muscle mass. Rationale Decreased muscle mass is noted if the cast is in place for a lengthy period of time. Exercise and routine activities will help in gradual return of function and appearance if there was no other trauma beforehand. The cast cutter works by vibration and cuts only the hard part of the cast; it does not cut the skin. There may be heat generation while cutting the cast, but it will not burn the skin. After the cast is removed, the underlying skin is covered with desquamated skin and sebaceous secretions. It should be removed by a simple soak in a bathtub; vigorous scrubbing may cause excoriation and bleeding.

unconscious child care

ABC priority, stabilize spine, treat shock, reduce ICP, continual/ frequent observation of LOC, pupillary reactions, size, equality response to vocal commands and painful stimuli, any abnormal movements, changes in muscle tone pain management -> bowel management after opioids; Quiet, dimly lit environment Limit visitors Prevent sudden, jarring movement Proper positioning Prevention of straining during coughing, vomiting, and defecating vital signs (child often in PICU/ intensive setting) temp q2-4 h supply adequate nutrition and hydration, provide drug therapy Signs of pain Changes in behavior (inc. agitation and rigidity) Alterations in vital signs and perfusion (inc. HR, RR, BP, dec. O2) elimination: foley, stool softeners bathe daily, mouth care, eye care position child to prevent aspiration or airway obstruction ( tracheostomy, suctioning, nasotracheal or orotrcheal tube or mechanical ventilation chest physiotherapy Q2H to prevent pulmonary complications monitor blood gasas for metabolic alkalosis

The health care provider has prescribed oxybutynin (Ditropan) 0.2 mg/kg/day divided bid for a child with myelomeningocele. The child weighs 33 lb. The nurse is preparing to administer the 0900 dose. Calculate the dose the nurse should administer in milligrams. Record your answer below using one decimal place.

ANS: 1.5 The correct calculation is: 33 lb/2.2 kg = 15 kg Dose of Ditropan is 0.2 mg/kg/day divided bid 0.2 mg ´ 15 = 3 mg 3 mg/2 = 1.5 mg

The health care provider has prescribed diazepam (Valium) 0.8 mg/kg/day PO divided q 6 hours for a child with cerebral palsy. The child weighs 110 lb. The nurse is preparing to administer the 1200 dose. Calculate the dose the nurse should administer in milligrams. Record your answer below in a whole number.

ANS: 10 The correct calculation is: 110 lb/2.2 kg = 50 kg Dose of Valium is 30 mg/kg/day divided q 6 hours 0.8 mg ´ 50 = 40 mg/day 40 mg/4 = 10 mg for one dose

The health care provider has prescribed dantrolene sodium (Dantrium) 0.5 mg/kg PO once a day for a child with cerebral palsy. The child weighs 55 lb. Calculate the dose the nurse should administer in milligrams. Record your answer below using one decimal place.

ANS: 12.5 The correct calculation is: 55 lb/2.2 kg = 25 kg Dose of Dantrium is 0.5 mg/kg given once a day 0.5 mg ´ 25 = 12.5 mg

The health care provider has prescribed valproic acid (Depakene) 30 mg/kg/day divided bid for a child with cerebral palsy having seizures. The child weighs 22 lb. The nurse is preparing to administer the 0900 dose. Calculate the dose the nurse should administer in milligrams. Record your answer below in a whole number.

ANS: 165 The correct calculation is: 22 lb/2.2 kg = 11 kg Dose of Depakene is 30 mg/kg/day divided bid 30 mg ´ 11 = 330 mg 330 mg/2 = 165 mg for one dose

The nurse is planning care for a 3-year-old boy who has Down syndrome and is on continuous oxygen. He recently began walking around furniture. He is spoon fed by his parents and eats some finger foods. What goal is the most appropriate to promote normal development? a. Encourage mobility. b. Encourage assistance in self-care. c. Promote oral-motor development. d. Provide opportunities for socialization.

ANS: A A major principle for developmental support in children with complex medical issues is that it should be flexible and tailored to the individual child's abilities, interests, and needs. This child is exhibiting readiness for ambulation. It is an appropriate time to provide activities that encourage mobility, for example, longer oxygen tubing. Parents should provide decreasing amounts of assistance with self-care as he is able to develop these skills. The boy is receiving oral foods and is eating finger foods. He has acquired this skill. Mobility is a new developmental task. Opportunities for socialization should be ongoing.

A child's parents ask the nurse many questions about their child's illness and its management. The nurse does not know enough to answer all the questions. What nursing action is most appropriate at this time? a. Tell them, "I don't know, but I will find out." b. Suggest that they ask the physician these questions. c. Explain that the nurse cannot be expected to know everything. d. Answer questions vaguely so they do not lose confidence in the nurse.

ANS: A Questions from parents should be answered in a straightforward manner. Stating "I don't know" or "I'll find out" is better than pretending to know or giving excuses. Suggesting that they ask the physician these questions is not supportive of the family. The nurse's role is to assist the parents in obtaining accurate information about their child's illness and its management. Although the nurse cannot be expected to know everything, it is an unprofessional attitude to state this. Nurses must provide accurate information to the extent possible. Vague answers are not helpful to the family.

A school-age child is admitted in vasoocclusive sickle cell crisis (pain episode). The child's care should include which therapeutic interventions? a. Hydration and pain management b. Oxygenation and factor VIII replacement c. Electrolyte replacement and administration of heparin d. Correction of alkalosis and reduction of energy expenditure

ANS: A The management of crises includes adequate hydration, pain management, minimization of energy expenditures, electrolyte replacement, and blood component therapy if indicated. Factor VIII is not indicated in the treatment of vasoocclusive sickle cell crisis. Oxygen may prevent further sickling, but it is not effective in reversing sickling because it cannot reach the clogged blood vessels. Also, prolonged oxygen can reduce bone marrow activity. Heparin is not indicated in the treatment of vasoocclusive sickle cell crisis. Electrolyte replacement should accompany hydration. The acidosis will be corrected as the crisis is treated. Energy expenditure should be minimized to improve oxygen utilization. Acidosis, not alkalosis, results from hypoxia, which also promotes sickling.

If an intramuscular (IM) injection is administered to a child who has Reye syndrome, the nurse should monitor for what? a. Bleeding b. Infection c. Poor absorption d. Itching at the injection site

ANS: A The nurse should watch for bleeding from the site. Because of related liver dysfunction with Reye syndrome, laboratory studies, such as prolonged bleeding time, should be monitored to determine impaired coagulation.

A toddler is diagnosed with chronic benign neutropenia. The parents are being taught about caring for their child. What information is important to include? a. Avoid large indoor crowds and people who are ill. b. Parenteral antibiotics are necessary to control disease. c. Frequent rest periods are needed during the daytime. d. List the side effects of corticosteroids used to decrease inflammation.

ANS: A The parents are taught to minimize risk of infection by avoiding crowded areas and individuals who are ill. Parents are also cautioned about when to notify their practitioner and administration of granulocyte colony-stimulating factor, if indicated. Antibiotics are not needed unless the child has an infection. The toddler does not need any additional rest as a result of the neutropenia. Corticosteroids are not indicated.

The nurse is planning care for a school-age child with bacterial meningitis. What intervention should be included? a. Keep environmental stimuli to a minimum. b. Have the child move her head from side to side at least every 2 hours. c. Avoid giving pain medications that could dull sensorium. d. Measure head circumference to assess developing complications.

ANS: A The room is kept as quiet as possible and environmental stimuli are kept to a minimum. Most children with meningitis are sensitive to noise, bright lights, and other external stimuli. The nuchal rigidity associated with meningitis would make moving the head from side to side a painful intervention. If pain is present, the child should be treated appropriately. Failure to treat can cause increased intracranial pressure. In this age group, the head circumference does not change. Signs of increased intracranial pressure would need to be assessed.

What nursing intervention is most appropriate in promoting normalization in a school-age child with a chronic illness? a. Give the child as much control as possible. b. Ask the child's peer to make the child feel normal. c. Convince the child that nothing is wrong with him or her. d. Explain to parents that family rules for the child do not need to be the same as for healthy siblings.

ANS: A The school-age child who is ill may be forced into a period of dependency. To foster normalcy, the child should be given as much control as possible. It is unrealistic for one individual to make the child feel normal. The child has a chronic illness, so it would be unacceptable to convince the child that nothing is wrong. The family rules should be similar for each of the children in a family. Resentment and hostility can arise if different standards are applied to each child.

A child develops syndrome of inappropriate antidiuretic hormone secretion (SIADH) as a complication to meningitis. What action should be verified before implementing? a. Forcing fluids b. Daily weights with strict input and output (I and O) c. Strict monitoring of urine volume and specific gravity d. Close observation for signs of increasing cerebral edema

ANS: A The treatment of SIADH consists of fluid restriction until serum electrolytes and osmolality return to normal levels. SIADH often occurs in children who have meningitis. Monitoring weights, keeping I and O and specific gravity of urine, and observing for signs of increasing cerebral edema are all part of the nursing care for a child with SIADH.

A 12-year-old child has failed several courses of chemotherapy. An experimental drug is available that his parents want him to receive. He has told his parents and the oncologists that he is ready to die and does not want any more chemotherapy. The nurse recognizes what to be true? a. Parents and child both need support in the decision making. b. Twelve-year-olds are minors and cannot give consent or refuse treatments. c. The oncologists needs to make the decision because the parents and child disagree. d. The parents have the right and responsibility to make decisions for their children younger than age 18 years.

ANS: A This is a family issue that requires support to help both parents and child resolve the conflict. Because the child has little chance of survival, many institutions support the child's right to refuse or assent to therapy. The institution can obtain a court order to support the child's decision if verified by the oncologists. Twelve-year-olds can give consent for therapy under certain conditions, including being an emancipated minor and receiving therapy for birth control and sexually transmitted infections. Right to self-determination is also accepted if the child is fully aware of the consequences of the actions. The practitioners cannot take the responsibility for decision making from the parent or child. Parents have the responsibility for decision making, but certain circumstances do limit their authority.

The nurse is teaching the parents of a 1-month-old infant with developmental dysplasia of the hip about preventing skin breakdown under the Pavlik harness. What statement by the parent would indicate a correct understanding of the teaching? a. "I should gently massage the skin under the straps once a day to stimulate circulation." b. "I will apply a lotion for sensitive skin under the straps after my baby has been given a bath to prevent skin irritation." c. "I should remove the harness several times a day to prevent contractures." d. "I will place the diaper over the harness, preferably using a superabsorbent disposable diaper that is relatively thin."

ANS: A To prevent skin breakdown with an infant who has developmental dysplasia of the hip and is in a Pavlik harness, the parent should gently massage the skin under the straps once a day to stimulate circulation. The parent should not apply lotions or powder because this could irritate the skin. The parent should not remove the harness, except during a bath, and should place the diaper under the straps

The potential effects of chronic illness or disability on a child's development vary at different ages. What developmental alteration is a threat to a toddler's normal development? a. Hindered mobility b. Limited opportunities for socialization c. Child's sense of guilt that he or she caused the illness or disability d. Limited opportunities for success in mastering toilet training

ANS: A Toddlers are acquiring a sense of autonomy, developing self-control, and forming symbolic representation through language acquisition. Mobility is the primary tool used by toddlers to experiment with maintaining control. Loss of mobility can create a sense of helplessness. Toddlers do not socialize. They are sensitive to changes in family routines. A sense of guilt is more likely to occur in a preschooler. Toilet training is not usually mastered until the end of the toddler period

The school nurse is informed that a child with human immunodeficiency virus (HIV) infection will be attending school soon. What is an important nursing intervention to include in the plan of care? a. Carefully follow universal precautions. b. Inform the parents of the other children. c. Determine how the child became infected. d. Reassure other children that they will not become infected.

ANS: A Universal precautions are necessary to prevent further transmission of the disease. Informing the parents of the other children would violate the child's right to privacy. It is not within the role of the school nurse to determine how the child became infected. Reassuring other children that they will not become infected violates the child's privacy. General health classes can discuss prevention of HIV transmission.

5. What measure is important in managing hypercalcemia in a child who is immobilized? a. Provide adequate hydration. b. Change position frequently. c. Encourage a diet high in calcium. d. Provide a diet high in calories for healing.

ANS: A Vigorous hydration is indicated to prevent problems with hypercalcemia. Suggested intake for an adolescent is 3000 to 4000 ml/day of fluids. Diuretics are used to promote the removal of calcium. Changing position is important for skin and respiratory concerns. Calcium in the diet is restricted when possible. A high-protein diet served as frequent snacks with favored foods is recommended. A high-calorie diet without adequate protein will not promote healing.

A 4-month-old with significant head lag meets the criteria for floppy infant syndrome. A diagnosis of progressive infantile spinal muscular atrophy (Werdnig-Hoffmann disease) is made. What should be included in the nursing care for this child? a. Infant stimulation program b. Stretching exercises to decrease contractures c. Limited physical contact to minimize seizures d. Encouraging parents to have additional children

ANS: A Werdnig-Hoffmann disease (spinal muscular atrophy type 1) is the most common paralytic form of floppy infant syndrome (congenital hypotonia). An infant stimulation program is essential. Frequent position changes, including changes in environment, provide the child with more physical contacts. Verbal, tactile, and auditory stimulation are also included. Contractures do not occur because of muscular atrophy. Sensation is normal in children with this disorder. Frequent touch is necessary as part of the stimulation. Werdnig-Hoffmann disease is inherited as an autosomal recessive trait. Parents should be referred for genetic counseling

What is important to incorporate in the plan of care for a child who is experiencing a seizure? a. Describe and record the seizure activity observed. b. Suction the child during a seizure to prevent aspiration. c. Place a tongue blade between the teeth if they become clenched. d. Restrain the child when seizures occur to prevent bodily harm.

ANS: A When a child is having a seizure, the priority nursing care is observation of the child and seizure. The nurse then describes and records the seizure activity. The child is not suctioned during the seizure. If possible, the child should be placed on the side, facilitating drainage to prevent aspiration.

What nursing intervention is most appropriate when providing comfort and support for a child when death is imminent? a. Limit care to essentials. b. Avoid playing music near the child. c. Whisper to the child instead of using a normal voice. d. Explain to the child the need for constant measurement of vital signs.

ANS: A When death is imminent, care should be limited to interventions for palliative care. Music may be used to provide comfort to the child. The nurse should speak to the child in a clear, distinct voice. Vital signs do not need to be measured frequently.

What functional goal should the nurse expect for a child who has a T1 to T10 spinal cord injury? (Select all that apply.) a. May be braced for standing b. Able to drive automobile with hand controls c. Can manage adapted public transportation d. Some able to use regular public transportation e. Ambulates well, often with short leg braces with or without cane

ANS: A, B, C A child with a T1 to T10 spinal cord injury may be braced for standing, is able to drive an automobile with hand controls, and can manage adapted public transportation. The ability to use regular public transportation and ambulation with bilateral long braces using four-point or swing-through crutch gait are functional goals for individuals with a T10 to L2 injury.

A child has a slight (26-40 dB) degree of hearing loss. The nurse recognizes this amount of hearing loss can have what effect? (Select all that apply.) a. No speech defects b. Difficulty hearing faint speech c. Usually is unaware of the hearing difficulty d. Can distinguish vowels but not consonants e. Unable to understand conversational speech

ANS: A, B, C A child with a slight degree of hearing loss has no speech defects, may have difficulty hearing faint speech, and is usually unaware of the hearing difficulty. The ability to distinguish vowels but not consonants is an effect of severe hearing loss and being unable to understand conversational speech is an effect of moderately severe hearing loss.

The nurse is teaching the family with a child with cerebral palsy (CP) strategies to prevent constipation. What should the nurse include in the teaching session? (Select all that apply.) a. Increase fluid intake. b. Increase fiber in the diet. c. Administer stool softeners daily as prescribed. d. Increase the amount of dairy products in the diet. e. Allow the child to decide when to try to have a bowel movement.

ANS: A, B, C A variety of factors, including decreased mobility, decreased fluid intake, a fear of toileting, poor positioning on the toilet, and lack of fiber intake may be responsible for constipation for a child with CP. Stool softeners, laxatives, and a bowel management program may be required to prevent chronic constipation. The child should be placed on the toilet or encouraged to have a bowel movement at the same time each day. Dairy products can cause constipation.

The nurse is preparing to admit a 5-year-old with an epidural hemorrhage. What clinical manifestations should the nurse expect to observe? (Select all that apply.) a. Headache b. Vomiting c. Irritability d. Cephalhematoma e. Pallor with anemia

ANS: A, B, C The classic clinical picture of an epidural hemorrhage is a lucid interval (momentary unconsciousness) followed by a normal period for several hours, and then lethargy or coma due to blood accumulation in the epidural space and compression of the brain. The child may be seen with varying degrees of impaired consciousness depending on the severity of the traumatic injury. Common symptoms in a child with no neurologic deficit are irritability, headache, and vomiting. In infants younger than 1 year of age, the most common symptoms are irritability, pallor with anemia, and cephalhematoma.

The nurse is preparing to admit a 4-year-old child with chronic benign neutropenia. What clinical features of chronic benign neutropenia should the nurse recognize? (Select all that apply.) a. Gingivitis is present. b. Anemia is not present. c. Monocytosis is present. d. It has an autosomal recessive pattern. e. Treatment is by bone marrow transplantation.

ANS: A, B, C The clinical features of chronic benign neutropenia include gingivitis, no anemia, and monocytosis. It is not inherited, and because it is benign, it does not require treatment except antibiotics as indicated.

The nurse is preparing to admit a 2-year-old child with spina bifida occulta. What clinical manifestations of spina bifida occulta should the nurse expect to observe? (Select all that apply.) a. Dark tufts of hair b. Skin depression or dimple c. Port-wine angiomatous nevi d. Soft, subcutaneous lipomas e. Bladder and sphincter paralysis

ANS: A, B, C, D Clinical manifestations of spina bifida occulta include dark tufts of hair; skin depression or dimple; port-wine angiomatous nevi; and soft, subcutaneous lipomas. Bladder and sphincter paralysis are present with spina bifida cystica but not occulta.

What are some of the associated disabilities seen with cerebral palsy? (Select all that apply.) a. Visual impairment b. Hearing impairment c. Speech difficulties d. Intellectual impairment e. Associated heart defects

ANS: A, B, C, D Some of the disabilities associated with CP are visual impairment, hearing impairment, behavioral problems, communication and speech difficulties, seizures, and intellectual impairment. Additional sensory deficits such as hypersensitivity, hyposensitivity, and balance difficulties may occur in children with CP.

The nurse is caring for a 12-year-old child with b-thalassemia. What clinical manifestations should the nurse expect to observe? (Select all that apply.) a. Anorexia b. Unexplained fever c. Enlarged spleen or liver d. Bronzed, freckled complexion e. Precocious sexual development

ANS: A, B, C, D The clinical manifestations of b-thalassemia include anorexia; unexplained fever; an enlarged spleen or liver; and a bronzed, freckled complexion. There is delayed sexual maturation, not precocious.

What are supportive interventions that can assist a preschooler with a chronic illness to meet developmental milestones? (Select all that apply.) a. Encourage socialization. b. Encourage mastery of self-help skills. c. Provide devices that make tasks easier. d. Clarify that the cause of the child's illness is not his or her fault. e. Discuss planning for the future and how the condition can affect choices.

ANS: A, B, C, D To encourage initiative, mastery of self-help skills should be encouraged, and devices should be provided that make tasks easier. To develop peer relationships, socialization should be encouraged. To develop body image, the fact that the cause of the child's illness is not the fault of the child should be emphasized. Discussing planning for the future and how the condition can affect choices is appropriate for an adolescent.

The nurse is assessing coping behaviors of a family with a child with a chronic illness. What indicates avoidance coping behaviors? (Select all that apply.) a. Refuses to agree to treatment b. Avoids staff, family members, or child c. Is unable to discuss possible loss of the child d. Recognizes own growth through a passage of time e. Makes no change in lifestyle to meet the needs of other family members

ANS: A, B, C, E Avoidance coping behaviors include refusing to agree to treatment; avoiding staff, family members, or child; unable to discuss possible loss of the child; and making no change in lifestyle to meet the needs of other family members. Recognizing one's own growth through a passage of time is an approach behavior.

The nurse is preparing to admit a neonate with bacterial meningitis. What clinical manifestations should the nurse expect to observe? (Select all that apply.) a. Jaundice b. Cyanosis c. Poor tone d. Nuchal rigidity e. Poor sucking ability

ANS: A, B, C, E Clinical manifestations of bacterial meningitis in a neonate include jaundice, cyanosis, poor tone, and poor sucking ability. The neck is usually supple in neonates with meningitis, and there is no nuchal rigidity.

The nurse is preparing to admit a 6-month-old infant with increased intracranial pressure (ICP). What clinical manifestations should the nurse expect to observe in this infant? (Select all that apply.) a. High-pitched cry b. Poor feeding c. Setting-sun sign d. Sunken fontanel e. Distended scalp veins f. Decreased head circumference

ANS: A, B, C, E Clinical manifestations of increased ICP in an infant include a high-pitched cry, poor feeding, setting-sun sign, and distended scalp veins. The infant would have a tense, bulging fontanel and an increased head circumference.

The nurse is administering a unit of blood to a child. What are signs and symptoms of a transfusion reaction? (Select all that apply.) a. Chills b. Shaking c. Flank pain d. Hypothermia e. Sudden severe headache

ANS: A, B, C, E Signs and symptoms of a transfusion reaction include chills, shaking, flank pain, and sudden severe headache. Hyperthermia, not hypothermia, occurs.

The nurse is preparing to admit an adolescent with encephalitis. What clinical manifestations should the nurse expect to observe? (Select all that apply.) a. Malaise b. Apathy c. Lethargy d. Hypoactivity e. Hypothermia

ANS: A, B, D The clinical manifestations of encephalitis include malaise, apathy, and lethargy. There is hyperactivity, not hypoactivity, and hyperthermia, not hypothermia.

The nurse is assisting with application of a synthetic cast on a child with a fractured humerus. What are the advantages of a synthetic cast over a plaster of Paris cast? (Select all that apply.) a. Less bulky b. Drying time is faster c. Molds readily to body part d. Permits regular clothing to be worn e. Can be cleaned with small amount of soap and water

ANS: A, B, D, E The advantages of synthetic casts over plaster of Paris casts are that they are less bulky, dry faster, permit regular clothes to be worn, and can be cleaned. Plaster of Paris casts mold readily to a body part, but synthetic casts do not mold easily to body parts

What characterizes a preschooler's concept of death? (Select all that apply.) a. Belief their thoughts can cause death. b. They have a concrete understanding of death. c. Death is seen as temporary and gradual. d. Death is seen as a departure, a kind of sleep. e. They usually have some sense of the meaning of death.

ANS: A, C, D, E A preschool child's concept of death includes believing that his or her thoughts can cause death, seeing death as temporary and gradual and a kind of sleep, and having some sense of the meaning of death. Having a concrete understanding of death is a characteristic of a school-age child's concept of death.

The nurse is preparing to admit a 10-year-old child with Duchenne muscular dystrophy. What clinical features of Duchenne muscular dystrophy should the nurse recognize? (Select all that apply.) a. Calf muscle hypertrophy b. Late onset, usually between 6 and 8 years of age c. Progressive muscular weakness, wasting, and contractures d. Loss of independent ambulation by 9 to 12 years of age e. Slowly progressive, generalized weakness during adolescence

ANS: A, C, D, E Clinical features of Duchenne muscular dystrophy include calf muscle hypertrophy; progressive muscular weakness; wasting and contractures; loss of independent ambulation by 9 to 12 years of age; and slowly progressive, generalized weakness during adolescence. The onset is early, not late, usually between 3 and 5 years of age.

What does the nurse recognize as physical signs of approaching death? (Select all that apply.) a. Mottling of skin b. Decreased sleeping c. Cheyne-Stokes respirations d. Loss of the sense of hearing e. Decreased appetite and thirst

ANS: A, C, E Physical signs of approaching death include mottling of skin, Cheyne-Stokes respirations, and decreased appetite and thirst. Sleeping increases, not decreases, and hearing is the last sense to fail.

The nurse is caring for a child with increased intracranial pressure (ICP). What interventions should the nurse plan for this child? (Select all that apply.) a. Avoid jarring the bed. b. Keep the room brightly lit. c. Keep the bed in a flat position. d. Administer prescribed stool softeners. e. Administer a prescribed antiemetic for nausea.

ANS: A, D, E Other measures to relieve discomfort for a child with ICP include providing a quiet, dimly lit environment; limiting visitors; preventing any sudden, jarring movement, such as banging into the bed; and preventing an increase in ICP. The latter is most effectively achieved by proper positioning and prevention of straining, such as during coughing, vomiting, or defecating. An antiemetic should be administered to prevent vomiting, and stool softeners should be prescribed to prevent straining with bowel movements. The head of the bed should be elevated 15 to 30 degrees.

The nurse is caring for a child immobilized because of Russel traction. What interventions should the nurse implement to prevent renal calculi? (Select all that apply.) a. Monitor output. b. Encourage the patient to drink apple juice. c. Encourage milk intake. d. Ensure adequate fluids. e. Encourage the patient to drink cranberry juice.

ANS: A, D, E To prevent renal calculi in a child who is immobilized, a nurse should monitor output; ensure adequate fluids; and encourage cranberry juice, which acidifies urine. Apple juice and milk alkalize the urine, so they should not be encouraged.

What characterizes a toddler's concept of death? (Select all that apply.) a. They are unable to comprehend an absence of life. b. They may recognize the fact of physical death. c. They understand the universality and inevitability of death. d. The are affected more by the change in lifestyle than the concept of death. e. They can only think about events in terms of their own frame of reference—living.

ANS: A, D, E Toddlers are egocentric and can only think about events in terms of their own frame of reference—living. Their egocentricity and vague separation of fact and fantasy make it impossible for them to comprehend absence of life. Instead of understanding death, this age group is affected more by any change in lifestyle. Toddlers do not understand the universality and inevitability of death and do not recognize the fact of physical death.

The nurse is caring for a 14-year-old child with disseminated intravascular coagulation (DIC). What clinical manifestations should the nurse expect to observe? (Select all that apply.) a. Petechiae b. Chronic diarrhea c. Hepatosplenomegaly d. Bleeding from openings in the skin e. Hypotension f. Purpura

ANS: A, D, E, F Some clinical manifestations of DIC are petechiae, bleeding from openings in the skin, hypotension, and purpura. Hepatosplenomegaly and chronic diarrhea are clinical manifestations of human immunodeficiency virus (HIV) infection in children.

The nurse is making a home visit 48 hours after the death of an infant from sudden infant death syndrome (SIDS). What intervention is an appropriate objective for this visit? a. Give contraceptive information. b. Provide information on the grief process. c. Reassure parents that SIDS is not likely to occur again. d. Thoroughly investigate the home situation to verify SIDS as the cause of death.

ANS: B A home visit after the death of an infant is an excellent time to help the parents with the grief process. The nurse can clarify misconceptions about SIDS and provide information on support services and coping issues. Giving contraceptive information is inappropriate unless requested by parents. Telling the parents that SIDS is not likely to occur again is a false reassurance to the family. Investigating the home situation to verify SIDS as the cause of death is not the nurse's role; this would have been done by legal and social services if there were a question about the infant's death.

A goal for children with spina bifida is to reduce the chance of allergy development. What is a priority nursing intervention? a. Recommend allergy testing. b. Provide a latex-free environment. c. Use only powder-free latex gloves. d. Limit use of latex products as much as possible.

ANS: B A latex-free environment is the goal. This includes eliminating the use of latex gloves and other medical devices containing latex. Allergy testing would provide information about whether the allergy has developed. It will not reduce the chances of developing the allergy. Although powder-free latex gloves are less allergenic, latex should not be used. Limiting the use of latex products is one component of providing a latex-free environment, but latex products should not be used.

What statement best describes a subdural hematoma? a. Bleeding occurs between the dura and the skull. b. Bleeding occurs between the dura and the cerebrum. c. Bleeding is generally arterial, and brain compression occurs rapidly. d. The hematoma commonly occurs in the parietotemporal region.

ANS: B A subdural hematoma is bleeding that occurs between the dura and the cerebrum as a result of a rupture of cortical veins that bridge the subdural space. An epidural hemorrhage occurs between the dura and the skull, is usually arterial with rapid brain concussion, and occurs most often in the parietotemporal region.

The nurse stops to assist an adolescent who has experienced severe trauma when hit by a motorcycle. The emergency medical system (EMS) has been activated. The first person who provided assistance applied a tourniquet to the child's leg because of arterial bleeding. What should the nurse do related to the tourniquet? a. Loosen the tourniquet. b. Leave the tourniquet in place. c. Remove the tourniquet and apply direct pressure if bleeding is still present. d. Remove the tourniquet every 5 minutes, leaving it off for 30 seconds each time.

ANS: B A tourniquet is applied only as a last resort, and then it is left in place and not loosened until definitive treatment is available. After the tourniquet is applied, skin and tissue necrosis occur below the site. Loosening or removing the tourniquet allows toxins from the tissue necrosis to be released into the circulation. This can induce systemic, deadly tourniquet shock.

The nurse is providing support to a family that is experiencing anticipatory grief related to their child's imminent death. What statement by the nurse is therapeutic? a. "Your other children need you to be strong." b. "You have been through a very tough time." c. "His suffering is over; you should be happy." d. "God never gives us more than we can handle."

ANS: B Acknowledging that the family has been through a very tough time validates the loss that the parents have experienced. It is nonjudgmental. After the death of a child, the parent recognizes the responsibilities to the rest of the family but needs to be able to experience the grief of the loss. Telling the parents what they should do is giving advice. The parent would not be happy that the child has died, and stating so is argumentative. The parents may be angry with God, or their religious beliefs may be unknown, so the nurse should not provide false reassurance by talking to them about God.

The middle school nurse is speaking to parents about prevention of injuries as a goal of the physical education program. How should the goal be achieved? a. Use of protective equipment at the family's discretion b. Education of adults to recognize signs that indicate a risk for injury c. Sports medicine program to help student athletes work through overuse injuries d. Arrangements for multiple sports to use same athletic fields to accommodate more children

ANS: B Adults close to sports activities need to be aware of the early warning signs of fatigue, dehydration, and risk for injury. School policy should require mandatory use of protective equipment. Proper sports medicine therapy does not support "working through" overuse injuries. Too many students involved in different activities create distractions, which contribute to the child losing focus. This is a contributing factor to injury.

What is the major health concern of children in the United States? a. Acute illness b. Chronic illness c. Congenital disabilities d. Nervous system disorders

ANS: B An estimated 18% of children in the United States have a chronic illness or disability that warrants health care services beyond those usually required by children. Chronic illness has surpassed acute illness as the major health concern for children. Congenital disabilities exist from birth but may not be hereditary. These represent a portion of the number of children with chronic illnesses. Mental and nervous system disorders account for approximately 17% of chronic illnesses in children.

An infant is born with one lower limb deficiency. When is the optimum time for the child to be fitted with a functional prosthetic device? a. As soon as possible after birth b. When the infant is developmentally ready to stand up c. At about ages 12 to 15 months, when most children are walking d. At about 4 years, when the healthy limb is not growing so rapidly

ANS: B An infant should be fitted with a functional prosthetic leg when the infant is developmentally ready to pull to a standing position. When the infant begins limb exploration, a soft prosthesis can be used. The child should begin using the prosthesis as part of his or her normal development. This will match the infant's motor readiness.

The nurse is caring for a family whose infant was just born with anencephaly. What is the most important nursing intervention? a. Implement measures to facilitate the attachment process. b. Help the family cope with the birth of an infant with a fatal defect. c. Prepare the family for extensive surgical procedures that will be needed. d. Provide emotional support so the family can adjust to the birth of an infant with problems.

ANS: B Anencephaly is the most serious neural tube defect. The infants have an intact brainstem and, if born alive, may be able to maintain vital functions for a few hours to several weeks. The family requires emotional support and counseling to cope with the birth of an infant with a fatal defect. The parents should be encouraged to hold their infant and provide comfort measures. This facilitates the grieving process because the infant has a limited life expectancy. Infants with anencephaly do not have cerebral hemispheres. There is no surgical correction available for this defect. Emotional support is needed as the family adjusts to the birth of a child who has a fatal defect.

Care for the child with acute idiopathic thrombocytopenic purpura (ITP) includes which therapeutic intervention? a. Splenectomy b. Intravenous administration of anti-D antibody c. Use of nonsteroidal anti-inflammatory drugs (NSAIDs) d. Helping child participate in sports

ANS: B Anti-D antibody causes an increase in platelet count approximately 48 hours after administration. Splenectomy is reserved for chronic severe ITP not responsive to pharmacologic management. NSAIDs are not used in ITP. Both NSAIDs and aspirin interfere with platelet aggregation. The nurse works with the child and parents to choose quiet activities while the platelet count is below 100,000/mm3.

A toddler is admitted to the pediatric unit with presumptive bacterial meningitis. The initial orders include isolation, intravenous access, cultures, and antimicrobial agents. The nurse knows that antibiotic therapy will begin when? a. After the diagnosis is confirmed b. When the medication is received from the pharmacy c. After the child's fluid and electrolyte balance is stabilized d. As soon as the practitioner is notified of the culture results

ANS: B Antimicrobial therapy is begun as soon as a presumptive diagnosis is made. The choice of drug is based on the most likely infective agent. Drug choice may be adjusted when the culture results are obtained. Waiting for culture results to begin therapy increases the risk of neurologic damage. Although fluid and electrolyte balance is important, there is no indication that this child is unstable. Antibiotic therapy would be a priority intervention.

What clinical manifestations suggest hydrocephalus in an infant? a. Closed fontanel and high-pitched cry b. Bulging fontanel and dilated scalp veins c. Constant low-pitched cry and restlessness d. Depressed fontanel and decreased blood pressure

ANS: B Bulging fontanels, dilated scalp veins, and separated sutures are clinical manifestations of hydrocephalus in neonates. A closed fontanel, high-pitched cry, constant low-pitched cry, restlessness, a depressed fontanel, and decreased blood pressure are not clinical manifestations of hydrocephalus, but all should be referred for evaluation.

The nurse is preparing a staff education in-service session for a group of new graduate nurses who will be working in a long-term care facility for children; many of the children have cerebral palsy (CP). What statement should the nurse include in the training? a. Children with dyskinetic CP have a wide-based gait and repetitive movements. b. Children with spastic pyramidal CP have a positive Babinski sign and ankle clonus. c. Children with hemiplegia CP have mouth muscles and one lower limb affected. d. Children with ataxic CP have involvement of pharyngeal and oral muscles with dysarthria.

ANS: B CP has a variety of clinical classifications. Spastic pyramidal CP includes manifestations such as a positive Babinski sign and ankle clonus; ataxic CP has a wide-based gait and repetitive movements; hemiplegia CP is characterized by motor dysfunction on one side of the body with upper extremity more affected than lower limbs; and dyskinetic CP involves the pharyngeal and oral muscles, causing drooling and dysarthria.

A child is on phenytoin (Dilantin). What should the nurse encourage? a. Fluid restriction b. Good dental hygiene c. A decrease in vitamin D intake d. Taking the medication with milk

ANS: B Chronic treatment with phenytoin may cause gum hypertrophy. Children taking phenobarbital or phenytoin should receive adequate vitamin D and folic acid because deficiencies of both have been associated with these drugs. The medication should not be taken with milk, and fluids should be encouraged, not restricted.

Iron overload is a side effect of chronic transfusion therapy. What treatment assists in minimizing this complication? a. Magnetic therapy b. Infusion of deferoxamine c. Hemoglobin electrophoresis d. Washing red blood cells (RBCs) to reduce iron

ANS: B Deferoxamine infusions in combination with vitamin C allow the iron to remain in a more chelatable form. The iron can then be excreted more easily. Use of magnets does not remove additional iron from the body. Hemoglobin electrophoresis is used to confirm the diagnosis of hemoglobinopathies; it does not affect iron overload. Washed RBCs remove white blood cells and other proteins from the unit of blood; they do not affect the iron concentration.

A child has just returned from surgery for repair of a fractured femur. The child has a long-leg cast on. The toes on the leg with the cast are edematous, but they have color, sensitivity, and movement. What action should the nurse take? a. Call the health care provider to report the edema. b. Elevate the foot and leg on pillows. c. Apply a warm moist pack to the foot. d. Encourage movement of toes.

ANS: B During the first few hours after a cast is applied, the chief concern is that the extremity may continue to swell to the extent that the cast becomes a tourniquet, shutting off circulation and producing neurovascular complications (compartment syndrome). One measure to reduce the likelihood of this problem is to elevate the body part and thereby increase venous return. The health care provider does not need to be notified because edema is expected and warm moist packs will not decrease the edema. The child should move the toes, but that will not help reduce the edema.

Families progress through various stages of reactions when a child is diagnosed with a chronic illness or disability. After the shock phase, a period of adjustment usually follows. This is often characterized by what response? a. Denial b. Guilt and anger c. Social reintegration d. Acceptance of the child's limitations

ANS: B For most families, the adjustment phase is accompanied by several responses, including guilt, self-accusation, bitterness, and anger. The initial diagnosis of a chronic illness or disability often is met with intense emotion and characterized by shock and denial. Social reintegration and acceptance of the child's limitations are the culmination of the adjustment process.

A child with sickle cell disease is in a vasoocclusive crisis. What nonpharmacologic pain intervention should the nurse plan? a. Exercise as a distraction b. Heat to the affected area c. Elevation of the extremity d. Cold compresses to the affected area

ANS: B Frequently, heat to the affected area is soothing. Cold compresses are not applied to the area because doing so enhances vasoconstriction and occlusion. Bed rest is usually well tolerated during a crisis, although the actual rest obtained depends a great deal on pain alleviation and the use of organized schedules of nursing care. Although the objective of bed rest is to minimize oxygen consumption, some activity, particularly passive range of motion exercises, is beneficial to promote circulation. Usually the best course is to let children determine their activity tolerance. Elevating the extremity will not help in sickle cell disease.

What finding is characteristic of fractures in children? a. Fractures rarely occur at the growth plate site because it absorbs shock well. b. Rapidity of healing is inversely related to the child's age. c. Pliable bones of growing children are less porous than those of adults. d. The periosteum of a child's bone is thinner, is weaker, and has less osteogenic potential compared to that of an adult.

ANS: B Healing is more rapid in children. The younger the child, the more rapid the healing process. Nonunion of bone fragments is uncommon except in severe injuries. The epiphyseal plate is the weakest point of long bones and a frequent site of injury during trauma. Children's bones are more pliable and porous than those of adults. This allows them to bend, buckle, and break. The greater porosity increases the flexibility of the bone and dissipates and absorbs a significant amount of the force on impact. The adult periosteum is thinner, is weaker, and has less osteogenic potential than that of a child.

In anticipation of the admission of a child with hereditary spherocytosis (HS) who is experiencing an aplastic crisis, what action should the nurse plan? a. Secure an isolation room. b. Prepare for a transfusion of packed red blood cells. c. Anticipate preoperative preparation for a splenectomy. d. Gather equipment and medication for treatment of shock.

ANS: B In hereditary spherocytosis, aplastic crisis results in a sudden cessation of RBC production by the bone marrow. Hemoglobin and hematocrit values drop rapidly, which results in severe anemia. Transfusion support may be needed, and close monitoring of the child's cardiovascular status is necessary. The nurse should prepare for a transfusion of packed red blood cells initially. An isolation room is not needed, splenectomy would not be done at this time, and the child will not be in shock.

The nurse has been visiting an adolescent with recently acquired tetraplegia. The teen's mother tells the nurse, "I'm sick of providing all the care while my husband does whatever he wants to, whenever he wants to do it." What reaction should be the nurse's initial response? a. Refer the mother for counseling. b. Listen and reflect the mother's feelings. c. Ask the father in private why he does not help. d. Suggest ways the mother can get her husband to help.

ANS: B It is appropriate for the nurse to reflect with the mother about her feelings, exploring solutions such as an additional home health aide to help care for the child and provide respite for the mother. It is inappropriate for the nurse to agree with the mother that her husband is not helping enough. This judgment is beyond the role of the nurse and can undermine the family relationship. Counseling, if indicated, would be necessary for both parents. A support group for caregivers may be indicated. The nurse should not ask the father in private why he does not help or suggest way the mother can get her husband to help. These interventions are based on the mother's perceptions; the father may have a full-time job and other commitments. The parents may need an unbiased third person to help them through the negotiation of their new parenting responsibilities

A child, age 3 years, has cerebral palsy (CP) and is hospitalized for orthopedic surgery. His mother says he has difficulty swallowing and cannot hold a utensil to feed himself. He is slightly underweight for his height. What is the most appropriate nursing action related to feeding this child? a. Bottle or tube feed him a specialized formula until he gains sufficient weight. b. Stabilize his jaw with caregiver's hand (either from a front or side position) to facilitate swallowing. c. Place him in a well-supported, semireclining position. d. Place him in a sitting position with his neck hyperextended to make use of gravity flow.

ANS: B Jaw control is compromised in many children with CP. More normal control is achieved if the feeder stabilizes the oral mechanisms from the front or side of the face. Bottle or tube feeding will not improve feeding without jaw support. The semireclining position and hyperextended neck position increase the chances of aspiration.

The nurse has attended a professional development program about palliative care for the pediatric population. What statement by the nurse should indicate a correct understanding of the program? a. "Palliative care provides interventions that hasten death." b. "Palliative care promotes the optimal functioning and quality of life." c. "Palliative care does not provide pain and symptom management like hospice care." d. "Palliative care is not well received in hospitals that provide end-of-life care for children."

ANS: B Palliative care is designed to promote optimal functioning and quality of life during the time the child has remaining. Palliative care does not provide interventions that are intended to hasten death. The care does provide pain and symptom management and is well received in hospitals that provide end-of-life care for children.

A 5-year-old child will be starting kindergarten next month. She has cerebral palsy, and it has been determined that she needs to be in a special education classroom. Her parents are tearful when telling the nurse about this and state that they did not realize her disability was so severe. What is the best interpretation of this situation? a. This is a sign the parents are in denial. b. This is a normal anticipated time of parental stress. c. The parents need to learn more about cerebral palsy. d. The parents' expectations are too high.

ANS: B Parenting a child with a chronic illness can be stressful. At certain anticipated times, parental stress increases. One of these identified times is when the child begins school. Nurses can help parents recognize and plan interventions to work through these stressful periods. The parents are not in denial; rather, they are responding to the child's placement in school. The parents are not exhibiting signs of a remembering deficit; this is their first interaction with the school system with this child.

The parents of a child with spastic cerebral palsy (CP) state that their child seems to have significant pain. In addition to systemic pharmacologic management, the nurse includes which teaching? a. Patterning b. Positions to reduce spasticity c. Stretching exercises after meals d. Topical analgesics for muscle spasms

ANS: B Parents and children are taught positions to assume while sitting and recumbent that reduce spasticity. The American Academy of Pediatrics has stated that patterning should not be used for neurologically disabled children. Patterning attempts to alter abnormal tone and posture and elicit desired movements through positional manipulation or other means of modifying or augmenting sensory output. Stretching should be done after appropriate analgesic medication has been given and is effective. Topical analgesia is not effective for the muscle spasms of spastic CP.

The nurse is caring for a school-age child with severe anemia and activity intolerance. What diversional activity should the nurse plan for this child? a. Playing a musical instrument b. Playing board or card games c. Participating in a game of table tennis d. Participating in decorating the hospital room

ANS: B Plan diversional activities that promote rest but prevent boredom and withdrawal. Because short attention span, irritability, and restlessness are common in anemia and increase stress demands on the body, plan appropriate activities such as playing board or card games. Playing a musical instrument, participating in a game of table tennis, or decorating the hospital room would cause undue exertion

A feeling of guilt that the child "caused" the disability or illness is especially common in which age group? a. Toddler b. Preschooler c. School-age child d. Adolescent

ANS: B Preschoolers are most likely to be affected by feelings of guilt that they caused the illness or disability or are being punished for wrongdoings. Toddlers are focused on establishing their autonomy. The illness fosters dependency. School-age children have limited opportunities for achievement and may not be able to understand limitations. Adolescents face the task of incorporating their disabilities into their changing self-concept.

A fifteen-year-old child presents to the emergency department with a severe head trauma, including herniation of the brain into the spinal column. The child is non-responsive and intubated. How will the nurse most correctly explain to the parents what is happening? "While the hydrocephalus persists, your child will not be responsive to your voice or painful stimuli." "Eventually your child will be able to squeeze your hand and will gradually begin to respond to touch." "The injury is severe, but with surgery and extended medical care in, a complete recovery can be expected." "Because of the type of injury sustained, the brain will not be able to maintain respiratory effort without the ventilator."

"Because of the type of injury sustained, the brain will not be able to maintain respiratory effort without the ventilator." The brain stem is essential to respiratory and cardiac function. Once it is compressed into the spinal column it can no longer recover the ability to maintain those vital functions.

A child with cystic fibrosis is preparing for discharge after being hospitalized with pneumonia. The parent tells the nurse that this is their 4th admission in the last year and asks the nurse how to prevent infection. Which statements by the nurse are most appropriate? Select all that apply. "Your child can share personal items with siblings at home." "Clean and disinfect your child's nebulizer on a regular basis." "Avoid going to crowded places such as malls, markets, or parks." "Washing your hands before touching your child is very important." "Limit the number of vaccinations your child receives because they can cause infection."

"Clean and disinfect your child's nebulizer on a regular basis." A nebulizer can be a source of respiratory infection if not properly cleaned and disinfected. Correct "Avoid going to crowded places such as malls, markets, or parks." Respiratory tract infections are often transmitted in crowded or congested places where close contact with infected people is very likely. Correct "Washing your hands before touching your child is very important." Meticulous hand hygiene by everyone providing care to the child will decrease the chance of infection.

The mother of a child with hemophilia calls the provider's office and states that the child was struck with a baseball during a game. Which question, by the nurse, is most appropriate? "Can you see any obvious bleeding?" "Why was he playing baseball, anyway?" "Take him to the emergency department to be seen." "Where on his body did he get hit with the baseball?"

"Where on his body did he get hit with the baseball?" The nurse should determine the location of the injury to help determine the best course of action. For example, a head injury may require transport by emergency medical services, while a leg injury may only require transport by the parents.

compartment syndrome

- A compartment is a group of muscle surrounded by tough, inelastic fascial tissue - Compartment syndrome is when the pressure in this space increases so much that circulation to the muscles and nerves is compromised - Causes: Tight dressings or casts, skin traction, hemorrhage, trauma, burns, surgery, venous obstruction, infiltrated IV infusion, exudate, external pressure - Assessment of the 5 p's of ischemia: Pain Pallor Pulselessness Paresthesia Paralysis - Tx: Immediate relief of the pressure, which sometimes means fasciotomy > DO NOT ELEVATE the limb because this will decrease circulation even more - Ischemia can cause muscle damage after 12-24 hours > severe deformity after 48 hours > fibrosis and contractors in 5-10 days > paralysis if untreated

Torticollis aka wry neck

- Limited neck motion R/T contracture of the sternocleidomastoid muscle - Congenital ( due to or a traumatic lesion of the muscle - limited neck motion (palpate a nontender mass in the middle of it) symptoms - neck in flexed position and the head drawn or tilted laterally to the affected side, chin pointed toward the opposite side abnormal positioning in utero) - Tx: stretching or surgical release of the muscle by 12-18 months - Encourage the child to turn the head in the direction desired for correction through feeding and play if not treated in time may cause permanent limitation of neck movement nurse should teach parents to encourage the child to turn head in the direction desired for correction through feeding and play

Legg-Calve-perthes disease

- Self limiting - Avascular necrosis of the femoral head > deformity - Usually boys 2-12 years - Sx: hip soreness, ache, stiffness, limping - may be constant or intermittent), pain, limited range of motion at the hip diagnosis - radiographic findigs - Rest, limited weight bearing, NSAIDs, PT, ROM, traction, braces or casting if this doesnt help surgical proximal/or pelvic femoral osteotomy self limiting - Best results in kids less than 5 years because their epiphyses is more cartilaginous, latter diagnosis, more damage

therapeutic management of myelomeningocele

- focusing on problems - hydrocephalus, paralysis, orthopedic deformities, GU abnormalities - fucus on acquired problems - surgical closure 24-74 hrs initial care: - preventing infection - performing neurologic assessment, onserving for anomalies - dealing with the impact on family - preventing joint contractions - correcting th existing deformity - preventing or minimizing effects of motor and sensory dificit - preventing skin breakdown - obtaining best possible function of accected lower extremities - in infant - preserve renal function; in older kids - preserve renal function and achieve optimum urinary continence - clean intermittent catherization with antspasmotic meds (oxybutenin) to ensure routine, regular bladder evacuation and preventing deleterious elevation of intravesicle pressure - vesicostomy - bladder brought to abdominal wall - mitrofanoff procedure ( uses appendix as alternate route for intermittent catherization - artificial urinary sphincter bowel control - diet modification, regular toilet habits,preventon of constipation and impaction, dietary fibers, laxatives, suppositories, enemas - antegrade continence enema

characteristics of complex and chronic diseases

- frequent and prolonged hospitalizations - complex and multisystem health and developmental needs - reliance on technology - care that cross hospital, clinic, and home settings

pathophysiology bacterial meningitis

- vascular dissemination from a focus of infection elsewhere - direct implantation after penetrating wounds, skull fractures, lumbar pundture, surgical procesures, spina bifida, external ventricular device, internal ventricula shunt once implanted, the organisms spread into the CSF infective process - inflammation, exudation, white blood cell accumulation, varying degrees of tissue damage

doll's head manuever

-rotate child's head quickly to one side then the other -conjugate (paired or working together) movement of eyes in the direction opposite of head rotation is normal

The nurse explains to Madison and her parents that which will occur during and after the cast is removed? (select all that apply)

-she may feel heat or a vibration or a tickle during the removal -the machine to remove the cast is very noisy -skin might be scaly or dry after the cast is removed

therapeutic management anemia

-use only breast milk or iron fortified formula -iron spplementation of 1 mg/kg/day ( up to 4-6 mo) -iron drops 2-3 mg /kg/day maximum 15 mg -limit amount of formula to no more than 1 l/day to -encourage to take iron rich foods universal screening for anemia at 12 mo iron given in 2 divided doses metween meals with citrus juice. do not give with tea, legumes, maize

Madison weighs 33lbs, and the prescribed dose is 0.2mL/kg and the vial is labeled 5mg/mL. How many mL of medication should the nurse administer?

0,6

Boston brace

An underarm orthosis customized from prefabricated plastic shells, with corrective forces for each patient supplied by lateral pads; prevent progression of curves in the spine:

A child with a terminal illness has died following complications. The nurse is following-up with the child's parents. What can the nurse expect to observe in response to their child's death? Any emotions that the parents are feeling may be expressed. The grief response will be different for each parent, and that is normal. The grief response could last months but should resolve within a year. Symptoms that might indicate one or both parents need a referral to a mental health care provider. A support group must be attended so that the parents can learn from other parents who have been in similar circumstances.

Any emotions that the parents are feeling may be expressed. Expression of emotion is important to help the parents grieve and not hold their emotions in. Correct The grief response will be different for each parent, and that is normal. Everyone grieves in an individual way and that is OK. The parents need to know that they may react differently to the loss of the same child. Symptoms that might indicate one or both parents need a referral to a mental health care provider. There is a normal expected grief response, but depending on the individual, that normal response can lead to more serious emotional problems that may require a referral.

Which care is appropriate for an infant with a myelomeningocele? Application of a sterile, dry, nonadherent dressing over the defect Application of a clean, moist, nonadherent dressing over the defect Application of a sterile, moist, nonadherent dressing over the defect Application of a sterile, moist, nonadherent dressing around the defect

Application of a sterile, moist, nonadherent dressing over the defect Rationale Before surgical closure, the myelomeningocele (1)(2) is prevented from drying by the application of a sterile, moist, nonadherent dressing over the defect.

Which similarity is appropriate for both supratentorial features of an epidural hematoma and a subdural hematoma? Usually unilateral High mortality rate Frequent retinal hemorrhages Associated with increased intracranial pressure

Associated with increased intracranial pressure Rationale Both epidural hematoma and subdural hematoma are associated with increased intracranial pressure. Epidural hematoma is usually unilateral, but subdural hematoma is mostly bilateral. Epidural hematoma has a relatively high mortality rate, but subdural hematoma usually has a low mortality rate. Retinal hemorrhages are uncommon in epidural hematoma but very frequent in subdural hematoma.

The nurse knows that developmental dysplasia of the hip differs between the infant and neonate in which way? Patellar subluxation Complete hip immobility Presence of a femur fracture Asymmetry of gluteal skinfolds

Asymmetry of gluteal skinfolds Infants beyond the neonatal period exhibit asymmetry of the gluteal skinfolds when lying with the legs extended against the examining table (or when the infant is held upright with the legs dangling).

The mother of a 10-month-old reports that the child has been pale, tires quickly, and sometimes has difficulty breathing. Which orders would the nurse anticipate for this patient? Select all that apply. Give an oral iron supplement. Collect blood for complete blood count (CBC). Maintain NPO (nothing by mouth) status for this patient. Prepare the patient for insertion of in intravenous line. Obtain vital signs, including pulse oximetry, and notify the provider of any concerning results.

Collect blood for complete blood count (CBC). To help confirm a diagnosis of iron deficiency anemia, the nurse should expect an order for a CBC. Prepare the patient for insertion of in intravenous line. The patient will likely require insertion on an IV line for the administration of fluids to increase volume and perfusion. Correct Obtain vital signs, including pulse oximetry, and notify the provider of any concerning results. The nurse would expect to collect a complete set of vital signs, including pulse oximetry, to determine the presence of impaired perfusion secondary to iron deficiency anemia.

anencephaly?

Congenital malformation in which both cerebral hemispheres are absent. Incompatible with life. Many affected infants are still born. They have a portion of the brainstem to maintain vital functions for a few hours to a few weeks but eventually succumb to respiratory failure. They are basically missing the top half of their brain. They usually die within 24-48 hours.

During a physical assessment the nurse evaluates the hip of an infant and finds the head of the femur is outside of the acetabulum. How would this dysplasia be classified? Fracture of the hip Instability of the hip Dislocation of the hip Subluxation of the hip

Dislocation of the hip Dislocation of the hip occurs when the head of the femur lies outside the acetabulum. In DDH the acetabulum is shallow which does not all the femur to fit adequately in the joint. This can occur as a late stage of developmental dysplasia of the hip and it can occur in children with certain neuromuscular disorders.

A school-aged patient has a lateral spine curvature of 25 degrees. The nurse anticipates which intervention according to these results? Education on use of a brace Immediate surgical repair Education on core strengthening Monitor for a year for changes

Education on use of a brace Nonoperative management is the preferred treatment for patients with spine curvature ≥ 25 degrees and who are still skeletally immature. The nurse would anticipate providing patient teaching on use of a brace.

The nurse is caring for a patient with suspected meningitis. Which interventions should need to be performed in caring for this patient? Elevate head of bed to 30 degrees. Begin intravenous (IV) normal saline infusion. Administer broad-spectrum antibiotic as prescribed. Draw blood sample for a white blood cell (WBC) count. Discuss with the patient's family the importance of vaccination.

Elevate head of bed to 30 degrees. Although simple, this can be an important part of caring for a patient with suspected meningitis. It can help to control an increased intracranial pressure (ICP). Correct Begin intravenous (IV) normal saline infusion. This can be started after antibiotic infuses to prevent dehydration and ensure tissue perfusion. Correct Administer broad-spectrum antibiotic as prescribed. Early antibiotic therapy reduces morbidity and mortality. Correct Draw blood sample for a white blood cell (WBC) count. This is important information and therefore the blood sample can be obtained after antibiotic is begun

What kind of results do I get from the SF if it's bacterial?

Elevated WBC's/neutrophils, elevated protein, decreased glucose, positive gram stain, cloudy or turbid fluid, elevated pressure. We usually check for protein, glucose, WBC's and gram stain. What do I see with WBC's in bacterial? In bacterial- increased WBC's. GOT TO KNOW THE DIFFERENCE. Increased WBC's Elevated Protein, Cloudy Fluid, Positive gram stain and DECREASED glucose. In VIRAL we see slightly elevated WBC's protein and glucose are both normal and gram stain is negative. Know the differences between those two and know the differences between bacterial and viral meningitis. Viral meningitis = WBC's slightly elevated, increased lymphocytes, normal or slightly increased protein content, normal glucose, negative gram stain, clear to slightly cloudy fluid, normal pressure.

Chelation therapy is begun on a child with alpha-thalassemia major. What is the purpose of this therapy? Treat the disease. Eliminate excess iron. Decrease risk of hypoxia. Manage nausea and vomiting

Eliminate excess iron. Correct Iron overload (hemosiderosis) is a complication of blood transfusions. Chelation therapy is necessary to minimize the development of hemosiderosis and hemochromatosis. Blood transfusions are the primary medical management. Chelation therapy removes iron; it does not affect the disease process.

The nurse is caring for a pediatric patient with abnormal laboratory values for rheumatoid factor (RF), C-reactive protein (CRP), and erythrocyte sedimentation rate (ESR). Which assessment findings correspond with the abnormal laboratory results? Select all that apply. Enlarged knee joints Limping when walking Pain when moving joints Hip rotates without resistance Accelerated closure of the epiphyseal plates

Enlarged knee joints Abnormal RF, CRP, and ESR laboratory results can be indicative of arthritis in children, and enlarged joints are a symptom of arthritis. Correct Limping when walking Abnormal RF, CRP, and ESR laboratory results can be indicative of arthritis in children, and limping when walking is a symptom of arthritis. Correct Pain when moving joints Abnormal RF, CRP, and ESR laboratory results can be indicative of arthritis in children, and pain when moving joints is a symptom of arthritis.

Which posturing finding is appropriate with severe dysfunction of the cerebral cortex or lesions to cortiocospinal tracts above the brainstem? Primitive Extension Flexion Unilateral decerebrate

Flexion Rationale Flexion or decorticate posturing is seen with severe dysfunction of the cerebral cortex or with lesions to cortiocospinal tracts above the brainstem. Primitive posturing is the term used to describe how brain dysfunction results in the loss of primitive postural reflexes. Extension posturing is a sign of dysfunction at the level of the midbrain or lesions to the brainstem. Unilateral decerebrate posturing is caused by tentorial herniation.

Which assessment findings indicate appropriate development of the musculoskeletal system of a 5-year-old patient? Select all that apply. Fused cranial sutures Smaller muscular size Normal curvature of the neck Incomplete lumbar curvature of the spine Decreased range of motion in the knee joint

Fused cranial sutures Fused cranial sutures are normal by the time the child is 12-18 months old and is an assessment finding that indicates appropriate development of the musculoskeletal system in a 5-year-old pediatric patient. Smaller muscular size Muscles grow in size as the child ages; smaller muscle size is an assessment finding that indicates appropriate development of the musculoskeletal system in a 5-year-old pediatric patient. Normal curvature of the neck Cervical curvature develops in the first few months of life; normal cervical curvature is an assessment finding that indicates appropriate development of the musculoskeletal system in a 5-year-old pediatric patient. Incomplete lumbar curvature of the spine Lumbar curvature may not be fully developed until age 8-10 years; incomplete lumbar curvature is an assessment finding that indicates appropriate development of the musculoskeletal system in a 5-year-old pediatric patient.

A twelve-year-old child has begun experiencing difficulty in playing at recess and states, "I just can't run as fast or throw and catch the ball like I could before, so I just don't play now." On what should the nurse focus the neurologic assessment? Primitive reflex presence Gross and fine motor skills Cognitive and learning skills Language and communication skills

Gross and fine motor skills Movement and coordination must be assessed. The nurse should suspect that the child's gross and fine motor skills are underdeveloped or regressing due to lack of myelin. A decrease in myelinization slows the action potentials of the central nervous system (CNS) and therefore prevents coordination of the movement of motor units. In adults, myelin naturally regresses in the elderly. In children, myelin should increase along with motor skills.

Match each term with its definition.

Hemostasis Prevention of blood loss Fibrinogen Protein necessary for clotting Thrombocyte Small, irregular shaped, non-nucleated fragments Megakaryocyte Precursor to platelets Vasoconstriction Blood vessels decrease in diameter

Which conditions can lead to anemia? Select all that apply. Mild concussion Liver laceration Iron-deficient diet Closed humerus fracture Heavy menstrual bleeding

Liver laceration A liver laceration can lead to a reduced volume of packed RBCs, and thus anemia. Iron-deficient diet An iron-deficient diet can cause a reduction in circulating hemoglobin, and thus anemia. Heavy menstrual bleeding Heavy menstrual bleeding can lead to a reduced volume of packed RBCs, and thus anemia.

Which finding is appropriate for a 2-month-old infant after a car accident whose Moro, tonic neck, and withdrawal reflexes are present? Neurologic health Decorticate posturing Severe brain damage Decerebrate posturing

Neurologic health Rationale The Moro, tonic neck, and withdrawal reflexes are usually present in infants under 3 to 4 months of age. Therefore the presence of these reflexes indicates neurologic health. The presence of the Moro, tonic neck, and withdrawal reflexes does not indicate severe brain damage. Decorticate posturing is indicative of severe dysfunction of the cerebral cortex and is not related to the presence of the Moro, tonic neck, or withdrawal reflexes. Decerebrate posturing is indicative of dysfunction at the level of the midbrain and is not related to the presence of the Moro, tonic neck, or withdrawal reflexes.

Which nursing action is appropriate for the toddler who has undergone surgery for a brain tumor and is becoming irritable and pupils are unequal and sluggish? Notifying the health care practitioner immediately Documenting the findings and obtain the child's vital signs Administering pain medication and assessing the response Continuing to observe the child for signs of increased intracranial pressure

Notifying the health care practitioner immediately Rationale The worsening of symptoms may indicate that the intracranial pressure (ICP) is increasing. The health care practitioner would be notified immediately because this is considered a medical emergency. The nurse has sufficient data to contact the health care provider (HCP). Determination of the level of consciousness should be done as part of the assessment. The nurse has noted possible signs of increased ICP; therefore, action would now be taken and the HCP should be notified. Pain medication should not be given because it can often mask the signs of increasing ICP.

Which level of consciousness is appropriate for the young child who is arousable with stimulation after a head injury? Stupor Lethargy Confusion Obtundation

Obtundation Rationale Obtundation is a level of consciousness described as arousable with stimulation. Stupor is marked by continued deep sleep, slow response to vigorous and repeated stimulation, and moaning responses to stimuli. Lethargy is marked by limited spontaneous movement, sluggish speech, drowsiness, and falling asleep quickly. Confusion is indicated by impaired decision-making.

AVPU stands for/ consciousness

Quickly assessing level of alertness A-alert able to maintain airway V-responds to verbal stim ? If air adjunct is needed to keep tongue from obstructing airway. P-pain ? May not b able to maintain airway. May need airway adjunct while further assessment is made to determine the need for intubation. U-unresponsive-announce for pulse to b check while assessing cause might b problem w airway. Reprioritize the ABCs if needed

The nurse stops to assist a child who has been hit by a car while riding a bicycle. Someone has activated the emergency medical system. Until paramedics arrive, the nurse would consider what in caring for this child who has experienced severe trauma? Rapid assessment should begin with ABC status: airway, breathing, and circulation. Assessment should begin with the area injured; assessment of other areas can wait. The possibility of spinal cord injury should be ruled out before transporting the child to the hospital. Temperature maintenance is more difficult than in adults because young children have a larger surface area related to body mass.

Rapid assessment should begin with ABC status: airway, breathing, and circulation. Correct The first priority is always airway, breathing, and circulation. Assessment of the injured area occurs after the child's cardiopulmonary status has been addressed. Transport can occur by immobilizing the cervical spine. The head is maintained in a neutral position, and movement of the head or body is not allowed in any direction. Infants have the greatest discrepancy in body surface areas. Children old enough to ride bikes have similar body proportions to adults.

Characteristics of DMD

Reach developmental milestones early First signs: issues with riding a bike, running, & climbing stairs Onset between 3 and 7 years old Progressive muscular weakness, contractures, & muscle wasting Calf muscle pseudohypertrophy Loss of independent ambulation by 9 - 12 years of age Slowly progressive, generalized weakness during teenage years boys will have waddling gait and lordosis (weakened pelvic muscles), fall frequently, rising from a sqatting or sitting position muscles in calves, thighs, upper arms enlarged and feel firm, woody on palpation

A child is brought to the emergency department following a motor vehicle accident and a Glasgow coma score (GCS) is obtained with the following results: Opens eyes spontaneously and obeys motor commands but seems confused. Which should be an appropriate step in nursing management of this patient? Administer intravenous (IV) fluids. Ask the parents to keep the child awake. Manage cerebrospinal fluid (CSF) pressure. Reassess neurologic status within thirty minutes.

Reassess neurologic status within thirty minutes. The nurse should continue to monitor the patient since the child is showing signs of a mild head injury.

overuse injury

Repetitive microtrauma that occur to particular atomic structure

The parents of a child with fragile X syndrome tell the nurse that they have researched the disease online and are concerned about their child's future. They ask the nurse how they will know what services their child will need and where to obtain the services. Which action is appropriate for the nurse to take? Ask the case manager to talk with the parents. Schedule a multidisciplinary team meeting with the parents. Ask the provider to review the plan of care with the parents. Review the support groups available in the community with the parents.

Schedule a multidisciplinary team meeting with the parents. The multidisciplinary team will provide the parents with all of the information related to the services needed for the child. All aspects of care will be addressed by the team.

subdural area

a potential space that normally contains only enough fluid to prevent adhesion between the arachnoid and the dura mater

Nursing considerations for the infant with SMA type 1 should include which of the following to promote developmental care? a. Feeding by nasogastric tube b. Using an infant walker to devleop muscle strength c. Incorporating verbal, tactile, and auditory stimulation d. Encouraging the parents to seek genetic couseling

c. Incorporating verbal, tactile, and auditory stimulation

common complications of duchenne muscular dystrophy

contractures and deformities of arge and small joints scoliosis - disuse atrophy - to minimize complication, daily goal 3 hrs ambulation pulmonary infection - antibiotic therapy, aggressive airway clearance ( due to muscular weakness kids cannot cough) , infections, obesity, cardiopulmonary problems progresses to diaphragm and axiliary muscles of respiration IQ 20 points below norm eventual cause of death - respirtory tract infection or cardiac failure obesity leads to premature loss of ambulation

medical interventions for fracture injury

control of pain, hemorrhage and edema 2. releif of muscle spasms 3. realignment of fracture fragments 4. promotion of bone healing 5. immobilization of fracture until adequate healing has began 6. prevention of secondary complications 7. limitation of disuse syndrome 8. restoration of function

Bone healing is characteristically more rapid in children than in adults because: a. Children have less constant muscle contraction associated with the fracture. b. Children's fractures are less severe than adults. c. Children have an active growth plate that helps speed repair with less likelihood of deformity. d. Children have thickened periosteum and a more generous blood supply

d. Children have thickened periosteum and a more generous blood supply.

Associated disabilities and problems related to the child with CP include which of the following? a. All children with CP have intelligence testing in the abnormal range b. A large number of eye cataracts are associated with CP and require surgical correction c. Seizures are a common occurrence among children with athetosis and diplegia d. Coughing and choking, especially while eating, predispose children with CP to aspiration

d. Coughing and choking, especially while eating, predispose children with CP to aspiration

Characteristic features of Duchenne muscular dystrophy include all of the following except: a. waddling gait b. lordosis c. calf muscle hypertrophy d. neurogenic bladde

d. neurogenic bladder

The nurse caring for the child with spinal cord injury and neurogentic bladder knows: a. to keep the patient's urine alkaline b. that administration of dicyclomine (Bentyl) will relax the bladder musculature but will not promote increased bladder capacity or more adequate emptying. c. that the bladder that empties periodically by reflex action will not need intermittent catheterization d. that pyelonephritis and renal failure are the most significant causes of death in longstanding paraplegic patients

d. that pyelonephritis and renal failure are the most significant causes of death in longstanding paraplegic patients

Place the steps of clot formation in the order in which they occur.

damaged tissue and platelets release factors that stimulate a series of reactions involving the activation of clotting factors prothrombin converted to thrombin fibrinogen converted to fibrin fibrin mesh forms, blood stops flowing clot retracts, site is sealed When tissue is damaged, platelets and factors are released that stimulate prothrombin to convert to thrombin and fibrinogen to convert to fibrin. Fibrin mesh forms over the area of damage, causing blood to stop flowing—this then retracts and seals the site.

clinical manifestations of bacterial meningitis/ infants and young children

fever poor feeding vomiting marked irritability frequent seizures ( often accompanied by a high-pitched cry) bulging fontanel nuchial rigidity possible brudzinski and kernig signs not helpful subdural empyema

adjustment

follows shock open admission that the condition exists accompanied by responses such as guilt and self-accusation

physical therapy for CP

for good skeletal alignment, training in purposeful acts, gait training stretching, passive active and resistance movement applied to specific muscle groups; strength, edurance

cognitive impairment

general term that encompasses any type of mental difficulty or deficiency

clinical manifestations of increased intracranial pressure in children

headache nausea forceful vomiting diplopia blurred vision seizures indifference drowsinezz decline in school performance diminished physical activity and motor performance increased sleeping inability to follow simple commnads lethargy

vital sign changes in ICP

hypo/hyperthermia pulse - rapid, slow, bounding, feeble cushing reflex- slow pulse with increase in BP (late sign) respirations-slow deep irregular or hyperventilation odor of breath - fruity, acetone, uremia, fetid, alcohol

DDH

idiopathic teratologic ( involves neuromuscular defect)

circulatory impairment

if the trauma or immobilization device restricts blood flow check always for capillary refill, pulses, skin color, temp inadequate blood supply - low Hmct

concept of functional burden

impact of the child with special needs familyt resources and ability to cope level of adjustment is influenced by it issues related to caring for and living with the child in relation to the family's resurces and ability to cope

Clinical manifestations of CP

include delayed gross motor development, altered motor performance, alterations of muscle tone and subsequent muscle contractures, abnormal posture, and associated disabilities such as seizures and sensory impairment

therapeutic management of scoliosis

observation with regular clinical and radiographic evaluation orthotic intervention (braces) surgical spinal fusion depends on - magnitude, location, type of curve, af=ge and skeletal maturity, underlying disease

If madison developed compartment syndrome In addition to notifying the healthcare provider of this development, what action should the nurse implement?

obtain equipment needed for cast removal

Dislocations

occurs whne the force of stress on the ligament is great enough to disrupt the normal position of the opposing bone ends or the bone end and its socket symptom - pain that increases with attempted passive or active movemnt of the extremity - Shoulder dislocation: immobilize with a sling or bandage that holds the arm to the chest until medical help - Hip dislocation: Common in children under 5 R/T falls and Down syndrome kids - Best chance for prevention of damage to head of femur R/T lack of blood supply is relocation within 60 minutes - Patella dislocations can happen spontaneously or R/T injury - They dislocate laterally - Therapy is immobilization for 3-4 w or surgery if it keeps happening - Most common dislocation: subluxation or partial dislocation of the radial head in the elbow aka pulled elbow or nursemaid's elbow - 1-3 years old - From jerking the arm

Nursing care of an unconscious child focuses

on ensuring respiratory management; performing neurologic assessment; monitoring ICP; supplying adequate nutrition and hydration; administering drug therapy as indicated; regulating temperature; promoting elimination, hygienic care, proper positioning, exercise, and stimulation; and providing family support.

injuries required immediate medical attention

open fractures compartment syndrome fractures associated with vascular or nerve injuries joint dislocations that cannot be reduced

The nurse is caring for a child who has decreased muscle strength and muscle tone. Which is the most appropriate nursing intervention to ensure sufficient venous return? Immobilize the child. Limit the hours of sleep. Use compression stockings. Ensure sufficient fluid intake.

use compression stockings. Rationale In the presence of decreased muscle strength and tone, the venous return to the heart is also reduced. This stagnation of the venous blood can lead to the formation of thromboembolism, especially in the lower limbs. The best intervention in this condition is the use of compression, or antiembolism, stockings. The nurse encourages the child to move to improve the venous return. Immobilizing the child aggravates the condition and increases the risk of thrombus formation. The hours of sleeping are not related to thrombus formation as long as there is sufficient movement of the limb during the daytime. Adequate fluid intake is essential for the maintenance of normal health but is not directly related to thrombus formation.

basal ganglia

the middle meningeal layer; a delicate, avascular, weblike structure that loosely surrounds the brain

A parent of a breastfed infant with iron deficiency anemia (IDA) questions the need for iron supplementation. Which response by the nurse is most appropriate? "Children's diets lack iron, so they have to take pills instead." "Your baby's bone marrow function is not fully developed yet." "Because the infant is growing so rapidly, his body needs extra iron." "Other vitamin supplements do not contain iron, so you need special pills instead."

"Because the infant is growing so rapidly, his body needs extra iron." Because children must produce additional red blood cells (RBCs) to accommodate for their physical growth, their need for iron, which is used in synthesis of new hemoglobin for RBC production, is increased.

The parents of a newborn diagnosed with fragile X syndrome ask the nurse about the cause of the syndrome. Which response by the nurse is appropriate? "Fragile X syndrome is caused by a missing part of the X chromosome." "Fragile X syndrome is caused by one defective gene on the X chromosome." "Fragile X syndrome is caused by an additional entire set of chromosomes in each cell." "Fragile X syndrome is caused by an additional chromosome on the X chromosome".

"Fragile X syndrome is caused by one defective gene on the X chromosome." Fragile X syndrome is caused by an underlying single gene defect on the X chromosome.

A patient with iron deficiency anemia (IDA) is preparing for discharge from the emergency department. During discharge teaching, which statement made by the parent indicates teaching has been effective? "Iron helps my child's red blood cells (RBCs) maintain a normal shape." "Iron helps my child's body make healthy RBCs with hemoglobin." "It's important that I include iron in my child's diet to prevent excessive bleeding." "Even though my child's iron is low now, it will increase with age, and the RBCs will return to normal."

"Iron helps my child's body make healthy RBCs with hemoglobin." IDA is a problem that stems from lack of iron that causes the bone marrow to produce RBCs without hemoglobin, the oxygen-carrying component of RBCs. With appropriate levels of iron, the bone marrow will produce healthy RBCs with hemoglobin.

The parents of a child with cystic fibrosis tell the nurse they are considering having more children and want to know the likelihood of having another child with cystic fibrosis. Which response by the nurse is appropriate? "Since only one parent needs to be a carrier, your child has a 1 in 4 chance of having cystic fibrosis." "Since cystic fibrosis is an autosomal dominant trait, every child will be affected with cystic fibrosis." "Since cystic fibrosis is an autosomal dominant trait, your child has a 1 in 4 chance of having cystic fibrosis." "Since cystic fibrosis is an autosomal recessive trait, your child has a 1 in 4 chance of having cystic fibrosis."

"Since cystic fibrosis is an autosomal recessive trait, your child has a 1 in 4 chance of having cystic fibrosis." Cystic fibrosis is an autosomal recessive trait that requires both parents to be carriers for the disease to be present. There is a 1 in 4 chance of the disease occurring in each pregnancy.

Parents of a child with type 1 diabetes tell the nurse, "Our child will never have the life we dreamed for him." Which response by the nurse is appropriate? "Your child will be able to live whatever life they choose." "You must remain positive or your child will become discouraged." "You are grieving the loss of your 'perfect' child. Tell me more about how you feel." "There have been great advances in treating type 1 diabetes. Let me share them with you."

"There have been great advances in treating type 1 diabetes. Let me share them with you." Education decreases fears and misconceptions and encourages appropriate interactions between family members, the child, and health care providers. This is an appropriate response by the nurse.

The nurse receives a report on multiple patients newly admitted to the pediatric unit. Which patient should the nurse assess first? 5-year-old with mild muscle weakness 6-month-old with a fractured left femur 3-year-old with dislocated right shoulder 14-month-old with palpable anterior suture line

6-month-old with a fractured left femur Fractures in children younger than 1 year are unusual and may indicate child abuse; the nurse should assess this child first to evaluate the family situation.

The nurse is caring for a child with an epidural hematoma. The nurse should assess for what signs that can indicate Cushing triad? (Select all that apply.) a. Fever b. Flushing c. Bradycardia d. Systemic hypertension e. Respiratory depression

ANS: C, D, E Cushing triad (systemic hypertension, bradycardia, and respiratory depression) is a late sign of impending brainstem herniation. Fever or flushing does not occur with Cushing triad.

The nurse is instructing the parents of a child with iron deficiency anemia regarding the administration of a liquid oral iron supplement. Which instruction should the nurse tell the parents? 1. administer the iron at mealtimes 2. administer the iron through a straw 3. mix the iron with cereal to administer 4. add the iron to formula for easy administration

Administer the iron through a straw; In iron deficiency anemia, iron stores are depleted, resulting in a decreased supply of iron for the manufacture of hemoglobin in red blood cells. An oral iron supplement should be administered through a straw or medicine dropper placed at the back of the mouth because the iron stains the teeth. The parents should be instructed to brush or wipe the child's teeth or have the child brush the teeth after administration. Iron is administered between meals because absorption is decreased if there is food in the stomach. Iron requires an acid environment to facilitate its absorption in the duodenum.

Which nursing information is appropriate for the parent of a 9-year-old girl with cerebral palsy (CP) wanting to promote socialization at school? After-school activities are not recommended for children with CP. Participation in after-school activities leads to lowered self-esteem in children with CP. Activities such as karate, basketball, and cheerleading are appropriate for children with CP. After-school activities often provide children with CP opportunities for socialization and recreation.

After-school activities often provide children with CP opportunities for socialization and recreation. Rationale After-school recreational outlets should be considered for the child who is unable to participate in athletic programs or other physically demanding activities. Activities such as karate, basketball, and cheerleading are not always appropriate for children with cerebral palsy (CP). Participation in after-school activities is not associated with lowered self-esteem in children with CP.

skin traction

Applied directly to the skin surface and indirectly to the skeletal structures. The pulling mechanism is attached to the skin with adhesive material or an elastic bandage. Both types are applied over soft, foam-backed traction straps to distribute the traction pull.

A child with sickle cell anemia (SCA) presents with pain and swelling in the right knee and complaints of abdominal pain. The nurse notes abdominal distention. Which action should the nurse take next? Palpate the abdomen. Apply ice to the affected area. Auscultate the patient's lung sounds. Assess vital signs and notify the health care provider.

Assess vital signs and notify the health care provider. The patient's symptoms indicate sickle cell crisis. The nurse would assess vital signs for a patient with sickle cell crisis and notify the health care provider to initiate treatment.

What feature identifies Madison's fracture as an open fracture?

Bone fragments protruding through the skin.

What type of fracture in children results when the porous bone is compressed? Buckle fracture Complete fracture Greenstick fracture Plastic deformation

Buckle fracture Rationale A buckle fracture results when porous bone is compressed. In a complete fracture, the bone fragments are divided. In a greenstick fracture, the bone is angulated beyond the limits of bending. Plastic deformation occurs when the bone is bent but not broken.

The nurse observes that the ten-year-old patient is becoming increasingly restless. Knowing that the child suffered a concussion playing football, what does the nurse do next? Calculate Glasgow coma score. Perform bilateral pupil examination. Inspect child's skull for size and shape. Ask patient about nausea and headache. Check vital signs and oxygen saturation.

Calculate Glasgow coma score. The child's GCS will offer information about the increased restlessness and whether it is neurologic in nature. Correct Perform bilateral pupil examination. The nurse assesses reasons for restlessness including signs of increased ICP: diplopia, papilledema, and poor pupillary response to light. Ask patient about nausea and headache. Nausea and headaches are common, early signs of increased intracranial pressure. If patient does have these, it helps direct the nurse's assessment. Correct Check vital signs and oxygen saturation. Hypoxia can be a cause of restlessness in any population. Anytime there is a new finding in a patient, vital signs should be reevaluated as indicators of overall cardiorespiratory status.

A patient with sickle cell anemia presents with pallor, lethargy, headache, and a history of fainting spells. Which provider order would the nurse anticipate? Assess pain level every shift. Assess exertional heart rate on a treadmill. Collect blood for type and cross-matching. Administer 3% saline solution intravenously at a rate of 25 mL/hr.

Collect blood for type and cross-matching. The patient is displaying signs of a vaso-occlusive crisis, and blood transfusion is an expected treatment. Before administration, the patient's blood type must be determined.

What is the goal for treatment of these kiddos?juvenile idiopathic arthritis?

Control pain, preserve ROM, minimize inflammation. Give them normal a life as possible. We want to keep those joints working as much as possible.

An appropriate nursing intervention for the care of a child with an extremity in a new cast is to: A. Keep the cast covered with a sheet B. Use the fingertips when handling the cast to prevent pressure areas C. Use heated fans or dryers to circulate air and speed the cast-drying process D. Turn the child at least every 2 hours to help dry the cast evenly

D. Turn the child at least every 2 hours to help dry the cast evenly

club feet

Deformity of the ankle and food. Bone deformity and soft tissue contracture. Serial casting. What do we call this serial casting? Ponseti method. Inward rotation of the feet. They get serial casting. This is called Ponseti Method. How long do we keep them in a cast? 5-8 weeks. I THINK ITS ABOUT 6 -10 (book says 5-8. She says 6-10) Every one to two weeks as the baby grows, they get a new cast. How about after the cast comes off? Anything special with their foot gear? They actually have special shoes. The bar goes inside the shoe to keep the foot aligned as they try to get the foot to go straight. Denis Browne bar with Ponseti sandals placed in abduction are fitted to maintain the correction and prevent recurrence. The foot abduction brace is used at nighttime for 3-5 years.

A child with a chronic health condition is having trouble coping with the hospital setting and becomes upset when informed of a room change. Which action by the nurse is most appropriate? Do not change the room for several hours and introduce the change to the child slowly Encourage the child to do some deep breathing to cope with the stress of having to change rooms Discuss comfort measures with the child's parents that might help the child cope with the distress Change the room while the child is off the floor so that the child does not have to deal with the stress of the room change

Discuss comfort measures with the child's parents that might help the child cope with the distress It is important for the nurse to collaborate with parents to identify the child's preferences and actions that may help to pacify the child during times of stress

Why does contracture deformity of the hips, knees, and ankles in a child with Duchenne muscular dystrophy occur? Fatty infiltration Weakened pelvic muscles Early selective muscle involvement Weakened gluteus medius and maximus muscles

Early selective muscle involvement Rationale Contracture deformities of the hips, knees, and ankles occur from early selective muscle involvement. Muscles in the calves, thighs, and upper arms become enlarged from fatty infiltration. Lordosis associated with Duchenne muscular dystrophy occurs as a result of weakened pelvic muscles. A waddling gait is a result of weakness in the gluteus medius and maximus muscles.

The nurse receives the following results of a first trimester prenatal screening: Triple maternal serum screen value—low alpha fetoprotein, low unconjugated estriol, and increased human gonadotropin levels. Which action is most important for the nurse to take? Teach patient about Down syndrome. Prepare patient for a level II ultrasound. Refer patient and her partner for genetic counseling. Educate the patient about amniocentesis or chorionic villi sampling (CVS).

Educate the patient about amniocentesis or chorionic villi sampling (CVS). Abnormal triple maternal serum screen prompts additional testing to be done. Prenatal testing includes amniocentesis or chorionic villus sampling.

A 5-year-old child is having trouble coping in the hospital and behaviors have become increasingly problematic to the nurse and staff. What actions by the nurse should help to alleviate this problem? Encourage the child to express feelings through drawings Give the child different ways to communicate needs such as a picture board Allow the child to help with some aspects of personal care and with keeping the room tidy Remind parents the child's behavior is a normal part of dealing with an illness in a school-aged child Keep the child in the hospital room as a time-out until the child is able to stop having disruptive behaviors

Encourage the child to express feelings through drawings Drawing provides a different outlet for frustration that can decrease problematic behaviors, and therefore this should be encouraged by the nurse. Correct Give the child different ways to communicate needs such as a picture board The nurse should be providing different ways to communicate needs that can decrease frustration. Correct Allow the child to help with some aspects of personal care and with keeping the room tidy Giving responsibilities and some chores is helpful to school-age children to maintain control and normalcy.

The nurse would make a referral for communication impairment in what situations? Dysfluency occurring at any age First words not uttered before age 2 years Speech largely unintelligible at 18 months Substitution of easily produced sounds for more difficult ones by age 3 years

First words not uttered before age 2 years Correct Usually, children are able to assign meaning to words by the first year of life. A delay until the second birthday is an indication for referral. A referral should be made if stuttering or other dysfluency persists past age 5 years. Referral is indicated if the child is unintelligible at age 3 years. The competency to substitute sounds is not expected until age 5 years.

Open, or compound fracture

Fracture with an open wound from which the bone has protruded:

Which aspect of care is appropriate for the child with newly diagnosed Duchenne (pseudohypertrophic) muscular dystrophy? Genetic counseling Therapeutic treatment Instruction on muscle immobilization Help in finding a nursing facility for long-term care

Genetic counseling Rationale Duchenne (pseudohypertrophic) muscular dystrophy is inherited as an X-linked recessive gene. Genetic counseling is recommended for parents, female siblings, maternal aunts, and their female offspring. No effective treatment exists at this time for childhood muscular dystrophy. Maintenance of optimal function of all muscles for as long as possible is the primary goal. It has been found that children who remain as active as possible are able to avoid wheelchair confinement for a longer time. Finding a nursing facility is inappropriate at the time of diagnosis. When the child becomes increasingly incapacitated, the family may consider home-based care, a skilled nursing facility, or respite care to provide necessary care.

The nurse is explaining blood components to an 8-year-old child. What is the nurse's best description and action of platelets? Make up the liquid portion of blood. Help keep germs from causing infection. Carry the oxygen you breathe from your lungs to all parts of your body. Help your body stop bleeding by forming a clot (scab) over the hurt area.

Help your body stop bleeding by forming a clot (scab) over the hurt area. Correct Platelets are involved in hemostasis. Plasma makes up the liquid portion of blood. White blood cells help keep germs from causing infection. Red blood cells carry the oxygen you breathe from your lungs to all parts of your body.

Transfusion Reactions

Hemolytic—the most severe, but rare Febrile reactions—fever, chills Allergic reactions—urticaria, pruritus, laryngeal edema Air emboli—may occur when blood is transfused under pressure Hypothermia Electrolyte disturbances—hyperkalemia from massive transfusions or with renal problems circulatory overload - too rapid transfusion or excessive quantity transmission of infection alloimmunization - antibody formation

The nurse is caring for two patients, one with a fracture that has not broken the skin and one with a fracture that has pierced the skin. The nurse will need to monitor for which complication that can occur with both fractures? A. Infection B. Hemorrhaging C. Impaired skin integrity D. Compartment syndrome

Hemorrhaging Hemorrhaging is a complication for both simple (fracture that has not broken the skin) and compound (fracture that has broken the skin) fractures. The patient with the simple fracture is at risk for internal hemorrhaging, while the patient with compound fracture is at risk for external hemorrhaging.

The nurse is performing a musculoskeletal examination of a pediatric patient. Which findings indicate normal function? Select all that apply. Unequal muscle size Involuntary muscle movement Hip rotation without resistance Increased movement in the sutural joints Unrestricted range of motion in the ball and socket joints

Hip rotation without resistance Joints, such as the hip, enable movement and indicate normal function of the musculoskeletal system. Unrestricted range of motion in the ball and socket joints Ball and socket joints should have near 360 degrees of movement; this would be a normal finding.

Hydrocephalus

Hydrocephalus is a group of conditions resulting from disturbances in the dynamics of cerebral circulation and cerebrospinal fluid (CSF) caused by either (1) impaired absorption of CSF fluid (nonobstructive or communicating hydrocephalus) or (2) obstruction to the flow of CSF through the ventricular system (obstructive or noncommunicating hydrocephalus). Surgical treatment is the therapy of choice in almost all cases of hydrocephalus; however, most children require a shunt to promote CSF drainage.

immobilization

Immobilization of the affected extremity with a synthetic cast is common treatment for a simple, nondisplaced fracture. Surgical pinning and either open or closed reduction may be used for complex fractures.

Neurologic physical examination

Includes observation of the size and shape of the head, spontaneous activity, postural reflex activity, sensory responses, symmetry of movement

The nurse develops a discharge plan for a teen who is overweight with slipped capital femoral epiphysis (SCFE). Which instructions should the nurse include in the plan? Select all that apply. A. Instruct the patient on the proper use of crutches. B. Teach the child how to perform isometric exercises. C. Instruct that patient to wear a Pavlik harness to protect the hip. D. Provide instructions regarding the use of an abduction orthosis. E. Provide information to help the patient develop good nutritional habits to reduce weight.

Instruct the patient on the proper use of crutches. The patient will require the use of crutches to limit weight bearing down on the affected hip. Crutches are typically used for 4-6 weeks. Correct Teach the child how to perform isometric exercises. The isometric exercises will improve strength in muscles that support the hip joint. Provide information to help the patient develop good nutritional habits to reduce weight. Weight reduction will help reduce the risk of SCFE. The nurse should help the patient identify high-calorie foods that should be avoided.

Which complication is appropriate as a result of a disruption in the descending sympathetic pathways with loss of vasomotor tone and sympathetic innervation of the cardiovascular system? Neurogenic shock Autonomic dysreflexia Spinal shock syndrome Spinal cord compression

Neurogenic shock Rationale Neurogenic shock may occur after a spinal cord injury as a result of disruption of the descending sympathetic pathways with loss of vasomotor tone and sympathetic innervation of the cardiovascular system. Autonomic dysreflexia or hyperreflexia replaces the paralytic nature of autonomic function when the lesion lies above the midthoracic level. Spinal shock syndrome is caused by a sudden disruption of central and autonomic pathways. Spinal cord compression is a form of cord trauma that results in a temporary neural dysfunction without visible damage to the cord.

which action would best help facilitate bond and ease Blancas hesitation to hold her daughter

Nurse should comment on maria's beautiful hair while holding maria

idiopathic scoliosis

Observation for idiopathic scoliosis is an important part of a routine physical assessment in preadolescents and adolescents. Idiopathic scoliosis is a complex spinal deformity in three planes, usually involving lateral curvature, spinal rotation causing rib asymmetry, and thoracic hypokyphosis. In most cases of adolescent scoliosis, there is no apparent cause. Scoliosis is managed by bracing or surgical correction. Bracing is not curative, and surgical intervention may be required for correction of severe curves (usually 40 degrees or more). The goals of surgical intervention are to correct the curvatures on the sagittal and coronal planes and to provide a solid, pain-free fusion in a well-balanced torso, with maximum mobility of the remaining spinal segments.

The nurse is working with a new graduate in developing a plan of care for a newborn infant with spina bifida (myelomeningocele) and hydrocephalus. The nurse reminds the graduate to monitor for increased intracranial pressure (ICP). Which assessment technique should be performed to detect the presence of an increase in ICP? Measure urine specific gravity Assess for increased muscle tone Observe anterior fontanel for bulging Monitor blood pressure for signs of hypotension

Observe anterior fontanel for bulging Excessive buildup of CSF in the newborn's brain will cause expansion and fullness. The nurse can observe this through bulging (secondary to pressure) in the fontanels.

Which description represents a common disease presentation for a child with aplastic anemia? Pallor and fatigue Generalized edema Abdominal distention Epistaxis and joint pain

Pallor and fatigue Pallor and fatigue are symptoms of aplastic anemia.

Which most common cause is appropriate for the diagnosis of cerebral palsy (CP)? Birth asphyxia Cerebral trauma Neonatal diseases Prenatal brain abnormalities

Prenatal brain abnormalities Rationale The most common currently identifiable cause of CP is existing brain abnormalities during the prenatal period. Birth asphyxia had previously been considered a factor in the development of CP. Neonatal diseases have previously been considered factors in the development of CP. Cerebral trauma has previously been considered a factor in the development of CP.

The nurse notes the vital signs of a patient with sickle cell anemia (SCA) after splenectomy to be as follows: heart rate, 122; respiratory rate, 24; blood pressure, 80/48; and temperature of 100.3° F. The patient appears drowsy but is easy to arouse. Based on this assessment, which initial action should the nurse take? Administer an antipyretic. Auscultate the patient's lung sounds. Contact the health care provider (HCP). Prepare the patient for insertion of an intravenous (IV) line.

Prepare the patient for insertion of an intravenous (IV) line. Tachycardia, tachypnea, and hypotension indicate possible fluid volume deficit from blood loss during surgery. The nurse would prepare the patient for IV fluid administration per protocol.

The pediatric nurse is caring for a patient with beta-thalassemia who has been transferred to the unit for treatment. Before admission, the patient was experiencing complications of the disorder. Which action should the nurse take in initiating the treatment process? Administer desferoxamine as ordered. Administer prophylactic antibiotics as ordered. Prepare the patient for possible blood transfusion, if ordered. Educate the patient and family about the need for a bone marrow transplant.

Prepare the patient for possible blood transfusion, if ordered. Preparation the patient for a blood transfusion is the immediate action in treatment initiation.

Which measure is most important in managing hypercalcemia in a child who is immobilized? Changing position frequently Promoting adequate hydration Encouraging a diet high in calcium Providing a diet high in protein and calories

Promoting adequate hydration Rationale Hydration is extremely important in helping remove excess calcium from the body. This can help prevent hypercalcemia. Changing the child's position frequently will help with managing skin integrity but will not affect the calcium level. The calcium will not be incorporated into bone because of the lack of weight bearing. The child is at risk for hypercalcemia. The child's metabolism is slower because of the immobilization. A diet with sufficient calories and nutrients for healing is important.

What do we do for that? Newborn baby comes out with a sac on his back full of nerves and spinal fluid what do we do?

Protect the sac. Every effort is made to prevent trauma to the protective covering. Assess neurologic involvement by noting movement of the extremities, skin response, especially anal reflex (will provide clues to degree of neurologic involvement) Application of sterile, moist, nonadherent dressing. Dressings are changed frequently, Q2-4 hours and the sac closely inspected for leaks, abrasions, irritation, and signs of infection. When an overhead warmer is used the dressings require more frequent moistening. Prone positioning to protect the sac.

During the medication reconciliation for a patient with Von Willebrand disease (vWD), the nurse notes the child has been taking ibuprofen as needed for headaches. Which action is most important for the nurse to take? Continue to give the ibuprofen as needed. Obtain an order for aspirin, as needed, for headache. Determine the frequency and severity of the patient's headaches. Provide additional education to the caregivers about avoiding medications that affect platelet function.

Provide additional education to the caregivers about avoiding medications that affect platelet function. Patients with vWD should be taught to avoid any medications that affect platelet function, including ibuprofen. Acetaminophen is an effective alternative.

Which intervention is appropriate when ensuring adequate nutrition of the preterm infant with cerebral palsy? Provide feedings by mouth. Provide enteral feedings. Provide parenteral feedings. Provide gastrostomy feedings.

Provide gastrostomy feedings. Rationale Scheduled rest periods and adequate nutrition help to meet the metabolic needs of a patient with cerebral palsy. Preterm infants with decreased oral feeding capacity are prescribed gastrostomy tube placement. The ability to consume food orally is not developed in preterm infants. Supplementing nutrients by gastrostomy feedings helps in adequate growth and meets the nutritional needs of infants. Enteral and parenteral feedings are not used in preterm infants with cerebral palsy and may not be effective in meeting their nutritional needs.

A 14-year-old patient with cancer asks the nurse about dying. The nurse knows that the parents don't want that information shared with the child. What should the nurse do? Pursue an ethics consult. Discuss the situation with the parents. Discuss the situation with the nurse manager. Be honest with the child and answer the question.

Pursue an ethics consult. When faced with conflict regarding upholding the desires of the parent and being honest with the patient, the nurse should consider consulting the ethics committee for assistance in developing an agreeable solution.

Therapeutic management ITP

Restrict activity when platelet count is low Acute - prednisone, IVIG, anti D antibody (cant be used when actively bleeding), infusion when administering anti D, observe child for 1 h and maintain patent IV line, obtain baseline vitals and again 2, 20, 60 min after beginning infusion if fever, chills, headache begins - give acetominophen, benadryl, hydrocortisone. observe patient 1 hr after infusion no participation in conact sportsd Self-limiting w/o major complications

Which order of pathological events describes the development of osteomyelitis?

Rupture of small capillaries Pus spreads to the outside of the bone (under periosteum) Connective tissue of the bone is displaced from the accumulation of pus Abscess forms

Which clinical manifestation is appropriate for increased intracranial pressure in infants? Photophobia Vomiting and diarrhea Shrill, high-pitched cry Pulsating anterior fontanel

Shrill, high-pitched cry Rationale A shrill, high-pitched cry is a common clinical manifestation of increased intracranial pressure (ICP) in infants. The characteristic cry occurs as a result of the pressure being placed on the meningeal nerves, which causes pain. Photophobia is not indicative of increased ICP in infants. A pulsating anterior fontanel is normal in infants. The infant with increased ICP would be seen with a bulging anterior fontanel. Vomiting is one of the signs of increased ICP in children, but when present with diarrhea it is more indicative of a gastrointestinal disturbance.

How is a child with extrapyramidal cerebral palsy different from one with pyramidal cerebral palsy? Exaggerated stretch reflexes Slow, wormlike, writhing movements Presence of positive Babinski reflex Impairment of fine and gross motor skills

Slow, wormlike, writhing movements Rationale While extrapyramidal cerebral palsy is a nonspastic cerebral palsy, pyramidal cerebral palsy is a spastic cerebral palsy characterized by increased muscle tone. Slow, wormlike, writhing movements are characteristic of dyskinetic cerebral palsy, which is a type of extrapyramidal cerebral palsy. Exaggerated stretch reflexes, positive Babinski reflex, and impairment of fine and gross motor skill are characteristics of pyramidal cerebral palsy.

Which statement is appropriate when describing bowel function in children born with a myelomeningocele? Incontinence cannot be prevented. Enemas and laxatives are contraindicated. Some degree of fecal continence can usually be achieved. A colostomy is usually required by the time the child reaches adolescence.

Some degree of fecal continence can usually be achieved. Rationale With diet modification and regular toilet habits (bowel training) to prevent constipation and impaction, some degree of fecal continence can be achieved. Although a lengthy process, continence can be achieved with modification of diet, use of stool softeners, enemas, or a combination thereof. Enemas and stool softeners are part of the strategy to achieve continence. Laxatives should be used only as a last resort. A colostomy is not indicated for the child with myelomeningocele.

The nurse is caring for a child who has a decreased metabolic rate. What food item does the nurse include in the child's diet? Butter Bananas Soybeans Strawberries

Soybeans Rationale Children with a decreased metabolic rate should consume a high-protein, high-fiber diet. Soybeans and other legumes are excellent sources of proteins and should be included in the child's diet. Butter is a high-fat food item and is not recommended. Bananas and strawberries are rich sources of vitamins, potassium, and other nutrients but are not very good sources of proteins.

The nurse suspects a child is having an adverse reaction to a blood transfusion. What is the initial action by the nurse? Notify the physician. Take the vital signs and blood pressure and compare them with baseline levels. Dilute infusing blood with equal amounts of normal saline. Stop transfusion and maintain a patent intravenous line with normal saline and new tubing

Stop transfusion and maintain a patent intravenous line with normal saline and new tubing. Correct Stopping the transfusion and maintaining a patent intravenous line with normal saline and new tubing is the priority nursing action. If an adverse reaction is occurring, it is essential to minimize the amount of blood that is infused. Notifying a physician and taking vital signs and blood pressure should be performed after the blood transfusion is stopped and infusion of normal saline has begun. Blood should not be diluted; it should be returned to the blood bank if an adverse reaction has occurred.

How are synthetic casts different from plaster of Paris casts? Synthetic casts mold easily to any body parts. Synthetic casts must be protected when the child is bathing. Synthetic casts have smooth exteriors that minimize snagging of clothes. Synthetic casts dry faster, which results in a more comfortable sensation.

Synthetic casts dry faster, which results in a more comfortable sensation. Rationale Both synthetic and plaster of Paris casts produce heat by chemical reactions, but synthetic casts dry faster, so the uncomfortable sensation lasts only a few minutes. Synthetic casts do not mold easily to some body parts and are not suitable in such cases, whereas plaster of Paris molds readily to any body parts. Some synthetic casts can be immersed in water if the practitioner gives permission for it, but they must be dried properly. Plaster of Paris casts are not waterproof and precautions are taken to keep them dry. Both synthetic casts and plaster of Paris casts have smooth exteriors.

What is a priority nursing consideration when caring for a child with sickle cell anemia? Refer the parents and child for genetic counseling. Teach the parents and child how to recognize the signs and symptoms of crises. Help the child and family adjust to a short-term disease. Observe for complications of multiple blood transfusions.

Teach the parents and child how to recognize the signs and symptoms of crises. Correct Parents need specific instructions on the need to watch for changes in the child's condition, including adequate hydration, and environmental concerns. Genetic counseling is important, but teaching care of the child is a priority. Sickle cell anemia is a long-term, chronic illness. Multiple blood transfusions are an option for some children with sickle cell disease. The priority for all children with this condition is properly preparing the parents to care for them.

Barlow test

Test for a hip that is dislocatable but not dislocated in infants. With infant supine and hip and knees flexed, push posteriorly in line with the shaft of femur. An unstable femoral head will dislocate posteriorly from acetabulum.

Therapeutic Management anemia

Treat underlying cause Transfusion after hemorrhage if needed Nutritional intervention for deficiency anemias (iron containing foods) Supportive care IV fluids to replace intravascular volume Oxygen Bed rest Family education

Neural Tubal Deficits Treatment

Treatment = prevention: Supplementation: folic acid 0.4 mg/day If history of NTD, 4.0 mg/day 1998: FDA fortified cereal grains with folic acid Begin preconception

How is subluxation of the hip joint different from acetabular dysplasia? Select all that apply. The ligamentum teres is stretched. The capsule of the hip joint is stretched. The roof of the acetabulum is oblique and shallow. The femoral head is in contact with the acetabulum. The femoral head is displaced posteriorly and superiorly.

The ligamentum teres is stretched. The capsule of the hip joint is stretched. Rationale Subluxation means incomplete dislocation. When there is subluxation of the hip joint, the ligamentum teres and the capsule of the hip joint are stretched. As a result, the head of the femur is partially displaced. The roof of the acetabulum is oblique and shallow in cases of dysplasia rather than subluxation. In both subluxation and dysplasia, the femoral head remains in contact with the acetabulum. The femoral head is displaced posteriorly and superiorly in cases of dislocation of the hip joint.

Which purpose is appropriate when performing a lumbar puncture (LP)? To analyze cerebrospinal fluid To rule out subdural effusions To detect electrical activity To relieve intracranial pressure

To analyze cerebrospinal fluid Rationale An LP is done to obtain cerebrospinal fluid (CSF) for laboratory analysis. A subdural tap is performed to rule out subdural effusions. It is also done to remove CSF to relieve pressure. Electrical activity or spikes are detected by an electroencephalography (EEG). This test indicates the potential for seizures. LP is contraindicated in patients with increased intracranial pressure. A subdural tap or ventricular puncture may be done to remove CSF to relieve pressure.

The nurse understands that which is accurate regarding the necessity of a bone marrow biopsy for the diagnosis of aplastic anemia? To obtain a complete blood count To assess for the presence of microcytic red blood cells To assess for the presence of infection in the blood To assess for the absence of precursors to mature red blood cells

To assess for the absence of precursors to mature red blood cells A bone marrow biopsy determines whether precursors to mature red blood cells are present or absent and helps rule out other causes of pancytopenia, such as cancer.

autonomic nervous system

the part of the nervous system that is composed of the sympathetic and parasympathetic systems, which provide automatic control of vital functions

autoregulation

the unique ability of the cerebral arterial vessels to change their diameter in response to fluctuation cerebral perfusion pressure

Five distinct family management styles have been identified

thriving, accommodating, enduring, struggling, and floundering

goal in working with the family of a child with chronic oe complex illenxx

to support the family's coping and promote their optimal functioning

risk of overuse injury

training error muscle tendon imbalance anatomic malalignment incorrect footwear associated disease state growth

What is the most common cause of cerebral palsy (CP)? a. Central nervous system (CNS) diseases b. Birth asphyxia c. Cerebral trauma d. Neonatal encephalopathy

ANS: D Approximately 80% of CP is caused by unknown prenatal causes. Neonatal encephalopathy in term and preterm infants is believed to play a significant role in the development of CP. CNS diseases such as meningitis or encephalitis can result in CP. Birth asphyxia does contribute to some cases of CP. Cerebral trauma, including shaken baby syndrome, can result in CP.

Emboli formation R/T fractures

- Pulmonary emboli may happen 6-8 weeks post fracture R/T post injury bleeding and clotting - Fat emboli may happen within 24 hours R/T fat droplets from the marrow - Sudden chest pain and dyspnea are Sx and 911 > elevate Head, give O2, and start an IV

A school-age child is admitted with osteomyelitis. Which statement by the parent will require further attention by the nurse and health care provider? "Our child has had several cavities." "Our child has been on antibiotics frequently." "Our child has poor coordination and falls often." "We have noticed a change in our child's energy levels."

"Our child has had several cavities." Children with poor oral hygiene, as may be indicated by several cavities, are at increased risk for systemic infections and illness, including osteomyelitis.

Nursing care of braces/orthoses

- Assessment of the skin for redness is crucial every 2-4 hours for the first few days - If redness doesn't go away after 1/2-1 hour of the brace being removed, contact the doc orthpses used to prevent deformities, increase the energy effeciency of gait, control alignment braces - facilitate walking, stabilize paralized or markedly weakened extremities

A pregnant woman asks the nurse why Down syndrome occurs. Which response by the nurse is appropriate? "Down Syndrome occurs because there is a chromosome missing on chromosome 21." "Down syndrome usually occurs because of a structural relocation of chromosome 21." "Down syndrome occurs because there is a fragile chromosome on the 21st chromosome." "Down syndrome occurs because there is an additional chromosome present on the 21st chromosome."

"Down syndrome occurs because there is an additional chromosome present on the 21st chromosome." Down syndrome is caused by three representatives of Chromosome 21 instead of two, and therefore this is the most appropriate response for the nurse to make.

During a well-child visit, a three-year-old patient being examined exhibits unsteady gait and poorly developed speech. The health care provider suspects cerebral palsy (CP). How should the nurse guide the parents? "Ensure that your home is free of sharp edges to protect the child in case of falling while walking." "Administer pain medication whenever the child shows signs of pain such as grimacing or moaning." "While we get further testing arranged, keep notes regarding any unusual behaviors or actions." "The speech-language pathologist will work with you to evaluate reasons for the poorly developed speech." "The physical therapist will show you some exercises to improve coordination and strengthen the child's muscles."

"Ensure that your home is free of sharp edges to protect the child in case of falling while walking." A three-year-old child with suspected cerebral palsy would be expected to have unsteadiness when walking due to abnormal development of the motor system. "The speech-language pathologist will work with you to evaluate reasons for the poorly developed speech." A three-year-old child with suspected cerebral palsy would be expected to have speech difficulties due to abnormal central nervous system development. Speech development can also be related to cognitive issues, autism, and hearing loss. Correct "The physical therapist will show you some exercises to improve coordination and strengthen the child's muscles." Physical therapy and occupational therapy will work with the child to further evaluate gait and develop a plan to prevent or reduce declining strength and coordination

A newborn infant failed to pass meconium in the first twenty-four hours of life. The parents ask the nurse if there is anything wrong. Which response by the nurse is appropriate? "Failure to pass meconium in the first day of life could be an indicator of inadequate breast milk intake." "Failure to pass meconium in the first day of life could be associated with a blockage caused by thick mucus secretions." "Failure to pass meconium in the first day of life could be associated with the large intestine not being formed correctly." "Failure to pass meconium in the first day of life could be associated with the small intestine not fully being developed

"Failure to pass meconium in the first day of life could be associated with a blockage caused by thick mucus secretions." The presence of a meconium ileus would cause a delay in the passage of the first meconium stool, which is associated with cystic fibrosis.

The parents of three children, including a toddler with hemophilia, are talking with the nurse and admit that the child with hemophilia is the center of attention in the family since they are more protective of the child who is ill. Which statements by the nurse should be an appropriate response to the parent's actions toward their child? Select all that apply. "Favoritism toward your toddler can negatively affect important boundaries and should be avoided." "Making the toddler the center of attention can actually cause some harm by decreasing age-appropriate development." "It is important for you as parents to spend time together to maintain your relationship as well so that the foundation of your family is strong." "Let's talk about ways to ensure that all of your children feel equal attention so that there are not negative feelings towards the ill child or parents." "The toddler needs extra support because of the chronic condition, so it is important to ensure that the toddler's required care is the focus of family activities." Correct Answer Exactly!

"Favoritism toward your toddler can negatively affect important boundaries and should be avoided." Maintaining boundaries in the family with a child who has a chronic condition is important so that all members of the family feel cared for equally and development can be fostered. "Making the toddler the center of attention can actually cause some harm by decreasing age-appropriate development." Overprotection can slow development, and therefore this is the appropriate response for the nurse to make. "It is important for you as parents to spend time together to maintain your relationship as well so that the foundation of your family is strong." Parents do need to foster a strong relationship between each other and with their children in order to cope with a chronic condition. Making the child who is ill the center of attention can cause problems in the family. "Let's talk about ways to ensure that all of your children feel equal attention so that there are not negative feelings towards the ill child or parents." Overindulging one child can cause animosity among the children and cause problems in sibling relationships.

What would cause a nurse to suspect that an infection has developed under a cast? Cold toes Increased respirations Complaint of paresthesia "Hot spots" felt on the cast surface

"Hot spots" felt on the cast surface Correct If hot spots are felt on the cast surface, they usually indicate infection beneath the area. This should be reported so that a window can be made in the cast to observe the site. Cold toes may indicate too tight a cast and need further evaluation. Increased respirations may indicate a respiratory infection or pulmonary emboli. This should be reported, and the child should be evaluated. The five Ps of ischemia from a vascular injury include pain, pallor, pulselessness, paresthesia, and paralysis. Paresthesia is an indication of vascular injury, not infection.

A preschool-aged child with cancer has begun crying for her pacifier, which she stopped using 6 months ago. The parents voice concerns to the nurse. How should the nurse respond to the parents? "How do you feel about allowing her to use the pacifier while in the hospital?" "Explain to the child that she is a 'big girl' and does not need a pacifier anymore." "Do not give her the pacifier because this could cause her not to develop appropriately." "Your daughter may not be dealing well with the illness and treatment. She may need to meet with a psychologist."

"How do you feel about allowing her to use the pacifier while in the hospital?" Regression is typical and expected in young children who are undergoing a major illness. The child is trying to cope with stress related to the hospitalization and/or cancer, so allowing a pacifier is acceptable and can decrease the child's stress. The nurse should investigate how the parents feel about the child using the pacifier during the hospital admission.

Immobility effects on the skeletal system

- Bone formation ceases from lack of stress on the bones and destruction (resorption) continues = no equilibrium > prone to fractures and increased calcium in the blood, bone calcium depleted, secretion of phosphorus and nitrogen is increased

The nurse suspects that a female adolescent patient has iron deficiency anemia because she complains of weakness, palpitations, and always feeling cold. Which follow-up questions should the nurse ask? Select all that apply. "When was your last bowel movement?" "Have you ever had a blood transfusion?" "How heavy is your monthly period?" "What types of foods do you eat on a typical day?" "How long have you experienced these symptoms?"

"How heavy is your monthly period?" Heavy blood loss during menstruation could lead to iron deficiency anemia. The nurse should obtain a menstrual history. "What types of foods do you eat on a typical day?" Lack of dietary iron can lead to iron deficiency anemia. The nurse should obtain a diet history. "How long have you experienced these symptoms?" The nurse should question the patient about the onset and duration of symptoms.

The nurse is providing education to a child and family during a sports physical examination to explain the differences in the pediatric musculoskeletal system compared to adults. Which statement by the child indicates correct understanding of the education? "My bones are more likely to break because they are growing." "I don't need to worry about fractures because my bones will heal faster." "As new muscles develop, I will be able to run faster while playing sports." "I am not as likely to sprain my ankle when playing sports, but I should be careful."

"I am not as likely to sprain my ankle when playing sports, but I should be careful." This statement indicates correct understanding of the differences between the pediatric and adult musculoskeletal system, because the resiliency of soft tissues makes sprain less likely than in adults.

A child has recently been diagnosed with Guillain- Barré syndrome. Which patient statements require follow-up by the nurse? "I had a nasty cold two weeks ago, but mom said it was not a big deal." "I had a red rash a while back with a fever. It only lasted about three days." "When I get well, I'm not shaking hands with anyone, not even the pastor!" "I haven't felt well for a few days and my toes are kind of numb feeling today." "I hate getting the flu-shot, but dad insisted this year since I've been sick so much."

"I had a nasty cold two weeks ago, but mom said it was not a big deal." Enterovirus causes cold and flu-type symptoms and is associated with the development of Guillain- Barré syndrome. The child needs teaching about hand-washing techniques to prevent respiratory illnesses. Correct "I had a red rash a while back with a fever. It only lasted about three days." A rubella infection has been associated with Guillain-Barré syndrome. The nurse follows up as the health care team works to determine a cause that could be prevented in the future. Also, if "a while back" means more than a month or so, the health care provider can rule it out as a cause. "I hate getting the flu-shot, but dad insisted this year since I've been sick so much." The influenza vaccine has a demonstrated link to the onset of Guillain-Barré. If this is the suspected cause, the child needs to avoid the flu-vaccine in the future

The 12-year-old patient with severe headaches is being discharged from the emergency department. Which statement by the patient indicates to the nurse the need for further teaching? "I hear my friends use marijuana for stress relief. I will try that." "If ibuprofen doesn't work at first, I take another one in two hours." "Since chocolate seems to trigger my headaches, I will eat less of it." "In my diary, lack of sleep is a trigger. I am going to go to bed earlier." "Football and the Spanish club are too much. I think I will drop them both."

"I hear my friends use marijuana for stress relief. I will try that." Smoke can increase headache occurrence, therefore this statement would indicate a need for further teaching. Also the nurse will want to teach about illegal substance use. Correct "If ibuprofen doesn't work at first, I take another one in two hours." Ibuprofen will cause bleeding and other injury if taken this way, therefore this statement indicates a need for further teaching. Correct "Since chocolate seems to trigger my headaches, I will eat less of it." The child needs to avoid, not just decrease, any trigger that has been identified. This statement would therefore indicate a need for more education on dietary habits and their connection to headaches "Football and the Spanish club are too much. I think I will drop them both." Encourage the child to consider ways to reduce fatigue and stress without increasing isolation by dropping both activities at the same time. Perhaps there is a way to reduce involvement with one of the activities or maybe consider dropping one to evaluate headache response.

A nurse is providing care for a thirteen-year-old patient with a history of headaches. Which statements by the patient indicate further teaching is required? Select all that apply. "I just try to stay away from everyone." "I keep a diary of what I eat and how I sleep." "I dropped one of my extracurricular groups." "When I feel stressed out, I play video games." "I alternate ibuprofen and acetaminophen all day."

"I just try to stay away from everyone." Isolation is not an effective therapeutic strategy for the long-term management of headache, and therefore this statement indicates a need for further teaching. "When I feel stressed out, I play video games." The use of stress management techniques is a therapeutic strategy for the long-term nursing management of headache. However, visual stimuli can induce headaches. Alternative stress management techniques must be taught, and therefore this statement indicates a need for further teaching. "I alternate ibuprofen and acetaminophen all day." The administration of medication is a therapeutic strategy for the long-term nursing management of headache. Constant self-medicating will lead to rebound headaches. This statement indicates a need for further teaching.

The nurse is educating a parent of a child newly diagnosed with sickle cell anemia. Which statements, made by the parent, indicate teaching was effective? Select all that apply. "My child should avoid range of motion exercises." "I keep bottles of water with me whenever we go out." "I made an appointment for the year 4 immunizations." "I will call our health care provider as soon as I notice any cough or fever." "We only play in the snow for about 20 minutes at a time."

"I keep bottles of water with me whenever we go out." Children with sickle cell anemia should stay well hydrated and therefore this statement does not indicate a need for further teaching. Correct "I made an appointment for the year 4 immunizations." Children with sickle cell anemia should be routinely vaccinated to prevent infection. Correct "I will call our health care provider as soon as I notice any cough or fever." Any sign of infection should be reported to the health care provider immediately. Correct "We only play in the snow for about 20 minutes at a time." Patients with sickle cell anemia should avoid prolonged exposure to the cold.

Over heating/heat-stress

- R/T greater surface area which leads to increased transfer of heat between the body and the environment, increased metabolic heat during exercise, decreased sweating capacity, decreased water intake during exercise, longer acclimation to hot conditions -

Which statement is appropriate when addressing the concerns of parents about their daughter's health when their son has X-linked Duchenne muscular dystrophy (DMD)? Select all that apply. "Your daughter is surely a carrier of the disease." "It can be a new mutation, and the mother need not be a carrier." "Your daughter may be a carrier and could develop cardiomyopathy." "It is a genetic disease caused by mutation of the gene that encodes dystrophin." "Your son has inherited the disease from both parents."

"It can be a new mutation, and the mother need not be a carrier." "Your daughter may be a carrier and could develop cardiomyopathy." "It is a genetic disease caused by mutation of the gene that encodes dystrophin." Rationale DMD is a genetic disease. It can be a new mutation that appears in any generation, and the mother need not be a carrier. In some cases, the daughter is a carrier and can later develop cardiomyopathy. In this mutation, the gene that encodes dystrophin is unable to produce the necessary protein. This condition follows an X-linked recessive inheritance pattern. When the father is unaffected and the mother is a carrier, there is a 50% chance that a son will be affected and a 50% chance that a daughter will be a carrier. Therefore the daughter is not necessarily a carrier of the disease. The son cannot inherit the disease from the father if the latter is not showing signs of muscular dystrophy.

A home care nurse is caring for a 4-month-old infant with developmental dysplasia of the hip (DDH). The baby is in a Pavlik harness. The baby's mother tells the nurse, "I don't think my baby will be able to sleep while wearing the harness." Which response by the nurse is most appropriate? "The harness can be removed during a short 30-minute nap." "You can reapply the harness after the baby falls asleep." "It is important for the harness to be worn continuously." "You can have the baby not take one of the daily naps."

"It is important for the harness to be worn continuously." Rationale The harness is worn continuously until the hip is proved stable on clinical and ultrasound examination, usually in 6 to 12 weeks. Some practitioners permit its removal for bathing. Removal or reapplication of the harness will probably awaken the infant. Babies should not be prevented from taking naps, because such naps are essential for good health.

Which statement requires correction regarding growth and development in children with cognitive impairment (CI)? "Children with CI need simple, one-step directions." "It is important to punish a child with CI when a task is not mastered." "Fading and shading can be used to help children with CI learn tasks." "Advances in technology can provide children with CI positive reinforcement."

"It is important to punish a child with CI when a task is not mastered." Rationale The child with CI should not be punished when a task is not mastered. This statement is erroneous and needs to be corrected. A child with CI has difficulty mastering tasks. Therefore, providing simple, one-step directions helps these children to be successful. Fading is physically helping the child through each step of a task to promote mastery. Shading involves waiting for the child to display success and then incorporating positive reinforcement when the child is successful. Advances in technology, such as computers, have improved the capabilities of children with cognitive impairment.

The parents of four children, one of which has a chronic illness, state, "My spouse and I spend as much time as we can with our sick child, but our family just seems to be stressed all the time." What should be the nurse's best response? "The children just need an outlet for their frustration. You could send them to a support group." "The other children are probably just reacting to the stress of having a sibling with a chronic illness." "It is possible to spend too much time with a child who is ill and not enough time with your other children." "Educate your other children on the need for you to spend extra time with the child who is ill to decrease their stress."

"It is possible to spend too much time with a child who is ill and not enough time with your other children." This statement is factual and provides an opportunity for the nurse to educate on the need for spending time equally among all the children.

A nurse is caring for a male child with Duchenne muscular dystrophy (DMD) and the mother shares that she is pregnant again. She expresses concern that her second child is a girl and asks about the possibility that her daughter will also have DMD. How does the nurse respond? "Your daughter will likely have DMD because your son has it and it is a genetic disorder." "Your daughter may have DMD because she needs just one copy of the affected X chromosomes to have the disorder." "There is a 50% chance that your daughter will have DMD because only one dominant gene is required for her to have it." "It is unlikely that your daughter will have DMD because she would need to have two 'X' chromosomes with the gene, making the condition much more likely in male children."

"It is unlikely that your daughter will have DMD because she would need to have two 'X' chromosomes with the gene, making the condition much more likely in male children." Duchenne muscular dystrophy is an X-linked disorder. Both X chromosomes would need to be afflicted with the DMD gene in order for the daughter to have the disease.

An adolescent who has had diabetes mellitus for ten years is able to check his blood glucose and maintain glucose levels while still participating in sports and maintaining a 'B' average. The adolescent tells the nurse, "I am so glad that I am finally able to deal with having diabetes." What is the nurse's best response? "You seem to be taking on a lot for someone with a chronic health care condition." "It seems that you have finally worked through your anger and frustration related to your illness. Keep up the good work." "You might be in denial that you have a chronic illness that requires you to make some changes in your life. This is common at your age." "It sounds like you feel as though you are coping well with diabetes and have reached acceptance of the illness. Just remember that at times you might struggle, and that it is OK to ask for help."

"It sounds like you feel as though you are coping well with diabetes and have reached acceptance of the illness. Just remember that at times you might struggle, and that it is OK to ask for help." This teen seems to be doing well, but setting unreasonable expectations can be a set-up for distress in the future. Acknowledging the hard work is important, but also pointing out that there are resources available to help if needed is equally important

Which statement can the nurse use to explain to parents why nonsurgical treatments are used instead of surgical treatments when treating clubfoot? A. "The surgical method takes too long to complete." B. "Nonsurgical methods allow for a faster correction of the deformity." C. "Nonsurgical treatment will limit long-term complications and recurrence." D. "The surgical method has a limited coverage by the insurance company."

"Nonsurgical treatment will limit long-term complications and recurrence." Because long-term complications and recurrence were found to occur after surgical correction, nonoperative treatment modalities are now used.

The nurse is assessing a 1-month-old infant for the presence of skeletal abnormalities. Which statement by the baby's mother suggests the presence of such an abnormality? "The baby always prefers sleeping while curled up." "The baby's feet look flat when I put on the booties." "It's difficult to put the diaper between the baby's legs." "When I try to stand my baby up the legs won't straighten."

"It's difficult to put the diaper between the baby's legs." Rationale Restricted abduction of hip on the affected side indicates the presence of developmental dysplasia of the hip (DDH). Flexion of the extremities is a young infant's typical position when sleeping. Flat feet are an expected finding in a young infant. Failure to straighten the legs is an expected finding in a young infant.

A single parent of a child with cancer comes into the care provider's office and states, "I am so frustrated. Lately the appointments are too much and my child's behavior has been terrible." Which statement by the nurse is appropriate? "Why exactly are you frustrated?" "You should be firmer when you discipline the child." "All the appointments are necessary for the child's health." "Let's talk about some helpful techniques that can assist you in working through the frustration you are having."

"Let's talk about some helpful techniques that can assist you in working through the frustration you are having." Teaching the parent relaxation techniques, such as deep breathing, provides an effective outlet for emotional tension and support for the current problem, and therefore this statement by the nurse would be appropriate.

A teenager with leukemia states, "I thought everything about this was going to be terrible, but at least my parents have stopped making me do chores and homework." The next time the nurse sees the parents, what should be the nurse's best response? "Your teen seems to be responding well to a decrease in stress at home from the elimination of the need to do homework and chores." "Let's talk about the importance of maintaining age-appropriate expectations with your teenager, even while your teen is battling leukemia." "Allowing your teen to neglect homework and chores is going to make it harder on going back to usual expectations after the battle with leukemia is over." "Decreasing expectations for a little while will minimize the behavioral and psychologic issues that often develop in children and teens experiencing a chronic health condition."

"Let's talk about the importance of maintaining age-appropriate expectations with your teenager, even while your teen is battling leukemia." Maintaining expectations and rules is important to the development of any child with a chronic illness. Although modifications may need to be made, eliminating chores and expectations can actually hurt the overall development of the teen.

The parent of a school-aged child states, "I am so proud of my daughter. She was diagnosed with asthma and yet she still does everything she was doing before. She processed this diagnosis more quickly than I thought she would." How should the nurse respond? "Let's talk to your daughter and see how she is doing." "It is great to hear that your daughter is coping so well." "She might not be doing as well as you think. There is a chance she is in denial that there is a problem." "Your daughter might be hiding her feelings from you and should speak with a mental health specialist for support."

"Let's talk to your daughter and see how she is doing." The nurse needs to assess the child to see if she is really reaching acceptance or if she is in denial with the asthma diagnosis. Having a discussion with the daughter will be the best action to take.

The parents of infant twins, both with developmental delays, ask the nurse about resources for caring for the children as they get a little older. Which information should the nurse share? "Your family will find a pattern of normalcy that will make caring for the children easier." "Medical homes can offer holistic and centralized care that include the patient and family as the primary focus." "There are support groups and community resources available to help cope with caring for children with special health care needs." "Having two children with developmental delays likely means that the children will need to be institutionalized to receive adequate care."

"Medical homes can offer holistic and centralized care that include the patient and family as the primary focus." With two children needing care, the family will need support and care that is centered on the family. A medical home can provide the care and community resources needed to care for two children with developmental delays.

A child with a fracture of the tibia is discharged from the hospital after application of a cast. The nurse explains to the parents about the cast care to be performed at home. Which statement by the child's parents indicates that they understand proper cast care? "My child needs to stand and walk every 30 minutes." "My child needs to keep the broken leg hanging down." "My child needs to elevate the affected limb whenever possible." "My child needs to use a small stick to relieve itching under the cast."

"My child needs to elevate the affected limb whenever possible." Rationale The affected part should always be elevated whenever possible. This reduces swelling and allows venous return. Standing and walking frequently are not recommended, because the injured area requires rest. Keeping the affected limb hanging down reduces the venous return. The child should not be allowed to insert sticks or anything else inside the cast to relieve itching. This can damage the cast or cause infection.

Which statement, made by an infant's parent, would indicate the infant is at risk for having inherited sickle cell anemia? Select all that apply. "My mother had sickle cell disease." "I have cousins with sickle cell disease." "My husband and I are both carriers of the trait." "I have sickle cell disease, but my husband does not and he is not a carrier." "My husband and I are not carriers, but my grandmother and my husband's grandfather both died of sickle cell disease."

"My mother had sickle cell disease." Sickle cell disease is an inherited autosomal recessive genetic disorder. A maternal grandmother with sickle cell disease indicates the child may carry the trait but does not confirm the child has the condition. "I have cousins with sickle cell disease." Sickle cell disease is an inherited autosomal recessive genetic disorder. A family history of sickle cell disease indicates the child may carry the trait but does not confirm the child has the condition. "My husband and I are both carriers of the trait." Sickle cell disease is an inherited autosomal recessive genetic disorder. If both parents are carriers, the child has a higher likelihood of having the condition.

Immobility effects on the GI

- Muscles atrophy > increased catabolic activity > negative nitrogen balance (negative because of increased breakdown of muscles = less muscle/protein but; this does not indicate a lab value) + diminished appetite = anorexia (lack of nutrients) - Risk for aspiration - Slowing of stool in the colon > fecal impaction - Embarrassed to use the bedpan, or weakened muscles of defecation > constipation

An adolescent who is training for a marathon develops shin splints. Which statement by the nurse is most appropriate to help the adolescent manage the condition? "Reduce your regular training time." "Take a complete rest for some time." "Run in the pool instead of on the road." "Take antiinflammatory medicines and continue usual activities."

"Reduce your regular training time." Rationale The adolescent with shin splints should not aggravate the injury by the repetitive stress that initiated the symptoms; however, the adolescent's training should be continued. The nurse can suggest alternative training, such as pool running, that involves the same movements as running, but without the weight bearing. The repetitive action that caused the injury should be avoided, but reducing training time is not recommended. The adolescent should not be advised to rest, because the adolescent is training for a marathon. The adolescent may take nonsteroidal antiinflammatory drugs to manage symptoms, but should engage in cross-training exercises

The adolescent has a large ecchymosis in the thigh area and limited range-of-motion in that extremity. Which instruction by the nurse is most appropriate? "Apply a range-of-motion brace." "Rest, ice, compress, and elevate." "Disregard the pain and work it out." "Take nonsteroidal antiinflammatory drugs."

"Rest, ice, compress, and elevate." Rationale The adolescent is suffering from a contusion, which is damage to the soft tissue, subcutaneous structures, and muscles. Application of cold is the immediate course of action. The adolescent should rest the leg, ice the area, apply compression, and elevate the extremity on pillows. A range-of-motion brace is applied for torn ligaments, especially those in the knee. Application of a brace is not indicated in this case. If the adolescent does not rest the leg and continues with regular activities, myositis ossificans may occur from deep contusion to the quadriceps muscles. In addition to giving rest to the extremity, the adolescent may take nonsteroidal antiinflammatory drugs to manage the pain.

The parents of a child with fragile X syndrome ask the nurse how they can prepare their child for school. They state that the child has poor verbal skills, is uncoordinated when walking, and cannot button or zip clothing items. Which response by the nurse is most appropriate? "You should get your child a tutor to help with schoolwork." "Sensorimotor integration therapy would benefit your child." "Working with an occupational therapist would help your child." "Special education services will assist your child when they start school.

"Sensorimotor integration therapy would benefit your child." Sensorimotor integration therapy is the intervention of choice for children with fragile X syndrome.

A parent asks the nurse why fragile X syndrome affects boys greater than girls. Which response by the nurse is appropriate? "Boys are genetically more susceptible to genetic mutations compared to girls." "Boys are considered carriers of the gene, and girls are not carriers of the gene." "Boys have a full mutation of the gene but girls only have a partial mutation of the gene." "Since boys are XY, rather than XX, there is no normal X chromosome to mask the abnormal one."

"Since boys are XY, rather than XX, there is no normal X chromosome to mask the abnormal one." Only males exhibit the full effects of the disorder because their single X chromosome has the abnormal gene.

The nurse is discussing the surgical closure of a myelomeningocele with the parents of a newborn patient. Which statement by the parents indicates the need for further teaching? "Surgically closing this defect will ensure my baby can walk at the right age." "This surgery makes me feel less like my baby is too fragile to hold and feed." "This surgery minimizes the problems my baby has over the course of a lifetime." "Surgically closing this defect reduces the chance my baby will develop infections."

"Surgically closing this defect will ensure my baby can walk at the right age." Surgery will decrease cord deterioration and allow for earlier physical therapy and developmental interventions. The child with spina bifida corrected by surgery still may require additional surgeries.

Which nursing instruction is appropriate to include when educating the parents of a child with Down syndrome? "Your child is at the stage of detachment." "The child will cling if you show enough care." "Swaddle your child in a blanket to provide warmth." "Wash your hands once a day when handling secretions."

"Swaddle your child in a blanket to provide warmth." Rationale The nurse would encourage the parents of a child with Down syndrome to swaddle the child to provide warmth. The nurse would explain that the child's lack of clinging is a physical characteristic of children with Down syndrome, and not a sign of detachment or rejection or the result of poor parenting. Children with Down syndrome have an underdeveloped nasal bone, which causes chronic drainage of mucus; the nurse would instruct the parents to perform good hand hygiene when handling secretions.

An adolescent going through cancer treatment lost her hair and missed prom six months ago. The teen is coming in for her usual check-up and starts crying. The nurse states, "All of your blood work looks good. You are fine." Which statement reflects how the nurse should have responded better? "Tell me what you are thinking about right now." "There is nothing to be concerned about right now. Everything looks good." "I think you are misunderstanding what your test results show. You are getting better." "I thought you had made it past your grief. You seemed so happy during your last appointment."

"Tell me what you are thinking about right now." The nurse needs to understand that anyone, especially an adolescent, may fluctuate through the grieving process. This teen has undergone a change in body image and has missed out on a significant milestone in her life. Allowing the child to talk openly would be best in this situation.

A 36-year-old newly pregnant patient asks the nurse the most significant risk factor for Down syndrome. Which response by the nurse is appropriate? "The most significant risk factor is the age of the woman at conception." "The most significant risk factor is having siblings with Down syndrome." "The most significant risk factor is living in the southern states of America." "The most significant risk factor is the age of the father at the time of conception."

"The most significant risk factor is the age of the woman at conception." The age of the mother at the time of conception is the most significant risk factor for Down syndrome, and therefore this response is the most appropriate.

Immobility effects on the cardiovascular system

- Orthostatic intolerance - Increased workload of the heart - Thrombus formation - Sx of excessive autonomic activity: pallor, sweating, restlessness, fainting, edema - Sx of DVT: sudden chest pain and dyspnea, SOB, air hunger, pain and swelling in the lower extremities spinal cord injury -> orthstatic intolerance

Which nursing information is appropriate for the parents of a child with cerebral palsy preparing for an intrathecal infusion of baclofen? Select all that apply. "The child will be discharged after 2 days." "The procedure has more side effects than oral baclofen." "The procedure is performed to reduce muscle spasticity." "A pump containing baclofen is programmed to provide a set dose." "An intrathecal catheter is connected to a pump containing baclofen." "A pump containing baclofen is placed in the subcutaneous space of the thorax."

"The procedure is performed to reduce muscle spasticity." "A pump containing baclofen is programmed to provide a set dose." "An intrathecal catheter is connected to a pump containing baclofen." Rationale The nurse explains to the parents that the procedure is performed to reduce muscle spasticity by infusing baclofen directly into the intrathecal space surrounding the spinal cord. A pump containing baclofen is programmed to provide a set dose using a telemetry wand and a computer. An intrathecal catheter is tunneled from the lumbar area to the abdomen and connected to the pump. The child is discharged after the dosage is adjusted and proper healing is ensured, which may take several days. This procedure has fewer side effects than oral baclofen. The pump containing baclofen is placed in the subcutaneous space of the mid-abdomen.

A pregnant patient whose fetus has trisomy 21 asks the nurse which medical conditions are associated with trisomy 21. Which responses by the nurse are appropriate? Select all that apply. "There could be a failure of the upper lip to completely form." "There could be a hole between the right and left side of the baby's heart." "There could be a problem with the baby's esophagus, in which it is not completely formed." "There could problems with the hands and feet where the fingers and toes are fused together." "There could be a lack of nerves going to the large intestine that would cause it not to work correctly."

"There could be a hole between the right and left side of the baby's heart." Atrial septal defect is a medical condition associated with trisomy 21, and therefore this is an appropriate response for the nurse to make. Correct "There could be a problem with the baby's esophagus, in which it is not completely formed." Tracheoesophageal fistula is a medical condition associated with trisomy 21, and therefore this statement would be appropriate. "There could be a lack of nerves going to the large intestine that would cause it not to work correctly." Hirschsprung's disease is a medical condition associated with trisomy 21, and therefore this is the response that the nurse would make.

Which nursing response is appropriate for the parent of a child with cognitive impairment who is using detailed verbal explanations to teach dressing skills? "Use demonstration while explaining." "The child will take a long time to learn." "Use short sentences to avoid confusion." "Explain very slowly and use comforting words."

"Use demonstration while explaining." Rationale A child with cognitive impairment understands concrete ideas better than abstract ideas. Therefore demonstration is a more effective way to teach the child than verbal explanations. Asking the parent to explain slowly and using comforting words does not ensure that the child comprehends the message. Telling the parent that the child will take a long time to learn is not encouraging. Using short sentences will not ensure that the child understands the message.

The nurse is completing discharge teaching of a patient who originally presented with symptoms of aplastic anemia. Which statement, made by the parents, indicates teaching was effective? "My child is a competitive wrestler." "Our favorite thing to do together is go to the mall." "We don't believe in vaccinations for our children." "We bought a new soft-bristle toothbrush yesterday."

"We bought a new soft-bristle toothbrush yesterday." Children with aplastic anemia should use soft toothbrushes to prevent bleeding from the gums. This statement indicates that teaching was effective.

The nurse cares for a five-year-old patient involved in a motor vehicle accident. The paralysis extends from the naval downward. In performing discharge teaching, the nurse knows further teaching is needed when the parents make which statements? "We need to catheterize him every 8 hours for urine." "We need to turn him in the bed at least every 2 hours." "We need to make sure he has a bowel movement often." "He will need to eat every meal that we prepare for him." "He will enjoy sitting outside all morning in his wheelchair."

"We need to catheterize him every 8 hours for urine." The child should not hold urine in his bladder for an extended period of time. Since the child cannot feel the urge to urinate, catheterization does need to occur on a schedule. If urine is left in the bladder, causing bladder distention, a kidney infection may result. Catheterization should occur instead every 4-6 hours. "We need to make sure he has a bowel movement often." The parents need to make sure that the child has a bowel movement every 1-2 days, rather than stating "often." The stool should be softly formed rather than hard or loose. With little innervation to the bowels, peristalsis will be limited and constipation quickly becomes an obstruction. Correct "He will need to eat every meal that we prepare for him." This is not the best way to make sure the child gets adequate nutrition. Five-year-olds normally ingest small portions and they can love a food one day and hate it the next. Nutritional teaching includes information about snacks, supplements, and nutrient-dense foods. Correct "He will enjoy sitting outside all morning in his wheelchair." The child should not sit up in the wheelchair for an extended period of time. This statement needs to be clarified. Sitting for more than an hour or two can produce pressure ulcers.

A nurse is caring for a young child who strained his ankle playing soccer at school that morning. After teaching the parents how to care for the ankle for the rest of the day, which statements by the parents indicate effective learning has occurred? Select all that apply. "We should administer NSAIDS for pain." "We should apply heat for 20 minutes at a time." "My child should rest his/her ankle as much as possible." "We should apply a compression bandage to support the ankle." "We should have him keep his ankle below the level of his heart to allow drainage of excess fluid."

"We should administer NSAIDS for pain." Analgesia and anti-inflammatory medications, such as NSAIDs, should be used in the treatment of soft tissue injury to relieve pain. "My child should rest his/her ankle as much as possible." Rest (R) prevents further damage or irritation to the injury site. "We should apply a compression bandage to support the ankle." Compression (C) will support the ligaments, muscles, and tendons in an injured joint.

Skeletal TB

- Primary TB infection > lymphohematogenous spread > bone and joint infection with TB - Usually happens to the vertebrae > destruction leads to hyperkyphosis symptoms - persistent or intermittent pain, joint swelling, stiffness, - TB skin test will be positive but MUST also isolate mycobacterium TB from the site - Tx: isoniazid + rifampin + pyrazinamide for 1 year isolation

The nurse is providing education to the parents of a child experiencing spinal shock after a spinal cord injury. Which statements by the nurse are correct? "We will not know what permanent injuries exist for one to two months." "Currently, the child appears to have no function below the level of injury." "Limbs that are currently flaccid will remain that way for about six months." "Nurses will perform passive range of motion exercises daily to maintain muscles." "Some complications, such as low blood pressure, will resolve within a few weeks."

"We will not know what permanent injuries exist for one to two months." The injuries associated with spinal shock can cause temporary loss of function, which means regaining function can occur. This statement could therefore be used by the nurse to address concerns regarding the recovery phase. Correct "Currently, the child appears to have no function below the level of injury." Spinal shock can cause deceased function of everything below the level of injury including musculoskeletal abilities, bladder and bowel control, and so on. When the shock resolves, some function may return. "Some complications, such as low blood pressure, will resolve within a few weeks." Complications related to spinal shock, such as inability to maintain blood pressure, control temperature, and manage heart rate should resolve when the spinal shock resolves, and therefore this information can be relayed to the parents.

A concerned mother brings her 11-year-old in because of repeated nosebleeds. The nurse notes pallor and tachycardia. The child has a platelet count of 45,000/mm3 and a granulocyte count of 750 mm3. After ensuring the child is stable, which questions would be important for the nurse to ask? Select all that apply. "How much water does your child drink on a typical day?" "What new medications has your child recently begun taking?" "Do you have a family history of sickle cell anemia or hemophilia?" "Has your child been exposed to any chemicals recently?" "What other symptoms, such as fever or rashes, have you noticed recently?"

"What new medications has your child recently begun taking?" Acquired aplastic anemia can stem from new medications. The nurse would question the mother about any new medications prescribed for the child. "Has your child been exposed to any chemicals recently?" Acquired aplastic anemia can stem from exposure to chemicals or substances in the environment. The nurse would question the mother about any recent exposures, which could indicate a cause. "What other symptoms, such as fever or rashes, have you noticed recently?" Patients with aplastic anemia often experience frequent illness. The nurse would question the mother about recent illnesses, fever, or rashes that could suggest the presence of leukopenia.

A nurse is caring for a child with a chronic illness who was recently hospitalized due to complications of the illness. After leaving the patient's room, the nurse notices a sibling standing outside the room looking very angry. The sibling says, "Today is my birthday and nobody even cares." Which response from the nurse is best? "You sound upset. Tell me more about how you are feeling." "Happy Birthday! Let's find you something to celebrate with." "Your parents care about you; let's try to be positive in this situation." "Let's go see your brother since he is sick, and then we can celebrate your birthday."

"You sound upset. Tell me more about how you are feeling." This statement acknowledges the feelings of the sibling and allows the opportunity for discussion of those feelings.

The nurse is caring for a patient with cystic fibrosis (CF) and observes a distended abdomen, fever, and weight loss. The parents ask the nurse what is wrong with their infant. Which statement does the nurse use to address the question? "Your baby is constipated and needs to have a bowel movement." "Your baby's body is not receiving enough calories to allow for weight gain." "Your baby's body may make mucus that is too thick so the intestines become blocked." "Your baby demonstrates signs of a bowel infection and requires intravenous fluids and medicines."

"Your baby's body may make mucus that is too thick so the intestines become blocked." Cystic fibrosis may present as failure to thrive or chronic respiratory infections. Symptoms of a bowel obstruction are present due to plugging of the intestine by thick, tenacious secretions and stool.

Nurses can advocate for methods that foster opportunities for parent empowerment. Through ongoing contact with the child, the nurse (

(1) observes the child's reactions to chronic illness or disability, ability to function, and adaptive behaviors within the environment and with significant others, (2) explores the child's own understanding of his or her illness, and (3) provides support while the child learns to cope with his or her feelings

Torticollis

(wry neck), which can be either congenital or acquired, is a condition of limited neck motion in which the neck is flexed and the head is drawn or tilted laterally to the affected side while the chin is pointed toward the opposite side. It is a manifestation rather than a disease entity and may be associated with a number of conditions, including a congenital abnormality of the cervical spine or a traumatic lesion of the sternocleidomastoid muscle. Congenital muscular torticollis may occur as a result of abnormal positioning in utero, which causes contracture of the sternocleidomastoid muscle. Infants with positional plagiocephaly may have muscular torticollis as well, which can be successfully treated with neck stretching exercises.

Traction use

*Not used very often (complex or severe injury) but when it is, the purposes: - Realign bone fragments to help them fuse properly - Rest for an extremity - Prevent/improve contracture - Correct joint deformity - Treat dislocation - Allow pre-op or post-op positioning and alignment -Immobilization of specific areas of the body -Reduce muscle spasms (uncommon in kids)

External fixation

- A device that lengthens bones, corrects angular or rotational deformities, and immobilizes fractures - Uses wires, rings, and telescoping rods that are surgically placed - Result: Up to 15 cm (5.9 in) - Allows for partial weight bearing and a need for crutches - After removal, a cast is placed for 4-6 weeks to prevent fracture - Involve children in their care = less discomfort - Prepare family for reactions from others

Juvenile idiopathic arthritis

- CHRONIC childhood arthritis, cause unknown, onset before 16 years, 90% have negative rheumatoid factor, complication: uveitis (inflammation of the iris) - Synovial inflammation > joint effusion > erosion and destruction of the joint cartilage - Sx: swelling, warm, tender, limited motion, no redness (that means infection), morning stiffness, limp diagnosis of exclusion, no definite test no cure - Goal: Control pain, preserve ROM, minimize inflammation, promote normal g/d, eye care to prevent vision loss - Meds: NSAIDs (check GI, renal, hepatic functions often), disease modifying anti rheumatic drugs to reduce inflammation, glucocorticoids to reduce inflammation, no opiates, iron supplement Naproxene causes skin fragility - wear sunscreen and report skin lesions - PT is great. Activity modifications may be needed but should keep exercising and being active so they don't lose muscle - 8-10 hours sleep at night, no daytime naps - Go to school even if having joint pain - Moist heat is helpful = baths, wet warm towels, swimming - Promote independence

Sudden death

- Cardiac arrest - More males than females, females survive more than males - Risk factors: certain sports (involving collision and frequent body contact), underlying medical problem (cardiac abnormalities), the environment especially idiopathic hypertrophic cardiomyopathy - severe chest pain, dizziness, dyspnea 2. congenital coronary artery malformation 3. valvular heart disease, atherosclerosis coronary artery disease, dialated cardiomyopathy, marfan syndrome, myocarditis, commotio cordis ( blunt blow causing ventricular fibrilation) environmental causes - playing conditions, clothing, equipment, rules used, outdoor temp

What are the most frequent causes for immobilization?

- Congenital defects (spina bifida) - Neuromuscular (muscular dystrophy, spinal muscular atrophy) - Prolonged mechanical ventilation and sedation - Infection and skin injuries (burns) - Musculoskeletal (fractures, osteomyelitis) - Neuro (spinal cord injury, Guillain-Barre, TBI, coma)

Contusions

- Damage to soft tissue, subq structures, and muscle - Tearing > hemorrhage > edema > pain and muscle spasm - Look black and blue (ecchymosis) - Myositis ossificans (the formation of bone tissue INSDIE muscle tissue R/T trauma to the area) from deep contusions to the biceps or quads and can cause a restriction of flexibility - Tx: Ice and rest until strength and ROM is equal to the unaffected side

Clubfoot aka talipes equinovarus

- Deformity of the ankle and foot > forefoot adduction, mid foot supination, hind foot cares, ankle equinus - A bone deformity + soft tissue contracture - Strong family tendency > 1 in 10 chance that an affected parent will have an affected kid - Dx: Observation/assessment > affected foot is usually smaller, shorter, with calf atrophy - Tx: Ponseti method = serial casting, manipulation, stretching, percutaneous heel cord tenotomy > long-leg cast at the very end for 3 w > Denis Browne bar with Ponseti sandals treatment 3 stages: 1. correction of the deformity 2. maintenance of the correction until normal muscle balance is regained 3. follow-up observation to avert possible recurence of the deformity nursing care - like of child with cast - skin, circulation, teaching parents

What makes school-age children and adolescents prone to injury?

- Difficulty controlling movement leads to physical injury - Trying to keep up with older kids and siblings by engaging in activities beyond their physical capabilities - Vulnerable to a dare > risk taking

Most common fracture locations in kids

- Distal forearm = radius, ulna - Clavicle *Fractures are more common in kids vs adults because kids' bones are more pliable and porous, but they heal faster. The younger the kid, the faster the healing process

Osteogenesis imperfecta

- Excessive fractures and bone deformity, with growth restriction - Other features: Deformity, fragility, blue sclerae, hearing loss, discolored teeth - A GENETIC disorder - NOT ACQUIRED - Focus of tx: decreasing number of fractures, improving bone metabolism, optimizing function via braces, rods, splints, PT - Handle with care! Even changing a diaper may cause a fracture > do not lift by ankles. Instead, support buttocks upwards or use pillows - Use a manual BP cuff, careful repositioning of the head to prevent flattening surgery to correct deformities for recurrent fractures - intramedullary rod reposition head to avoid flattening skin care is essential oral dental care - careful, fragile teeth teach parents about suitable activity bisphosphonate (-nate) therapy for increasing vertebral bone density - observe respir. infection, give immunizations, but no live virus vaccines

How does immobilization affect a chid?

- It is natural for children to move around. Their growth and development are influenced by mobility. Therefore, they are affected physically, socially, psychologically, and emotionally. The entire family is affected. major consequences of immobilization: - Decreased muscle strength/mass (atrophy), decreased metabolism, bone demineralization (osteoporosis), loss of joint mobility/contracture > passive and active ROM within 3-7 days

Helping the child get through cast application

- Kids don't realize that their limbs still exist beneath the cast, so saying things like "this will help your arm get better" don't work because they don't understand cause and effect - Explain that it will feel warm at first but not hot - Use a doll or stuffed animal to explain the procedure - Distract with bubbles and talking about pets

Idiopathic scoliosis

- Lateral curvature, spinal rotation causing rib asymmetry, and hypokyphosis classified according to age onset - congenital, infintile(birth to 3yo), juvinile (3-10yo), adolescent - Curves of less than 25 degrees do not need tx - Dx: xray - Bracing for moderate 25-45 degrees > goal: slow progression of curvature to allow skeletal growth and maturity > not totally effective - Surgery for severe > 45 degrees - Nursing care: Support the child's body image and mental health - Postop care: Log roll, neuro checks, walking on day 2 or 3

Metatarsus adducts aka pigeon-toed

- Medial adduction of the toes and forefoot. The feet might be kidney shaped too. - Angle occurs at the tarsometatarsal joint while the heel and ankle stay neutral type 1 - forefoot is flexible and corrects easily into abduction with manipulation type 2 - forefoot is only partially flexible and corrects only to neutral position with active manipulation type 3 - forefoot is rigid and will not stretch to a neutral position with manipulation - R/T abnormal intrauterine positioning management depends on the flexibility of the deformity mostly will correct spontaneously over 4 years - Stage 1 and 2 tx with stretching and manipulation, stage 3 tx with serial manipulation, casting, and corrective shoes surgery only for kids over 6yo with pain ambulation nursing care - teaches parents to stretch the forefoot, or cast care

Immobility effects on mental health

- Removal of power to be active deprives them of necessary input and a natural outlet for their feelings and fantasies - Effects on body image - Sensory deprivation leads to feelings of isolation, boredom, and being forgotten - Feeling dependent, depression, anxiety - React with protest, anger, aggression, quiet, passive, submissive - Some think its a form of punishment for misbehavior

Signs of steroid use

- Severe acne, a sudden increase in strength/muscle mass, a sudden decrease in body fat, male pattern baldness, water retention, deepening voice in females - male pattern hair growth and ∂eepenign voice - MANY SIDE EFFECTS that are dangerous/unhealthy 1. virilization in felames 2. oligospermia 3. prostatic hypertrophy 4. miocardiac infaction, stroke 5. testicular atrophy, infertility, gynecomastia 6. increased cholesterol, hypertension, hepatocellular carçinoma, mood çhanges.aggresiveness, chnges in libido, depression, anxiety, psychosis

What do you do if spinal cord injury is suspected or a possibility?

- Spinal cord injury is suspected if the pt has head, trunk, or multi system trauma - Treat the pt as if the injury is present until diagnostic tests are done to rule out > MRI, CT, radiography - Calm the child, reassure them, and tell them not to move. Do not move them until they are stabilized. Cervical spine is stabilized via a rigid cervical collar. Body is placed on a rigid immobilization board. Infants are not removed from car seats.

Sprain vs strain

- Sprain: Ligament stretch or tear Grade 1: Mild, over-stretching or micro tear, no hemorrhage or increased instability, swelling later Grade 2: Moderate, partial tearing with some ligamentous continuity remaining, immediate pain and swelling Grade 3: Severe, total loss of ligamentous continuity, pain, swelling is minimal because hemorrhage extravasates into soft tissues symptoms - something comes apart, snap, pain, rapid diffused swelling, reluctance to use affected extremity - Strain: Musculotendinous micro tear painful to touch, swallen same grading, but no laxity - Tx: RICE - rest, ice, compression, elevation

Taking off the cast

- The blade makes a loud noise and uses vibration to cut the cast. Littles might be terrified of the noise and think their arm is going to come off with the cast. Reassure them that everything is going great and accept their reaction, because they think of cast as part of them - The limb will be covered with desquamated skin and sebaceous secretions. Soaking in water will remove this but it might take a few days. Warn from scrubbing which can cause excoriation and bleeding. reassure child that resuming ecercises will return function and appearaance

Slipped capital femoral epiphysis

- The proximal femoral epiphysis spontaneously displaces in a posterior and inferior direction - Happens before or during accelerated growth and onset of puberty - Risk factor: obesity - Sx: Limp, pain in the groin/hip/thigh/knee often lies stiff with the lower extremity flexed, abducted and externally rotated range of motion restriction of flexion and extension - Goal: To prevent further slipping - Tx: 911 > non weight bearing, surgery asap nursing care - preparing child and family for surgery, postoperative hemodynamic stabilization, pain management, assessment of complication, teach prper use crutches, avoiding weght bearing activity

Types of casts

- Upper extremity cast to immobilize the wrist or elbow - Lower extremity cast to immobilize the ankle or knee - Spica cat to immobilize the hip and knee

Immobility effects on the renal system

- Urinary system relies on gravity > supine leads to urinary stasis > calculi formation or infection - Can't handle the increased metabolites from protein breakdown and bone demineralization > renal calculi > clue: hematuria difficulty relaxing the perenial muscles and external spchincter to initiate voiding

Osteomyelitis

- Usually seen in 10 years or younger - C.A.: S. aureus, group B strep in neonates, Salmonella in sickle cell anemia, Neisseria gonorrhoeae in sexually active - Bone infection > most common bones to become infected are foot, femur, tibia, pelvis - Process: bacteria > inflammation > edema > vascular congestion > small-vessel thrombosis > bone destruction > abscess formation > dead bone (sequestra) - In infants: blood vessels cross the growth plate into the epiphysis and joint - In children: infection is contained by the growth plate and joint infections is less likely - Sx: Severe pain, fever, irritability, tenderness, flexing/guarding, maybe inflammation - Dx: Culture of bone, pus, blood, joint fluid, infected skin, MRI, CT - IV antibiotics for at least 4 weeks; shorter duration of IV followed by oral sometimes - Support the limb to prevent movement, pain meds, sedation - As infection subsides, PT to ensure restoration of optimum function important to monitor hematologic, renal, hepatic, ototoxic and other side effects

Hydrocephalus early infancy

-Abnormally rapid head growth -Bulging fontanels (tense and non-pulsatile) -Dilated scalp veins -Separated sutures -Macewen sign (cracked pot) -Thinning of skull bones Hydrocephalus Severe Cases in infants -Frontal enlargement/bossing -Depressed eyes -Setting sun sign (downward rotation of eyes) -Sluggish pupils with unequal response to light Hydrocephalus - clinical manifestations: infancy (general) -Irritability -Lethargy -Seizure activity -Infant cries when held or rocked, quiets when allowed to lay still -Not meeting developmental milestones -Changes in LOC -Spasticity in lower extremities -Vomiting -Difficulty sucking and feeding -Shrill, high pitch cry -Cardiopulmonary arrest Hydrocephalus - clinical manifestations: -Headache on awakening, improves after emesis or taking a position of upright posture -Papilledema -Strabismus -Ataxia -Irritability -Lethargy -Apathy -Confusion -Incoherence -Vomiting

Hydrocephalus

-Imbalance in the production and absorption of CSF in the ventricular system. -With accumulation of CSF, ventricles dilate and compress brain tissue imbalance in the production and absorption of CSF in the ventricular system -Impaired absorption of CSF (communicating hydrocephalus) -Obstruction to the flow of CSF (non-communicating hydrocephalus) Causes of hydrocephalus -developmental defects -neoplasms -infections -trauma hemorrhage

Amputation nursing care

-bandaging using a figure 8 to decrease edema, control hemorrhage, aid in developing contour - The stump can be elevated for the first 24 hours but not longer than that because contractures may develop in the proximal joint - Turn the patient on all four sides, eventually remaining prone for up to an hour - Arm exercises to build up arm strength in prep of using crutches; use trapeze, full range of motion exercises above amputation (active and isotonic) - Skin checks (skin irritation, breakdown, infection) and skin care every day (unscented soap and water are good, powder is fine, alcohol is bad) and after the prosthesis is removed - Phantom pain is real R/T nerve- brain connections that remain present for a while > treat with analgesics

surgical intervention for scoliosis

-more than 45 degrees -the one that doesnt respond to conservative bracing - neuromascular, dysplastic, congenital curves surgical techniques: 1. realignment and straightening of the spine with internal fixation combined with bony fusion 2. vertebral body stapling 3. anterior tetheringxx

Therapeutic management hydrocephalus

-relief of ventricular pressure - treatment of the cause ofthe ventriculomegaly - treatment of associated complication - management of problems realted to the effect of the disorder on psychomotor development treatment mostly surgical - removal of obstruction, shunt procedure to provide primary drainage of the CSF from the ventricles to an extracranial compartment ( peritoneum) tablets and magnetic toys interfere with shunt programming, keep it at leats 2.5 cm away

The nurse is reviewing a health care provider's prescriptions for a child with sickle cell anemia who was admitted to the hospital for the treatment of vaso-occlusive crisis. Which prescriptions documented in the child's record should the nurse question? Select all that apply. 1. Restrict fluid intake 2. Position for comfort 3. Avoid strain on painful joints 4. Apply nasal oxygen at 2 L/min 5. Provide a high-calorie, high-protein diet 6. Give meperidine (Demerol) 25 mg intravenously every 4 hours for pain

1, 6 Sickle cell anemia is one of a group of diseases termed hemoglobinopathies, in which hemoglobin A is partly or completely replaced by abnormal sickle hemoglobin S. It is caused by the inheritance of a gene for a structurally abnormal portion of the hemoglobin chain. Hemoglobin S is sensitive to changes in the oxygen content of the RBC; insufficient oxygen causes the cells to assume a sickle shape, and the cells become rigid and clumped together, obstructing capillary blood flow. Oral and intravenous fluids are an important part of treatment. Meperidine (Demerol) is not recommended for a child with sickle cell disease because of the risk for normeperidine-induced seizures. Normeperidine, a metabolite of meperidine, is a central nervous system stimulant that produces anxiety, tremors, myoclonus, and generalized seizures when it accumulates with repetitive dosing. The nurse would question the prescription for restricted fluids and meperidine for pain control. Positioning for comfort, avoiding strain on painful joints, oxygen, and a high-calorie and high-protein diet are also important parts of the treatment plan.

Hemophilia Assessment

1. Abnormal bleeding in response to trauma or surgery (sometimes is detected after circumcision) 2. Epistaxis (nosebleeds) 3. Joint bleeding causing pain, tenderness, swelling, and limited range of motion 4. Tendency to bruise easily 5. results of tests that measure platelet function are normal; results of tests that measure clotting factor may be abnormal.

Emergency tx: Trauma

1. Assess environment > any potential threats to rescuers and bystanders? 2. Observe the scene to see what caused the injury > this helps determine course of action in treating the child 3. Only move the child if they are in danger or potential for further injuries. If you do have to move them, stabilize the cervical spine to avoid more injury 4. Assess LOC > alert? responsive to verbal still? responsive to painful stimuli? unresponsive to all stimuli? 5. Open airway > if their is suspected spinal injury, use the modified jaw thrust 6. Activate EMS 7. CPR if no response/pulse - Compress at 100/min - 30 compressions:2 breaths (give each breath over 1 second) 8. Turn AED on but avoid interrupting compressions for more than 10 seconds 9. After 5 cycles, check pulses - Carotid artery in children 1 and older - Brachial artery in infants 10. If pulse present, assess breathing and begin rescue breathing 11. Control bleeding > pressure to wound site and appropriate arterial pressure point, elevation, tourniquet as last resort because you apply it you can't remove it!!! 12. Do not remove objects protruding from body 13. Assess pain, pulses distal to injury, maintain body heat, identify child

cerebrospinal fluid analysis/ viral meningitis

WBC - slightly elevated, increased lymphocytes protein content - normal or slightly increased glucose content - normal gram stain; bacteria culture - negative color - clear or slightly cloudy opening pressure - normal

Hemophilia Description

1. Hemophilia refers to a group of bleeding disorders resulting from a deficiency of specific coagulation proteins. 2. Identifying the specific coagulation deficiency is important so that definitive treatment with the specific replacement agent can be implemented; aggressive replacement therapy is initiated to prevent the chronic crippling effects from joint bleeding. 3. The most common types are factor VIII deficiency (hemophilia A or classic hemophilia) and factor IX deficiency (Hemophilia B or christmas disease) 4. Hemophilia is transmitted as an X-linked recessive disorder (it may also occur as a result of a gene mutation) 5. carrier females pass on to the defect to affected males; female offspring are rarely born with the disorder, but may be if they inherit an affected gene from their mother and are offspring of a father with hemophilia. 6. The primary treatment is replacement of the missing clotting factor; additional medications, such as agents to relieve pain or corticosteroids may be prescribed depending on the source of the bleeding from the disorder.

spastic CP

1. Spastic - the most common type when patients have increased muscle tone, stiff muscles and awkward movements. It might affect various body parts: a. Spastic diplegia/diparesis stiffness of legs). Patients have difficulty walking, because legs might turn inwards and cross at the knees (scissoring). all extremities affected; lower more than upper b. Spastic hemiplegia/hemiparesis - affects only one side of the body and arms most affected more c. Spastic quadriplegia/quadriparesis - affects all four limbs, the trunk, and the face. Patients cannot walk, might have intellectual disabilities seizures, problems with vision, speech, hearing. d. tetraplegia - all 4 extremities involved: legs and trunks, mouth, pharynx, and tongue e. triplegia - three limbs involved f. monoplegia - only one limb involved persisten primitive reflexes, positive babinski reflex, ankle clonus, exxagerated stretch reflexes, eventual development of contractures

von Willebrand interventions

1. Treatment and care are similar to measures implemented for hemophilia, including administration of clotting factors. 2. Provide emotional support to the child and parents, especially if the child is experiencing an episode of bleeding

cerebral palsy assessment

1. extreme irritability and crying 2. feeding difficulties 3. abnormal motor performance 4. alterations of muscle tone; stiff and rigid arms or legs 5. delayed developmental milestones 6. persistence of primitive reflexes (moro, tonic neck) after 6 months (most primitive reflexes disappear by 3 to 4 months of age) 7. abnormal posturing such as opisthotonos (exaggerated arching of back) 8. seizures may occur.

abormal signs requiring immediate follow up of a headache

1. headache progresses in fequency and severity over a brief period (2-3 weeks) 2. awakens the child from sleep 3. occurs in early morning 4. worse on arising 5. characterized by persistent, occipital or frontal pain 6. accompanied by unexplained vomiting 7. associated with a chnage in gait, personality or behavior 8. exacerbated by valsava maneur ( intesified by lowering head and straining, like bowel movement)

skin care AFO, KAFO, HKAFO

1. if child has decreased sensation - check condition checke more often than Q4H 2. if the child complains of burning sensation under braces - remove brace and observe for redness, if more than once contact HCP 3. if small blister appears, cover with a sterile bandage, do not put alcohol. avoid wearing device until skin heals if no sign of healing after 3 days, contact HCP 4. lotions and creams use only if the skin dryes - clean braces with soap and water and dry throughrougly - check screws and fasteners if brace broken or out of alignment - contact orthotist

hydrocephalus assessment

1. infant a. increased head circumference b. thin, widely separated bones of the head that produce a cracked-pot sound upon percussion (macewens sign) c. anterior fontanel tense, bulging, and non pulsating; sutures will separate prior to fontanel bulging d. dilated scalp veins e. frontal bossing f. setting sun eyes 2. child a. behavior changes such as irritability and lethargy b. headache on awakening c. nausea and vomiting d. ataxia e. nystagmus 3. late signs: high, shrill cry and seizures

categories of events, leading to cognitive impairment

1. infection and intoxication ( rubella, syphilis, maternal drug consumption, chronic lead ingection, kernicterus 2. trauma or physical agent ( injury to the brain, experienced during prenatal, perinatal, postnatal period) 3. inadequate nutrition and metabolic disorders (PKU or congenital hypothyrodism) 4. gross postnatal brain disease, such as neurofibromatosis and tuberus sclerosis) 5. unknown prenatal influence, including cerebral and cranial malformation, such as microcephaly and hydrocephalus 6 chromosomal abnormalities resulting from radiation, viruses, chemical, parental age, genetic mutation ( down syndrom, FXS) 7. gestational disorders ( prematurity, low birth weight, postmaturity) 8. psychiatric disorders that have onset during the child's developmental period up to age 16 yo, such as autism spectrum disorders 9. environmental influences, including evidence of a deprived environment associated with a history of intellectual disability among parents and siblings

cast care

1. keep the casted part elevated on pillows 2. expose the plaster cast to air until fry 3. a wet plaster should be lifted and supported with the palms to avoid indenting and creating pressure points 4. observe fingers or toes for any evidence of swelling or discoloration and contact HCP if noted 5. check movement and sensation of fingers and toes frequently and contact regarding any chages 6. encourage frequent rest for a few first days 7. fo no allow the affected limb to hang in a dependent position for longer than 30 min 8. elevate injured leg when the child is sitting and avoid standing for too long 9. do not allow the child to pu anything inside the cast, so keep small items away 10. examine skin at the cast edges to detect breakdow, pad cast 11. avoid placing cast in water 12. if child is incontinent protect the cast with plastic

cerebral palsy interventions

1. the goal of management is early recognition and interventions to maximize child's abilities. 2. an interprofessional team approach is implemented to meet the many needs of the child. 3. therapeutic management includes physical therapy, occupational therapy, speech therapy, education, and recreation. 4. assess the child's developmental level and intelligence. 5. encourage early intervention and participation in school programs. 6. prepare for using mobilizing devices to help prevent or reduce deformities. 7. encourage communication and interaction with the child on his or her developmental level, rather than chronological age level 8. provide a safe environment by removing sharp objects, using a protective helmet if the child falls frequently, and implementing seizure precautions if necessary. 9. provide safe, appropriate toys for the child's age and developmental level. 10. position the child upright after meals. 11. medications may be prescribed to relieve muscle spasms which cause intense pain; antiseizure medications may also be prescribed. 12. provide the parents with information about the disorder and treatment plan; encourage support groups for parents.

von Willebrand's disease description

1. von Willebrand's disease is a hereditary bleeding disorder that is characterized by a deficiency of or a defect in a protein termed von Willebrand factor. 2. The disorder causes platelelts to adhere to damaged endothelium; the von Willebrand factor protein also serves as a carrier protein for factor VIII 3. It is characterized by an increased tendency to bleed from mucous membranes.

Which score is appropriate for a Glasgow Coma Scale (GCS) documented as a moderate head injury? 8 11 13 15

11 Rationale If the score of GCS is 9 to 12, then the nurse documents the head injury as a moderate one. Therefore, 11 is the correct answer. If the score of GCS is 8 or less, then the nurse documents the head injury as a severe one. If the score of GCS is 13 to 15, then the nurse documents the head injury as a mild one.

A 10 y/o child with hemophilia A has slipped on the ice and bumped his knee. The nurse should prepare to administer which prescription? 1. Injection of factor X 2. Intravenous infusion of iron 3. Intravenous infusion of factor VIII 4. Intramuscular injection of iron using the Z-track method

3 Hemophilia refers to a group of bleeding disorders resulting from a deficiency of specific coagulation proteins. The primary treatment is replacement of the missing clotting factor; additional medications, such as agents to relieve pain, may be prescribed depending on the source of bleeding from the disprder. A child with hemophilia A is at risk for joint bleeding after a fall. Factor VIII would be prescribed intravenously to replace the missing clotting factor and minimize the bleeding. Factor X and iron are not used to treat children with hemophilia A.

The parents of a school-aged child newly diagnosed with asthma are receiving discharge education. What information should the nurse emphasize to address the need for follow-up appointments? Asthma is an illness that is unique because it requires close monitoring. The child with asthma will receive routine follow-up care during the annual check-up. The illness trajectory of asthma indicates that the child will need to follow-up more than other chronic illnesses usually require. A child with a chronic illness needs regular care monitoring and follow-up and should work to develop a positive relationship with the health care team.

A child with a chronic illness needs regular care monitoring and follow-up and should work to develop a positive relationship with the health care team. The need for regular assessment, monitoring, and follow-up is much greater in children with chronic illnesses than it is for the general population of children. Detecting issues in this population early is important to decrease long-term issues.

Hemophilia

A group of hereditary bleeding disorders that result from deficiencies of specific clotting factors Bleeding no NSAIDS no aspirin ever Elevation to help prevent the bleeding Vasoconstrict is ice it Kidney trauma if they get hit in the back

The nurse assesses vital signs for the 1-year-old patient with a severe head trauma. Which findings are most concerning? An oxygen saturation of 90% A temperature of 99.9 Fahrenheit A heart rate of 52 beats per minute Cheyne-Stokes respiratory pattern Blood pressure of 210/132 mm Hg

A heart rate of 52 beats per minute Bradycardia is a manifestation of Cushing's response, which can indicate an impending brain herniation. Correct Cheyne-Stokes respiratory pattern A change in the respiratory rate and pattern is a manifestation of Cushing's response, which can indicate an impending brain herniation. Correct Blood pressure of 210/132 mm Hg Increased systolic blood pressure and a widening pulse pressure is a manifestation of Cushing's response, which can indicate an impending brain herniation.

von Willebrand Disease (vWD)

A hereditary bleeding disorder involving deficiency of von Willebrand factor (a plasma protein and the carrier for factor VIII) von Willebrand factor needed for platelet adhesion Transmitted as autosomal dominant trait Occurs in both males and females The gene for the disease is located on chromosome 12 prolonged bleeding time because platelets fail to adhere to the walls of the raptured vessel to form a platelet plug Factor 8

At a followup appointment to assess the healing of a fracture, the nurse notes a bump at the fracture site. The nurse informs the child's father that the bump is a callus that serves which purpose? Use of the injured part Means for adequate blood supply Sufficient support for weight bearing A means of holding bone fragments together

A means of holding bone fragments together Rationale New bone cells are formed in large numbers and stimulated to maximum activity. They are found at the site of the injury. In time, calcium salts are absorbed to form the callus. Functional use cannot occur until the fracture site is stable. Sufficient support for weight bearing is not possible until the fracture site is stable. The callus does not provide an adequate blood supply.

developing successful parent-professional partnership

promote primary nursing; in nonhospital settings 2. acknowledge parents' overall competence and their unique expertise with their child 3. respect parents time 4. explain or define any medical, technical, or discipline-specific terms 5. tell families I am not sure, I don't know when appropriate 6. fascilitate family's effectiveness in team meetings

Major consequences of immobility in the pediatric patient include which one of the following? A. Bone demineralization leading to osteoporosis B. Orthostatic hypotension C. Dependent edema in the lower extremities D. Decrease in the metabolic rate

A. Bone demineralization leading to osteoporosis

bone healing stage 2/ cellular proliferation

AFTER 24 HRS - blood supply increases, bringing available calcium, phosphate and fibroblasts - cells proliferate at ends of bone fragments and differentiate into cartilage and connective tissue NEXT FEW DAYS - hematoma becomes granulaton tissue, which develops into a framework for bone forming substances - fibroblasts convert to osteoblasts 2-3 DAYS - halisteresis ( softening of bone ends) occurs for 1/8-1/4 inch; bone cells are resorbed

A health care provider prescribes morphine sulfate, 0.2 mg/kg IV every 2 to 4 h as needed for pain for a child with a terminal illness. The child weighs 10 kg. The medication label states: "Morphine sulfate 5 mg/ml." The nurse prepares to administer one dose. How many milliliters will the nurse prepare to administer one dose? Fill in the blank. Record your answer using one decimal place.

ANS: 0.4 Follow the formula for dosage calculation. Multiply 0.2 mg ´ 10 kg to get the dose = 2 mg Desired ———- ´ Volume = ml per dose Available 2 mg ———- ´ 1 ml = 0.4 ml 10 mg

A health care provider prescribes morphine sulfate (Roxanol), 10 mg PO every 4 h as needed for pain for a child with a terminal illness. The medication label states: "Morphine sulfate (Roxanol) 20 mg/1 ml." The nurse prepares to administer one dose. How many milliliters will the nurse prepare to administer one dose? Fill in the blank. Record your answer using one decimal place.

ANS: 0.5 Follow the formula for dosage calculation. Desired ———- ´ Volume = ml per dose Available 10 mg ———- v 1 ml = 0.5 ml 20 mg

A health care provider prescribes OxyContin (oxycodone), 3 mg PO every 4 to 6 h as needed for pain for a child with a terminal illness. The medication label states: "OxyContin 5 mg/1 ml." The nurse prepares to administer one dose. How many milliliters will the nurse prepare to administer one dose? Fill in the blank. Record your answer using one decimal place.

ANS: 0.6 Follow the formula for dosage calculation. Desired ———- ´ Volume = ml per dose Available 3 mg ———- ´ 1 ml = 0.6 ml 5 mg

The health care provider has prescribed neostigmine (Prostigmin) 0.04 mg/kg/per dose SC q 4 to 6 hrs PRN for a child with myasthenia gravis. The child weighs 77 lb. The nurse is preparing to administer a dose. Calculate the dose the nurse should administer in milligrams. Record your answer below using one decimal place.

ANS: 1.4 The correct calculation is: 77 lb/2.2 kg = 35 kg Dose of Prostigmin is 0.04 mg/kg/dose 0.04 mg ´ 35 = 1.4 mg

The health care provider has prescribed carbamazepine (Tegretol) 20 mg/kg/day divided bid for a child with cerebral palsy having seizures. The child weighs 44 lb. The nurse is preparing to administer the 0900 dose. Calculate the dose the nurse should administer in milligrams. Record your answer below in a whole number.

ANS: 200 The correct calculation is: 44 lb/2.2 kg = 20 kg Dose of Tegretol is 20 mg/kg/day divided bid 20 mg ´ 20 = 400 mg 400 mg/2 = 200 mg

The health care provider has prescribed gabapentin (Neurontin) 30 mg/kg/day divided q 8 hours for a child with cerebral palsy having seizures. The child weighs 110 lb. The nurse is preparing to administer the 1200 dose. Calculate the dose the nurse should administer in milligrams. Record your answer below in a whole number.

ANS: 500 The correct calculation is: 110 lb/2.2 kg = 50 kg Dose of Neurontin is 30 mg/kg/day divided every 8 hours 30 mg ´ 50 = 1500 mg/day 1500 mg/3 = 500 mg for one dose

The family and child have decided that hospice care best meets their needs during the terminal phase of illness. The nurse recognizes that the parents understand the principles of this care when they make which statement? a. "It will be good to be at home and care for our child." b. "What a relief it will be not to need any more medicines." c. "We are going to miss the support of the hospice team when our child dies." d. "We know that once hospice care starts, we will not be able to return to the hospital if the care is difficult."

ANS: A A major principle of hospice care is that the family members are the principal caregivers and are supported by a team of professionals. Pain and symptom management is a priority. The family and visiting nurses administer medications to keep the child as pain and symptom free as possible. The hospice team provides bereavement support to help the family in the postdeath adjustment. This may last for up to a year or more. If the family decides they can no longer care for the child at home, readmission to a freestanding hospice or hospital is possible.

A 17-year-old patient is returning to the surgical unit after Luque instrumentation for scoliosis repair. In addition to the usual postoperative care, what additional intervention will be needed? a. Position changes are made by log rolling. b. Assistance is needed to use the bathroom. c. The head of the bed is elevated to minimize spinal headache. d. Passive range of motion is instituted to prevent neurologic injury.

ANS: A After scoliosis repair using a Luque procedure, the adolescent is turned by log rolling to prevent damage to the fusion and instrumentation. The patient is kept flat in bed for the first 12 hours and is not ambulatory until the second or third postoperative day. A urinary catheter is placed. The head of the bed is not elevated until the second postoperative day. Range of motion exercises are begun on the second postoperative day.

A 5-year-old child is admitted to the hospital in a sickle cell crisis. The child has been alert and oriented but in severe pain. The nurse notes that the child is complaining of a headache and is having unilateral hemiplegia. What action should the nurse implement? a. Notify the health care provider. b. Place the child on bed rest. c. Administer a dose of hydrocodone (Vicodin). d. Start O2 per the hospital's protocol.

ANS: A Any number of neurologic symptoms can indicate a minor cerebral insult, such as headache, aphasia, weakness, convulsions, visual disturbances, or unilateral hemiplegia. Loss of vision is usually the result of progressive retinopathy and retinal detachment. The nurse should notify the health care provider.

In which condition are all the formed elements of the blood simultaneously depressed? a. Aplastic anemia b. Sickle cell anemia c. Thalassemia major d. Iron deficiency anemia

ANS: A Aplastic anemia refers to a bone marrow failure condition in which the formed elements of the blood are simultaneously depressed. Sickle cell anemia is a hemoglobinopathy in which normal adult hemoglobin is partly or completely replaced by abnormal sickled hemoglobin. Thalassemia major is a group of blood disorders characterized by deficiency in the production rate of specific hemoglobin chains. Iron-deficiency anemia results in a decreased amount of circulating red cells.

The nurse is talking to the parent of a child with special needs. The parent has expressed worry about how to support the siblings at home. What suggestion is appropriate for the nurse to give to the parent? a. "You should help the siblings see the similarities and differences between themselves and your child with special needs." b. "You should explain that your child with special needs should be included in all activities that the siblings participate in even if they are reluctant." c. "You should give the siblings many caregiving tasks for your child with special needs so the siblings feel involved." d. "You should intervene when there are differences between your child with special needs and the siblings."

ANS: A Appropriate information to give to a parent who wants to support the siblings of a child with special needs includes helping the siblings see the differences and similarities between themselves and the child with special needs to promote an understanding environment. The parent should be encouraged to allow the siblings to participate in activities that do not always include the child with special needs, to limit caregiving responsibilities, and to allow the children to settle their own differences rather than step in all the time.

The nurse is planning care for an adolescent with acquired immunodeficiency syndrome. What is the priority nursing goal? a. Prevent infection. b. Prevent secondary cancers. c. Identify source of infection. d. Restore immunologic defenses.

ANS: A As a result of the immunocompromise that is associated with human immunodeficiency virus (HIV) infection, the prevention of infection is paramount. Although certain precautions are justified in limiting exposure to infection, these must be balanced with the concern for the child's normal developmental needs. Preventing secondary cancers is not currently possible. Case finding is not a priority nursing goal in planning care for an individual. Current drug therapy is affecting the disease progression; although not a cure, these drugs can suppress viral replication, preventing further deterioration but not actually restoring immunologic defenses.

A 12-year-old child with Guillain-Barré syndrome (GBS) is admitted to the pediatric intensive care unit. She tells you that yesterday her legs were weak and that this morning she was unable to walk. After the nurse determines the current level of paralysis, which should the next priority assessment be? a. Swallowing ability b. Parental involvement c. Level of consciousness d. Antecedent viral infections

ANS: A Assessment of swallowing is essential. Both pharyngeal involvement and respiratory function are usually involved at the same time. The child may require ventilatory support. The inability to swallow also contributes to aspiration pneumonia. Parental involvement is important after the physiologic assessment is complete. The child is answering questions and describing the onset of the illness, which demonstrates she is alert and oriented. Information regarding antecedent viral infections can be obtained after the child is assessed and stabilized.

nursing care of braces

protective clothing under braces to protect from friction and presure assessment of areas i contact every 1-2 hrs for the first few days if area is red, remove braces for 30 min, if still red contact HCP safety is important

A 10-year-old child, without a history of previous seizures, experiences a tonic-clonic seizure at school that lasts more than 5 minutes. Breathing is not impaired. Some postictal confusion occurs. What is the most appropriate initial action by the school nurse? a. Stay with child and have someone else call emergency medical services (EMS). b. Notify the parent and regular practitioner. c. Notify the parent that the child should go home. d. Stay with the child, offering calm reassurance.

ANS: A Because this is the child's first seizure and it lasted more than 5 minutes, EMS should be called to transport the child, and evaluation should be performed as soon as possible. The nurse should stay with the recovering child while someone else notifies EMS.

What condition can result from the bone demineralization associated with immobility? a. Osteoporosis b. Pooling of blood c. Urinary retention d. Susceptibility to infection

ANS: A Bone demineralization leads to a negative calcium balance, osteoporosis, pathologic fractures, extraosseous bone formation, and renal calculi. Pooling of blood is a result of the cardiovascular effects of immobilization. Urinary retention is secondary to the effect of immobilization on the urinary tract. Susceptibility to infection can result from the effects of immobilization on the respiratory and renal systems.

What explanation provides the rationale for why iron-deficiency anemia is common during infancy? a. Cow's milk is a poor source of iron. b. Iron cannot be stored during fetal development. c. Fetal iron stores are depleted by 1 month of age. d. Dietary iron cannot be started until 12 months of age.

ANS: A Children between the ages of 12 and 36 months are at risk for anemia because cow's milk is a major component of their diet, and it is a poor source of iron. Iron is stored during fetal development, but the amount stored depends on maternal iron stores. Fetal iron stores are usually depleted by ages 5 to 6 months. Dietary iron can be introduced by breastfeeding, iron-fortified formula, and cereals during the first 12 months of life.

The nurse is teaching a preschool child with a cognitive impairment how to throw a ball overhand. What teaching strategy should the nurse use for this child? a. Demonstrate how to throw a ball overhand. b. Explain the reason for throwing a ball overhand. c. Show pictures of children throwing balls overhand. d. Explain to the child how to throw the ball overhand.

ANS: A Children with cognitive impairment have a deficit in discrimination, which means that concrete ideas are much easier to learn effectively than abstract ideas. Therefore, demonstration is preferable to verbal explanation, and the nurse should direct learning toward mastering a skill rather than understanding the scientific principles underlying a procedure. Demonstrating how to throw the ball is the best teaching strategy.

A young child's parents call the nurse after their child is bitten by a raccoon in the woods. The nurse's recommendation should be based on what knowledge? a. Antirabies prophylaxis must be initiated immediately. b. The child should be hospitalized for close observation. c. No treatment is necessary if thorough wound cleaning is done. d. Antirabies prophylaxis must be initiated as soon as clinical manifestations appear.

ANS: A Current therapy for a rabid animal bite consists of a thorough cleansing of the wound and passive immunization with human rabies immunoglobulin (HRIG) as soon as possible. Hospitalization is not necessary. The wound cleansing, passive immunization, and immunoglobulin administration can be done as an outpatient. The child needs to receive both HRIG and rabies vaccine.

The parents of a child born with disabilities ask the nurse for advice about discipline. The nurse's response should be based on remembering that discipline is which? a. Essential for the child b. Not needed unless the child's behavior becomes problematic c. Best achieved with punishment for misbehavior d. Too difficult to implement with a special needs child

ANS: A Discipline is essential for the child. It provides boundaries on which she can test out her behavior and teaches her socially acceptable behaviors. The nurse should teach the parents ways to manage the child's behavior before it becomes problematic. Punishment is not effective in managing behavior.

The nurse is preparing a school-age child for computed tomography (CT) scan to assess cerebral function. The nurse should include what statement in preparing the child? a. "The scan will not hurt." b. "Pain medication will be given." c. "You will be able to move once the equipment is in place." d. "Unfortunately no one can remain in the room with you during the test."

ANS: A For CT scans, the child must be immobilized. It is important to emphasize to the child that at no time is the procedure painful. Pain medication is not required; however, sedation is sometimes necessary. The child will not be allowed to move and will be immobilized. Someone is able to remain with the child during the procedure.

What technique facilitates lip reading by a hearing-impaired child? a. Speak at an even rate. b. Avoid using facial expressions. c. Exaggerate pronunciation of words. d. Repeat in exactly the same way if child does not understand.

ANS: A Help the child learn and understand how to read lips by speaking at an even rate. Avoiding using facial expressions, exaggerating pronunciation of words, and repeating in exactly the same way if the child does not understand interfere with the child's understanding of the spoken word.

The parents of a child on a ventilator tell the nurse that their insurance company wants the child to be discharged. They explain that they do not want the child home "under any circumstances." What principle should the nurse consider when working with this family? a. Desire to have the child home is essential to effective home care. b. Parents should not be expected to care for a technology-dependent child. c. Having a technology-dependent child at home is better for both the child and the family. d. Parents are not part of the decision-making process because of the costs of hospitalization.

ANS: A Home care requires the family to manage the child's illness, including providing daily hands-on care, monitoring the child's medical condition, and educating others to care for the child. The child's home environment with the child's family is perceived as the best place for the child to be cared for. If the family does not want to or is not able to assume these responsibilities, other arrangements need to be investigated. The family is an essential part of the decision-making process. Without family involvement and support, the technology-dependent child will not be well cared for at home.

Which snack selection is the best choice for Madison while she is immobilized?

pudding

An 18-month-old child is brought to the emergency department after being found unconscious in the family pool. What does the nurse identify as the primary problem in drowning incidents? a. Hypoxia b. Aspiration c. Hypothermia d. Electrolyte imbalance

ANS: A Hypoxia is the primary problem because it results in global cell damage, with different cells tolerating variable lengths of anoxia. Neurons sustain irreversible damage after 4 to 6 minutes of submersion. Severe neurologic damage occurs from hypoxia in 3 to 6 minutes. Aspiration of fluid does occur, resulting in pulmonary edema, atelectasis, airway spasm, and pneumonitis, which complicate the anoxia. Hypothermia occurs rapidly, except in hot tubs. Electrolyte imbalances do result, but they are not a major cause of morbidity and mortality.

The nurse is doing a neurologic assessment on a 2-month-old infant after a car accident. Moro, tonic neck, and withdrawal reflexes are present. How should the nurse interpret these findings? a. Neurologic health b. Severe brain damage c. Decorticate posturing d. Decerebrate posturing

ANS: A Moro, tonic neck, and withdrawal reflexes are three reflexes that are present in a healthy 2-month-old infant and are expected in this age group.

The nurse is caring for a 10-year-old child who has an acute head injury, has a pediatric Glasgow Coma Scale score of 9, and is unconscious. What intervention should the nurse include in the child's care plan? a. Elevate the head of the bed 15 to 30 degrees with the head maintained in midline. b. Maintain an active, stimulating environment. c. Perform chest percussion and suctioning every 1 to 2 hours. d. Perform active range of motion and nontherapeutic touch every 8 hours.

ANS: A Nursing activities for children with head trauma and increased intracranial pressure (ICP) include elevating the head of the bed 15 to 30 degrees and maintaining the head in a midline position. The nurse should try to maintain a quiet, nonstimulating environment for a child with increased ICP. Chest percussion and suctioning should be performed judiciously because they can elevate ICP. Range of motion should be passive and nontherapeutic touch should be avoided because both of these activities can increase ICP.

For case management to be most effective, who should be recognized as the most appropriate case manager? a. Nurse b. Panel of experts c. Multidisciplinary team d. Insurance company

ANS: A Nursing case managers are ideally suited to provide the care coordination necessary. Care coordination is most effective if a single person works with the family to accomplish the many tasks and responsibilities that are necessary. The family retains the role as primary decision maker. Most likely the insurance company will have a case manager focusing on the financial aspects of care. This does not include coordination of care to assist the family.

What rationale explains why prolonged use of oxygen should be discouraged in a child with anemia? a. Prolonged use of oxygen can decrease erythropoiesis. b. Prolonged use of oxygen can interfere with iron production. c. Prolonged use of oxygen interferes with a child's appetite. d. Prolonged use of oxygen can affect the synthesis of hemoglobin

ANS: A Oxygen administration is of limited value, because each gram of hemoglobin is able to carry a limited amount of the gas. In addition, prolonged use of supplemental oxygen can decrease erythropoiesis. Prolonged use of oxygen does not interfere with iron production, a child's appetite, or affect the synthesis of hemoglobin.

What manifestation observed by the nurse is suggestive of parental overprotection? a. Gives inconsistent discipline b. Facilitates the child's responsibility for self-care of illness c. Persuades the child to take on activities of daily living even when not able d. Encourages social and educational activities not appropriate to the child's level of capability

ANS: A Parental overprotection is manifested when the parents fear letting the child achieve any new skill, avoid all discipline, and cater to every desire to prevent frustration. Overprotective parents do not allow the child to assume responsibility for self-care of the illness. The parents prefer to remain in the role of total caregiver. The parents do not encourage the child to participate in social and educational activities.

The nurse is teaching the family of an infant with cerebral palsy how to administer a diazepam (Valium) pill by gastrostomy tube. What should the nurse include in the teaching session? a. The pill should be crushed and mixed with a small amount of water. b. The pill should be crushed and mixed with the infant's formula. c. After administering the medication, flush the tube with air. d. Before administering the medication, check the placement of the tube.

ANS: A Pills may be crushed and mixed with small amounts of water but not other liquids, such as formula or elixir medications, because these may act together to form a sludge that can interfere with gastrostomy tube function. When crushed pills or tablets are administered, flush the feeding tube with more water after instilling the dissolved pill in water. The tube should not be flushed with air, and placement does not need to be checked because it is directly into the stomach.

The community nurse is planning prevention measures designed to avoid conditions that can cause cognitive impairment. Taking folic acid supplements during pregnancy to prevent neural tube defects is which type of prevention strategy? a. Primary b. Secondary c. Tertiary d. Rehabilitative

ANS: A Primary prevention strategies are those designed to avoid conditions that cause cognitive impairment. Use of folic acid supplements during pregnancy to prevent neural tube defects is a primary prevention strategy. Secondary prevention activities are those designed to identify the condition early and initiate treatment to avert cerebral damage. Tertiary prevention strategies are those concerned with treatment to minimize long-term consequences. Rehabilitation services is an example of tertiary prevention.

An adolescent whose leg was crushed when she fell off a horse is admitted to the emergency department. She has completed the tetanus immunization series, receiving the last tetanus toxoid booster 8 years ago. What care is necessary for therapeutic management of this adolescent to prevent tetanus? a. Tetanus toxoid booster is needed because of the type of injury. b. Human tetanus immunoglobulin is indicated for immediate prophylaxis. c. Concurrent administration of both tetanus immunoglobulin and tetanus antitoxin is needed. d. No additional tetanus prophylaxis is indicated. The tetanus toxoid booster is protective for 10 years.

ANS: A Protective levels of antibody are maintained for at least 10 years. Children with serious "tetanus-prone" wounds, including contaminated, crush, puncture, or burn wounds, should receive a tetanus toxoid booster prophylactically as soon as possible. This adolescent has circulating antibodies. The immunoglobulin is not indicated.

What statement is most descriptive of a school-age child's reaction to death? a. Very interested in funerals and burials b. Little understanding of words such as "forever" c. Imagine the deceased person to be still alive d. Can explain death from a religious or spiritual point of view

ANS: A School-age children are interested in naturalistic and physiologic explanations of why death occurs and what happens to the body. School-age children do have an established concept of forever and have a deeper understanding of death in a concrete manner. Adolescents may explain death from a religious or spiritual point of view.

A child is upset because, when the cast is removed from her leg, the skin surface is caked with desquamated skin and sebaceous secretions. What technique should the nurse suggest to remove this material? a. Soak in a bathtub. b. Vigorously scrub the leg. c. Carefully pick material off the leg. d. Apply powder to absorb the material.

ANS: A Simply soaking in the bathtub is usually sufficient for removal of the desquamated skin and sebaceous secretions. Several days may be required to eliminate the accumulation completely. The parents and child should be advised not to scrub the leg vigorously or forcibly remove this material because it may cause excoriation and bleeding. Oil or lotion, but not powder, may provide comfort for the child.

A young girl has just injured her ankle at school. In addition to notifying the child's parents, what is the most appropriate, immediate action by the school nurse? a. Apply ice. b. Observe for edema and discoloration. c. Encourage child to assume a position of comfort. d. Obtain parental permission for administration of acetaminophen or aspirin.

ANS: A Soft tissue injuries should be iced immediately. In addition to ice, the extremity should be rested, be elevated, and have compression applied. The nurse observes for the edema while placing a cold pack. The applying of ice can reduce the severity of the injury. Maintaining the ankle at a position elevated above the heart is important. The nurse helps the child be comfortable with this requirement. The nurse obtains parental permission for administration of acetaminophen or aspirin after ice and rest are assured.

Therapeutic management of a 6-year-old child with hereditary spherocytosis (HS) should include which therapeutic intervention? a. Perform a splenectomy. b. Supplement the diet with calcium. c. Institute a maintenance transfusion program. d. Increase intake of iron-rich foods such as meat.

ANS: A Splenectomy corrects the hemolysis that occurs in HS. The splenectomy is generally reserved for children older than age 5 years with symptomatic anemia. Supplementation with calcium does not affect the HS. Additional folic acid can prevent deficiency caused by the rapid cell turnover. A maintenance transfusion program suppresses red blood cell formation. At this time, the risks of transfusion are greater than those of a splenectomy. Iron supplementation does not influence the course of HS.

A student athlete was injured during a basketball game. The nurse observes significant swelling. The player states he thought he "heard a pop," that the pain is "pretty bad," and that the ankle feels "as if it is coming apart." Based on this description, the nurse suspects what injury? a. Sprain b. Fracture c. Dislocation d. Stress fracture

ANS: A Sprains account for approximately 75% of all ankle injuries in children. A sprain results when the trauma is so severe that a ligament is either stretched or partially or completely torn by the force created as a joint is twisted or wrenched. Joint laxity is the most valid indicator of the severity of a sprain. A fracture involves the cross-section of the bone. Dislocations occur when the force of stress on the ligaments disrupts the normal positioning of the bone ends. Stress fractures result from repeated muscular contraction and are seen most often in sports involving repetitive weight bearing such as running, gymnastics, and basketball.

The mother of a young child with cognitive impairment asks for suggestions about how to teach her child to use a spoon for eating. The nurse should make which recommendation? a. Do a task analysis first. b. Do not expect this task to be learned. c. Continue to spoon feed the child until the child tries to do it alone. d. Offer only finger foods so spoon feeding is unnecessary.

ANS: A Successful teaching begins with a task analysis. The endpoint (self-feeding, toilet training, and so on) is broken down into the component steps. The child is then guided to master the individual steps in sequence. Depending on the child's functional level, using a spoon for eating should be an achievable goal. The child requires demonstration and then guided training for each component of the self-feeding. Feeding finger foods so spoon feeding is unnecessary eliminates some of the intermediate steps that are necessary to using a fork and spoon. For socialization purposes, it is desirable that a child use feeding implements.

The nurse should suspect a hearing impairment in an infant who fails to demonstrate which behavior? a. Babbling by age 12 months b. Eye contact when being spoken to c. Startle or blink reflex to sound d. Gesturing to indicate wants after age 15 months

ANS: A The absence of babbling or inflections in voice by at least age 7 months is an indication of hearing difficulties. Lack of eye contact is not indicative of a hearing loss. An infant with a hearing impairment might react to a loud noise but not respond to the spoken word. The child with hearing impairment uses gestures rather than vocalizations to express desires at this age.

. A child with hemophilia A is scheduled for surgery. What precautions should the nurse institute with this child? a. Handle the child gently when transferring to a cart. b. Caution the child not to brush his teeth before surgery. c. Use tape sparingly on postoperative dressings. d. Do not administer analgesics before surgery.

ANS: A The goal of prevention of bleeding episodes is directed toward decreasing the risk of injury. The child should be handled carefully when transferring to a cart. Brushing teeth, use of tape, and giving analgesics will not risk a bleeding episode.

The nurse is caring for a neonate born with a myelomeningocele. Surgery to repair the defect is scheduled the next day. What is the most appropriate way to position and feed this neonate? a. Prone with the head turned to the side b. On the side c. Supine in an infant carrier d. Supine, with defect supported with rolled blankets

ANS: A The prone position with the head turned to the side for feeding is the optimum position for the infant. It protects the spinal sac and allows the infant to be fed without trauma. The side-lying position is avoided preoperatively. It can place tension on the sac and affect hip dysplasia if present. The infant should not be placed in a supine position.

The nurse notes that the parents of a critically ill child spend a large amount of time talking with the parents of another child who is also seriously ill. They talk with these parents more than with the nurses. How should the nurse interpret this situation? a. Parent-to-parent support is valuable. b. Dependence on other parents in crisis is unhealthy. c. This is occurring because the nurses are unresponsive to the parents. d. This has the potential to increase friction between the parents and nursing staff.

ANS: A Veteran parents share experiences that cannot be supplied by other support systems. They have known the stress related to diagnosis, have weathered the many transition times, and have a practical remembering of resources. The parents can be mutually supportive during times of crisis. Nursing staff cannot provide the type of support that is realized from other parents who are experiencing similar situations. Friction should not exist between the nursing staff and the family of the child who is critically ill.

The nurse is conducting preoperative teaching to parents and their child about an external fixation device. What should the nurse include in the teaching session? (Select all that apply.) a. Pin care b. Crutch walking c. Modifications in activity d. Observing pin sites for infection e. Full weight bearing will be allowed after 24 hours

ANS: A, B, C, D The device is attached surgically by securing a series of external full or half rings to the bone with wires. Children and parents should be instructed in pin care, including observation for infection and loosening of pins. Partial weight bearing is allowed, and the child needs to learn to walk with crutches. Alterations in activity include modifications at school and in physical education. Full weight bearing is not allowed until the distraction is completed and bone consolidation has occurred.

The nurse is preparing an education program on hearing impairment for a group of new staff nurses. What concepts should be included? (Select all that apply.) a. A child with a slight hearing loss is usually unaware of a hearing difficulty. b. A clinical manifestation of a hearing impairment in children is avoidance of social interaction. c. A child with a severe hearing loss may hear a loud voice if nearby. d. Children with sensorineural hearing loss can benefit from the use of a hearing aid. e. A clinical manifestation of hearing impairment in an infant is lack of the startle reflex. f. Identification of a hearing loss after the first year is essential to facilitate language development in children.

ANS: A, B, C, E When discussing hearing impairment in children, the nurse should include information about differences in hearing losses, such as with a slight hearing loss, the child is usually unaware of a hearing difficulty, and with a severe loss, the child may hear a loud noise if it is nearby. An infant with a hearing loss may lack the startle response, and a hearing impaired child may avoid social interaction. Children with a sensorineural hearing loss would not benefit from a hearing aid. Identification of a hearing loss is imperative in the first 3 to 6 months to facilitate language and educational development for children.

The nurse is assessing coping behaviors of a family with a child with a chronic illness. What indicates approach coping behaviors? (Select all that apply.) a. Plans realistically for the future b. Verbalizes possible loss of the child c. Uses magical thinking and fantasy d. Realistically perceives the child's condition e. Does not share the burden of the disorder with others

ANS: A, B, D Approach coping behaviors include planning realistically for the future, verbalizing possible loss of a child, and realistically perceiving the child's behavior. Using magical thinking and fantasy is an avoidance behavior. The family should share the burden of the disorder with others as an approach behavior.

What are supportive interventions that can assist an infant with a chronic illness to meet developmental milestones? (Select all that apply.) a. Encourage consistent caregivers. b. Encourage periodic respite from demands of care. c. Encourage one family member to be the primary caretaker. d. Encourage parental "rooming in" during hospitalization. e. Withhold age-appropriate developmental tasks until the child is older.

ANS: A, B, D To develop trust, consistent caretakers and parents "rooming in" should be encouraged. To develop a sense of separateness from parents, periodic respites from caregiving should be encouraged. All members of the family, not one primary caretaker, should be encouraged to participate in care. Age-appropriate developmental tasks should be encouraged, not withheld until an older age.

The nurse is preparing to admit a 7-year-old child with ataxic cerebral palsy. What clinical manifestations of ataxic cerebral palsy should the nurse expect to observe? (Select all that apply.) a. Wide-based gait b. Rapid, repetitive movements performed poorly c. Slow, twisting movements of the trunk or extremities d. Hypertonicity with poor control of posture, balance, and coordinated motion e. Disintegration of movements of the upper extremities when the child reaches for objects

ANS: A, B, E Clinical manifestations of ataxic cerebral palsy include a wide-based gait; rapid, repetitive movements performed poorly; and disintegration of movements of the upper extremities when the child reaches for objects. Slow, twisting movements of the trunk are seen with dyskinetic cerebral palsy, and hypertonicity with poor control of posture, balance, and coordinated motion are seen with spastic cerebral palsy.

What are supportive interventions that can assist a toddler with a chronic illness to meet developmental milestones? (Select all that apply.) a. Give choices. b. Provide sensory experiences. c. Avoid discipline and limit setting. d. Discourage negative and ritualistic behaviors. e. Encourage independence in as many areas as possible.

ANS: A, B, E To encourage autonomy, choices should be given and independence encouraged in as many areas as possible. Sensory experiences should be encouraged to help the toddler to learn through sensorimotor experiences. Age-appropriate discipline and limit setting should be initiated. Negative and ritualistic behaviors are normal and should be allowed.

The nurse should suspect a child has cerebral palsy (CP) if the parent says what? a. "My 6-month-old baby is rolling from back to prone now." b. "My 4-month-old doesn't lift his head when on his tummy." c. "My 8-month-old can sit without support." d. "My 10-month-old is not walking."

ANS: B Delayed gross motor development is a universal manifestation of CP. The child shows a delay in all motor accomplishments, and the discrepancy between motor ability and expected achievement tends to increase with successive developmental milestones as growth advances. The infant who does not lift his head when on the tummy is showing a gross motor delay, as that is seen at 0 to 3 months. The other statements are within normal growth and development expectations.

In teaching a 16-year-old adolescent who was recently diagnosed with systemic lupus erythematosus (SLE), what statements should the nurse include? (Select all that apply.) a. "You should use a moisturizer with a sun protection factor (SPF) of 30." b. "You should avoid pregnancy because this can cause a flare-up." c. "You should not receive any immunizations in the future." d. "You may need to be on a low-protein, high-carbohydrate diet." e. "You should expect to lose weight while taking steroids." f. "You may need to modify your daily recreational activities."

ANS: A, B, F Teaching for an adolescent with SLE should foster adaptation and self-advocacy and include using a moisturizer with an SPF of 30, avoiding pregnancy because it can produce a flare-up, and modifying recreational activities but continuing with daily exercise as an essential part of the treatment plan. The adolescent should continue to receive immunizations as scheduled, should expect to gain weight while on steroid therapy, and would not have a specialized diet.

The clinic nurse is evaluating causes for iron deficiency caused by inadequate supply of iron. What should the nurse recognize as causes for iron deficiency caused by an inadequate iron supply? (Select all that apply.) a. Prematurity b. Slow growth rate c. Excessive milk intake d. Severe iron deficiency in the mother e. Exclusive breastfeeding of infant from birth to 3 months

ANS: A, C, D Causes for iron deficiency caused by an inadequate supply of iron include prematurity, excessive milk intake, and severe iron deficiency in the mother. Rapid growth rate, not slow, and exclusive breastfeeding of infant after 6 months, not from birth to 3 months, can be causes of inadequate supply of iron.

What are common respiratory symptoms dying children experience? (Select all that apply.) a. Cough b. Eupnea c. Wheezing d. Shortness of breath e. Decrease in secretions

ANS: A, C, D Common respiratory symptoms dying children experience include cough, wheezing, and shortness of breath. Eupnea is normal breathing, and secretions increase not decrease.

What are supportive interventions that can assist an adolescent with a chronic illness to meet developmental milestones? (Select all that apply.) a. Encourage activities appropriate for age. b. Avoid discussing planning for the future. c. Provide instruction on interpersonal and coping skills. d. Emphasize good appearance and wearing of stylish clothes. e. Understand that the adolescent will not have the same sexual needs.

ANS: A, C, D To achieve independence from family, instruction on interpersonal and coping skills should be provided. To promote heterosexual relationships, activities appropriate for age should be encouraged, and a good appearance and wearing of stylish clothes should be emphasized. Plans for the future should be discussed, and the adolescent will have the same sexual needs as adolescents without a chronic illness.

The nurse is caring for a child with meningitis. What acute complications of meningitis should the nurse continuously assess the child for? (Select all that apply.) a. Seizures b. Cerebral palsy c. Cerebral edema d. Hydrocephalus e. Cognitive impairments

ANS: A, C, E Acute complications of meningitis include syndrome of inappropriate antidiuretic hormone (SIADH), subdural effusions, seizures, cerebral edema and herniation, and hydrocephalus. Long-term complications include cerebral palsy, cognitive impairments, learning disorder, attention deficit hyperactivity disorder, and seizures.

What activity should the school nurse recommend for a child with hemophilia A? (Select all that apply.) a. Golf b. Soccer c. Rugby d. Jogging e. Swimming

ANS: A, D, E Children and adolescents with severe hemophilia can participate in noncontact sports such as swimming, golf, walking, jogging, fishing, and bowling. Contact sports such as football, boxing, hockey, soccer, and rugby are strongly discouraged because the risk of injury outweighs the physical and psychosocial benefits of participating in these sports.

The nurse is teaching parents of a child with a cognitive impairment signs that indicate the child is developmentally ready for dressing training. What signs should the nurse include that indicate the child is developmentally ready for dressing training? (Select all that apply.) a. Can follow verbal commands b. Can sit quietly for 1 to 2 minutes c. Can master every task of dressing d. Can follow physical gestures or cues e. Can relate clothing to the appropriate body part

ANS: A, D, E Children are considered developmentally ready for dressing training if they can sit quietly for 3 to 5 minutes (not 1 to 2) while working on a task; can follow physical gestures or cues; can follow verbal commands; and can relate clothing to the appropriate body part, such as socks to feet. As with other self-help skills, the child may not be able to master every task but should be evaluated for evidence of willingness to participate at his or her level of readiness.

The nurse is preparing to admit a 10-year-old child with absence seizures. What clinical features of absence seizures should the nurse recognize? (Select all that apply.) a. There is no aura. b. There is a postictal state. c. They usually last longer than 30 seconds. d. There is a brief loss of consciousness. e. There is an occasional clonic movement.

ANS: A, D, E Clinical features of absence seizures include no auras, a brief loss of consciousness, and an occasional clonic movement. There is no postictal state, and the seizures rarely last longer than 30 seconds.

The clinic nurse is assessing an infant. What are early signs of cognitive impairment the nurse should discuss with the health care provider? (Select all that apply.) a. Head lag at 11 months of age b. No pincer grasp at 4 months of age c. Colicky incidents at 3 months of age d. Unable to speak two to three words at 24 months of age e. Unresponsiveness to the environment at 12 months of age

ANS: A, D, E Early signs of cognitive impairment include gross motor delay (head lag should be established by 6 months, and head lag still present at 11 months is a delay), language delay (normal language development is speaking two to three words by age 12 months; if unable to speak two to three words at 24 months, that is a delay), and unresponsiveness to the environment at 12 months. No pincer grasp at 4 months of age is normal (palmar grasp is the expected finding), and colicky incidents at 3 months of age is a normal finding.

A mother tells the clinic nurse that she often puts honey on her infant's pacifier to soothe the infant. What response should the nurse make to the mother? a. "That is a good way to soothe your baby." b. "Honey does not have any soothing effects." c. "There is still a risk for infant botulism from honey." d. "Honey is OK, but it should not be put on the pacifier."

ANS: C Although the precise source of Clostridium botulinum spores has not been identified as originating from honey in many cases of infant botulism in the United States, it is still recommended that honey not be given to infants younger than 12 months because the spores have been found in honey.

Nursing strategies to improve the growth and development of the child with human immunodeficiency virus (HIV) infection should include what? a. Provide only those foods that the child feels like eating. b. Fortify foods with nutritional supplements to maximize quality of intake. c. Weigh the child and measure height and muscle mass on a daily basis. d. Provide high-fat and high-calorie meals and snacks to meet body requirements for growth.

ANS: B HIV infection often leads to marked failure to thrive and multiple nutritional deficiencies. Nutritional management may be difficult because of recurrent illness, diarrhea, and other physical problems. The nurse should implement intensive nutritional interventions if the child's growth begins to slow or weight begins to decrease. Fortifying foods with nutritional supplements will maximize quality of intake. The child does not need to be weighed daily, and high-fat meals and snacks should not be encouraged.

A 2-year-old child starts to have a tonic-clonic seizure. The child's jaws are clamped. What is the most important nursing action at this time? a. Place a padded tongue blade between the child's jaws. b. Stay with the child and observe his respiratory status. c. Prepare the suction equipment. d. Restrain the child to prevent injury.

ANS: B It is impossible to halt a seizure once it has begun, and no attempt should be made to do so. The nurse must remain calm, stay with the child, and prevent the child from sustaining any harm during the seizure. The nurse should not move or forcefully restrain the child during a tonic-clonic seizure and should not place a solid object between the teeth. Suctioning may be needed but not until the seizure has ended.

The clinic nurse is evaluating lab results for a child. What recorded hematocrit (Hct) result is considered within the normal range? a. 30% b. 40% c. 50% d. 60%

ANS: B Normal hematocrit (Hct) is 35% to 45%.

What statement is true concerning osteogenesis imperfecta (OI)? a. It is easily treated. b. It is an inherited disorder. c. Braces and exercises are of no therapeutic value. d. Later onset disease usually runs a more difficult course.

ANS: B OI is a heterogeneous, autosomal dominant disorder characterized by fractures and bone deformity. Treatment is primarily supportive. Several investigational therapies are being evaluated. The primary goal of therapy is rehabilitation. Lightweight braces and splints help support limbs, prevent fractures, and aid in ambulation. The disease is present at birth. Prognosis is affected by the type of OI.

What is the rationale for orthopedic surgery for a child with cerebral palsy? a. To cure spasticity b. To improve function c. For cosmetic purposes d. To prevent the need of physical therapy

ANS: B Orthopedic surgery is used primarily to improve function rather than for cosmetic purposes and is followed by physical therapy. It will not cure spasticity.

The nurse has been assigned as a home health nurse for a child who is technology dependent. The nurse recognizes that the family's background differs widely from the nurse's own. The nurse believes some of their lifestyle choices are less than ideal. What nursing intervention is most appropriate to institute? a. Change the family. b. Respect the differences. c. Assess why the family is different. d. Determine whether the family is dysfunctional.

ANS: B Respect for varied family structures and for racial, ethnic, cultural, and socioeconomic diversity among families is essential in home care. The nurse must assess and respect the family's background and lifestyle choices. It is not appropriate to attempt to change the family. The nurse is a guest in the home and care of the child. The family and the values held by the cultural group prevail. The nurse may assess why the family is different to help the nurse and other health professionals understand the differences. It is not appropriate to determine whether the family is dysfunctional.

A 16-year-old boy with a chronic illness has recently become rebellious and is taking risks such as missing doses of his medication. What should the nurse explain to his parents? a. That he needs more discipline b. That this is a normal part of adolescence c. That he needs more socialization with peers d. That this is how he is asking for more parental control

ANS: B Risk taking, rebelliousness, and lack of cooperation are normal parts of adolescence, during which young adults are establishing independence. If the parents increase the amount of discipline, he will most likely be more rebellious. More socialization with peers does not address the problem of risk-taking behavior.

An adolescent comes to the school nurse after experiencing shin splints during a track meet. What reassurance should the nurse offer? a. Shin splints are expected in runners. b. Ice, rest, and nonsteroidal antiinflammatory drugs (NSAIDs) usually relieve pain. c. It is generally best to run around and "work the pain out." d. Moist heat and acetaminophen are indicated for this type of injury.

ANS: B Shin splints result when the ligaments tear away from the tibial shaft and cause pain. Actions that have an antiinflammatory effect are indicated for shin splints. Ice, rest, and NSAIDs are the usual treatment. Shin splints are rarely serious, but they are not expected, and preventive measures are taken. Rest is important to heal the shin splints. Continuing to place stress on the tibia can lead to further damage.

What condition occurs when the normal adult hemoglobin is partly or completely replaced by abnormal hemoglobin? a. Aplastic anemia b. Sickle cell anemia c. Thalassemia major d. Iron deficiency anemia

ANS: B Sickle cell anemia is one of a group of diseases collectively called hemoglobinopathies, in which normal adult hemoglobin is replaced by abnormal hemoglobin. Aplastic anemia is a lack of cellular elements being produced. Thalassemia major refers to a variety of inherited disorders characterized by deficiencies in production of certain globulin chains. Iron-deficiency anemia affects red blood cell size and depth of color but does not involve abnormal hemoglobin.

What type of cerebral palsy (CP) is the most common type? a. Ataxic b. Spastic c. Dyskinetic d. Mixed type

ANS: B Spastic CP is the most common clinical type. Early manifestations are usually generalized hypotonia, or decreased tone that lasts for a few weeks or may extend for months or as long as 1 year. It is replaced by increased stretch reflexes, increased muscle tone, and weakness. Ataxic, dyskinetic, and mixed type are less common forms of CP.

A child with a hip spica cast is being prepared for discharge. Recognizing that caring for a child at home is complex, the nurse should include what instructions for the parents' discharge teaching? a. Turn every 8 hours. b. Specially designed car restraints are necessary. c. Diapers should be avoided to reduce soiling of the cast. d. Use an abduction bar between the legs to aid in turning.

ANS: B Standard seat belts and car seats may not be readily adapted for use by children in some casts. Specially designed car seats and restraints meet safety requirements. The child must have position changes much more frequently than every 8 hours. During feeding and play activities, the child should be moved for both physiologic and psychosocial benefit. Diapers and other strategies are necessary to maintain cleanliness. The abduction bar is never used as an aid for turning. Putting pressure on the bar may damage the integrity of the cast.

The nurse is teaching the parent of a 4-year-old child with a cast on the arm about care at home. What statement by the parent indicates a correct understanding of the teaching? a. "I should have the affected limb hang in a dependent position." b. "I will use an ice pack to relieve the itching." c. "I should avoid keeping the injured arm elevated." d. "I will expect the fingers to be swollen for the next 3 days."

ANS: B Teaching the parent to use an ice pack to relieve the itching is an important aspect when planning discharge for a child with a cast. The affected limb should not be allowed to hang in a dependent position for more than 30 minutes. The affected arm should be kept elevated as much as possible. If there is swelling or redness of the fingers, the parent should notify the health care provider.

The nurse knows that parents need further teaching with regard to the treatment of congenital clubfoot when they state what? a. "We'll keep the cast dry." b. "We're happy this is the only cast our baby will need." c. "We'll watch for any swelling of the foot while the cast is on." d. "We're getting a special car seat to accommodate the cast."

ANS: B The common approach to clubfoot management and treatment is the Ponseti method. Serial casting is begun shortly after birth. Weekly gentle manipulation and stretching of the foot along with placement of serial long-leg casts allow for gradual repositioning of the foot. The extremity or extremities are casted until maximum correction is achieved, usually within 6 to 10 weeks. If parents state that this is the only cast the infant will need, they need further teaching.

What is a primary goal in caring for a child with cognitive impairment? a. Developing vocational skills b. Promoting optimum development c. Finding appropriate out-of-home care d. Helping child and family adjust to future care

ANS: B The goal for children with cognitive impairment is the promotion of optimum social, physical, cognitive, and adaptive development as individuals within a family and community. Vocational skills are only one part of that goal. The focus must also be on the family and other aspects of development. Out-of-home care is considered part of the child's development. Optimum development includes adjustment for both the family and child.

What statement is descriptive of most cases of hemophilia? a. X-linked recessive deficiency of platelets causing prolonged bleeding b. X-linked recessive inherited disorder in which a blood clotting factor is deficient c. Autosomal dominant deficiency of a factor involved in the blood-clotting reaction d. Y-linked recessive inherited disorder in which the red blood cells become moon shaped

ANS: B The inheritance pattern in 80% of all the cases of hemophilia is X-linked recessive. The two most common forms of the disorder are factor VIII deficiency (hemophilia A, or classic hemophilia) and factor IX deficiency (hemophilia B, or Christmas disease). The disorder involves coagulation factors, not platelets. The disorder does not involve red blood cells or the Y chromosome.

A 7-year-old child has just had a cast applied for a fractured arm with the wrist and elbow immobilized. What information should be included in the home care instructions? a. No restrictions of activity are indicated. b. Elevate casted arm when both upright and resting. c. The shoulder should be kept as immobile as possible to avoid pain. d. Swelling of the fingers is to be expected. Notify a health professional if it persists more than 48 hours.

ANS: B The injured extremity should be kept elevated while resting and in a sling when upright. This will increase venous return. The child should not engage in strenuous activity for the first few days. Rest with elevation of the extremity is encouraged. Joints above and below the cast on the affected extremity should be moved. Swelling of the fingers may indicate neurovascular damage and should be reported immediately. Permanent damage can occur within 6 to 8 hours.

The nurse is assessing the coping behaviors of the parents of a child recently diagnosed with a chronic illness. What behavior should the nurse consider an "approach behavior" that results in movement toward adjustment? a. Being unable to adjust to a progression of the disease or condition b. Anticipating future problems and seeking guidance and answers c. Looking for new cures without a perspective toward possible benefit d. Failing to recognize the seriousness of the child's condition despite physical evidence

ANS: B The parents who anticipate future problems and seek guidance and answers are demonstrating approach behaviors. These are positive actions in caring for their child. Being unable to adjust, looking for new cures, and failing to recognize the seriousness of the child's condition are avoidance behaviors. The parents are moving away from adjustment or exhibiting maladaptation to the crisis of a child with chronic illness or disability.

What is a principle of palliative care that can be included in the care of children? a. Maintenance of curative therapy b. Child and family as the unit of care c. Exclusive focus on the spiritual issues the family faces d. Extensive use of opiates to ensure total pain control

ANS: B The principles of palliative care involve a multidisciplinary approach to the management of a terminal illness or the dying process that focuses on symptom control and support rather than on cure or life prolongation in the absence of the possibility of a cure. In pediatric palliative care, the focus of care is on the family. Palliative care requires the transition from curative to palliative care. The transition occurs when the likelihood of cure no longer exists. Spiritual issues are just one of the foci of palliative care. The multidisciplinary team focuses on physical, emotional, and social issues as well. Pain control is a priority in palliative care. The use of opiates is balanced with the side effects caused by this class of drugs.

The most important nursing intervention when caring for an infant with myelomeningocele in the preoperative stage is which? a. Take vital signs every hour. b. Place the infant on the side to decrease pressure on the spinal sac. c. Watch for signs that might indicate developing hydrocephalus. d. Apply a heat lamp to facilitate drying and toughening of the sac.

ANS: B The spinal sac is protected from damage until surgery is performed. Early surgical closure is recommended to prevent local trauma and infection. Monitoring vital signs and watching for signs that might indicate developing hydrocephalus are important interventions, but preventing trauma to the sac is a priority. The sac is kept moist until surgical intervention is done.

The nurse is talking with a 10-year-old boy who wears bilateral hearing aids. The left hearing aid is making an annoying whistling sound that the child cannot hear. What intervention is the most appropriate nursing action? a. Ignore the sound. b. Suggest he reinsert the hearing aid. c. Ask him to reverse the hearing aids in his ears. d. Suggest he raise the volume of the hearing aid.

ANS: B The whistling sound is acoustic feedback. The nurse should have the child remove the hearing aid and reinsert it, making sure no hair is caught between the ear mold and the ear canal. Ignoring the sound or suggesting he raise the volume of the hearing aid would be annoying to others. The hearing aids are molded specifically for each ear.

The nurse is caring for a 14-year-old child with juvenile idiopathic arthritis (JIA). What clinical manifestations should the nurse expect to observe? (Select all that apply.) a. Erythema over joints b. Soft tissue contractures c. Swelling in multiple joints d. Morning stiffness of the joints e. Loss of motion in the affected joints

ANS: B, C, D, E Whether single or multiple joints are involved, stiffness, swelling, and loss of motion develop in the affected joints in JIA. The swelling results from soft tissue edema, joint effusion, and synovial thickening. The affected joints may be warm and tender to the touch, but it is not uncommon for pain not to be reported. The limited motion early in the disease is a result of muscle spasm and joint inflammation; later it is caused by ankylosis or soft tissue contracture. Morning stiffness of the joint(s) is characteristic and present on arising in the morning or after inactivity. Erythema is not typical, and a warm, painful, red joint is always suspect for infection.

The clinic nurse is evaluating causes for iron deficiency due to impaired iron absorption. What should the nurse recognize as causes for iron deficiency due to impaired iron absorption? (Select all that apply.) a. Gastric acidity b. Chronic diarrhea c. Lactose intolerance d. Absence of phosphates e. Inflammatory bowel disease

ANS: B, C, E Causes for iron deficiency due to impaired iron absorption include chronic diarrhea, lactose intolerance, and inflammatory bowel disease. Gastric alkalinity, not acidity, and the presence, not absence, of phosphates can be causes of impaired iron absorption.

A school-age child has sustained a head injury and multiple fractures after being thrown from a horse. The child's level of consciousness is variable. The parents tell the nurse that they think their child is in pain because of periodic crying and restlessness. What is the most appropriate nursing action? a. Explain that analgesia is contraindicated with a head injury. b. Have the parents describe the child's previous experiences with pain. c. Consult with a practitioner about what analgesia can be safely administered. d. Teach the parents that analgesia is unnecessary when the child is not fully awake and alert.

ANS: C A key nursing role is to provide sedation and analgesia for the child. Consultation with the appropriate practitioner is necessary to avoid conflict between the necessity to monitor the child's neurologic status and to promote comfort and relieve anxiety. Analgesia can be safely used in individuals who have sustained head injuries. The child's previous experiences with pain should be obtained as part of the assessment, but because of the severity of the injury, analgesia should be provided as soon as possible. Analgesia can decrease anxiety and resultant increased intracranial pressure.

A toddler with spastic cerebral palsy needs to be transported to the radiology department. What transportation method should the nurse use to take the toddler to the radiology department? a. A stretcher b. A wheelchair c. A wagon with pillows d. Carried in the nurse's arms

ANS: C A wagon with pillows would support the child with spastic cerebral palsy better than a stretcher or wheelchair. A wagon would give the child a "wheelchair" experience, so the nurse should not carry the child.

The nurse is discussing sexuality with the parents of an adolescent girl who has a moderate cognitive impairment. What factor should the nurse consider when dealing with this issue? a. Sterilization is recommended for any adolescent with cognitive impairment. b. Sexual drive and interest are very limited in individuals with cognitive impairment. c. Individuals with cognitive impairment need a well-defined, concrete code of sexual conduct. d. Sexual intercourse rarely occurs unless the individual with cognitive impairment is sexually abused.

ANS: C Adolescents with moderate cognitive impairment may be easily persuaded and lack judgment. A well-defined, concrete code of conduct with specific instructions for handling certain situations should be defined for the adolescent. Permanent contraception by sterilization presents moral and ethical issues and may have psychologic effects on the adolescent. It may be prohibited in some states. The adolescent needs to have practical sexual information regarding physical development and contraception. Cognitively impaired individuals may desire to marry and have families. The adolescent needs to be protected from individuals who may make intimate advances.

A 10-year-old boy on a bicycle has been hit by a car in front of a school. The school nurse immediately assesses airway, breathing, and circulation. What should be the next nursing action? a. Place the child on his side. b. Take the child's blood pressure. c. Stabilize the child's neck and spine. d. Check the child's scalp and back for bleeding.

ANS: C After determining that the child is breathing and has adequate circulation, the next action is to stabilize the neck and spine to prevent any additional trauma. The child's position should not be changed until the neck and spine are stabilized. Blood pressure is a later assessment. A less urgent but important assessment is inspection of the scalp for bleeding.

What condition is defined as reduced visual acuity in one eye despite appropriate optical correction? a. Myopia b. Hyperopia c. Amblyopia d. Astigmatism

ANS: C Amblyopia, or lazy eye, is reduced visual acuity in one eye. Amblyopia is usually caused by one eye not receiving sufficient stimulation. The resulting poor vision in the affected eye can be avoided with the treatment of the primary visual defect such as strabismus. Myopia, or nearsightedness, refers to the ability to see objects clearly at close range but not a distance. Hyperopia, or farsightedness, is the ability to see objects at a distance but not at close range. Astigmatism is unequal curvatures in refractive apparatus.

A critically injured child has died and is being removed from a ventilator in the pediatric intensive care unit. What is a priority nursing intervention for the family at this time? a. Ensure that parents are in the waiting room while the ventilator is removed. b. Help the parents understand that the child is already dead and no further interventions are necessary. c. Control the environment around the child and family to provide privacy. d. Encourage them to wait to see their child until the funeral home has prepared the body.

ANS: C Around the time of death, nursing care can be invaluable to the parents. The nurse should attempt to control the environment to ensure that the family and child have privacy. Other individuals such as clergy can be present if the family wishes. Attention to religious and cultural rituals may be important to them. The family should decide where they would like to be during removal from the ventilator. The family should be allowed to be with the child if they wish rather than waiting until the funeral home has prepared the body. Explain all interventions used for the child before death.

What is a major premise of family-centered care? a. The child is the focus of all interventions. b. Nurses are the authorities in the child's care. c. Parents are the experts in caring for their child. d. Decisions are made for the family to reduce stress.

ANS: C As parents become increasingly responsible for their children, they are the experts. It is essential that the health care team recognize the family's expertise. In family-centered care, consistent attention is given to the effects of the child's chronic illness on all family members, not just the child. Nurses are adjuncts in the child's care. The nurse builds alliances with parents. Family members are involved in decision making about the child's physical care.

A preadolescent has been diagnosed with scoliosis. The planned therapy is the use of a thoracolumbosacral orthotic. The preadolescent asks how long she will have to wear the brace. What is the appropriate response by the nurse? a. "For as long as you have been told." b. "Most preadolescents use the brace for 6 months." c. "Until your vertebral column has reached skeletal maturity." d. "It will be necessary to wear the brace for the rest of your life."

ANS: C Bracing can halt or slow the progress of most curvatures. They must be used continuously until the child reaches skeletal maturity. Telling the child "for as long as you have been told" does not answer the child's question and does not promote involvement in care. Six months is unrealistic because skeletal maturity is not reached until adolescence. When skeletal growth is complete, bracing is no longer effective.

The sibling of a 4-year-old girl dies from sudden infant death syndrome. The parents are concerned because the 4-year-old girl showed more outward grief when her cat died than now. How should the nurse explain this reaction to the parents? a. The child is not old enough to have a concept of death. b. This suggests maladaptive coping, and referral is needed for counseling. c. The death may be so painful and threatening that the child must deny it for now. d. The child is not old enough to have formed a significant attachment to her sibling.

ANS: C Children of this age believe that their thoughts can cause death. The child may feel guilty and responsible. The loss may be so deep, painful, and threatening that the child needs to deny it for a time. Denial is within the range of a normal response to the death of a sibling. Counseling is not indicated at this time. Denial is also characteristic of the child's developmental level. These children do have a concept of death, seeing it as a separation. The child also would have formed an attachment to the sibling, who was in the house and sharing the parents' time and attention.

The parents of an infant with cerebral palsy (CP) ask the nurse if their child will have cognitive impairment. The nurse's response should be based on which knowledge? a. Affected children have some degree of cognitive impairment. b. Around 20% of affected children have normal intelligence. c. About 45% of affected children have normal intelligence. d. Cognitive impairment is expected if motor and sensory deficits are severe.

ANS: C Children with CP have a wide range of intelligence, and 40% to 50% are within normal limits. A large percentage of children with CP do not have mental impairment. Many individuals who have severely limiting physical impairment have the least amount of intellectual compromise.

In a child with sickle cell anemia (SCA), adequate hydration is essential to minimize sickling and delay the vasoocclusion and hypoxia-ischemia cycle. What information should the nurse share with parents in a teaching plan? a. Encourage drinking. b. Keep accurate records of output. c. Check for moist mucous membranes. d. Monitor the concentration of the child's urine.

ANS: C Children with SCA have impaired kidney function and cannot concentrate urine. Parents are taught signs of dehydration and ways to minimize loss of fluid to the environment. Encouraging drinking is not specific enough for parents. The nurse should give the parents and child a target fluid amount for each 24-hour period. Accurate monitoring of output may not reflect the child's fluid needs. Without the ability to concentrate urine, the child needs additional intake to compensate. Dilute urine and specific gravity are not valid signs of hydration status in children with SCA.

The nurse asks the mother of a child with a chronic illness many questions as part of the assessment. The mother answers several questions, then stops and says, "I don't know why you ask me all this. Who gets to know this information?" The nurse should respond in what manner? a. Determine why the mother is so suspicious. b. Determine what the mother does not want to tell. c. Explain who will have access to the information. d. Explain that everything is confidential and that no one else will know what is said.

ANS: C Communication with the family should not be invasive. The nurse needs to explain the importance of collecting the information, its applicability to the child's care, and who will have access to the information. The mother is not being suspicious and is not necessarily withholding important information. She has a right to understand how the information she provides will be used. The nurse will need to share, through both oral and written communication, clinically relevant information with other involved health professionals.

What statement best describes Duchenne (pseudohypertrophic) muscular dystrophy (DMD)? a. It has an autosomal dominant inheritance pattern. b. Onset occurs in later childhood and adolescence. c. It is characterized by presence of Gower sign, a waddling gait, and lordosis. d. Disease stabilizes during adolescence, allowing for life expectancy to approximately age 40 years.

ANS: C DMD is characterized by a waddling gait and lordosis. Gower sign is a characteristic way of rising from a squatting or sitting position on the floor. DMD is inherited as an X-linked recessive gene. Genetic counseling is recommended for parents, female siblings, maternal aunts, and their female offspring. Onset occurs usually between ages 3 and 5 years. DMD has a progressive and relentless loss of muscle function until death by respiratory or cardiac failure.

An adolescent with a spinal cord injury is admitted to a rehabilitation center. Her parents describe her as being angry, hostile, and uncooperative. The nurse should recognize that this is suggestive of which psychosocial state? a. Normal phase of adolescent development b. Severe depression that will require long-term counseling c. Normal response to her situation that can be redirected in a healthy way d. Denial response to her situation that makes rehabilitative efforts more difficult

ANS: C During the rehabilitation phase, it is desirable for adolescents to begin to express negative feelings toward the situation. The rehabilitation team can redirect the negative energy toward learning a new way of life. The injury has interrupted the normal adolescent process of achieving independence, triggering these negative behaviors. Severe depression can occur, but it indicates that the child is no longer in denial. Long-term therapy is not indicated. Being angry, hostile, and uncooperative are behaviors that are indications that the adolescent understands the severity of the injury and need for rehabilitation.

The nurse is caring for a hospitalized adolescent whose femur was fractured 18 hours ago. The adolescent suddenly develops chest pain and dyspnea. The nurse should suspect what complication? a. Sepsis b. Osteomyelitis c. Pulmonary embolism d. Acute respiratory tract infection

ANS: C Fat emboli are of greatest concern in individuals with fractures of the long bones. Fat droplets from the marrow are transferred to the general circulation, where they are transported to the lung or brain. This type of embolism usually occurs within the second 12 hours after the injury. Sepsis would manifest with fever and lethargy. Osteomyelitis usually is seen with pain at the site of infection and fever. A child with an acute respiratory tract infection would have nasal congestion, not chest pain.

Spastic cerebral palsy (CP) is characterized by which clinical manifestations? a. Athetosis, dystonic movements b. Tremors, lack of active movement c. Hypertonicity; poor control of posture, balance, and coordinated motion d. Wide-based gait; poor performance of rapid, repetitive movements

ANS: C Hypertonicity and poor control of posture, balance, and coordinated motion are part of the classification of spastic CP. Athetosis and dystonic movements are part of the classification of dyskinetic or athetoid CP. Tremors and lack of active movement may indicate other neurologic disorders. A wide-based gait and poor performance of rapid, repetitive movements are part of the classification of ataxic CP.

What condition is an acquired hemorrhagic disorder that is characterized by excessive destruction of platelets? a. Aplastic anemia b. Thalassemia major c. Idiopathic thrombocytopenic purpura d. Disseminated intravascular coagulation

ANS: C Idiopathic thrombocytopenic purpura is an acquired hemorrhagic disorder characterized by an excessive destruction of platelets, discolorations caused by petechiae beneath the skin, and normal bone marrow. Aplastic anemia refers to a bone marrow failure condition in which the formed elements of the blood are simultaneously depressed. Thalassemia major is a group of blood disorders characterized by deficiency in the production rate of specific hemoglobin chains. Disseminated intravascular coagulation is characterized by diffuse fibrin deposition in the microvasculature, consumption of coagulation factors, and endogenous generation of thrombin and plasma.

After a tonic-clonic seizure, what symptoms should the nurse expect the child to experience? a. Diarrhea and abdominal discomfort b. Irritability and hunger c. Lethargy and confusion d. Nervousness and excitability

ANS: C In the postictal phase, after a tonic-clonic seizure, the child may remain semiconscious and difficult to arouse. The average duration of the postictal phase is usually 30 minutes. The child may remain confused or sleep for several hours. He or she may have mild impairment of fine motor movements. The child may have visual and speech difficulties and may vomit or complain of headache.

The nurse is caring for a 4-year-old child with cerebral palsy (CP). The child, developmentally, is at an infant stage. Appropriate developmental stimulation for this child should be what? a. Playing "pat-a-cake" with the child b. None so the child does not become overstimulated c. Putting a colorful mobile with music on the bed d. Giving the child a coloring book and crayons

ANS: C Incorporating play into the therapeutic program for a child with CP often requires great ingenuity and inventiveness from those involved in the child's care. Objects and toys are chosen for the child's developmental stage to provide needed sensory input using a variety of shapes, forms, and textures. Nurses can help parents integrate therapy into play activities in natural ways.

What statement best describes b-thalassemia major (Cooley anemia)? a. It is an acquired hemolytic anemia. b. Inadequate numbers of red blood cells (RBCs) are present. c. Increased incidence occurs in families of Mediterranean extraction. d. It commonly occurs in individuals from West Africa.

ANS: C Individuals who live near the Mediterranean Sea and their descendants have the highest incidence of thalassemia. Thalassemia is inherited as an autosomal recessive disorder. An overproduction of RBCs occurs. Although numerous, the red blood cells are relatively unstable. Sickle cell disease is common in blacks of West African descent.

A child is admitted for revision of a ventriculoperitoneal shunt for noncommunicating hydrocephalus. What is a common reason for elective revision of this shunt? a. Meningitis b. Gastrointestinal upset c. Hydrocephalus resolution d. Growth of the child since the initial shunting

ANS: D An elective revision of a ventriculoperitoneal shunt would most likely be done to accommodate the child's growth. Meningitis would require an emergent replacement or revision of the shunt. Gastrointestinal upset alone would not indicate the need for shunt revision. Noncommunicating hydrocephalus will not resolve without surgical intervention.

A 2-week-old infant with Down syndrome is being seen in the clinic. His mother tells the nurse that he is difficult to hold, that "he's like a rag doll. He doesn't cuddle up to me like my other babies did." What is the nurse's best interpretation of this lack of clinging or molding? a. Sign of detachment and rejection b. Indicative of maternal deprivation c. A physical characteristic of Down syndrome d. Suggestive of autism associated with Down syndrome

ANS: C Infants with Down syndrome have hypotonicity of muscles and hyperextensibility of joints, which complicate positioning. The limp, flaccid extremities resemble the posture of a rag doll. Holding the infant is difficult and cumbersome, and parents may feel that they are inadequate. A lack of clinging or molding is characteristic of Down syndrome, not detachment. There is no evidence of maternal deprivation. Autism is not associated with Down syndrome, and it would not be evident at 2 weeks of age.

During a well-child visit, the mother tells the nurse that her 4-month-old infant is constipated, is less active than usual, and has a weak-sounding cry. The nurse suspects botulism and questions the mother about the child's diet. What factor should support this diagnosis? a. Breastfeeding b. Commercial formula c. Infant cereal with honey d. Improperly sterilized bottles

ANS: C Ingestion of honey is a risk factor for infant botulism in the United States. Honey should not be given to children younger than the age of 1 year. Botulism is not found with the use of commercial infant cereals. Although there is a slight increase in botulism in breastfed infants when compared with formula-fed infants, there is not sufficient evidence to support formula feeding as prevention. Thoroughly cleaning bottles used for formula feeding is sufficient for botulism prevention. Inadequate sterilization of home-canned foods can contribute to botulism.

Parents bring a 7-year-old child to the clinic for evaluation of an injured wrist after a bicycle accident. The parents and child are upset, and the child will not allow an examination of the injured arm. What priority nursing intervention should occur at this time? a. Send the child to radiology so radiography can be performed. b. Initiate an intravenous line and administer morphine for the pain. c. Calmly ask the child to point to where the pain is worst and to wiggle fingers. d. Have the parents hold the child so that the nurse can examine the arm thoroughly.

ANS: C Initially, assessment is the priority. Because the child is alert but upset, the nurse should work to gain the child's trust. Initial data are gained by observing the child's ability to move the fingers and to point to the pain. Other important observations at this time are pallor and paresthesia. The child needs to be sent for radiography, but initial assessment data need to be obtained. Sending the child for radiography will increase the child's anxiety, making the examination difficult. It is inappropriate to ask parents to restrain their child. These parents are upset about the injury. If restraint is indicated, the nurse should obtain assistance from other personnel.

The nurse is often the individual who is in the optimum position to suggest tissue donation to a family (after consultation with the practitioner). What will occur if a family chooses organ or tissue donation? a. The funeral will be delayed. b. Cremation is the preferred method of burial. c. Written consent is required for tissue or organ donation. d. An open casket cannot be used subsequent to this procedure.

ANS: C Organ and tissue donation cannot proceed without the family's written informed consent. There is usually no delay in the funeral. Organs are usually retrieved before actual death, and tissue must be removed soon after. No obvious disfigurement of the body occurs, and an open casket can be used for the funeral.

The nurse is teaching the family of a child, age 8 years, with moderate hemophilia about home care. What should the nurse tell the family to do to minimize joint injury? a. Administer nonsteroidal anti-inflammatory drugs (NSAIDs). b. Administer DDAVP (synthetic vasopressin). c. Provide intravenous (IV) infusion of factor VIII concentrates. d. Encourage elevation and application of ice to the involved joint.

ANS: C Parents are taught home infusion of factor VIII concentrate. For moderate and severe hemophilia, prompt IV administration is essential to prevent joint injury. NSAIDs are effective for pain relief. They must be given with caution because they inhibit platelet aggregation. A factor VIII level of 30% is necessary to stop bleeding. DDAVP can raise the factor VIII level fourfold. Moderate hemophilia is defined by a factor VIII activity of 4.9. A fourfold increase would not meet the 30% level. Ice and elevation are important adjunctive therapy, but factor VIII is necessary.

The nurse is teaching the parents of a 3-year-old child who has been diagnosed with tonic-clonic seizures. What statement by the parent should indicate a correct understanding of the teaching? a. "I should attempt to restrain my child during a seizure." b. "My child will need to avoid contact sports until adulthood." c. "I should place a pillow under my child's head during a seizure." d. "My child will need to be taken to the emergency department [ED] after each seizure."

ANS: C Parents should try to place a pillow or folded blanket under the child's head for protection. The parent should not try to restrain the child during the seizure. The child does not need to go to the ED with each seizures; the nurse can teach parents certain criteria for when their child would need to be seen. Discussing what will happen in adulthood is not appropriate at this time.

The nurse uses the five Ps to assess ischemia in a child with a fracture. What finding is considered a late and ominous sign? a. Petaling b. Posturing c. Paresthesia d. Positioning

ANS: C Paresthesia distal to the injury or cast is an ominous sign that requires immediate notification of the practitioner. Permanent muscle and tissue damage can occur within 6 hours. The other signs of ischemia that need to be reported are pain, pallor, pulselessness, and paralysis. Petaling is a method of placing protective or smooth edges on a cast. Posturing is not a sign of peripheral ischemia. Finding a position of comfort can be difficult with a fracture. It would not be an ominous sign unless pain was increasing or uncontrollable.

The nurse is teaching the girls' varsity sports teams about the "female athlete triad." What is essential information to include? a. They should take low to moderate calcium to avoid hypercalcemia. b. They have strong bones because of the athletic training. c. Pregnancy can occur in the absence of menstruation. d. A diet high in carbohydrates accommodates increased training.

ANS: C Sexually active teenagers, regardless of menstrual status, need to consider contraceptive precautions. Increased calcium (1500 mg) is recommended for amenorrheic athletes. The decreased estrogen in girls with the female athlete triad, coupled with potentially inadequate diet, leads to osteoporosis. Diets high in protein and calories are necessary to avoid potentially long-term consequences of intensive, prolonged exercise programs in pubertal girls.

A 14-year-old girl is in the intensive care unit after a spinal cord injury 2 days ago. What nursing intervention is a priority for this child? a. Minimizing environmental stimuli b. Administering immunoglobulin c. Monitoring and maintaining systemic blood pressure d. Discussing long-term care issues with the family

ANS: C Spinal cord injury patients are physiologically labile, and close monitoring is required. They may be unstable for the first few weeks after the injury. Increased blood pressure may be an indication of autonomic dysreflexia. It is not necessary to minimize environmental stimuli for this type of injury. Spinal cord injury is not an infectious process. Immunoglobulin is not indicated. Discussing long-term care issues with the family is inappropriate. The family is focusing on the recovery of their child. It will not be known until the rehabilitation period how much function the child may recover.

Parents tell the nurse they do not want to let their school-age child know his illness is terminal. What response should the nurse make to the parents? a. "Have you discussed this with your health care provider?" b. "I would do the same thing in your position; it is better the child doesn't know." c. "I understand you want to protect your child, but often children realize the seriousness of their illness." d. "I praise you for that decision; it can be so difficult to be truthful about the seriousness of your son's illness."

ANS: C Terminally ill children develop an awareness of the seriousness of their diagnosis even when protected from the truth. Acknowledging parents feelings but giving them truthful information is the appropriate response. Asking about discussing this with the health care provider is avoiding the issue. Sharing your own feelings by stating "I would do the same thing" and giving praise for the decision is nontherapeutic.

What statement is most accurate in describing tetanus? a. Inflammatory disease that causes extreme, localized muscle spasm. b. Disease affecting the salivary gland with resultant stiffness of the jaw. c. Acute infectious disease caused by an exotoxin produced by an anaerobic spore-forming, gram-positive bacillus. d. Acute infection that causes meningeal inflammation resulting in symptoms of generalized muscle spasm.

ANS: C Tetanus results from an infection by the anaerobic spore-forming, gram-positive bacillus Clostridium tetani. The organism forms two exotoxins that affect the central nervous system to produce the clinical manifestations of the disease. Tetanus is not an inflammatory process. The toxin acts at the neuromuscular junction to produce muscular stiffness and to lower the threshold for reflex excitability. It is usually a systemic disease. Initial symptoms are usually a progressive stiffness and tenderness of the muscles of the neck and jaw. The sustained contraction of the jaw-closing muscles provides the name lockjaw. Meningeal inflammation is not the cause of the muscle spasms.

The American Association on Intellectual and Developmental Disabilities (AAIDD), formerly the American Association on Cognitive Impairment, classifies cognitive impairment based on what parameter? a. Age of onset b. Subaverage intelligence c. Adaptive skill domains d. Causative factors for cognitive impairment

ANS: C The AAIDD has categorized cognitive impairment into adaptive skill domains. The child must demonstrate functional impairment in at least two of the following adaptive skill domains: communication, self-care, home living, social skills, use of community resources, self-direction, health and safety, functional academics, leisure, and work. Age of onset before 18 years is part of the former criteria. Low intelligence quotient (IQ) alone is not the sole criterion for cognitive impairment. Etiology is not part of the classification.

What is a priority of care when a child has an external ventricular drain (EVD)? a. Irrigation of drain to maintain flow b. As-needed dressing changes if dressing becomes wet c. Frequent assessment of amount and color of drainage d. Maintaining the EVD below the level of the child's head

ANS: C The EVD is inserted into the child's ventricle. Frequent assessment is necessary to determine amount of drainage and whether an infection is present. The EVD is a closed system and is not opened for irrigation. Antibiotics may be administered through the drain, but this is usually done by the neuropractitioner. The dressing is not changed. If it becomes wet, then the practitioner should be notified that cerebrospinal fluid (CSF) may be leaking. Unless ordered, maintaining the EVD below the level of the child's head position will create too much pressure and potentially drain too much CSF.

What intervention is most appropriate to facilitate social development of a child with a cognitive impairment? a. Provide age-appropriate toys and play activities. b. Avoid exposure to strangers who may not understand cognitive development. c. Provide peer experiences, such as infant stimulation and preschool programs. d. Emphasize mastery of physical skills because they are delayed more often than verbal skills.

ANS: C The acquisition of social skills is a complex task. Initially, an infant stimulation program should be used. Children of all ages need peer relationships. Parents should enroll the child in preschool. When older, they should have peer experiences similar to those of other children such as group outings, Boy and Girl Scouts, and Special Olympics. Providing age-appropriate toys and play activities is important, but peer interactions facilitate social development. Parents should expose the child to individuals who do not know the child. This enables the child to practice social skills. Verbal skills are delayed more often than physical skills.

A mother states that she brought her child to the clinic because the 3-year-old girl was not keeping up with her siblings. During physical assessment, the nurse notes that the child has pale skin and conjunctiva and has muscle weakness. The hemoglobin on admission is 6.4 g/dl. After notifying the practitioner of the results, what nursing priority intervention should occur next? a. Reduce environmental stimulation to prevent seizures. b. Have the laboratory repeat the analysis with a new specimen. c. Minimize energy expenditure to decrease cardiac workload. d. Administer intravenous fluids to correct the dehydration.

ANS: C The child has a critically low hemoglobin value. The expected range is 11.5 to 15.5 g/dl. When the oxygen-carrying capacity of the blood decreases slowly, the child is able to compensate by increasing cardiac output. With the increasing workload of the heart, additional stress can lead to cardiac failure. Reduction of environmental stimulation can help minimize energy expenditure, but seizures are not a risk. A repeat hemoglobin analysis is not necessary. The child does not have evidence of dehydration. If intravenous fluids are given, they can further dilute the circulating blood volume and increase the strain on the heart.

The clinic nurse is evaluating lab results for a child. What recorded hemoglobin (Hgb) result is considered within the normal range? a. 9 g/dl b. 10 g/dl c. 11 g/dl d. 12 g/dl

ANS: D Normal hemoglobin (Hgb) determination is 11.5 to 15.5 g/dl.

A school-age child is diagnosed with a life-threatening illness. The parents want to protect their child from knowing the seriousness of the illness. The nurse should provide which explanation? a. This attitude is helpful to give parents time to cope. b. This will help the child cope effectively by denial. c. Terminally ill children know when they are seriously ill. d. Terminally ill children usually choose not to discuss the seriousness of their illness.

ANS: C The child needs honest and accurate information about the illness, treatments, and prognosis. Because of the increased attention of health professionals, children, even at a young age, realize that something is seriously wrong and that it involves them. Thus, denial is ineffective as a coping mechanism. The nurse should help parents understand the importance of honesty. Parents may need professional support and guidance from a nurse or social worker in this process. Children will usually tell others how much information they want about their condition.

An 8-year-old child is hit by a motor vehicle in the school parking lot. The school nurse notes that the child is responding to verbal stimulation but is not moving his extremities when requested. What is the first action the nurse should take? a. Wait for the child's parents to arrive. b. Move the child out of the parking lot. c. Have someone notify the emergency medical services (EMS) system. d. Help the child stand to return to play.

ANS: C The child was involved in a motor vehicle collision and at this time is not able to move his extremities. The child needs immediate attention at a hospital for assessment of the possibility of a spinal cord injury. Because the child cannot move his extremities, the child should not be moved until his cervical and vertebral spines are stabilized. The EMS team can appropriately stabilize the spinal column for transport. Although it is important to notify the parents, the EMS system should be activated and transport arranged for serious injuries. The only indication to move the child is to prevent further trauma.

Parents ask the nurse, "When should palliative care be initiated?" What is the best response by the nurse? a. "When curative care is not feasible." b. "When the child's prognosis is uncertain." c. "It should be included along the continuum of care." d. "It should begin when curative treatments are no longer appropriate."

ANS: C The current approach by palliative care experts promotes the inclusion of palliative care along the continuum of care from diagnosis through treatment, not merely at the end of life. It should not wait to be initiated when curative care is not feasible, the child's prognosis is uncertain, or curative treatments are no longer appropriate.

An adolescent has just been brought to the emergency department with a spinal cord injury and paralysis from a diving accident. The parents keep asking the nurse, "How bad is it?" The nurse's response should be based on which knowledge? a. Families adjust better to life-threatening injuries when information is given over time. b. Immediate loss of function is indicative of the long-term consequences of the injury. c. Extent and severity of damage cannot be determined for several weeks or even months. d. Numerous diagnostic tests will be done immediately to determine extent and severity of damage.

ANS: C The extent and severity of damage cannot be determined initially. The immediate loss of function is caused by anatomic and impaired physiologic function, and improvement may not be evident for weeks or months. It is essential to provide information about the adolescent's status to the parents. Immediate treatment information should be provided. Long-term rehabilitation and prognosis can be addressed after the child is stabilized. During the immediate postinjury period, physiologic responses to the injury make an accurate assessment of damage difficult.

A child with severe anemia requires a unit of red blood cells (RBCs). The nurse explains to the child that the transfusion is necessary for which reason? a. Allow her parents to come visit her. b. Fight the infection that she now has. c. Increase her energy so she will not be so tired. d. Help her body stop bleeding by forming a clot (scab).

ANS: C The indication for RBC transfusion is risk of cardiac decompensation. When the number of circulating RBCs is increased, tissue hypoxia decreases, cardiac function is improved, and the child will have more energy. Parental visiting is not dependent on transfusion. The decrease in tissue hypoxia will minimize the risk of infection. There is no evidence that the child is currently infected. Forming a clot is the function of platelets.

What immunoglobulin pattern does the nurse expect in a child recently diagnosed with Wiskott-Aldrich syndrome? a. Diminished levels of IgG b. Diminished levels of IgA c. Diminished levels of IgM d. Diminished levels of IgE

ANS: C The level of IgM is diminished early in the course of the disease, but levels of IgG, IgA, and IgE may be elevated initially.

What is an important nursing consideration when caring for a child with juvenile idiopathic arthritis (JIA)? a. Apply ice packs to relieve acute swelling and pain. b. Administer acetaminophen to reduce inflammation. c. Teach the child and family correct administration of medications. d. Encourage range of motion exercises during periods of inflammation.

ANS: C The management of JIA is primarily pharmacologic. The family should be instructed regarding administration of medications and the value of a regular schedule of administration to maintain a satisfactory blood level in the body. They need to know that nonsteroidal antiinflammatory drugs should not be given on an empty stomach and to be alert for signs of toxicity. Warm, moist heat is best for relieving stiffness and pain. Acetaminophen does not have antiinflammatory effects. Range of motion exercises should not be done during periods of inflammation.

When communicating with other professionals about a child with a chronic illness, what is important for nurses to do? a. Ask others what they want to know. b. Share everything known about the family. c. Restrict communication to clinically relevant information. d. Recognize that confidentiality is not possible in home care.

ANS: C The nurse needs to share, through both oral and written communication, clinically relevant information with other involved health professionals. Asking others what they want to know and sharing everything known about the family are inappropriate measures. Patients have a right to confidentiality. Confidentiality permits the disclosure of information to other health professionals on a need-to-know basis.

The nurse observes that a seriously ill child passively accepts all painful procedures. The nurse should recognize that this is most likely an indication that the child is experiencing what emotional response? a. Hopefulness b. Chronic sorrow c. Belief that procedures are a deserved punishment d. Understanding that procedures indicate impending death

ANS: C The nurse should be particularly alert to a child who withdraws and passively accepts all painful procedures. This child may believe that such acts are inflicted as deserved punishment for being less worthy. A child who is hopeful is mobilized into goal-directed actions. This child would actively participate in care. Chronic sorrow is the feeling of sorrow and loss that recurs in waves over time. It is usually evident in the parents, not in the child. The seriously ill child would actively participate in care. Nursing interventions should be used to minimize the pain.

What information should the nurse include when teaching the mother of a 9-month-old infant about administering liquid iron preparations? a. Give with meals. b. Stop immediately if nausea and vomiting occur. c. Adequate dosage will turn the stools a tarry green color. d. Allow preparation to mix with saliva and bathe the teeth before swallowing.

ANS: C The nurse should prepare the mother for the anticipated change in the child's stools. If the iron dose is adequate, the stools will become a tarry green color. A lack of color change may indicate insufficient iron. The iron should be given in two divided doses between meals when the presence of free hydrochloric acid is greatest. Iron is absorbed best in an acidic environment. Vomiting and diarrhea may occur with iron administration. If these occur, the iron should be given with meals, and the dosage reduced and gradually increased as the child develops tolerance. Liquid preparations of iron stain the teeth; they should be administered through a straw and the mouth rinsed after administration.

A 5-year-old girl sustained a concussion when she fell out of a tree. In preparation for discharge, the nurse is discussing home care with her mother. What sign or symptom is considered a manifestation of postconcussion syndrome and does not necessitate medical attention? a. Vomiting b. Blurred vision c. Behavioral changes d. Temporary loss of consciousness

ANS: C The parents are advised of probable posttraumatic symptoms that may be expected. These include behavioral changes, sleep disturbances, emotional lability, and alterations in school performance. If the child is vomiting, has blurred vision, or has temporary loss of consciousness, she should be seen for evaluation.

What needs to be included as essential teaching for adolescents with systemic lupus erythematosus (SLE)? a. High calorie diet because of increased metabolic needs b. Home schooling to decrease the risk of infections c. Protection from sun and fluorescent lights to minimize rash d. Intensive exercise regimen to build up muscle strength and endurance

ANS: C The photosensitive rash is a major concern for individuals with SLE. Adolescents who spend time outdoors need to use sunscreens with a high SPF, hats, and clothing. Uncovered fluorescent lights can also cause a photosensitivity reaction. The diet should be sufficient in calories and nutrients for growth and development. The use of steroids can cause increased hunger, resulting in weight gain. This can present additional emotional issues for the adolescent. Normal functions should be maximized. The individual with SLE is encouraged to attend school and participate in peer activities. A balance of rest and exercise is important; excessive exercise is avoided.

A preschooler is found digging up a pet bird that was recently buried after it died. What is the best explanation for this behavior? a. He has a morbid preoccupation with death. b. He is looking to see if a ghost took it away. c. He needs reassurance that the pet has not gone somewhere else. d. The loss is not yet resolved, and professional counseling is needed.

ANS: C The preschooler can recognize that the pet has died but has difficulties with the permanence. Digging up the bird gives reassurance that the bird is still present. This is an expected response at this age. If the behavior persists, intervention may be required.

A newborn assessment shows a separated sagittal suture, oblique palpebral fissures, a depressed nasal bridge, a protruding tongue, and transverse palmar creases. These findings are most suggestive of which condition? a. Microcephaly b. Cerebral palsy c. Down syndrome d. Fragile X syndrome

ANS: C These are characteristics associated with Down syndrome. An infant with microcephaly has a small head. Cerebral palsy is a diagnosis not usually made at birth; no characteristic physical signs are present. The infant with fragile X syndrome has increased head circumference; long, wide, or protruding ears; a long, narrow face with a prominent jaw; hypotonia; and a high-arched palate.

A 14-year-old is admitted to the emergency department with a fracture of the right humerus epiphyseal plate through the joint surface. What information does the nurse know regarding this type of fracture? a. It will create difficulty because the child is left handed. b. It will heal slowly because this is the weakest part of the bone. c. This type of fracture requires different management to prevent bone growth complications. d. This type of fracture necessitates complete immobilization of the shoulder for 4 to 6 weeks.

ANS: C This type of fracture (Salter type III) can cause problems with growth in the affected limb. Early and complete assessment is essential to prevent angular deformities and longitudinal growth problems. The difficulty for the child does not depend on the location at the epiphyseal plate. Any fracture of the dominant arm presents obstacles for the individual. Healing is usually rapid in the epiphyseal plate area. Complete immobilization is not necessary. Often these injuries are surgically repaired with open reduction and internal fixation.

What most accurately describes bowel function in children born with a myelomeningocele? a. Incontinence cannot be prevented. b. Enemas and laxatives are contraindicated. c. Some degree of fecal continence can usually be achieved. d. Colostomy is usually required by the time the child reaches adolescence.

ANS: C With a combination of dietary modification, regular toilet habits, and prevention of constipation and impaction, some degree of fecal continence can usually be achieved. Incontinence can be minimized with the development of a regular bowel training program. A surgical intervention can assist with continence. Enemas and laxatives are part of a bowel training program. Colostomies are not indicated in children with myelomeningocele.

The nurse is teaching parents of a child being discharged from the hospital after a splenectomy about the risk of infection. What should the nurse include in the teaching session? (Select all that apply.) a. Avoid obtaining the pneumococcal vaccination for the child. b. Avoid obtaining the meningococcal vaccination for the child. c. The child should receive prophylactic penicillin for certain procedures. d. Have the child immunized with the Haemophilus influenzae type b vaccination. e. Notify the health care provider if your child develops a fever of 38.5° C (101.3° F).

ANS: C, D, E Because of the risk of life-threatening bacterial infection after splenectomy, these children are immunized with the pneumococcal, meningococcal, and H. influenzae type b vaccines before surgery and receive prophylactic penicillin for several years after splenectomy. The parents should be instructed in the importance of seeking immediate medical attention if their child develops a fever of 38.5° C (101.3° F) or higher as a common sign of infection or postsplenectomy sepsis.

The nurse is assessing a child with Down syndrome. The nurse recognizes that which are possible comorbidities that can occur with Down syndrome? (Select all that apply.) a. Diabetes mellitus b. Hodgkin's disease c. Congenital heart defects d. Respiratory tract infections e. Acute megakaryoblastic leukemia

ANS: C, D, E Children with Down syndrome often have multiple comorbidities, contributing to numerous other conditions. Respiratory tract infections are prevalent; when combined with cardiac anomalies, they are the chief cause of death, particularly during the first year. The incidence of leukemia is several times more frequent than expected in the general population, and in about half of the cases, the type is acute megakaryoblastic leukemia.

The nurse is preparing to admit a 7-year-old child with complex partial seizures. What clinical features of complex partial seizures should the nurse recognize? (Select all that apply.) a. They last less than 10 seconds. b. There is usually no aura. c. Mental disorientation is common. d. There is frequently a postictal state. e. There is usually an impaired consciousness.

ANS: C, D, E Clinical features of complex partial seizures include the following: it is common to have mental disorientation, there is frequently a postictal state, and there is usually an impaired consciousness. These seizures last longer than 10 seconds (usually longer than 60 seconds), and there is usually an aura.

The nurse is caring for a child with a subdural hematoma. The nurse should assess for what signs that can indicate brainstem compression? (Select all that apply.) a. Coma b. Lethargy c. Hemiplegia d. Hemiparesis e. Unequal pupils

ANS: C, D, E Hemiparesis, hemiplegia, and anisocoria (unequal pupils) are signs of brainstem compression and require emergency treatment targeted at decreasing increased intracranial pressure. Coma and lethargy are seen with a subdural hematoma but do not indicate a brainstem compression.

The nurse is teaching the parents of a child with a seizure disorder about the triggers that can cause a seizure. What should the nurse include in the teaching session? (Select all that apply.) a. Cold b. Sugared drinks c. Emotional stress d. Flickering lights e. Hyperventilation

ANS: C, D, E The most common factors that may trigger seizures in children include emotional stress, sleep deprivation, fatigue, fever, and physical exercise. Other precipitating factors include sleep, flickering lights, menstrual cycle, alcohol, heat, hyperventilation, and fasting. Cold and sugared drinks are not triggers for seizures.

What refers to a hernial protrusion of a saclike cyst of meninges, spinal fluid, and a portion of the spinal cord with its nerves through a defect in the vertebral column? a. Rachischisis b. Meningocele c. Encephalocele d. Myelomeningocele

ANS: D A myelomeningocele has a visible defect with an external saclike protrusion, containing meninges, spinal fluid, and nerves. Rachischisis is a fissure in the spinal column that leaves the meninges and the spinal cord exposed. Meningocele is a hernial protrusion of a saclike cyst of meninges with spinal fluid but no neural elements. Encephalocele is a herniation of brain and meninges through a defect in the skull, producing a fluid-filled sac.

An 8-year-old girl with moderate cerebral palsy (CP) recently began joining a regular classroom for part of the day. Her mother asks the school nurse about joining the after-school Girl Scout troop. The nurse's response should be based on which knowledge? a. Most activities such as Girl Scouts cannot be adapted for children with CP. b. After-school activities usually result in extreme fatigue for children with CP. c. Trying to participate in activities such as Girl Scouts leads to lowered self-esteem in children with CP. d. Recreational activities often provide children with CP with opportunities for socialization and recreation.

ANS: D After-school and recreational activities serve to stimulate children's interest and curiosity. They help the children adjust to their disability, improve their functional ability, and build self-esteem. Increasing numbers of programs are adapted for children with physical limitations. Almost all activities can be adapted. The child should participate to her level of energy. Self-esteem increases as a result of the positive feedback the child receives from participation.

What intervention should be beneficial in reducing the risk of Reye syndrome? a. Immunization against the disease b. Medical attention for all head injuries c. Prompt treatment of bacterial meningitis d. Avoidance of aspirin for children with varicella or those suspected of having influenza

ANS: D Although the etiology of Reye syndrome is obscure, most cases follow a common viral illness, either varicella or influenza. A potential association exists between aspirin therapy and the development of Reye syndrome, so use of aspirin is avoided. No immunization currently exists for Reye syndrome. Reye syndrome is not correlated with head injuries or bacterial meningitis.

A child has a seizure disorder. What test should be done to gather the most specific information about the type of seizure the child is having? a. Sleep study b. Skull radiography c. Serum electrolytes d. Electroencephalogram (EEG)

ANS: D An EEG is obtained for all children with seizures and is the most useful tool for evaluating a seizure disorder. The EEG confirms the presence of abnormal electrical discharges and provides information on the seizure type and the focus. The EEG is carried out under varying conditions—with the child asleep, awake, awake with provocative stimulation (flashing lights, noise), and hyperventilating. Stimulation may elicit abnormal electrical activity, which is recorded on the EEG. Various seizure types produce characteristic EEG patterns: high-voltage spike discharges are seen in tonic-clonic seizures, with abnormal patterns in the intervals between seizures; a three-per-second spike and wave pattern is observed in an absence seizure; and absence of electrical activity in an area suggests a large lesion, such as an abscess or subdural collection of fluid.

What physiologic defect is responsible for causing anemia? a. Increased blood viscosity b. Depressed hematopoietic system c. Presence of abnormal hemoglobin d. Decreased oxygen-carrying capacity of blood

ANS: D Anemia is a condition in which the number of red blood cells or hemoglobin concentration is reduced below the normal values for age. This results in a decreased oxygen-carrying capacity of blood. Increased blood viscosity is usually a function of too many cells or of dehydration, not of anemia. A depressed hematopoietic system or abnormal hemoglobin can contribute to anemia, but the definition depends on the decreased oxygen-carrying capacity of the blood.

The nurse is discussing long-term care with the parents of a child who has a ventriculoperitoneal shunt. What issues should be addressed? a. Most childhood activities must be restricted. b. Cognitive impairment is to be expected with hydrocephalus. c. Wearing head protection is essential until the child reaches adulthood. d. Shunt malfunction or infection requires immediate treatment.

ANS: D Because of the potentially severe sequelae, symptoms of shunt malfunction or infection must be assessed and treated immediately. Limits should be appropriate to the child's developmental age. Except for contact sports, the child will have few restrictions. Cognitive impairment depends on the extent of damage before the shunt was placed.

How might the quality of life for a terminally ill child and his family be enhanced by nurses? a. Tell the family what is best. b. Leave the family alone to deal with their tragedy. c. Remain objective and uninvolved with family grieving. d. Advocate for and implement pain and symptom relief measures.

ANS: D By increasing personal remembering, the nurse can advocate for and provide the best possible care for the child and family. This is supportive for the family and helps the nurse reduce the stress of caregiving. If the nurse tells the family what is best, this removes the decision making from the parents. It also increases pressure on the nurse to be the expert. The nurse is in a supportive role. The nurse should not leave the family alone to deal with their tragedy. Becoming involved is an objective, deliberate choice. Ideally, the nurse achieves detached concern, which allows sensitive, understanding care because the nurse is sufficiently detached to make objective, rational decisions.

What finding by the nurse is most characteristic of chronic sorrow? a. Lack of acceptance of child's limitation b. Lack of available support to prevent sorrow c. Periods of intensified sorrow when experiencing anger and guilt d. Periods of intensified sorrow at certain landmarks of the child's development

ANS: D Chronic sorrow is manifested by feelings of sorrow and loss that recur in waves over time. The sorrow is a response to the recognition of the child's limitations. The family should be assessed in an ongoing manner to provide appropriate support as their needs change. The sorrow is not preventable. The chronic sorrow occurs during the reintegration and acknowledgment stage.

What factor is most important for parents implementing do not resuscitate (DNR) orders? a. Parents' beliefs about euthanasia b. Presence of other children in the home c. Experiences of the health care team with other children in this situation d. Acknowledgment by health care team that child has no realistic chance for cure

ANS: D Earlier implementation of DNR orders, use of less aggressive therapies, and greater provision of palliative care measures are associated with an honest appraisal of the child's condition. Euthanasia involves an action carried out by a person other than the patient to end the life of the patient suffering from a terminal condition. DNR orders do not involve euthanasia but give permission for health care providers to allow the child to die without intervention. Parents state that regardless of the number of children they have, the death of a child is a new experience and nothing can prepare them for it. Health professionals may base their discussions with families on prior experiences, but families base their decision on an honest appraisal of their child's condition.

One of the techniques that has been especially useful for learners having cognitive impairment is called fading. What description best explains this technique? a. Positive reinforcement when tasks or behaviors are mastered b. Repeated verbal explanations until tasks are faded into the child's development c. Negative reinforcement for specific tasks or behaviors that need to be faded out d. Gradually reduces the assistance given to the child so the child becomes more independent

ANS: D Fading is physically taking the child through each sequence of the desired activity and gradually fading out the physical assistance so the child becomes more independent. Positive reinforcement when tasks or behaviors are mastered is part of behavior modification. An essential component is ignoring undesirable behaviors. Verbal explanations are not as effective as demonstration and physical guidance. Consistent negative reinforcement is helpful, but positive reinforcement that focuses on skill attainment should be incorporated.

What nursing intervention is especially helpful in assessing feelings of parental guilt when a disability or chronic illness is diagnosed? a. Ask the parents if they feel guilty. b. Observe for signs of overprotectiveness. c. Talk about guilt only after the parents mention it. d. Discuss the meaning of the parents' religious and cultural background.

ANS: D Guilt may be associated with cultural or religious beliefs. Some parents are convinced that they are being punished for some previous misdeed. Others may see the disorder as a trial sent by God to test their religious beliefs. The nurse can help the parents explore their religious beliefs. On direct questioning, the parents may not be able to identify the feelings of guilt. It would be appropriate for the nurse to explore their adjustment responses. Overprotectiveness is a parental response during the adjustment phase. The parents fear letting the child achieve any new skill and avoid all discipline.

The mother of a 1-month-old infant tells the nurse she worries that her baby will get meningitis like the child's younger brother had when he was an infant. The nurse should base a response on which information? a. Meningitis rarely occurs during infancy. b. Often a genetic predisposition to meningitis is found. c. Vaccination to prevent all types of meningitis is now available. d. Vaccinations to prevent pneumococcal and Haemophilus influenzae type B meningitis are available.

ANS: D H. influenzae type B meningitis has been virtually eradicated in areas of the world where the vaccine is administered routinely. Bacterial meningitis remains a serious illness in children. It is significant because of the residual damage caused by undiagnosed and untreated or inadequately treated cases. The leading causes of neonatal meningitis are the group B streptococci and Escherichia coli organisms. Meningitis is an extension of a variety of bacterial infections. No genetic predisposition exists. Vaccinations are not available for all of the potential causative organisms.

The nurse is caring for a child with tetanus during the acute phase. What should the nurse plan in the care for this child? a. Playing music on a radio b. Giving frequent back rubs c. Providing bright lighting in the room d. Clustering nursing care to limit distractions

ANS: D In caring for a child with tetanus during the acute phase, every effort should be made to control or eliminate stimulation from sound, light, and touch. Although a darkened room is ideal, sufficient light is essential so that the child can be carefully observed; light appears to be less irritating than vibratory or auditory stimuli. The infant or child is handled as little as possible, and extra effort is expended to avoid any sudden or loud noise to prevent seizures.

What statement best describes iron deficiency anemia in infants? a. It is caused by depression of the hematopoietic system. b. Diagnosis is easily made because of the infant's emaciated appearance. c. It results from a decreased intake of milk and the premature addition of solid foods. d. Clinical manifestations are related to a reduction in the amount of oxygen available to tissues.

ANS: D In iron-deficiency anemia, the child's clinical appearance is a result of the anemia, not the underlying cause. Usually the hematopoietic system is not depressed. The bone marrow produces red blood cells that are smaller and contain less hemoglobin than normal red blood cells. Children who have iron deficiency from drinking excessive quantities of milk are usually pale and overweight. They are receiving sufficient calories but are deficient in essential nutrients. The clinical manifestations result from decreased intake of iron-fortified solid foods and an excessive intake of milk.

A 6-year-old child is admitted for revision of a ventriculoperitoneal shunt for noncommunicating hydrocephalus. What sign or symptom does the child have that indicates a revision is necessary? a. Tachycardia b. Gastrointestinal upset c. Hypotension d. Alteration in level of consciousness

ANS: D In older children, who are usually admitted to the hospital for elective or emergency shunt revision, the most valuable indicators of increasing intracranial pressure are an alteration in the child's level of consciousness, complaint of headache, and changes in interaction with the environment.

What functional ability should the nurse expect in a child with a spinal cord lesion at C7? a. Complete respiratory paralysis b. No voluntary function of upper extremities c. Inability to roll over or attain sitting position d. Almost complete independence within limitations of wheelchair

ANS: D Individuals who sustain injuries at the C7 level are able to achieve a significant level of independence. Some assistance is needed with transfers and lower extremity dressing. Patients are able to roll over in bed and to sit and eat independently. Patients with injuries at C3 or higher have complete respiratory paralysis. Those with injuries at C4 or higher do not have voluntary function of higher extremities. Injuries at C5 or higher prevent rolling over or sitting.

What statement is correct regarding sports injuries during adolescence? a. Conditioning does not help prevent many sports injuries. b. The increase in strength and vigor during adolescence helps prevent injuries related to fatigue. c. More injuries occur during organized athletic competition than during recreational sports participation. d. Adolescents may not possess insight and judgment to recognize when a sports activity is beyond their capabilities.

ANS: D Injuries occur when the adolescent's body is not suited to the sport or when he or she lacks the insight and judgment to recognize that an activity exceeds his or her physical abilities. More injuries occur when an adolescent's muscles and body systems (respiratory and cardiovascular) are not conditioned to endure physical stress. Injuries do not occur from fatigue but rather from overuse. All sports have the potential for injury to the participant, whether the youngster engages in serious competition or in sports for recreation. More injuries occur during recreational sports than during organized athletic competition.

For children who do not have a matched sibling bone marrow donor, the therapeutic management of aplastic anemia includes what intervention? a. Antibiotics b. Antiretroviral drugs c. Iron supplementation d. Immunosuppressive therapy

ANS: D It is thought that aplastic anemia may be an autoimmune disease. Immunosuppressive therapy, including antilymphocyte globulin, antithymocyte globulin, cyclosporine, granulocyte colony-stimulating factor, and methylprednisone, has greatly improved the prognosis for patients with aplastic anemia. Antibiotics are not indicated as the management. They may be indicated for infections. Antiretroviral drugs and iron supplementation are not part of the therapy.

The majority of children in the United States with human immunodeficiency virus (HIV) infection acquired the disease by which means? a. Through sexual contact b. From a blood transfusion c. By using intravenous (IV) drugs d. Perinatally from their mothers

ANS: D More than 90% of the children with HIV under 13 years who were reported to the Centers for Disease Control and Prevention acquired the infection during the perinatal period. With intervention, the number of children infected can be decreased. Sexual contact and IV drug use are the leading causes of infection in the 14- to 19-year age group. This number is less than the number of cases in the under 13-year age group. Transfusion has accounted for 3% to 6% of all pediatric acquired immunodeficiency syndrome cases to date. Before 1985 and routine screening of donated blood products, children with hemophilia were at great risk from pooled plasma products.

Neuropathic bladder disorders are common among children with which disorder? a. Plagiocephaly b. Meningocele c. Craniosynostosis d. Myelomeningocele

ANS: D Myelomeningocele is one of the most common causes of neuropathic bladder dysfunction among children. Plagiocephaly is the flattening of a side of the child's head. This is not associated with neuropathic bladder. Children with meningocele usually do not have neuropathic bladder. Craniosynostosis is the premature closure of one or more cranial sutures. It is not associated with neuropathic bladder.

A 12-year-old boy is in the final phase of dying from leukemia. He tells the nurse who is giving him opiates for pain that his grandfather is waiting for him. How should the nurse interpret this situation? a. The boy is experiencing side effects of the opiates. b. The boy is making an attempt to comfort his parents. c. He is experiencing hallucinations resulting from brain anoxia. d. He is demonstrating readiness and acceptance that death is near.

ANS: D Near the time of death, many children experience visions of "angels" or people and talk with them. The children mention that they are not afraid and that someone is waiting for them. If the child has built a tolerance to the opioids, side effects are not likely. At this time, many children do begin to comfort their families and tell them that they are not afraid and are ready to die, but the visions usually precede this stage. There is no evidence of tissue hypoxia.

A 23-month-old child is admitted to the hospital with a diagnosis of meningitis. She is lethargic and very irritable with a temperature of 102° F. What should the nurse's care plan include? a. Observing the child's voluntary movement b. Checking the Babinski reflex every 4 hours c. Checking the Brudzinski reflex every 1 hour d. Assessing the level of consciousness (LOC) and vital signs every 2 hours

ANS: D Observation of vital signs, neurologic signs, LOC, urinary output, and other pertinent data is carried out at frequent intervals on a child with meningitis. The nurse should avoid actions that cause pain or increase discomfort, such as lifting the child's head, so the Brudzinski reflex should not be checked hourly. Checking the Babinski reflex or child's voluntary movements will not help with assessing the child's status.

The nurse is caring for a child with hemophilia A. The child's activity is as tolerated. What activity is contraindicated for this child? a. Ambulating to the cafeteria b. Active range of motion c. Ambulating to the playroom d. Passive range of motion exercises

ANS: D Passive range of motion exercises should never be part of an exercise regimen after an acute episode because the joint capsule could easily be stretched and bleeding could recur. Active range of motion exercises are best so that the patient can gauge his or her own pain tolerance. The child can ambulate to the playroom or the cafeteria.

The nurse is caring for a child with severe head trauma after a car accident. What is an ominous sign that often precedes death? a. Delirium b. Papilledema c. Flexion posturing d. Periodic or irregular breathing

ANS: D Periodic or irregular breathing is an ominous sign of brainstem (especially medullary) dysfunction that often precedes complete apnea. Delirium is a state of mental confusion and excitement marked by disorientation for time and place. Papilledema is edema and inflammation of the optic nerve. It is commonly a sign of increased intracranial pressure. Flexion posturing is seen with severe dysfunction of the cerebral cortex or of the corticospinal tracts above the brainstem.

The nurse is preparing a community outreach program about the prevention of iron-deficiency anemia in infants. What statement should the nurse include in the program? a. Whole milk can be introduced into the infant's diet in small amounts at 6 months. b. Iron supplements cannot be given until the infant is older than 1 year of age. c. Iron-fortified cereal should be introduced to the infant at 2 months of age. d. Breast milk or iron-fortified formula should be used for the first 12 months.

ANS: D Prevention, the primary goal in iron-deficiency anemia, is achieved through optimal nutrition and appropriate iron supplements. The American Academy of Pediatrics recommends feeding an infant only breast milk or iron-fortified formula for the first 12 months of life. Whole cow's milk should not be introduced until after 12 months, iron supplements can be given during the first year of life, and iron-fortified cereals should not be introduced until the infant is 4 to 6 months old.

Parents ask for help for their other children to cope with the changes in the family resulting from the special needs of their sibling. What strategy does the nurse recommend? a. Explain to the siblings that embarrassment is unhealthy. b. Encourage the parents not to expect siblings to help them care for the child with special needs. c. Provide information to the siblings about the child's condition only as requested. d. Invite the siblings to attend meetings to develop plans for the child with special needs.

ANS: D Siblings should be invited to attend meeting to be part of the care team for the child. They can learn about an individualized education plan and help design strategies that will work at home. Embarrassment may be associated with having a sibling with a chronic illness or disability. Parents must be able to respond in an appropriate manner without punishing the sibling. The parents may need assistance with the care of the child. Most siblings are positive about the extra responsibilities. Parents need to inform the siblings about the child's condition before a nonfamily member does so. The parents do not want the siblings to fantasize about what is wrong with the child.

Several nurses tell their nursing supervisor that they want to attend the funeral of a child for whom they had cared. They say they felt especially close to both the child and the family. The supervisor should recognize that attending the funeral serves what purpose? a. It is improper because it increases burnout. b. It is inappropriate because it is unprofessional. c. It is proper because families expect this expression of concern. d. It is appropriate because it can assist in the resolution of personal grief.

ANS: D Some nurses find shared remembrance rituals useful in resolving grief. Attending funeral services can be a supportive act for both the family and the nurse. Burnout is a state of physical, emotional, and mental exhaustion. It results from prolonged involvement with individuals in situations that are emotionally demanding. Attending the funeral of a child can be an effective coping measure. Attending funerals does not detract from the professionalism of care. Although it is important to consider the family's expectations, the act of attending the funeral provides a sense of closure with the family and facilitates the grief process for the nurse.

A young adolescent experiences infrequent migraine episodes. What pharmacologic intervention is most likely to be prescribed? a. Opioid b. Lorazepam c. Ergotamine d. Sumatriptan

ANS: D Sumatriptan is a serotonin agonist at specific vascular serotonin receptor sites and causes vasoconstriction in large intracranial arteries. Opioids are used infrequently because they rarely work on the mechanism of pain. Lorazepam is a benzodiazepine that acts as an anxiolytic and sedative. It is not indicated for treatment of migraine episodes. Ergotamine, an a-adrenergic blocker, is used for adult vascular headaches, but it is not used in adolescents because of the side effects.

A 7-year-old child is in the end stages of cancer. The parents ask you how they will know when death is imminent. What physical sign is indicative of approaching death? a. Hunger b. Tachycardia c. Increased thirst d. Difficulty swallowing

ANS: D The child begins to have difficulty swallowing as he or she approaches death. The child's appetite will decrease, and he or she will take only small bites of favorite foods or sips of fluids in the final few days. The pulse rate will slow.

The nurse is preparing to administer a unit of packed red blood cells to a hospitalized child. What is an appropriate action that applies to administering blood? a. Take the vital signs every 15 minutes while blood is infusing. b. Use blood within 1 hour of its arrival from the blood bank. c. Administer the blood with 5% glucose in a piggyback setup. d. Administer the first 50 ml of blood slowly and stay with the child.

ANS: D The nurse should administer the first 50 ml of blood or initial 20% of volume (whichever is smaller) slowly and stay with the child. Vitals signs should be taken 15 minutes after initiation and then every hour, not every 15 minutes. Blood should be used within 30 minutes, not 1 hour. Normal saline, not 5% glucose, should be the IV solution.

An 8-year-old girl is receiving a blood transfusion when the nurse notes that she has developed precordial pain, dyspnea, distended neck veins, slight cyanosis, and a dry cough. These manifestations are most suggestive of what complication? a. Air embolism b. Allergic reaction c. Hemolytic reaction d. Circulatory overload

ANS: D The signs of circulatory overload include distended neck veins, hypertension, crackles, a dry cough, cyanosis, and precordial pain. Signs of air embolism are sudden difficulty breathing, sharp pain in the chest, and apprehension. Urticaria, pruritus, flushing, asthmatic wheezing, and laryngeal edema are signs and symptoms of allergic reactions. Hemolytic reactions are characterized by chills, shaking, fever, pain at infusion site, nausea, vomiting, tightness in chest, flank pain, red or black urine, and progressive signs of shock and renal failure.

Spina Bifida Occulta "Tethered cord"

Abnormal adhesion to bony or fixed structure Puts traction on the cord Altered gait Bowel and bladder problems Foot deformities May not be seen in early infancy

How are the clinical manifestations of lower motor neuron syndrome different from those of upper motor neuron syndrome? Select all that apply. Absence of flexor spasms Presence of flaccid paralysis Presence of intact reflex arcs Presence of skin and tissue changes Absence of wastage of muscle mass

Absence of flexor spasms Presence of flaccid paralysis Presence of skin and tissue changes Rationale Flexor spasms are absent in lower motor neuron syndrome. Lower motor neuron syndrome manifests as flaccid paralysis caused by muscle atonia. Skin and tissue changes can be seen in lower motor neuron syndrome. Intact reflex arcs can be seen in upper motor neuron syndrome, whereas reflex arcs are permanently damaged in lower motor neuron syndrome. There is no wastage of muscle mass in upper motor neuron syndrome, because of increased muscle tone.

When a child with mild cognitive impairment reaches the end of adolescence, what characteristics would be expected? Achieves a mental age of 5 to 6 years. Achieves a mental age of 8 to 12 years. Is unable to progress in functional reading or mathematics. Acquires practical skills and useful reading and mathematics to an eighth-grade level.

Achieves a mental age of 8 to 12 years. Correct By the end of adolescence, the child with mild cognitive impairment can acquire practical skills and useful reading and math skills to a third- to sixth-grade level. A mental age of 8 to 12 years is obtainable, and the child can be guided toward social conformity. A mental age of 5 to 12 years is characteristic of children with moderate cognitive impairment. Practical skills and useful reading and mathematics at an eighth-grade level are not descriptive of cognitive impairment.

promotion of normal development: early childhood

Achieving basic trust (satisfying, consistent relations with parents, more parental support and expression of affection) parents need help to meet infant's needs achieving separation from parents, autonomy, initiative. natural response is overprotection, so need help with understanding importance of separation, create social experience outsde home , beginning independence and discipline ( boundaries, teaching them socially acceptable behavior) parents need to teach behavior before it becomes a problem

A family wants to begin oral feeding of their 4-year-old son, who is ventilator dependent and currently tube fed. They ask the home health nurse to feed him the baby food orally. The nurse recognizes a high risk of aspiration and an already compromised respiratory status. What is the most appropriate nursing action? Refuse to feed the child orally because the risk is too high. Explain the risks involved and then let the family decide what should be done. Feed the child orally because the family has the right to make this decision for him. Acknowledge their request, explain the risks, and explore with the family the available options.

Acknowledge their request, explain the risks, and explore with the family the available options. Correct Parents want to be included in the decision making for their child's care. The nurse should discuss the request with the family to ensure this is the issue of concern, and then potential options can be explored. Refusing to feed the child orally does not determine why the parents want oral feedings to begin and does not involve them in problem-solving. The decision to begin oral feedings or not to change the feedings should be a collaborative one, made in consultation with the family, the nurse, and an appropriate member of the health care team.

Home care is being considered for a young child who is ventilator dependent. What factor is most important in deciding whether home care is appropriate? Level of education of parents Adequate family training Presence of two parents in the home Family's ability to assume all health care costs

Adequate family training Correct One of the essential elements is the family's preparation and ability to care for the child. The family must be able to demonstrate all aspects of care for the child because in many areas nursing care may not be available on a continual basis. The amount of formal education reached by the parents is not the important issue. The determinant is the family's ability to care adequately for the child in the home. At least two family members should learn and demonstrate all aspects of the child's care in the hospital, but it does not have to be two parents. Few families can assume all health care costs. Creative financial planning, including negotiating arrangements with the insurance company or public programs, may be required.

A child is experiencing phantom limb pain after a total knee amputation. Which is the best measure to control the pain? Administer analgesics. Check for neuroma in the stump. Tell the child that the pain is unreal. Suggest exercises for muscle strengthening.

Administer analgesics. Rationale The child may experience phantom limb pain. It is real and should be treated with analgesics and other measures of pain management. If the limb pain increases with ambulation, it may indicate a neuroma at the free nerve endings in the stump and should be evaluated immediately. The parents and caregivers should always show their support to the child and always listen to the child; they should treat the phantom limb pain as a real pain. Phantom limb pain does not go away with muscle strengthening exercises; the sensation fades away gradually.

A 14-year-old patient with sickle cell disease presents with reports of severe joint, chest, and abdominal pain after having the flu for 3 days. What orders should the nurse anticipate receiving for this patient? Select all that apply. Administer intravenous fluids. Administer the flu vaccine. Have the patient remain in bed. Provide intravenous pain medication. Encourage use of an incentive spirometer. Perform passive range-of-motion exercises with affected joint.

Administer intravenous fluids. Severe joint pain and abdominal pain are signs of sickle cell crisis. Patients in sickle cell crisis require hydration and fluid resuscitation. Provide intravenous pain medication. Severe joint pain and abdominal pain are signs of sickle cell crisis. Patients in sickle cell crisis are given IV pain medication to promote comfort. Encourage use of an incentive spirometer. Severe joint, chest, and abdominal pain indicate sickle cell crisis. Some patients in crisis develop respiratory compromise caused vaso-occlusion in the lung vessels and therefore incentive spirometer is encouraged. Perform passive range-of-motion exercises with affected joint. Passive range-of-motion exercises are indicated to promote and maintain circulation in the affected area.

Which nursing intervention is appropriate when preventing antithymocyte globulin (ATG) from accumulating in the extravascular space of an aplastic anemia patient causing extravasation? Administer oxygen immediately. Closely monitor the child for fever and rashes. Administer medication to prevent tissue damage. Immediately administer systemic corticosteroids.

Administer medication to prevent tissue damage. Rationale Aplastic anemia is primarily treated with immunosuppressive agents such as antithymocyte globulin (ATG), which makes the child susceptible to infections. Extravasation of the drug may cause tissue damage, so medication needs to be administered. Oxygen may be required if anaphylaxis occurs. Fever and skin rashes immediately after ATG administration are normal reactions. Serum sickness is a delayed reaction due to ATG therapy. It is usually seen within 7 to 14 days of ATG administration. Serum sickness may be prevented by administering systemic corticosteroids.

Which nursing interventions are appropriate for the child undergoing a blood transfusion? Select all that apply. Administer the first 50 mL of blood slowly. Administer blood through an appropriate filter. Shake the blood container frequently while the transfusion is in progress. Check the blood group and blood type of the child and the donor before the transfusion. Take vital signs only before initiation, 15 minutes after initiation, and after completion of transfusion. Use blood within 30 minutes of its arrival from the blood bank, or store it in a regular unit refrigerator.

Administer the first 50 mL of blood slowly. Administer blood through an appropriate filter. Shake the blood container frequently while the transfusion is in progress. Check the blood group and blood type of the child and the donor before the transfusion. Rationale The first 50 mL of blood would be administered slowly, and the nurse should stay with the child to check for any reaction. Blood would be administered through an appropriate filter to eliminate particles in the blood. During the process, gently shake the blood container frequently to prevent precipitation. Always check the donor and recipient's blood group and blood type before the transfusion, to prevent any complications. Once blood arrives from the blood bank, it should be used within 30 minutes. If not used, it would be returned to the blood bank, not stored in a regular unit refrigerator. Vital signs should be taken prior to transfusion, 15 minutes after initiation, hourly while the transfusion is in progress, and after the transfusion is complete.

A child is brought to the emergency department with a suspected spinal cord injury at the level of C2. What is the immediate priority in the nursing care of a patient with this injury? Administer ventilatory support Assess child's visual field for spots Manage hypertension and bradycardia Provide permanent cervical stabilization

Administer ventilatory support A spinal cord injury at the level of C2 will cause the patient to be apneic and require ventilator support which is the immediate priority in the nursing care plan in this patient.

Which priority nursing consideration is appropriate for a child with suspected bacterial meningitis? Supporting the family Instituting standard precautions Administering antibiotics as soon as possible Administering pain medications around the clock

Administering antibiotics as soon as possible Rationale The priority nursing consideration in a child with suspected bacterial meningitis is to administer antibiotics as soon as possible. Supporting the family is important, but the priority nursing consideration for a child with suspected bacterial meningitis is to administer antibiotics as soon as possible. Early isolation, rather than standard precautions, is recommended. Administering pain medications around the clock is important for children who are in pain, but the priority nursing consideration for a child with suspected bacterial meningitis is to administer antibiotics as soon as possible.

Which nursing intervention is appropriate for a child who develops a severe vasoocclusive crisis? Administration of analgesic to relieve pain Application of cold compress to the affected area Administration of oxygen to reverse sickled red blood cells Administration of prophylactic antibiotics to prevent infection

Administration of analgesic to relieve pain Rationale Vasoocclusive crisis is a very painful condition. The first effort should be to relieve pain. Applying a cold compress to the affected area enhances vasoconstriction and occlusion, thereby aggravating the situation. Oxygen administration is not very beneficial, unless hypoxemia is present. Prophylactic antibiotics may be administered later in order to prevent infections.

What statement is correct regarding sports injuries during adolescence? Rapidly growing bones, muscles, joints, and tendons offer some protection from unusual strain. The increase in strength and vigor during adolescence helps prevent injuries related to fatigue. More injuries occur during organized athletic competition than during recreational sports participation. Adolescents may not possess the insight and judgment to recognize when an activity is beyond their capabilities.

Adolescents may not possess the insight and judgment to recognize when an activity is beyond their capabilities. Correct Children and adolescents may not possess the insight and judgment to recognize when an activity is beyond their capabilities. Rapidly growing bones, muscles, joints, and tendons are especially vulnerable to unusual strain. The increase in strength and vigor in adolescence may tempt adolescents to overextend themselves. More injuries occur during recreational sports participation than in organized athletic competition.

Which nursing response is appropriate for the mother asking about her 8-year-old son with moderate cerebral palsy (CP) joining the after-school Scout troop? <p>Which nursing response is appropriate for the mother asking about her 8-year-old son with moderate cerebral palsy (CP) joining the after-school Scout troop?</p> Most activities such as Scouting cannot be adapted for children with CP. After-school activities usually result in extreme fatigue for children with CP. Trying to participate in activities such as Scouting leads to lowered self-esteem in children with CP. After-school activities often provide children with CP with opportunities for socialization and recreation.

After-school activities often provide children with CP with opportunities for socialization and recreation. Rationale Recreational outlets and after-school activities should be considered for the child who is unable to participate in athletic programs as a means of promoting socialization. Most activities can be adapted for children with cerebral palsy. The child, family, and activity director should assess the degree of activity to ensure that it matches the child's capabilities. A supportive environment associated with after-school activities will add to the child's self-esteem.

The nurse is caring for a 16-year-old admitted for complications of hemophilia. Which psychosocial interventions are important for the nurse to implement? Select all that apply. Allow friends to visit. Allow the patient to express feelings. Administer factor replacement as ordered. Assess the patient's understanding of safety precautions. Place padding over the bed rails, head board, and foot board. Ask the patient if he or she would like to be introduced to other patients of the same age with a similar diagnosis.

Allow friends to visit. Adolescents are greatly influenced by their relationships with peers. The nurse should allow friends to visit the patient to maintain current important relationships. Correct Allow the patient to express feelings. Adolescents should be encouraged to verbalize their feelings to promote effective coping. Assess the patient's understanding of safety precautions. The nurse should assess the patient's understanding of necessary safety precautions and the risk for injury. Ask the patient if he or she would like to be introduced to other patients of the same age with a similar diagnosis. Adolescents are greatly influenced by their relationships with peers. The nurse should help to identify a support system for a patient with hemophilia.

The nurse notes that an adolescent that has been in the hospital for over a week has recently stopped talking to the staff when they enter the room. Which response by the nurse is most appropriate? Have the adolescent call a friend to talk about the illness and increase communication Allow the adolescent to make choices regarding how and when certain parts of care are completed Encourage the adolescent to have some peers come and visit during the hospitalization to decrease loneliness Tell the adolescent that interacting with staff and communicating needs must be done if treatment is going to be effective

Allow the adolescent to make choices regarding how and when certain parts of care are completed A lack of communication or change in communication can be an indication of a decrease in self-esteem and independence related to dealing with an illness. Allowing the adolescent to make some decisions regarding care can increase independence and self-esteem.

The nurse senses sadness in a parent with a child who has terminal cancer. Which action is the most appropriate for the nurse take in this situation? Place a referral to the hospital's chaplain. Allow the parent to talk and express emotions. Place a referral to a psychotherapist for grief counseling. Provide the parent with a list of support groups in the area.

Allow the parent to talk and express emotions. Allowing the parent to talk and express emotions provides the opportunity for the parent to grieve and express grief in a comfortable manner, and therefore this is the most appropriate action for the nurse to take.

To prepare a 9-year-old child with muscular dystrophy for school, the school nurse should establish goals to help the child achieve maximum potential for normal socialization. What action will assist the school nurse in achieving these goals? Limiting recess activities to 30 minutes per day Providing the child with an assistant for all activities during school hours Keeping the child in a special education classroom with other children with similar disabilities Allowing the child to use assistive devices to move about independently through the school

Allowing the child to use assistive devices to move about independently through the school Allowing the child to use assistive devices for mobility encourages independence and allows the child opportunities to socialize and interact with peers.

Which statements describe functions of the musculoskeletal system? Select all that apply. Allows movement Protects the heart Regulates mineral imbalances Produces hormones for growth Provides a skeletal framework

Allows movement Providing movement is a function of the musculoskeletal system. Protects the heart Protecting vital organs, including the heart, is a function of the musculoskeletal system. Provides a skeletal framework Providing a skeletal framework to support the body is a function of the musculoskeletal system.

Which drug is appropriate for abortive treatment of migraine in a 15-year-old adolescent? Almotriptan Sumatriptan Antiepileptics Tricyclic antidepressants

Almotriptan Rationale Almotriptan is the drug of choice for abortive treatment of migraine in an adolescent aged 15 years because it is the only approved triptan medication in United States that can be used in adolescents aged 12 to 17 years. Triptan is a serotonin agonist that is effective in the abortive treatment of migraines. Sumatriptan is commonly used for migraine treatment among children in Europe and the United Kingdom, but it is not approved for use in children in the United States. Antiepileptics and tricyclic antidepressants are prophylactic medications that can be used in children who experience frequent or refractory migraines.

Female athlete triad

Amenorrhea Osteoporosis Low caloric intake cardiovascular disease may be 4th element disordered eating - food restrictions, rigid food pattern, fasting, vomiting, use of diet pills, laxatives -> poor protein intake, law fat intake, inadequate caloric inatek eating disorders - bulimia, anorexia Tx: Long term, nutritional plan, psychologic health interventions

A patient is being treated for osteomyelitis. What symptoms would the nurse report to the health care provider? Select all that apply. A. An elevated temperature B. Increased range of motion C. Complaints of localized pain D. Pain radiating away from site E. Increased redness and swelling around site

An elevated temperature An elevated temperature could indicate the antibiotic is not working. Pain radiating away from site Radiating pain to an adjacent joint necessitates an assessment for possible septic arthritis. Correct Increased redness and swelling around site If signs and symptoms get worse, the health care provider should be notified.

What is included in the therapeutic management of a child with tetanus? Nonsteroidal antiinflammatory drugs (NSAIDs) to reduce inflammation Muscle stimulants to counteract muscle weakness Bronchodilators to prevent respiratory complications Antibiotics to control bacterial proliferation at the site of injury

Antibiotics to control bacterial proliferation at the site of injury Correct Antibiotics are administered to control the proliferation of the vegetative forms of the organism at the site of infection. Tetanus toxin acts at the myoneural junction to produce muscular stiffness and lowers the threshold for reflex excitability. NSAIDs are not routinely used. Sedatives or muscle relaxants are used to help reduce titanic spasm and prevent seizures. Respiratory status is carefully evaluated for any signs of distress because muscle relaxants, opioids, and sedatives that may be prescribed may cause respiratory depression. Bronchodilators would not be used unless specifically indicated.

Which characteristic symptoms of rabies suggest that the disease causes hydrophobia? Select all that apply. Apnea Anoxia Cyanosis Anorexia Sore throat Hypersensitivity

Apnea Anoxia Cyanosis Rationale Apnea, anoxia, and cyanosis are the characteristic symptoms experienced by a rabies patient when the patient tries to swallow. Swallowing causes spasms of respiratory muscles and results in apnea, anoxia, and cyanosis. Therefore, the term hydrophobia is used. Anorexia and sore throat are the initial flu-like symptoms of rabies. Hypersensitivity causing increased reaction to external stimuli is seen in the excitement phase of rabies. However, the term hydrophobia is not based on those symptoms.

A female patient with Von Willebrand disease (vWD) presents with an open leg fracture after falling at cheerleading practice. The nurse notes no other fractures, but the bleeding continues after 35 minutes. Which action should the nurse take first? Assess the patient's hemoglobin. Administer an intravenous (IV) normal saline bolus. Apply a pressure dressing to the affected area. Initiate a transfusion of Von Willebrand factor.

Apply a pressure dressing to the affected area. A pressure dressing should be applied to help stop the bleeding.

A child is brought to the hospital with a suspected fracture. The child is crying and refuses to walk. What intervention helps relieve the child from pain and discomfort? Administer an anesthetic agent as prescribed. Apply a hot bag to the affected area and rub gently. Apply a splint that covers the joint above and below the fracture. Massage the affected area, apply a local anesthetic, and bandage.

Apply a splint that covers the joint above and below the fracture. Rationale Fractures involve broken bones, and the pain increases with increased movement. Therefore it is extremely important to immobilize the fractured bone. Splinting from the joint above and below the fractured part helps to avoid movement and provides rest and support to the injured part. Administering an anesthetic may not help in relieving the pain. Hot bag applications are not recommended in the initial management of fractures, because they lead to vasodilation and can increase discomfort. Massaging the area increases pain.

After the circumcision of a newborn male, the nurse notes prolonged bleeding. Which actions should the nurse take? Select all that apply. Administer an intravenous (IV) fluid bolus. Obtain a partial thromboplastin time (PTT). Administer oral analgesic medications. Apply gentle pressure for 10 to 15 minutes. Notify the provider about the prolonged bleeding time.

Apply gentle pressure for 10 to 15 minutes. If the nurse notes prolonged bleeding at the circumcision site, applying pressure will help to promote vaso-occlusion and stop the bleeding. Notify the provider about the prolonged bleeding time. The health care provider should be notified of prolonged bleeding so that further orders can be obtained.

The child presents to the school nurse's office after twisting an ankle in physical education class. Which is the nurse's first action? Observe for edema. Apply ice on the site. Elevate the extremity. Administer analgesics

Apply ice on the site. Rationale A child with a twisted ankle has suffered a soft tissue injury. The area should be iced immediately to reduce pain and to promote healing. With severe injury, the area is swollen. Instead of waiting to observe for edema, the nurse first should apply ice. Next, the nurse should apply an elastic wrap to provide compression and to keep the ice pack in place, and then elevate the extremity. Elevating the extremity helps reduce the edema formation in the injured area. Analgesics may be given afterwards to relieve the discomfort.

A 12-year-old in the clinic reports "severe headaches." The nurse should include which focused assessments for this patient? Correct Ask about medication history. Auscultate for a bruit in the head. Assess mental status and level of consciousness. Evaluate gait, coordination, and muscle strength. Anticipate health care provider examination of optic disks for papilledema.

Ask about medication history. The nurse should ask about medication history of this child as part of evaluating this patient. Daily use of analgesics may cause rebound headaches. Correct Auscultate for a bruit in the head. The nurse should auscultate for a bruit as an indication of an aneurysm as part of evaluating patient. Correct Assess mental status and level of consciousness. The nurse should assess mental status of child as part of evaluating patient. Anticipate health care provider examination of optic disks for papilledema. The nurse should anticipate health care provider examination of optic disks for papilledema in child as part of evaluating patient.

The nurse is caring for a school-aged child with a chronic illness that requires the child to wear a mask when leaving the hospital room. The child does not want to attend social activities on the unit. Which action by the nurse can help encourage the child to participate? Ask the child why the hesitation to wear the mask when leaving the hospital room Bargain with the child to attend at least one activity a week to help increase the child's socialization on the unit Encourage the child to attend the social activities on the unit without the mask so that the child does not feel singled out Allow the child to decline attendance and participation in the social activities on the unit to decrease emotional distress and embarrassment

Ask the child why the hesitation to wear the mask when leaving the hospital room The nurse should question the child about the reason for not wanting to wear the mask and should allow the child to verbalize feelings.

A husband and wife are speaking to the nurse about their infant who has cerebral palsy. The father seems anxious when interacting with the infant. What is the nurse's best response to this situation? Have both parents work together to care for the infant so that neither parent feels singled out Ask the father to assist the nurse with bathing the infant and encourage the father to ask questions Ensure that the father is not left alone with the child because the father seems uncomfortable and unsafe Address the education primarily to the mother because she seems to feel more comfortable caring for the infant

Ask the father to assist the nurse with bathing the infant and encourage the father to ask questions Specifically involving the father in the care of the child and encouraging questions is important to increase the comfort level of the father.

A 16-year-old patient with cerebral palsy is having a birthday party and receiving a lot of attention from family and the home health care staff. A younger sibling is at the birthday party but looking unhappy. Which actions by the home health nurse are appropriate in this situation? Ask the sibling about a special personal birthday event. Talk with the parents about reading a book to the sibling every night. Talk with the parents about taking the sibling to a movie later that day. Encourage the sibling to help cut the birthday cake and scoop ice cream. Encourage the sibling to avoid being negative and to be a positive support for the sibling.

Ask the sibling about a special personal birthday event. This action provides sibling-centered interactions that foster communication and sense of importance in the sibling because it acknowledges important events valued by the sibling. Correct Talk with the parents about reading a book to the sibling every night. Reading a book together daily provides a consistent time for interaction with the parents. This fosters positive relationships between the parents and the sibling. Correct Talk with the parents about taking the sibling to a movie later that day. Encouraging the parents to spend some time every day with the sibling is essential for the family to maintain positive relationships. Correct Encourage the sibling to help cut the birthday cake and scoop ice cream. Becoming involved with activities allows the sibling to interact with the parents and ill child. It is a source of attention for the sibling.

The parents of a 2-year-old report that the child has had episodes of epistaxis three times in the last week. On assessment, the nurse notes bruising across the chest and back. Which actions are important for the nurse to take? Select all that apply. Ask whether there is any gum bleeding when teeth are brushed. Administer a one-time fluid bolus of lactated Ringer's solution. Observe the parent-child interaction for signs of abuse. Determine length of time the bleeding lasts with each nosebleed. While obtaining the health history, ask specifically how the bruises occurred.

Ask whether there is any gum bleeding when teeth are brushed. The nurse should ask the patient and family about a history of bleeding from other locations, as this can help indicate the severity of the disorder. Observe the parent-child interaction for signs of abuse. Bruising in multiple stages of healing is a sign of physical abuse. The nurse should carefully assess the child to distinguish signs of abuse from bleeding related to Von Willebrand disease. Correct Determine length of time the bleeding lasts with each nosebleed. The nurse should question the patient and family about the duration and onset of epistaxis. Correct While obtaining the health history, ask specifically how the bruises occurred. The nurse should question the patient and family about the mechanism of injury and the history of present illness to help identify a cause of the current problem.

A child with Down syndrome may be screened for what before participating in some sports? Hyperflexibility Cutis marmorata Atlantoaxial instability Speckling of the iris (Brushfield spots)

Atlantoaxial instability Correct Children with Down syndrome are at risk for atlantoaxial instability. Before they participate in sports that put stress on the head and neck, a radiologic examination should be done. Hyperflexibility, cutis marmorata, and speckling of the iris (Brushfield spots) are characteristic of Down syndrome, but they do not affect the child's ability to participate in sports.

The nurse is caring for a child who has sustained an acceleration-deceleration head injury. Which actions should the nurse take in assessing this patient? Check child's gag reflex. Assess child for retinal injury. Check child for burns and bruising. Assess for associated extremity sprain. Contact health care provider because child needs head computed tomography (CT).

Assess child for retinal injury. An acceleration-deceleration head injury occurs with "shaken baby" syndrome, which is associated with retinal tears and hemorrhaging. This child will need to be assessed for retinal damage. Correct Check child for burns and bruising. Acceleration-deceleration injuries are associated with child abuse, and so checking for other signs of abuse will be necessary. Correct Assess for associated extremity sprain. Extremity sprains are not an associated condition occurring with acceleration-deceleration head injuries. They may however exist separately if child has been abused. Correct Contact health care provider because child needs head computed tomography (CT). The child has suffered from an acceleration-deceleration head injury and may have an epidural hematoma that will need to be diagnosed by a head CT scan.

The nurse assesses a pediatric patient and finds deficits in speech. What additional assessment does the nurse perform to gather more data about the patient's speech deficit? Assess for drowsiness and jitteriness. Evaluate for hearing loss and deafness. Determine if oral-motor weakness is present. Observe interactions between child and parents. Inquire about potential nonaccidental injury since deficits in speech usually indicate history of abuse.

Assess for drowsiness and jitteriness. As part of Glasgow coma scale, the nurse will first assess if child's cognition is intact before assessing other reasons for speech deficit. If GCS is low, patient may require immediate intervention. Correct Evaluate for hearing loss and deafness. Once GCS is determined to be adequate, nurse can assess for other issues such as hearing impairment. Correct Determine if oral-motor weakness is present. Occasionally, there will be a muscular weakness or a nerve not functioning properly that can cause a speech deficit. Correct Observe interactions between child and parents. A complete assessment of speech dysfunction will include listening to and observing interactions. This may be a clue to many issues that could cause speech deficit, including autism.

The nurse is caring for a child with hepatosplenomegaly related to beta-thalassemia. Which actions should the nurse take to minimize the risks related to this disorder? Select all that apply. Assess for signs of bleeding. Limit the number of visitors. Administer analgesics as ordered. Tell the patient to avoid contact sports. Monitor patient for signs of abdominal trauma

Assess for signs of bleeding. Patients with hepatosplenomegaly are at risk for bleeding related to increased friability of the enlarged liver and spleen. The nurse should observe the patient for signs of bleeding. Tell the patient to avoid contact sports. Patients with hepatosplenomegaly are at risk for bleeding related to increased friability of the enlarged liver and spleen. They should avoid contact sports to prevent trauma to the liver and spleen. Monitor patient for signs of abdominal trauma. Patients with hepatosplenomegaly are at risk for bleeding related to increased friability of the enlarged liver and spleen. They should be monitored for signs of abdominal trauma that can indicate and/or lead to bleeding.

A teenager is having an acute exacerbation of cystic fibrosis. The parents of the teen state, "We were doing so well coping before this episode. We don't even know what to do." What should the nurse do to help the parents? Discuss the need to remain consistent in their activities to maintain normalcy in the family Help the parents to determine why they are not able to cope with the teen's illness like they have in the past Assist the parents in identifying what usually helps the family to cope to see if that can be used during the exacerbation of the chronic illness Explain the illness trajectory of cystic fibrosis so that the parents know that they need to be prepared for the exacerbations to continue to happen

Assist the parents in identifying what usually helps the family to cope to see if that can be used during the exacerbation of the chronic illness Under times of acute stress, some families may simply need gentle reminders and support to maintain their resiliency.

A nurse is caring for a four-year-old child who must have surgery. The child will require an IV and an indwelling urinary catheter. How can the nurse best prepare the child for this experience? Select all that apply. Assure the child that a nurse or parent will be caring for him or her at all times. Let the child know that everything will be fine and that worrying is not necessary. Ensure that the child is told about the IV and urinary catheter right before going to surgery. Show the child a doll that has an IV and a urinary catheter in place while explaining what each is for. Give simple explanations about the surgery and what the child should expect a few days before the surgery.

Assure the child that a nurse or parent will be caring for him or her at all times. Preschool-aged children often fear being left alone, so accurately assuring the child that someone will be with him or her would be an important nursing intervention. Show the child a doll that has an IV and a urinary catheter in place while explaining what each is for. Preschool-aged children need more than just a verbal explanation. They need visual aids such as dolls or pictures to understand information. Give simple explanations about the surgery and what the child should expect a few days before the surgery. Simple explanations are important and should be started days before a major procedure to allow the preschool-aged child to process the information.

Which of the following statements about septic arthritis is true? a. the most common causative agent in children under 2 years of age is Haemophilus influenzae b. knees, hips, ankles, hands, and feet are the joints most commonly affected c. early radiographic findings show soft tissue swelling and erosions of the bone d. IV antibiotics use is based on Gram stain and clinical presentation

d. IV antibiotics use is based on Gram stain and clinical presentation

The nurse knows that the hyperventilating patient is experiencing which intracranial processes to maintain proper oxygenation? Select all that apply. Autoregulation Cerebral vasodilation Cerebral vasoconstriction Decreased cerebral blood flow Decreased intracranial pressure

Autoregulation A patient who is hyperventilating will autoregulate to maintain a steady blood flow to the brain. Cerebral vasoconstriction A patient who is hyperventilating will have decreased PaCO2 and therefore have vasoconstriction. The endothelial cells of the vasculature will constrict in the presence of increased levels of oxygen but not carbon dioxide. Decreased cerebral blood flow A patient who is hyperventilating will have decreased cerebral blood flow because of decreased PaCO2. The decrease in cerebral blood flow occurs because as the vasculature vasoconstricts, it decreases the caliber of the vessel for blood to flow through.

Interventions

Avoid aspirin or NSAIDs (increase bleeding time and inhibit platelet function) Manage bleeding episodes with prompt infusion therapy Children with vWD have normal life expectancy if well managed

Which nursing intervention is appropriate to prevent increased intracranial pressure in an unconscious child? Frequent suctioning Providing environmental stimulation Turning the head from side to side every hour Avoiding activities that result in pain or crying

Avoiding activities that result in pain or crying Rationale Nursing interventions would be focused on assessment and interventions to minimize pain. These activities can cause the intracranial pressure (ICP) to increase. Suctioning is a distressing procedure. In addition, the resultant decrease in carbon dioxide can increase ICP. Environmental stimulation should be minimized because it can increase ICP. The child's head would not be turned from side to side. If the jugular vein is compressed, the ICP may increase.

A patient's chest x-ray reveals inflammation of the bronchi and a sputum culture is positive for streptococcus. Which white blood cells may be active? Select all that apply. B cells T cells Monocytes Lymphocytes Megakaryocyte Natural killer cells

B cells B cells participate in both inflammatory and immune actions. Correct T cells T cells participate in both inflammatory and immune actions. Correct Monocytes Monocytes participate in both inflammatory and immune actions. Correct Lymphocytes Lymphocytes participate in both inflammatory and immune actions. Natural killer cells Natural killer cells participate in both inflammatory and immune actions.

Julie age 10, has just returned from surgery for repair of an open fracture; she has a dressing and elastic bandage wrap on her leg from upper thigh to mid-calf. The nurse immediately notifies the physician if assessment findings include which of the following A. Appearance of blood stained area the size of a dime on the dressing B. 2+ pedal pulse C. Inability to move the toes D. Report of pain level of 4/10 on Wong-Baker faces pain rating scale

C. Inability to move the toes

Cerebral palsy (CP)

is a group of permanent disorders of the development of movement and posture, causing activity limitation, that are attributed to nonprogressive disturbances that occurred in the developing fet al or infant brain. One of the major risk factors for development of CP is preterm birth.

Epidural (extradural) hematoma

is a hemorrhage into the space between the dura and the skull. As the hematoma enlarges, the dura is stripped from the skull; this accumulation of blood results in a mass effect on the brain, forcing the underlying brain contents downward and inward as it expands.

A nursing student is preparing to conduct a clinical conference regarding osteomyelitis. Which event triggering the inflammatory process should be included in the discussion? Pus accumulates under the periosteum. Bacteria enter the bone through blood vessels. Pus moves from the metaphyseal area into a joint. Pus accumulates and moves toward the medullary canal.

Bacteria enter the bone through blood vessels. In the initial stage, bacteria enter the metaphysis by small capillaries, and the inflammatory process begins. The presence of bacteria alerts the body's immunological defense system to react, resulting in an inflammatory reaction.

Which condition is appropriate when explaining to parents that there is a left shift on the complete blood count for their child? Jaundice Bacterial infection Sickle cell anemia Iron-deficiency anemia

Bacterial infection Rationale Shift to the left is the common term used to indicate that there is an abnormal complete blood count. This shows that there are immature neutrophils present in the peripheral blood film, which usually occurs due to increased bone marrow function. One of the reasons for the hyperfunction of bone marrow is bacterial infection. Jaundice is said to be present when serum bilirubin levels increase. In sickle cell anemia, abnormal red blood cells are seen in the blood film, and in iron-deficiency anemia, hematocrit levels decrease.

A newborn had a breech intrauterine presentation and has a family history of hip dysplasia. Which is the most reliable assessment to diagnose this infant with hip dysplasia? Hip abduction Detection of Galeazzi sign Barlow and Ortolani maneuvers Asymmetric thigh and gluteal folds

Barlow and Ortolani maneuvers Rationale A newborn with a breech intrauterine presentation and with a family history of hip dysplasia may develop developmental dysplasia of the hip (DDH). The Barlow and Ortolani maneuvers are most reliable from birth to 4 weeks of age. These maneuvers demonstrate the joint subluxation and its tendency to dislocate. After 10 weeks of age, the most sensitive test is limited hip abduction. Other signs of determining DDH are checking for Galeazzi sign, which is shortening of the thigh on the affected side, and asymmetric thigh and gluteal folds, but these are not the most reliable methods to check for DDH.

Which condition is appropriate for the child with a head injury that has leakage of cerebrospinal fluid (CSF) from the nose, subcutaneous bleeding over the mastoid process, subcutaneous bleeding around the orbit, and blood behind the tympanic membrane? Contusion Concussion Basilar fracture Growing fracture

Basilar fracture Rationale Basilar fractures involve the bones at the base of the skull in either the posterior or the anterior region. They have distinct clinical features such as leakage of CSF from the nose (CSF rhinorrhea), subcutaneous bleeding over the mastoid process (Battle sign), subcutaneous bleeding around the orbit (raccoon eyes), and blood behind the tympanic membrane (hemotympanum). Contusion describes the actual bruising or tearing of the cerebral tissue and causes focal disturbances in strength, sensation, or visual awareness. The symptoms of contusion range from mild, transient weakness of limbs to prolonged unconsciousness and even paralysis. Concussion is a change in the mental status with or without loss of consciousness and occurs right after a head injury. Growing fractures result from a skull fracture with an underlying tear in the dura that fails to heal properly. The clinical symptoms may not surface for months to years after the initial skull fracture and include headaches, seizures, or asymmetric cranial growth.

Which condition is appropriate for the child with a head injury that has leakage of cerebrospinal fluid (CSF) from the nose, subcutaneous bleeding over the mastoid process, subcutaneous bleeding around the orbit, and blood behind the tympanic membrane? Contusion Concussion Basilar fracture Growing fracture

Basilar fracture Rationale Basilar fractures involve the bones at the base of the skull in either the posterior or the anterior region. They have distinct clinical features such as leakage of CSF from the nose (CSF rhinorrhea), subcutaneous bleeding over the mastoid process (Battle sign), subcutaneous bleeding around the orbit (raccoon eyes), and blood behind the tympanic membrane (hemotympanum). Contusion describes the actual bruising or tearing of the cerebral tissue and causes focal disturbances in strength, sensation, or visual awareness. The symptoms of contusion range from mild, transient weakness of limbs to prolonged unconsciousness and even paralysis. Concussion is a change in the mental status with or without loss of consciousness and occurs right after a head injury. Growing fractures result from a skull fracture with an underlying tear in the dura that fails to heal properly. The clinical symptoms may not surface for months to years after the initial skull fracture and include headaches, seizures, or asymmetric cranial growth.

Which nursing recommendation is appropriate for the parents of a cognitively impaired child asking for guidance with discipline? Behavior modification is an excellent form of discipline. Discipline is ineffective with cognitively impaired children. Discipline is not necessary for cognitively impaired children. Physical punishment is the most appropriate form of discipline.

Behavior modification is an excellent form of discipline. Rationale Positive behaviors and desirable actions should be reinforced with cognitively impaired children. Behavior modification with positive reinforcement is effective in children with cognitive impairment. Discipline is essential in assisting the child in developing boundaries. Most children with cognitive impairment will not be able to understand the reason for the physical punishment, and the behavior will not change. Physical punishment is not an acceptable form of discipline.

The nurse is preparing an adolescent girl for surgery to treat scoliosis. What would the nurse include? Blood administration may be an option. Ambulation will not be allowed for up to 3 months. Surgery eliminates the need for casting and bracing. Discomfort can be controlled with nonpharmacologic methods

Blood administration may be an option. Correct Spinal surgery usually involves considerable blood loss. Several options are considered for blood replacement. Ambulation is allowed as soon as possible. Depending on the instrumentation used, most patients are walking by the second or third postoperative day. Casting and bracing are required postoperatively. The child usually has considerable pain for the first few days after surgery. Intravenous opioids should be administered on a regular basis.

A patient is admitted with Neisseria meningitidis. What should the nurse monitor for that indicates infection progression? Fatigue that is not relieved by rest Blood glucose levels every 6 hours Purpura over the trunk of the patient Blood pressure and heart rate changes Vomiting, diarrhea, and abdominal cramps

Blood glucose levels every 6 hours Adrenal insufficiency is a condition that can develop with Neisseria meningitidis infection, and therefore the nurse must monitor for hypoglycemia and symptoms of it such as sweating and nausea. Correct Purpura over the trunk of the patient Purpura and petechiae are manifestations of Neisseria meningitidis that occur at the time of presentation or shortly thereafter. Correct Blood pressure and heart rate changes Septic shock indicates the infection is progressing rapidly. In addition to common symptoms of fever, hypotension and tachycardia are indicative of septicemia.

When caring for a fourteen-year-old child with traumatic brain injury, which complications must be addressed immediately? Select all that apply. Blood pressure 80/40 mm Hg Decreased muscle tone bilaterally Weak deep tendon reflex responses Oxygen saturation of 88% on room air Respiratory rate of 8 breaths per minute

Blood pressure 80/40 mm Hg The care for a child with traumatic brain injury includes aggressively managing hypotension of the patient because the injury could affect centers of the brain that control basic body functions such as vascular tension. Weak deep tendon reflex responses Weak deep tendon reflexes are concerning for syndrome of inappropriate secretion of antidiuretic hormone (SIADH) and for progressive increased intracranial pressure (ICP). Oxygen saturation of 88% on room air The care for a child with traumatic brain injury includes monitoring for and correcting hypoxemia of the patient that could develop because centers of the brain that control basic body functions such as breathing are altered. Respiratory rate of 8 breaths per minute The care plan for a child with traumatic brain injury includes monitoring for hypoventilation of the patient because the injury could affect the centers of the brain that control basic body functions such as breathing.

Which typical differences in clinical manifestations are appropriate for bacterial meningitis in adolescents and in infants? Select all that apply. Fever is seen in infants. Vomiting is seen in adolescents. Seizures are seen in adolescents. Bulging fontanels are observed in infants. Positive Brudzinski sign is seen in adolescents

Bulging fontanels are observed in infants. Positive Brudzinski sign is seen in adolescents. Rationale Bulging fontanel can be observed in infants with bacterial meningitis but not in adolescents. Brudzinski sign is positive in adolescents; whereas, in infants, Brudzinski sign is not helpful in diagnosis because it is difficult to elicit and evaluate in infants. Fever, vomiting, and seizures can be seen both in infants and adolescents with bacterial meningitis.

A patient in the intensive care unit (ICU) had a nasogastric tube (NGT) placed for feeding. If the patient develops disseminated intravascular coagulation (DIC), what aspect of the tube needs to be checked? Placement in the abdomen Insertion site into the nares Feeding pump settings Connection to suction canister

Insertion site into the nares When caring for patients with DIC, the nurse should assess this the insertion site of all invasive lines and tubes for bleeding.

One nursing goal is to assess which families are at lesser or greater risk for succumbing to the effects of the crisis. A number of variables, including

available support systems, reactions to the child, perception of the illness or disability, coping mechanisms, available financial resources, and concurrent stresses within the family, may affect the resolution of a crisis

The recommended treatment for DDH in an infant 2 months old is: A. Surgical fixation B. Hip Spica cast C. Pavlic harness D. hip abduction orthosis

C. Palvic harness

The nurse is providing home care instructions to the parents of a 10-year-old child with hemophilia. Which sport activity should the nurse suggest for this child? a. soccer b. basketball c. swimming d. field hockey

C. swimming Children with hemophilia must avoid contact sports and as such, swimming is the safest activity for them.

Which nursing intervention is appropriate for the child with von Willebrand disease who suddenly develops epistaxis at school? Apply a cold pack to the bridge of the nose. Administer 1-deamino-8-D-arginine vasopressin. Apply pressure to the nose with the thumb and forefinger. Calm the child and have the child sit up and lean forward.

Calm the child and have the child sit up and lean forward. Rationale In case of epistaxis, the very first measure to be taken is to make the child sit up and lean forward to keep the airway patent. A cold pack may be applied to the bridge of the nose if bleeding persists. When local measures are not successful in stopping the bleeding, 1-deamino-8-D-arginine vasopressin is administered. After making the child sit up and lean forward, pressure is applied to the nose with thumb and forefinger for at least 10 minutes. This can control the bleeding, because most of the nosebleed originates from the anterior part of the nasal septum.

Diagnostic Evaluation Hemophilia

Can be diagnosed through amniocentesis Genetic testing of family members is needed to identify carriers Diagnosis is made on the basis of the history, lab studies and the examination Lab studies = low levels of factor VIII or IX and a prolonged PTT, platelet count, PT and fibrinogen levels are normal

Which drug is appropriate for the child with cerebral palsy and epilepsy? Baclofen Reserpine Levodopa Carbamazepine

Carbamazepine Rationale Seizures are common in patients with cerebral palsy. Patients experiencing frequent seizures are given antiepileptic drugs for effective illness management. Carbamazepine is the preferred drug for patients with seizures. Baclofen is used for pain management in patients for postoperative care. Reserpine is used to treat chorea and athetosis. Levodopa is used to treat muscular disorders like dystonia.

von Willebrand disease (vWD) i

is a hereditary bleeding disorder characterized by a deficiency of or defect in a protein called von Willebrand factor (vWF). The vWF protein contributes to the adherence of platelets to damaged endothelium and serves as a carrier protein for factor VIII. The disease results in prolonged bleeding time because platelets fail to adhere to the walls of the ruptured vessel to form a platelet plug.

Iron Deficiency Anemia

Caused by inadequate supply of dietary iron Generally is preventable Iron-fortified cereals and formulas for infants Special needs of premature infants Periods of rapid growth Excessive milk intake Premature infants at risk due to low fetal supply 12-36 months at risk due to ingestion of large amounts of cows milk and diet Adolescents at risk due to rapid growth and poor eating habits Pathophysiology Infants: last trimester of pregnancy, iron is transferred from the mother of the fetus. If dietary iron is not supplied to meet the infants growth. Demands after the fetal. Iron stores are depleted, iron deficiency anemia results Toddlers "milk babies" milk poor sources of iron -fecal loss of blood, chubby, pale, poor muscle development, prone to infection low HG, stool analysis for occult blood, low serum iron concentration Therapeutic management Infants iron fortified formula, cereal Children provide iron rich foods & vitamin c (helps w/absorption of iron) Dietary versus ferrous iron supplements Prognosis Very good if there is early recognition and adherence Nursing considerations Appropriate education with administration of iron and diet changes

Dyskinetic CP

Characterized by abnormal involuntary movement, such as athetosis-slow, wormlike, writhing movements of the upper extremities that usually involve the extremities, trunk, neck, facial muscles, and tongue

Cerebral Palsy (CP)

Characterized by early onset from static brain injury and impaired movement and posture also involves disturbances of sensation, perception, communication, cognition, behavior, muskuloskeletal problems, epilepsy Incidence 2.4 to 3.6 per 1000 live births Most common permanent physical disability in childhood

The nurse cares for a ten-year-old recovering from meningitis. The nurse is most concerned about which assessment finding? Child reports a persistent headache. Child fails to respond when called by name. Parents report the child is not engaging in activities. Parents insist the child complete missed schoolwork.

Child fails to respond when called by name. The child may be experiencing hearing loss secondary to neurologic damage sustained during the meningitis episode. This will not improve with time and needs to be addressed.

The nurse assesses a two-year-old child with papilledema related to hydrocephalus. Which finding causes the nurse the most concern? Child is holding head and crying Child is lethargic, responding to voice Child projectile vomits when sitting up Child has an increased head circumference

Child has an increased head circumference Sutures and fontanels close by 18 months of age. If sutures and fontanels are closed, child should not have an enlarged head circumference. This finding indicates there is an additional problem that would be most concerning to the nurse.

A 5-year-old child is admitted with complications related to an Arnold-Chiari malformation and myelomeningocele. What assessment findings cause the nurse to be concerned? Child is underweight for age. Child reports neck pain and stiffness. Child reports burning with urination. Child is experiencing severe headache.

Child is experiencing severe headache. Arnold-Chiari can result in hydrocephalus. A headache may indicate increasing ICP. If the child has a shunt, this may indicate a malfunction.

The parents of a 10-year-old child with Down syndrome tell the nurse that their child currently participates in soccer and basketball. They state that the child will start football in the next month. Which does the nurse consider before advising this patient? Children with Down syndrome have issues on the cervical area, making them prone to injury. Children with Down syndrome often have muscle strength issues, making football a difficult sport to play. Children with Down syndrome often have coordination issues making football a difficult sport for them to play. Children with Down syndrome have issues with buttoning and tying, so the parents may need to dress the child in the protective gear.

Children with Down syndrome have issues on the cervical area, making them prone to injury. Children with DS are more susceptible to having atlantoaxial subluxation or dislocations, so an x-ray of the neck is needed before playing in any active sports program.

Which nursing supportive care measures are appropriate for the child with Down syndrome who has nasal congestion, causing the child to breathe through the mouth? Select all that apply. Clear the child's nose using bulb-type syringes. Rinse the child's mouth after every feeding. Ask the child to blow the nose frequently with force. Use a cool-mist vaporizer to moisten the secretions. Avoid entry of any fluid, including water, in child's nose.

Clear the child's nose using bulb-type syringes. Rinse the child's mouth after every feeding. Use a cool-mist vaporizer to moisten the secretions. Avoid entry of any fluid, including water, in child's nose. Rationale Children with Down syndrome often suffer from various respiratory problems due to inadequate drainage. In the case of a stuffy nose, the nurse would ensure that the nose is clear by clearing the nose with bulb syringes. The child's mouth would be rinsed after every meal or feeding to avoid development of infection. Stuffy nose can also be relieved by inhaling cool mist from a vaporizer, which helps to liquefy the secretions. Asking the child to blow the nose hard does not relieve the problem, because the secretions are dry. The child's nose would be clear and free of any fluids.

Which nursing intervention is appropriate for the child diagnosed with Down syndrome? Decrease the fluid intake of the child. Clear the nose using a bulb-type syringe. Discourage the child from changing positions. Administer antibiotics until the symptoms of the infection disappear.

Clear the nose using a bulb-type syringe. Rationale Children with Down syndrome often have a stuffy nose due to an underdeveloped nasal bone. The nurse would use a bulb syringe to drain the mucus from the nose to provide some relief. The nurse would encourage the parents to increase the child's intake of fluids. Changing positions frequently is a helpful measure. The nurse would administer the entire course of antibiotics prescribed, not just until the symptoms are eradicated.

A 13-year-old patient is admitted with a fractured femur. For which signs and symptoms should the nurse evaluate the patient to identify potential systemic complications? Select all that apply. A. Arrhythmia B. Immobility C. Color changes D. Axillary petechiae E. Shallow rapid breathing

Color changes Color changes can be a symptom of a blocked vessel (emboli) or shock and should be further investigated. Correct Axillary petechiae Axillary petechiae can be an indication of fat emboli, a systemic complication of a fracture (particularly a femur fracture). When a bone is fractured, fat tissue from the bone marrow can leak into the blood. Long bones (like the femur) especially have more marrow than shorter bones, increasing the risk of fat emboli. Correct Shallow rapid breathing A fractured femur will result in a large amount of bleeding that can lead to hypovolemic shock, which may be indicated by shallow, rapid breathing.

Which statements would the nurse use to explain the relationship between complex fractures and external fixators? Select all that apply. Complex fractures may require external fixators to lengthen bones. External fixators keep the bone ends separated and in alignment. External fixators shorten the healing time by decreasing the bone length in complex fractures. External fixators correct angular deformities that involve bone and soft tissue in complex fractures. External fixators do not allow for periodic changes in alignment and bone length which is necessary for complex fractures

Complex fractures may require external fixators to lengthen bones. External fixators are pins or wires inserted through skin, soft tissue, and bone and secured to a rigid metal frame to lengthen bones. External fixators keep the bone ends separated and in alignment. The external fixator keeps the bone ends separated and in alignment, which promotes proper healing. External fixators correct angular deformities that involve bone and soft tissue in complex fractures. Because the external fixator secures the bone to a frame, it can correct angular deformities that involve bone and soft tissue.

Which reason is appropriate to take a child that underwent a submersion injury immediately to the hospital? Hypoxia can cause global cell damage. Neurons often sustain irreversible damage. Complications can occur even after 24 hours. Fluid absorption in the pulmonary circulation causes pulmonary edema.

Complications can occur even after 24 hours. Rationale The child is admitted to the hospital for observation because complications such as respiratory compromise and cerebral edema may occur even 24 hours after the occurrence of injury. As the child struggles to stay above water, breath holding leads to air hunger, causing hypoxia. Hypoxia is the primary problem of submersion injury, resulting in global cell damage. Neurons sustain irreversible damage after 4 to 6 minutes of submersion. Reflex inspiration causes aspiration and fluid is absorbed in the pulmonary circulation, causing pulmonary edema. All these conditions usually produce immediate symptoms, which is not the case in the present scenario.

A young child has recently been fitted with a knee, ankle, and foot orthosis (brace). What would be included in care of the skin? Apply lotion or cream to soften the skin. Contact a practitioner or orthotist if skin redness does not disappear. Place padding between the skin and brace if the child experiences a burning sensation under the brace. If a small blister develops, apply rubbing alcohol and place padding between the skin and the brace.

Contact a practitioner or orthotist if skin redness does not disappear. Correct Redness is a sign of skin irritation from the brace. The brace needs to be adjusted to be functional. The skin should not be softened. The brace is specially designed for the child. Padding may alter the alignment of the brace. Rubbing alcohol would be painful. If the brace causes blisters, it needs to be adjusted.

A nurse is making an initial home care visit for a child with muscular dystrophy. What should be part of her assessment? Select all that apply. Coping skills Presence of pets Height and weight Food in the pantry Width of the doorways

Coping skills It is important for the nurse to assess how the patient and family are coping with the child's diagnosis. The nurse may need to provide support and referrals as needed. Height and weight The patient's height and weight should be measured due to the risk of obesity. Correct Food in the pantry Dietary changes are often needed in patients with muscular dystrophy. The nurse should assess what foods are typical in the family diet. Correct Width of the doorways The nurse should assess the house to see if it could accommodate a wheelchair or other assistive devices.

What is the goal of treatment for a newborn with clubfoot? Correction of the deformity with a single cast worn for a year and followup observation Surgical correction of the deformity and maintenance of the correction until normal muscle balance is regained Correction of the deformity, maintenance of the correction until normal muscle balance is regained, and followup observation Correction of the deformity with casting for 3 to 8 weeks and maintenance of the correction until normal muscle balance is regained

Correction of the deformity, maintenance of the correction until normal muscle balance is regained, and followup observation Rationale The goal of treatment for clubfoot is to achieve a painless, plantigrade, and stable foot by correcting the deformity; maintenance of the correction until normal muscle balance is regained; and followup observation. Serial casts, rather than a single cast worn for a year, are the typical treatment protocol. Surgical correction is rarely required. Casting lasts 6 to 10 weeks for maximal correction, rather than 3 to 8 weeks.

What are the appropriate procedures to follow while placing the child in a cast for a lower extremity fracture? Select all that apply. Use heated fans to dry the cast after application. Counsel before casting according to the developmental age of the child. Check skin surface of the extremity for any abrasions or cuts. Pull underlying edge of the stockinette over the plaster cast to protect the skin. Allow movement of joints above the site of injury while immobilizing those below.

Counsel before casting according to the developmental age of the child. Check skin surface of the extremity for any abrasions or cuts. Pull underlying edge of the stockinette over the plaster cast to protect the skin. Rationale Information should be provided according to the developmental age of the child. Various play methods are used while counseling the child. Before placing the extremity into a cast, the nurse should examine the skin surface for any cuts or abrasions. When a plaster cast is used, the underlying stockinette is pulled over the raw edge of the plaster and is secured with a layer casting material to smooth the edge of the plaster. Heated fans should not be used because the cast may dry on the outside but remain wet on the inside, and heated fans may generate heat and may damage the underlying tissue. The movement in the joints both above and below the injured site may cause displacement at the site. Hence, in many situations, they are immobilized.

Which nursing response is appropriate for the parent of a 2-month-old formula-fed baby that has been advised by a friend to give fresh cow's milk to the baby instead of formula milk because it has high nutritional value? Fresh cow's milk is the best source of nutrition for a 2-month-old baby. The mother can start giving cow's milk to her baby at 3 months of age. Cow's milk should be avoided before 12 months of age, because it may cause sickle cell anemia. Cow's milk should be avoided before 12 months of age, because it may cause iron-deficiency anemia.

Cow's milk should be avoided before 12 months of age, because it may cause iron-deficiency anemia. Rationale It is important for a nurse to educate the parents about appropriate measures to be taken to prevent iron-deficiency anemia. Fresh cow's milk contains a heat-labile protein that can induce gastrointestinal bleeding in children younger than 12 months. It can also cause gastrointestinal mucosal damage in these children, leading to bleeding. Therefore, fresh cow's milk should not be given to children before 12 months of age, because it may cause iron-deficiency anemia due to gastrointestinal bleeding. Sickle cell anemia is not caused by cow's milk; instead, it is an inherited genetic disease.

Which organism causes aseptic meningitis? Tubercle bacillus Pneumococcus Meningococcus Cytomegalovirus

Cytomegalovirus Rationale Aseptic meningitis refers to the onset of meningeal symptoms, fever, and pleocytosis without bacterial growth from cerebrospinal fluid cultures. It is caused by many different viruses, including cytomegalovirus, arbovirus, herpes simplex virus (HSV), adenovirus, and human immunodeficiency virus (HIV). Tuberculous meningitis is caused by the tubercle bacillus. Bacterial or pyogenic meningitis is caused by pus-forming bacteria, especially Pneumococcus, Meningococcus, and Haemophilus organisms.

Medical Management hemophilia

DDAVP (desmopressin) IV Causes 2 to 4 times increase in factor VIII activity Used for mild hemophilia Replace missing clotting factors Transfusions At home with prompt intervention to decrease complications Following major or minor hemorrhages Medications - corticosteroids, NSAIDs and Amicar Corticosteroids are given for hematuria, acute hemarthrosis and chronic synovitis NSAID's are effective in relieving pain caused by synovitis but used with caution because the inhibit platelet function Amir prevents clot destruction (limited to the use on mouth or trauma surgery with a dose of factor concentrate given first) Exercise and PT —- physical activity decreases spontaneous bleeding episodes desmopressin A synthetic form of vasopressin treats mild hemophilia IS NOT EFFECTIVE in the treatment of severe hemophilia A, severe vWD or ANY form of hemophilia B prophylaxis therapy -periodic factor replacement

List the steps of RBC production in the proper order.

Decrease in circulating oxygen Kidneys produce erythropoietin RBC precursors produced in the bone marrow Rapid maturation leads to development of RBCs Hypoxia and hypoxemia stimulates the release of erythropoietin in the kidneys. Erythropoietin, produced in the kidneys, stimulates the bone marrow to produce RBC precursors that mature quickly.

Signs of brainstem involvement

Deep, rapid, or intermittent and gasping respirations wide fluctuations or noticeable slowing of pulse widening pulse pressure or extreme fluctuations in blood pressure sluggish, dilated, or unequal pupils

Hydrocephalus Prognosis

Depends on: -Rate at which hydrocephalus develops -Duration of the increased ICP -Frequency of complications -Underlying cause of the hydrocephalus

Which nursing intervention is appropriate in the care of a child experiencing a seizure? Describing and documenting the seizure activity observed Suctioning the child during the seizure to prevent aspiration Restraining the child when a seizure occurs to prevent bodily harm Placing a padded tongue blade between the teeth if they become clenched

Describing and documenting the seizure activity observed Rationale The priority nursing intervention is to observe the child and document the seizure activity. To prevent aspiration, the child would be placed on the side if possible to facilitate drainage. The child would not be restrained, because this could cause an injury. Nothing would be placed in the child's mouth, because this could cause injury not only to the child but to the nurse as well.

Just a quickie not on fractures since were in bony stuff. Is a fracture through the child's growth plate of the bone different or treated the same as any other fracture?

Detection of physeal injury is critical to determine whether bone growth will be affected. Damage to the growth plate can cause shortening and often a progressive angular deformity (box 33.3 pg. 1260) Whenever they get a fracture in the growth plate we worry. Because what can happen if their growth is retarded in the bone? Deformity/disability/limited ROM/asymmetry. Right, if that fracture doesn't heal well, the bone stops growing, and you can get all kinds of things. there are four levels of fracture through the growth plate. You do not need to know all of the different levels. They're called a Salter- Harris classification. You don't need to know all those. What I want you to know is the complication is the bones don't grow and they can end up with a lot of problems because of that. Remember that children can get osteoporosis. Believe it or not they can.

What are the priorities when developing a nursing plan of care for a ten-year-old patient with Guillain-Barré syndrome (GBS)? Select all that apply. Assess daily for fever, stiff neck, or confusion. Determine bilateral lower extremity strength. Check vital signs and trends every four hours. Perform tests of lower and upper sensory function. Observe chewing and swallowing of solids and liquids. Evaluate respiratory rate and use of accessory muscles.

Determine bilateral lower extremity strength. Neuromuscular impairment can be a finding in patients affected by Guillain-Barré. Assessing the patient's lower extremity strength will therefore be a priority. Check vital signs and trends every four hours. Autonomic instability may cause dizziness or the inability to alter heart rate and is a priority when developing a nursing care plan for a patient with Guillain-Barré syndrome. Perform tests of lower and upper sensory function. Guillain-Barré syndrome often presents with limb paresthesia, which is defined as altered sensation from the patient's limbs. This will be a priority in developing a care plan for this patient. Observe chewing and swallowing of solids and liquids. The patient with Guillain-Barré syndrome may develop cranial nerve dysfunction that could inhibit the ability to swallow. This will therefore be a priority in developing a nursing care plan for the patient. Evaluate respiratory rate and use of accessory muscles. The phrenic nerve may be affected in a patient with Guillain-Barré syndrome resulting in respiratory failure of the patient. Evaluating respiratory status will be a priority in caring for this patient.

Which patients are at risk for impaired clotting? Select all that apply. Acute kidney failure Impaired lung function Diagnosis of osteosarcoma Diagnosis of liver failure cirrhosis A patient with Von Willebrand disease

Diagnosis of osteosarcoma Megakaryocytes are produced in the bone marrow. Conditions that affect the bones can lead to impaired clotting. Correct Diagnosis of liver failure cirrhosis Conditions that affect liver function can lead to impaired clotting because the liver produces the clotting factors necessary for hematopoiesis. Correct A patient with Von Willebrand disease Dairy is often a primary source of calcium, which is necessary for clot formation. A lack of dietary calcium can contribute to impaired clotting.

A nurse is presenting an inservice on fragile X syndrome. Which information is important for the nurse to present regarding who should receive diagnostic testing? Select all that apply. Children with unexplained eyesight abnormalities should receive diagnostic testing Diagnostic testing should be completed on children who have long, narrow faces with large ears. Diagnostic testing should be completed on children with a family history of autism spectrum disorder. Diagnostic testing should be completed on children with learning disabilities that have no known cause. Diagnostic testing should be completed on children who stumble when walking and have language delays.

Diagnostic testing should be completed on children who have long, narrow faces with large ears. A long narrow face with large ears is a physical characteristic of fragile X syndrome; diagnostic testing may be indicated. Diagnostic testing should be completed on children with learning disabilities that have no known cause. Children with intellectual disability of unknown cause or learning disabilities, together with manifestations of FXS, should be considered for fragile X testing. Correct Diagnostic testing should be completed on children who stumble when walking and have language delays. Children with stumbling and language delays, which are motor and sensory characteristics of fragile X syndrome, may undergo diagnostic testing. This should be considered when there are either intellectual disabilities of unknown cause or learning disabilities.

Which condition is appropriate for the child with a neural tube defect who has herniation of the brain and meninges, through a defect in the skull, producing a fluid-filled sac? Exencephaly Anencephaly Cranioschisis Encephalocele

Encephalocele Rationale Encephalocele is the condition in which the brain and meninges are herniated through a defect in the skull, producing a fluid-filled sac. Exencephaly is the condition in which the brain is totally exposed or extruded through an associated skull defect. Anencephaly is the condition in which both cerebral hemispheres are absent; it occurs if a fetus with exencephaly survives and the brain degenerates to a spongiform mass with no bony covering. Cranioschisis is a skull defect through which various tissues protrude.

During the musculoskeletal assessment of a pediatric patient, the child complains of pain at the elbow when the nurse passively pronates the left radius/ulna. Which diagnostic test would the nurse expect the provider to order? Radiography Creatine phosphokinase Radionuclide scintigraphy Magnetic resonance imaging

Magnetic resonance imaging The child is complaining of joint pain with movement; the nurse would anticipate magnetic resonance imaging (MRI) being used to assess for soft tissue damage, including bones, ligaments, and joints.

The parents of a preschooler with a developmental delay come into the health care provider's office for help with recent tantrums. The parents want to know how they should discipline the child. How should the nurse respond? The child should be left alone to calm down and gain control. Discipline might not be the answer because the child may be trying to communicate frustration. Punishment is important to decrease the child's use of tantrums to gain attention and get what is wanted. The tantrum might indicate that the child has a need that is not being met, and disciplining the child could make this situation worse. Ignoring the behavior and withdrawing attention from the child will help to decrease the tantrums and help the child learn to talk about needs. Instead of disciplining the child, help the child to better communicate needs by using a picture board that can be used to initiate getting the child's personal needs met.

Discipline might not be the answer because the child may be trying to communicate frustration. The child with a chronic illness may experience many frustrations with no known way to communicate them. Discipline may increase frustration because it does not address what is frustrating the child. The tantrum might indicate that the child has a need that is not being met, and disciplining the child could make this situation worse. The use of punishment in a developmentally delayed preschool-aged child could increase the frustration level in the child and make behaviors worse. Children this age may act out when they do not know how to get their needs met. The parents should determine if the child is trying to communicate a need first. Instead of disciplining the child, help the child to better communicate needs by using a picture board that can be used to initiate getting the child's personal needs met. If the tantrums are indicating frustration, making communication easier can decrease frustration. Let the child gain some independence by being able to initiate getting own personal needs met.

Which nursing approach is appropriate when discussing family participation during hospitalization with the parents of a child with cerebral palsy who is having corrective surgery? Assigning the parents specific times to visit their child in the hospital Telling the parents to room-in and take part in all aspects of the child's care Scheduling nursing care when the parents are out of the child's hospital room Discussing the parents' desire to participate in the care of the child

Discussing the parents' desire to participate in the care of the child Rationale The most family-centered approach to promoting parent participation in the care of the child with cerebral palsy is to discuss the parents' desire to participate in the care of their child as a means of discovering the right level of involvement for the family. Assigning the parents specific times to visit their child in the hospital, telling the parents to room-in and take part in all aspects of the child's care, and scheduling nursing care when the parents are out of the child's hospital room are not the most therapeutic responses that the nurse could give the family.

Which condition is appropriate for the 15-month-old child who has been hospitalized with neck pain and is not able to roll over, sit alone, crawl, and creep? Down syndrome Visual impairment Fragile X syndrome Hearing impairment

Down syndrome Rationale The nurse would suspect that the child has Down syndrome. Children with Down syndrome begin to roll over, sit alone, crawl, or creep by the age of 22 months; other healthy children are able to develop these activities at an earlier age. Children with visual impairment would experience walking into objects, headaches, and nausea. Children with fragile X syndrome would experience short attention spans and be hyperactive. Children with hearing impairments may not comprehend verbal directions and lack a startle reflex.

How is a complex partial seizure different from an absence seizure? Select all that apply. It frequently shows automatisms. Duration is usually 60 seconds. Frequency per day is multiple times. Occasionally is associated with clonic movements. Frequently is associated with postictal impairment. Commonly is associated with mental disorientation.

Duration is usually 60 seconds. Frequently is associated with postictal impairment. Commonly is associated with mental disorientation. Rationale The duration of a complex partial seizure is usually greater than 60 seconds, but the duration of an absence seizure is usually less than 10 seconds. A complex partial seizure frequently is associated with postictal impairment, but an absence seizure never is. A complex partial seizure commonly is associated with mental disorientation but in an absence seizure, mental disorientation is unusual. Both complex partial seizure and absence seizure are frequently associated with automatisms. The frequency per day of complex partial seizures is rarely over one to two times, but the frequency per day of absence seizure is multiple times. Both complex partial seizure and absence seizure are occasionally associated with clonic movements.

A child presents to the clinic with a sprained ankle. The parent asks why the child needs to elevate the leg. What is the most appropriate response from the nurse? Elevating a sprained ankle helps reduce edema. Elevating a sprained ankle helps raise the pain threshold. Elevating a sprained ankle helps produce deep tissue vasodilation. Elevating a sprained ankle helps increase metabolism in the tissues.

Elevating a sprained ankle helps reduce edema. Rationale Elevating the extremity makes it possible for gravity to facilitate venous return and reduces edema. Elevation of an extremity after a soft tissue injury should have no effect on the pain threshold. Venous return to the heart is facilitated when a sprained ankle is elevated; this is not a result of vasodilation. Elevation of an extremity after a soft tissue injury will not affect metabolism.

Which supportive measure is appropriate for the child with hemophilia after an arm injury until factor replacement therapy can be instituted? Applying warm, moist compresses Applying pressure for at least 1 minute Elevating the area above the level of the heart Beginning passive range-of-motion exercise unless the pain is severe

Elevating the area above the level of the heart Rationale The initial response would include elevation of the arm to minimize bleeding. Cold should be applied to the arm. This will aid in vasoconstriction, which will in turn minimize blood loss. Pressure is effective in small areas but would not be as effective for an extremity. Passive range-of-motion exercise is not recommended. The child may perform active range-of-motion exercise after the bleeding episode has resolved.

What is the rationale for elevating an extremity after a soft tissue injury such as a sprained ankle? Elevation reduces edema formation. Elevation raises the child's pain threshold. Elevation produces deep tissue vasodilation. Elevation increases metabolism in the tissues.

Elevation reduces edema formation. Rationale Elevating the extremity uses gravity to facilitate venous return to reduce edema. Elevation should have no significant effect on the pain threshold. Elevation should not affect metabolism. Venous return to the heart, not vasodilation, is facilitated by elevation.

The home health nurse is planning care for a 3-year-old boy who has Down syndrome and is on continuous oxygen. He recently began walking around furniture. He is spoon fed by his parents and eats some finger foods. What is the most appropriate goal to promote normal development? Encourage mobility. Encourage assistance in self-care. Promote oral-motor development. Provide opportunities for socialization.

Encourage mobility. Correct A major principle for developmental support in children with complex medical issues is that it should be flexible and tailored to the individual child's abilities, interests, and needs. This child is exhibiting readiness for ambulation. It is an appropriate time to take steps that encourage mobility (e.g., providing longer oxygen tubing). The parents should provide decreasing amounts of assistance with self-care as the child is able to develop these skills. He is receiving oral foods and is eating finger foods; therefore, he has already acquired oral-motor skills. Mobility is a new developmental task. Opportunities for socialization should be ongoing.

The nurse is caring for a 2-year-old girl who is unconscious but stable after a car accident. Her parents are staying at the bedside most of the time. What is an appropriate nursing intervention? Suggest that the parents go home until she is alert enough to know they are present. Encourage the parents to hold, talk, and sing to her as they usually would. Use ointment on her lips but do not attempt to cleanse her teeth until swallowing returns. Position her with proper body alignment and the head of the bed lowered 15 degrees.

Encourage the parents to hold, talk, and sing to her as they usually would. Correct The parents should be encouraged to interact with their daughter. Senses of hearing and tactile perception may be intact, and stimulation of these senses is important. The daughter may be able to hear that they are present. Oral care is essential in an unconscious child. Mouth care should be done at least twice daily. The head of the bed should be elevated, not lowered.

The infant with congestive heart failure needs to have an IV inserted. To increase the comfort of the baby, what should the nurse do? Encourage the parents to stay with the child as long as they are comfortable. Do not allow the anxious parents to leave the room because this can cause the infant to have separation anxiety. Ensure that the infant has the IV started in the crib because that is where the child is most comfortable and feels safest. The nurse should give a detailed explanation to the parents to decrease anxiety of what steps are involved in inserting the IV.

Encourage the parents to stay with the child as long as they are comfortable. The parents should stay if they are comfortable to decrease the infant's anxiety. If they are not comfortable, then they can transmit their anxiety to the infant.

Which nursing intervention is appropriate for the parents that stay at the bedside of a 2-year-old child who is unconscious but in stable condition after a car accident? Encouraging the parents to hold, talk to, and sing to the child as they usually would Using ointment on the lips but not attempting to cleanse the teeth until swallowing returns Suggesting that the parents go home until the child is alert enough to know that they are present Positioning the child with proper body alignment and the head of the bed lowered to 15 degrees

Encouraging the parents to hold, talk to, and sing to the child as they usually would Rationale The parents would be encouraged to interact with the child. The senses of hearing and tactile perception may be intact, and stimulation is important in the child's recovery. Suggesting that the parents go home until the child is awake is not recommended. The child may be able to hear that they are present, and this stimulation may assist in recovery. Oral care is essential in the unconscious child. Mouth care would be performed at least twice daily to prevent oral infections. The head of the bed would be elevated, not lowered, in a child with neurologic involvement.

Which neurologic diagnostic procedure is appropriate to identify shifts in midline structures from their normal positions resulting from intracranial lesions? Radiography Lumbar puncture Nuclear brain scan Endocephalography

Endocephalography Rationale Endocephalography involves the pulsing of ultrasonic waves through the head to identify shifts in midline structures from their normal positions as a result of intracranial lesions. Radiography is used to identify fractures, dislocations, spreading suture lines, or degenerative changes. Lumbar puncture is used to measure spinal fluid pressure and obtain cerebral spinal fluid. Nuclear brain scanning is used to identify focal brain lesions.

A nurse is discussing disease management options with a family who is coping with chronic illness. The nurse notices a sibling who is withdrawn. What should the nurse do to help the sibling feel more involved? Engage with and ask if the sibling has any questions. Enquire about how much time the sibling spends with the parents. Ask whether the sibling would like to help change the child's bandage. Encourage the sibling to discuss any negative feelings with the child who is ill. Express interest and ask about the extracurricular activities the sibling is involved in.

Engage with and ask if the sibling has any questions. Engaging with the sibling by asking if there are any questions demonstrates an interest in the sibling and helps to develop a good rapport and helps the sibling to feel more involved. Correct Enquire about how much time the sibling spends with the parents. Assessing how much time the sibling spends with the parents provides information as to whether the sibling's needs are being met. Correct Ask whether the sibling would like to help change the child's bandage. By asking the sibling to assist with providing treatment to the ill child, the sibling can become more involved. Express interest and ask about the extracurricular activities the sibling is involved in. Asking about sibling activities demonstrates an interest and gives the sibling a chance to talk about personal accomplishments.

Which initial nursing action is appropriate for the 10-year-old child who falls down the stairs at school and shows symptoms of major spinal injury? Select all that apply. Ensure that the child has adequate initial stabilization of the entire spinal column. Roll the child back and forth to check for lacerations on the back. Provide a cervical collar with supportive blocks on a rigid backboard to support the spine. Ask the child's friends to help the child stand and take medications. Move the child to the nursing section and place a pillow under the child's head.

Ensure that the child has adequate initial stabilization of the entire spinal column. Provide a cervical collar with supportive blocks on a rigid backboard to support the spine. Rationale A child who has sustained a spinal injury must be immobilized immediately, and proper support must be provided to the spine to avoid additional trauma. This is done with the help of a cervical collar and rigid backboard. This minimizes additional injury. It is not advisable to roll the child because this can injure the spinal cord. The child also should not be allowed to stand up. Placing a pillow under the head is contraindicated because the vertebral column and spinal cord might move, resulting in additional damage to the spinal cord.

The nurse is performing the initial assessment of a 6-year-old pediatric patient. Which components of the nursing process should be included in the musculoskeletal assessment? Select all that apply. Evaluating motor development Checking proper movement of the eyes Measuring the child's head circumference Plotting the child's height on a growth chart Comparing child's current weight to previous results

Evaluating motor development Evaluating the motor developmental milestones of children is a critical component of the musculoskeletal assessment to evaluate for potential musculoskeletal disorders. Correct Checking proper movement of the eyes Movement of the eyes would be part of the eye evaluation when assessing extraocular muscle movement. Plotting the child's height on a growth chart Plotting the child's height on a growth chart is a critical component of the musculoskeletal assessment to evaluate for variations in height. Correct Comparing child's current weight to previous results Comparing the child's current weight to previous results is a critical component of the musculoskeletal assessment to evaluate for variations in weight that could impact the musculoskeletal system.

Which nursing intervention is appropriate while performing magnetic resonance imaging (MRI) for a 6-year-old patient who has come with the parents? Explain to the child about the procedure and the equipment. Administer a sedative to the child before performing the procedure. Get acquainted with the equipment to help the child in a better way. Assess vital signs and position the child to prevent aspiration of secretions.

Explain to the child about the procedure and the equipment. Rationale The appropriate intervention would be to explain to the child about the procedure and the equipment, such as why it is being done, why contrast material is injected, what the child will experience, and how the child can help. Sedation may be required in children, but it is not compulsory. The nurse would already be acquainted with the equipment and environment in which the test is performed so that the nurse can better explain the procedure to children before performing the test. If the child is sedated to perform the test, then the vital signs are assessed after performing the test and the child is positioned to prevent aspiration of secretions.

Neural Tube Defects

Failed closure of neural tube May involve entire length of the neural tube or small portion Incidence: Affects more girls than boys Incidence declining d/t folic acid added to cereal grain products 50% or more: folic acid deficiency Other cases: multifactorial

Which manifestations of the clonic phase differ from those of the tonic phase in a child who has generalized tonic-clonic seizures? Becoming cyanotic Foaming at the mouth Falling to the floor if standing Uttering a peculiar piercing cry

Foaming at the mouth Rationale During the clonic phase of generalized tonic-clonic seizures, the child cannot control oral secretions and may foam at the mouth. During the tonic phase, the child is apneic and may become cyanotic. If cyanotic, the child may fall to the floor if standing and may utter a peculiar piercing cry.

What do we want women of childbearing years to do to prevent NTD's?

Folic acid. Okay, how much should they take? 0.4 mg. For women who have had a previous pregnancy affected by NTD or defect has been identified in utero, folic acid is increased to 4 mg per day beginning 1 month prior to planned pregnancy and through the 1 st trimester.

Transverse fracture

Fracture that is crosswise, at right angles to the long axis of the bone:

The nurse is caring for a child with a spinal cord injury. Which intervention is a priority? Assess each shift for characteristics of stool. Administer an antacid medication twice daily. Bathe, dry, and provide skin care to the patient. Monitor the patient's temperature and skin changes.

Monitor the patient's temperature and skin changes. A spinal cord injury can disrupt the patient's sympathetic nervous system which controls dermal vasodilation. Hyper or hypothermia causes increased stress on the healing of the neurologic system, and monitoring temperature and skin changes will be a priority.

During pediatric care rounds, the nursing staff discusses a toddler with sragile X Syndrome who is nonverbal, unable to hold a crayon, and crawls instead of walks. Which cause and effect relationship does the nurse consider before starting a care plan for this patient? Fragile X syndrome causes intellectual decline that affects the child's ability to speak, write, and move. Fragile X syndrome causes prefrontal lobe dysfunction that affects the child's ability to speak, write, and move. Fragile X syndrome causes physical feature abnormalities that affect the child's ability to speak, write, and move. Fragile X syndrome causes speech and motor dysfunction that affects the child's ability to speak, write, and move.

Fragile X syndrome causes speech and motor dysfunction that affects the child's ability to speak, write, and move. Speech and motor dysfunction directly affect the child's ability to speak, write, and move and, therefore the nurse would consider this cause and effect relationship in caring for this patient.

A patient is nervous about having the pins used for traction removed and asks the nurse what to expect. Which statement is correct about pin removal? A. General anesthesia is used for skeletal traction removal; the patient won't be conscious during the procedure. B. The patient will receive local anesthetics to relieve pain and the area will be draped to prevent visualization of the procedure. C. General anesthesia is used for the procedure but the patient will be conscious for immediate mobility assessment. D. Topical analgesics will be used for the procedure. The bone does not contain pain receptors so the patient feels no pain as the pin is removed.

General anesthesia is used for skeletal traction removal; the patient won't be conscious during the procedure. Pin removal can be scary for a patient. Inform the patient of the use of general anesthesia to decrease anxiety.

Which acquired neuromuscular disorder is characterized by progressive, usually ascending flaccid paralysis? Tetanus Botulism Guillain-Barré syndrome Duchenne (muscular dystrophy)

Guillain-Barré syndrome Rationale Guillain-Barré syndrome is characterized by progressive, usually ascending flaccid paralysis. Tetanus is painful muscular rigidity primarily involving the masseter and neck muscles. Botulism is an acute flaccid paralysis caused by the toxin produced by Clostridium botulinum. Duchenne, the most severe and most common form of muscular dystrophy in children, is characterized by pseudohypertrophy of muscles and atrophy in the later stages.

Which routine vaccines are appropriated for reducing the incidence of bacterial meningitis during infancy? Select all that apply. E. coli vaccine H. influenza vaccine Pneumococcal vaccine Staphylococcal vaccine Meningococcal vaccine

H. influenza vaccine Pneumococcal vaccine Rationale H. influenza vaccine (Hib) and pneumococcal vaccine are routinely recommended for all children beginning at 2 months of age. There is no routine and effective vaccine for E. coli, meningitis, or staphylococcus. Meningococcal vaccine is not administered before infancy; it is administered to children between 2 to 11 years of age who are at risk of developing meningococcal meningitis.

An 8-year-old patient is having problems adjusting to daily radiation treatments. Which is the most appropriate action for the nurse to take to help with the adjustment? Collaborate with the oncology radiologist. Encourage the parents to comfort the child. Have a discussion with the child life specialist. Educate the child on the importance of the treatments.

Have a discussion with the child life specialist. Use of the child life specialist uses methods that are age-appropriate, educational, supportive, and therapeutic and may include medical play and art and therapeutic play and art. Therefore, this would be the most appropriate action.

Which method is an appropriate way to stop an occasional episode of epistaxis? Having the child sit up and lean forward Applying ice under the nose and above the lip Having the child lie down quietly with the feet elevated Applying continuous pressure to the nose with the thumb and forefinger for 1 minute

Having the child sit up and lean forward Rationale Having the child sit up and lean forward is the intervention used to prevent the child from aspirating blood. Pressure, not ice, is indicated for an occasional episode of epistaxis. Having the child lie with the feet elevated could lead to aspiration. Continuous pressure for 10 minutes is recommended; one minute would not be long enough.

heat exhaustion

Heat exhaustion R/T vigorous exercise in a hot environment and excessive loss of fluids > first symptoms - thirst, headache, fatigue, dizziness, anxiety, nausea, vomiting, may be disoriented, temp may be normal or slightly elevated, profuse sweting secondary symptoms - sweating, tachycardia, hypotension, syncope > move to cool area, give fluids, IV fluids if vomiting, rest

heat stroke

Heatstroke (temp higher than 104) > headache, weakness, disorientation, agitation, confusion, lethargy, loss of consciousness > cool off, clothes off, wet towels or immersion in cold water, fans, hospital > no antipyretics because liver isn't working already and liver and renal failure are common in heatstroke supportive care 0 supplemental oxygen cautious fluid and electrolyte relacemnt rapid cooling till 102

Which explanation is appropriate for an 8-year-old child when describing what platelets are and their purpose? Make up the liquid portion of blood Help keep germs from causing infection Carry the oxygen we breathe from the lungs to all parts of the body Help the body stop bleeding by forming a clot (scab) over the hurt area

Help the body stop bleeding by forming a clot (scab) over the hurt area Rationale Platelets are involved in clotting. Keeping germs from causing infections is the function of white blood cells. Plasma is the liquid portion of blood. Carrying oxygen is the function of the red blood cells.

Which nursing intervention is appropriate when performing a blood transfusion? Transfuse blood slowly for initial 20% of blood volume. Identify donor and recipient blood types and groups. Save donor blood for recrossmatch with patient's blood. Send the patient's blood and urine sample to the laboratory.

Identify donor and recipient blood types and groups. Rationale The nurse would first check the identification of the recipient and donor's blood group and type regardless of the blood product being used. This is the first step to ensure a safe blood transfusion without hemolytic reactions. After verifying with another nurse, the blood should be transfused slowly for the initial 20% of blood volume. The nurse would obtain the vital signs of the patient at frequent intervals and monitor for any reaction in the patient. In case of a reaction, the transfusion should be stopped and donor blood saved to recrossmatch with the patient's blood. In addition, with a reaction, the patient's blood and urine samples are sent to the laboratory to detect the presence of hemoglobin.

A patient is learning to cope with a newly diagnosed chronic illness. To help the patient adjust, which actions should the nurse take? Select all that apply. Initiate a standardized plan of care for the child Help the child with incorporating the illness into each stage of life Provide nursing interventions during acute exacerbations of the illness Provide support as the child accepts the illness as a part of each stage of life Educate the child in a way that there is an understanding that illness will be present long-term

Help the child with incorporating the illness into each stage of life Helping the child incorporate the illness appropriately into each stage of growth and development will be a necessary part of nursing care. Provide support as the child accepts the illness as a part of each stage of life Assisting the child and family to accept the illness appropriately into each stage of growth and development, regardless of the child's age at diagnosis, is important for the nurse to address. Educate the child in a way that there is an understanding that illness will be present long-term Nursing care includes assisting the child and family to accept, understand, and incorporate the illness appropriately into each stage of growth and development, regardless of the child's age at diagnosis.

Which cerebral palsy is appropriate in the child that is crawling asymmetrically using only one arm and leg to propel on either buttocks or abdomen? Diplegic Dystonic Athetoid Hemiplegic

Hemiplegic Rationale Children with hemiplegic cerebral palsy have an asymmetric crawl, using the unaffected arm and leg to propel themselves on either the buttocks or the abdomen. Children with diplegic cerebral palsy have abnormal crawling with propulsion by hand movements only. Lower extremities and hips are hiked along such as a bunny hop. Children with dystonic cerebral palsy have abnormal posture and slow and twisting movements of the trunk or extremities. Children with athetoid cerebral palsy have facial grimacing and writhing movements of the tongue, fingers, and toes.

Which labs are appropriate when reporting abnormal blood work? Select all that apply. Hematocrit (Hct) of 36% Hemoglobin (Hgb) of 9.5 Platelet count of 120,000 White blood cell count of 17,500 Red blood cell count of 3.0 million

Hemoglobin (Hgb) of 9.5 Platelet count of 120,000 White blood cell count of 17,500 Red blood cell count of 3.0 million Rationale The hemoglobin (Hgb) of 9.5 is abnormal, because average values are 11.5 to 15.5. The platelet count of 120,000 is abnormally low, because the average value is 150,000 to 400,000. The white blood cell count is abnormally high, because the average value is 4,500 to 13,500. The red blood cell count is abnormally low, because the average count should be 4.5 to 5.5 million. These values should all be reported. The hematocrit of 36% is within the normal range of 35 to 45%.

Which condition is appropriate for the child that complains of a sudden severe headache after 30 minutes of a blood transfusion? Hepatitis Febrile reaction to the blood transfusion Delayed reaction to the blood transfusion Hemolytic reaction to the blood transfusion

Hemolytic reaction to the blood transfusion Rationale A sudden, severe headache in a child receiving a blood transfusion is a symptom of a hemolytic reaction to the blood transfusion. Hepatitis would result in a toxic reaction, high fever, severe headache or substernal pain, hypotension, intense flushing, vomiting, or diarrhea. A febrile reaction would result in fever. In a delayed reaction, symptoms would take longer to present.

Types of Hemophilia /A

Hemophilia A-Factor VIII Deficiency Classic hemophilia Deficiency of factor VIII Accounts for 80% of cases of hemophilia Occurrence—1 in 5000 males

Which funduscopic findings are appropriate for the patient diagnosed with papilledema? Select all that apply. Hemorrhage Optic disc swelling Tortuosity of vessels Raised venous pulsations Well-defined optic disc margins

Hemorrhage Optic disc swelling Tortuosity of vessels Rationale Papilledema refers to the swelling of the optic disc. The characteristic features of papilledema are hemorrhage, optic disc swelling, and tortuosity of vessels. These are caused by an increase in intracranial pressure. In papilledema, venous pulsations are usually absent and the optic disc margins are indistinct and not well demarcated.

An infant is brought to the clinic for a well-child visit. The nurse notices that the infant is tightly swaddled in blankets, as she was during an earlier visit. The nurse teaches correct swaddling of the infant, with the hips placed in slight flexion and abduction during swaddling. What complication is the nurse trying to prevent? Hip dysplasia Pressure sore Limb fracture Muscle contracture

Hip dysplasia Rationale There is evidence that tight swaddling makes an infant prone to hip dysplasia. The nurse is primarily trying to prevent developmental dysplasia of the hip. A tight wrap for a longer period of time may result in a pressure sore. If the wrapping is too tight, it may result in soft tissue injury. Limb fracture may happen if there is any trauma involved. A contracture of hip adductor and flexor muscles is a sign of developmental dysplasia of the

Prognosis

Historically, most died by age 5 years Now those with mild to moderate hemophilia live near-normal lives Gene therapy for the future Infuse carrier organisms into patient; these act on target cells to promote manufacture of deficient clotting factor

What is one of the major physical characteristics of a child with Down syndrome? Excessive height Spots on the palms Hypotonic musculature Inflexibility of the joints

Hypotonic musculature Correct Hypotonic musculature is one of the major characteristics. Children with Down syndrome have short stature. Children with Down syndrome have a transverse palmar crease. Hyperflexibility is a characteristic of Down syndrome.

Which medication administration is avoided when caring for a child with epilepsy who is being treated with phenytoin? Phenytoin with folic acid IV phenytoin with glucose Phenytoin intravenously at over 20 mg/min Phenytoin if seizures are absent for a month

IV phenytoin with glucose Rationale IV phenytoin would not be given with glucose because the drug precipitates when mixed with glucose. Phenytoin use is associated with deficiency of folic acid; therefore, folic acid intake should be adequate with phenytoin use. Phenytoin would be administered through a slow intravenous push at a rate that does not exceed 50 mg/min. Antiepileptic medications such as phenytoin would be continued until the child is seizure free for 2 years.

Head injury

is a pathologic process involving the scalp, skull, meninges, or brain as a result of mechanical force. The most common head injury is concussion, which is defined as an alteration in mental status with or without loss of consciousness. Fractures resulting from head injuries may be classified as linear, comminuted, depressed, open, basilar, and growing fractures.

The nurse provides education on home care for a child in a cast. Which of the following statements demonstrate parental understanding regarding indications to contact the health care provider? Select all that apply. If the child has persistent inflammation, contact the health care provider. If the cast feels hot or has an unusual odor, contact the health care provider. If the child can move the digits distal to the cast, contact the health care provider. If the child develops a sporadic itch under the cast, contact the health care provider. If the child complains of pain, burning, numbness, or tingling, contact the health care provider.

If the child has persistent inflammation, contact the health care provider. Contact the health care provider if inflammation persists. Inflammation should peak 24-28 hours after the application of the cast. If the cast feels hot or has an unusual odor, contact the health care provider. Contact the physician if a cast feels warm or hot or has an unusual smell because this may indicate neurovascular compromise and infection. If the child complains of pain, burning, numbness, or tingling, contact the health care provider. Contact the physician if the child complains of pain, burning, numbness, or tingling because this may indicate neurovascular compromise.

Which nursing consideration when to seek medication attention is appropriate for the parents of a child with simple febrile seizures? Within seconds of a seizure If the seizure lasts more than 5 minutes If the seizure lasts more than 10 minutes To place the child who is actively having a seizure in the car and drive to the nearest emergency department

If the seizure lasts more than 5 minute Rationale A priority nursing consideration for the parents of a child with simple febrile seizures is to seek medical attention if the seizure lasts more than 5 minutes, not immediately after the seizure or if it lasts more than 10 minutes. The priority is to seek medical attention if the seizure lasts more than 5 minutes, not to place the child experiencing an active seizure in the car and drive to the nearest emergency department.

The 4-year-old child presents to the emergency department following a fall that dislocated the hip. The nurse anticipates assisting with which treatment for this patient? Surgical open reduction of the joint Monitoring for spontaneous reduction of the joint Temporary restriction of the joint with a spica cast Immediate hip relocation performed with sedation

Immediate hip relocation performed with sedation Rationale In a child under 5 years of age, the hip can be dislocated after a fall. A physician can do a closed hip reduction within 60 minutes of the injury. The procedure can be done with the child under mild sedation and under local anesthesia. There may be an irreversible damage if the hip is not relocated within a certain period of time. In hip dislocation, spontaneous reduction does not take place. If closed reduction does not help, then surgical open reduction is required. After the hip relocation, the child is placed under immobilization with a spica cast.

The school nurse is observing a gym class. The nurse is most concerned about the risk for injury in which children? In children given extra fluids on a hot day In children who are physically fit but over tired In children who take the appropriate safety precautions In children who are not physically prepared for the activity

In children who are not physically prepared for the activity Rationale Children who are not physically prepared for a particular sporting activity are more likely to sustain a serious injury than are those who are prepared for the activity. Children who are physically fit but tired are not more likely to suffer a serious sports injury than children who are not physically prepared. The administration of extra fluids on hot days is not associated with an increased risk of serious sports injuries. Appropriate safety precautions do not increase a child's risk of sustaining a serious sports injury.

Which hazard is appropriate to have concern for when caring for an infant with hydrocephalus who has just undergone surgical placement of a ventriculoperitoneal shunt? Infection Hemorrhage Poor feeding Seizure disorder

Infection Rationale Infection is the greatest hazard of the postoperative period in an infant who has just been fitted with a ventriculoperitoneal shunt for hydrocephalus. Although bleeding may occur, hemorrhage is not the greatest hazard of the postoperative period. A seizure disorder is not expected in the postoperative period. Poor feeding can occur after a ventriculoperitoneal shunt is placed as a result of excess fluid on the brain, but the greatest hazard of the postoperative period is infection.

A patient presents with Guillain-Barré syndrome (GBS). What does the nurse anticipate finding in the history and physical? Select all that apply. Tuberculosis skin test that was positive Toe infection positive for staphylococcus Influenza vaccine received one month ago Upper respiratory infection two weeks ago History of frequent urinary tract infections

Influenza vaccine received one month ago The flu shot has been demonstrated to have a link to the autoimmune process triggered in Guillain-Barré. Upper respiratory infection two weeks ago Upper respiratory infection has been associated with the subsequent development of Guillain-Barré syndrome.

Implementation of which nursing intervention will reduce the risk?

Initiate hourly assessment of Madison's foot distal to the fracture site.

Which method is appropriate when injecting parenteral iron preparations? Injection into the deltoid muscle Injection into a large muscle with the use of the Z-track method Injection into a large muscle with the use of the air-lock method Injection into a large muscle followed by massage to increase absorption

Injection into a large muscle with the use of the Z-track method Rationale The best way to administer parenteral iron preparations is injecting the medication into a large muscle with the use of the Z-track method. Never inject iron into the deltoid muscle. It is not appropriate to massage the site after injection of iron because this may worsen skin staining and irritation. When iron is being injected into the muscle, the Z-track method is preferred over the air-lock method.

Which nursing role is appropriate in supporting the parents of an infant born with physical defects? Select all that apply. Instruction on modifying their home for the care of their child Providing information about the diagnosis on an ongoing basis Care that is primarily focused on the physical needs of the child Clarification on what they can expect in the hospital and home Supporting the family in coping with the emotional aspects of the disorder

Instruction on modifying their home for the care of their child Providing information about the diagnosis on an ongoing basis Clarification on what they can expect in the hospital and home Supporting the family in coping with the emotional aspects of the disorder Rationale When a child is born with physical defects, the family needs the nurse to provide support by coordinating care for the child and family to help the family cope with the emotional aspects of the disorder. They also need ongoing information about the diagnosis, clarification of what they can expect in the hospital and home environment, and instruction on modifying their home for the care of their child. Care would be offered in a holistic way that is focused on the physical, emotional, and spiritual needs of the child rather than just the physical needs of the child.

Which first therapeutic step is appropriate in the management of a child with bacterial meningitis? Administration of dexamethasone Intravenous infusion of cephalosporin Restoration of circulating blood volume Isolation of the child in an intensive care unit

Intravenous infusion of cephalosporin Rationale While caring for a child with meningitis, the nurse should administer antibiotics as soon as they are ordered. Cephalosporin antibiotics are superior for prompt sterilization of cerebrospinal fluid and also reduce the risk of severe hearing impairment. Dexamethasone is used to initially manage increased intracranial pressure and cerebral herniation, but its long-term effectiveness to reduce complications is doubtful. Restoration of circulating blood volume and maintenance of electrolyte balance is needed to treat shock. The child is isolated from other children, usually in an intensive care unit for close observation.

Which method of administration is appropriate to ensure safety for an infant being treated with a liquid iron preparation as a supplement? Select all that apply. With a dropper toward the back of mouth With meals for better absorption By educating the parents that iron supplementation will cause transient diarrhea Iron supplementation will turn stools to a black color Avoid administering with milk products in order to increase absorption

Iron supplementation will turn stools to a black color Avoid administering with milk products in order to increase absorption With a dropper toward the back of mouth Rationale Iron supplementation is given to promote growth and development. However, it may stain the teeth on exposure. Therefore, iron supplements should be administered with a dropper towards the back of the mouth to prevent staining of the teeth. Iron supplements turn the stools to a black or tarry green color. Intake of iron supplements with milk products causes a decrease in iron absorption, because milk products bind with iron and prevent absorption. Iron supplements should be taken between meals so as to increase their absorption. Iron supplements cause transient constipation but not diarrhea.

What is a clinical manifestation of increased intracranial pressure (ICP) in infants? Irritability Photophobia Vomiting and diarrhea Pulsating anterior fontanel

Irritability Correct Irritability is one of the changes that may indicate increased ICP. Photophobia is not indicative of increased ICP in infants. A pulsing anterior fontanel is normal. Vomiting is one of the signs of increased ICP in children, but when present with diarrhea, it is indicative of a gastrointestinal disturbance.

Which reason is appropriate for replacing intravenous (IV) phenytoin with fosphenytoin in a patient with seizures? Select all that apply. Is suitable with saline solutions Is suitable with glucose solutions Has reduced allergic manifestations Can be administered at a higher push rate Can be given by the intramuscular route.

Is suitable with saline solutions Is suitable with glucose solutions Can be administered at a higher push rate Can be given by the intramuscular route.

Which nursing statements are appropriate when explaining to the parents of a child with muscular dystrophy about mechanical cough in-exsufflator (MIE)? Select all that apply. It is also referred to as cough assist. It can be connected directly to a tracheostomy. It delivers negative inspiratory pressures at a set rate. It is designed so that insufflation mimics cough reflex. It can be used in the daily management of respiratory function. It involves coordination between exsufflation and the breathing rhythm.

It is also referred to as cough assist. It can be connected directly to a tracheostomy. It can be used in the daily management of respiratory function. It involves coordination between exsufflation and the breathing rhythm. Rationale MIE is also referred to as cough assist. It can be connected directly to a tracheostomy. It is safe and effective in the daily management of respiratory function. The negative pressure exsufflation is in coordination with the breathing rhythm of the patient. It delivers positive inspiratory pressures at a set rate. It is designed such that exsufflation mimics cough reflex so that mucus can be effectively cleared.

Which nursing information is appropriate for the patient having a positron emission tomography (PET) scan following the removal of a tumor in the skull when discussing the importance of this scan? Select all that apply. It measures the blood flow to the brain. It detects biochemical changes in tissue. It detects the metabolic activity in the brain. It detects the electrical activity of the brain. It visualizes the morphologic features of target tissue.

It measures the blood flow to the brain. It detects biochemical changes in tissue. It detects the metabolic activity in the brain. Rationale PET scans measure and detect the blood volume and flow to the brain. A PET scan is also helpful in assessing the biochemical changes in the target tissue. The metabolic activity of the brain may also be assessed by PET scan. An electroencephalogram is useful in assessing the electrical activity of the brain. The morphologic features of the brain are assessed by magnetic resonance imaging (MRI) and computed tomography (CT) scan.

A 3-year-old patient is diagnosed with hydrocephalus. A ventricular shunt was placed to relieve the pressure. What is the nursing care priority for a patient who had a ventricular shunt put into place? Assessing for proper bowel movements Assessing neurologic status every two hours Monitoring for fluid leaking from the incision Monitoring for headaches when the patient sits up

Monitoring for fluid leaking from the incision The main priority in the nursing care of a patient who had a ventricular shunt placed is the prevention or early detection of shunt infection, malformation, and malfunction. A leaking incision presents a high risk of infection.

How is Becker muscular dystrophy different from Duchenne muscular dystrophy? Select all that apply. It progresses much slower. Its age of onset is over 7 years. It is treated with physical therapy. Its inheritance pattern is X-linked recessive. It is manifested by lordosis and waddling gait.

It progresses much slower. Its age of onset is over 7 years. Rationale Becker muscular dystrophy has much slower progression than Duchenne muscular dystrophy. The age of onset for Becker muscular dystrophy is over 7 years of age, whereas the age of onset for Duchenne muscular dystrophy is 3 to 5 years of age. Becker muscular dystrophy and Duchenne muscular dystrophy are treated with physical therapy to prevent disuse atrophy of unaffected muscles. The inheritance pattern of both Becker muscular dystrophy and Duchenne muscular dystrophy is X-linked recessive. Both Becker muscular dystrophy and Duchenne muscular dystrophy are manifested by lordosis and a waddling gait.

Kids with NTD. What is one thing they are at huge risk for based on what they come in contact with?

Latex allergy from chronic exposure in hospital. We have to be really careful with the equipment we use with these kids. Make sure it is latex free. Because they are in the hospital and get so many treatments and so forth, they are very prone to develop latex allergies. We really really want to make sure were careful with the equipment we use. We really want to make sure we teach the parents because there is a lot of crossovers in food, toys, household items, especially anything that might come in contact with their mucous membranes. So, we really gotta do a lot of education with the parents on that. So, our goals of care would be, obviously what you guys said: prevent infection, prevent aspiration, and prevent a latex allergy

The nurse is caring for a fourteen-year-old patient suffering from a migraine headache. Which findings cause the nurse the most concern? Select all that apply. Left arm weakness Projectile vomiting Nausea and dizziness Urinary incontinence Sound and light intolerance

Left arm weakness This is a sign of a stroke that can occur with a severe migraine because it is a vascular headache. Though rare, this can be a complication of migraine that warrants immediate attention. Weakness should not be confused with arm tingling, which can be part of the patient's aura. Urinary incontinence This could be a sign of a seizure or even a stroke. Though rare, this can be a complication of migraine that warrants immediate attention.

A child has a painful limp and the X-ray was negative for fracture and dislocation. An MRI reveals increased bone density of the femoral head. Based on these findings, which does the nurse suspect as the pathology for this patient? Osteomyelitis Osgood-Schlatter disease Legg-Calvé-Perthes disease Slipped Capital Femoral Epiphysis (SCFE)

Legg-Calvé-Perthes disease Legg-Calvé-Perthes disease occurs when blood flow to the femoral head is diminished which leads to eventual necrosis. During this process, x-rays may be negative but MRI will reveal the damage to the femoral head.

The nurse is caring for a child with osteosarcoma who recently underwent amputation of the lower right leg. The child is complaining of severe pain and itching in the right foot. What is the best response by the nurse? Give the child a mild opioid. Explain to the child that the child is just angry about losing the foot. Let the child know that the sensations are real and not imagined. Tell the child that these sensations are a drug-induced hallucination.

Let the child know that the sensations are real and not imagined. Rationale Letting the child know that the sensations are real, not imagined, is the best response by the nurse. Phantom limb pain, in which the patient experiences tingling, itching, and pain in the amputated limb, can develop after amputation. Giving the child a mild opioid is not the most effective way to reduce the pain. Amitriptyline is the drug of choice for phantom limb pain. Severe pain and itching in the amputated extremity is not a manifestation of anger over losing the foot or a drug-induced hallucination.

Which findings are appropriate when describing a postoperative child's level of consciousness (LOC) as "lethargic"? Select all that apply. Sluggish speech Confused about place Eyes follow objects only by reflex Limited spontaneous movement Drowsy and falls asleep quickly

Limited spontaneous movement Drowsy and falls asleep quickly Sluggish speech Rationale Level of consciousness is determined by observations of the child's responses to the environment. The "lethargic" child has sluggish speech and limited spontaneous movement. The child is drowsy and falls asleep quickly. The child is not fully conscious. A "disoriented" child has decreased LOC and is confused about the time and place. In the "persistent vegetative state" the child has permanently lost function of the cerebral cortex. The eyes of this child follow objects only by reflex.

Which clinical manifestation indicates aversive seizure in a child with partial seizure? Loss of consciousness Salivating and has arrested speech Tonic-clonic movements Sequential clonic movements

Loss of consciousness Rationale Aversive seizure is the most common motor seizure in children with partial seizures. The child may be aware of the movements or may have a loss of consciousness. In Rolandic or Sylvan seizure, the child manifests salivation, arrested speech, and has tonic-clonic movements. In Jacksonian march, an orderly, sequential progression of clonic movements begins in one body part, and moves to adjacent body parts.

What clinical manifestations in an infant would be suggestive of spinal muscular atrophy (Werdnig-Hoffmann disease)? Hypertonicity Lying in the frog position Hyperactive deep tendon reflexes Motor deficits on one side of body

Lying in the frog position Correct The infant lies in the frog position with the legs externally rotated, abducted and flexed at knees. The child has hypotonia and inactivity as the most prominent features. The deep tendon reflexes are absent. The motor deficits are bilateral.

nonunion

bone fragments cannot maintain in correct alignment due to inadequate reduction or poor immobilization Causes: - separation of bone fragments at fracture site - Tissue wedges its way in between bone fragments - Loss of bone tissue from necrosis - Infection - Poor nutrition - Interruption of blood supply - Diseases that influence Ca metabolism (Vit D def) - Bone cancer - Corticosteroids treatment - bone grafting from tibia or iliac cres postsurgical immobilization is crucial

A child presents to the emergency department with increased intracranial pressure. Which strategies should be included in the nurse's care plan to help manage the pressure? Select all that apply. Maintain a comfortable room temperature. Administer intravenous fluids (IV) as prescribed. Elevate the child's head to 30 degrees at all times. Administer anticonvulsant medications as needed. Administer insulin for elevated blood glucose levels as prescribed.

Maintain a comfortable room temperature. Normothermia should be maintained in children with increased intracranial pressure because hyperthermia and hypothermia cause stress in the brain as it tries to maintain normothermia. Administer intravenous fluids (IV) as prescribed. Normovolemia is necessary. The nurse ensures that there is not hypervolemia or hypovolemia because decreased or increased blood flow causes stress on the brain. Elevate the child's head to 30 degrees at all times. The child with increased intracranial pressure should have the head elevated to decrease pressure caused by positioning. Administer insulin for elevated blood glucose levels as prescribed. Normal blood glucose is essential to prevent increased metabolic demands placing stress on the brain.

school age

Major task is achieving a sense of industry and activity and enter the school child schoul return to school asap as diagnosis is established structured plan of care during school days prevent schoolophobia due to often absences prepare to return to school - tutor, role play of child and "schoolmates", preparation of classmates by the teacher expressive activities are very helpful

An adolescent with a seizure disorder is admitted to the hospital after a fall. The child is increasingly withdrawn and depressed. Which action should the nurse take to best care for this adolescent? Sit with the patient on the nurse's breaks and lunch period to decrease the teen's withdraw Accept the changes in behavior as a temporary regression in an adolescent patient that will quickly resolve on their own Encourage the family to visit with the adolescent more often and help make decisions for the teen to decrease the teen's stress level Make arrangements for more peer interaction opportunities that might include increased visitation, social media, or other opportunities to connect

Make arrangements for more peer interaction opportunities that might include increased visitation, social media, or other opportunities to connect The child is showing signs of depression, which could be from missed socialization with peers. If the nurse can accommodate extra visitors safely, then it should be done with permission from administration. If peers are not allowed in the hospital unit, a phone may be provided to allow the child to speak with friends.

A 10-year-old patient presents to the pediatric clinic with maxillary hyperplasia, malocclusion, hypochromic anemia, and elevated aspartate aminotransferase (AST) and alanine aminotransferase (ALT) levels. Which additional assessments should the nurse perform? Select all that apply. Administer desferoxamine. Measure oxygen saturation. Perform a complete abdominal assessment. Evaluate peripheral pulses in all extremities. Evaluate parent's understanding of treatment.

Measure oxygen saturation. Maxillary hyperplasia, malocclusion, and hypochromic anemia indicate extramedullary erythropoiesis related to prolonged beta-thalassemia, which can affect tissue perfusion. Oxygen saturation is suggestive of peripheral tissue perfusion. Correct Perform a complete abdominal assessment. Elevated AST and ALT levels indicate liver dysfunction. It is important for the nurse to assess for hepatosplenomegaly, a complication of beta-thalassemia and other disease processes. Correct Evaluate peripheral pulses in all extremities. Maxillary hyperplasia, malocclusion, and hypochromic anemia indicate extramedullary erythropoiesis related to prolonged beta-thalassemia, which can affect tissue perfusion. Peripheral pulses are suggestive of peripheral tissue perfusion. Correct Evaluate parent's understanding of treatment. Maxillary hyperplasia, malocclusion, and hypochromic anemia indicate extramedullary erythropoiesis related to prolonged beta-thalassemia, which requires strict compliance with treatment and follow-up requirements. The nurse should evaluate the parent's understanding of treatment to determine the need for further teaching.

The nurse is performing a musculoskeletal assessment on a pediatric patient. Which action is the most appropriate? Schedule a radiography Review laboratory tests Ask the child draw a circle Measure the child's height

Measure the child's height Measuring the child's height should be done at every well-check visit and is the most appropriate intervention to assess for potential musculoskeletal disorders by comparing height measurements over time.

Which nursing action is appropriate when caring for the child with a myelomeningocele in the preoperative stage? Applying a heat lamp to facilitate drying and toughening of the sac Placing the child on one side to decrease pressure on the spinal cord Keeping the skin clean and dry to prevent irritation from diarrheal stools Measuring the head circumference and examining the fontanels for signs of developing hydrocephalus

Measuring the head circumference and examining the fontanels for signs of developing hydrocephalus Rationale Obstructive hydrocephalus is frequently associated with myelomeningocele. Assessment of the fontanels and daily measurements of the head circumference will aid in early detection of associated increased intracranial pressure. Before surgery the child is kept in a prone position to decrease tension on the sac and reduce the risk of trauma or sac tearing. The sac must be kept moist. Sterile, moist, nonadherent dressings are placed over the sac as prescribed by the health care provider. Most infants do not have diarrheal stools. The sac area, though, should be kept clean and dry and out of contact with urine and stools.

strain

Microscopic tear to the musculotendinous unit:

Therapeutic Management

Mobilization - anckle-foot orthoses, wheeled scooter boards, wheeled go-carts Surgical interventions - whose spasticity causes more deformities, or doesnt resond to orthotics,it corrects contructures, provide stability and alligns bones; neurosurgical are rare Medications Botulinum toxin type A (Botox) to reduce contractures, reduce spaspicity and can ambulate with AFO Baclofen - diaphoresis and constipation, hallucinations, mood changes, seizures, nausea and urinary incontinen. can implant pump for meds (baclofen + antibiotics), come to hospital to refil pump Oral Implanted pump for intrathecal administration Dantrolene sodium (Dantrium) - decreasing spasticity, hepatotoxic, drowsinss, fatigue, muscle weakness Diazepam (Valium) for older kids Increase their calories Seizures Technical aids

Boston Brace

Molded plastic jacket. Used for low thoracolumbar and lumbar curves.

Which nursing focus is appropriate for an infant who has recently undergone surgery to correct myelomeningocele? Select all that apply. Monitoring vital signs, nourishment, signs of infection, and pain management Caring for the operative site and monitoring signs of cerebrospinal fluid leakage Educating the parents on hydrocephalus, and developing cognitive skills Practicing stretches and exercises to minimize muscle contractures and deformity Educating parents on positioning, feeding, skin care, and exercise

Monitoring vital signs, nourishment, signs of infection, and pain management Caring for the operative site and monitoring signs of cerebrospinal fluid leakage Educating parents on positioning, feeding, skin care, and exercise Rationale Postoperative care for a child who has undergone a surgery to correct myelomeningocele includes monitoring vital signs, nourishment, signs of infection, and managing pain. It is also important to take adequate steps in caring for the operative site and monitoring signs of cerebrospinal fluid leakage. The nurse also needs to educate the parents on positioning, feeding, skin care, and range-of-motion exercises after the child returns home. Parents are not taught about hydrocephalus or cognitive skill development. Similarly, they are not taught about stretches and exercises to minimize the muscle contractures and deformity.

Duchenne Muscular Dystrophy (DMD)

Most severe & most common MD in childhood X linked recessive trait- short arm of the X chromosome males affected almost exclusively females migh be crrier, but no muscles weakness, but may develop cardiomyopathy dystrophin is absent in muscles (protein product) lack of dystrofin in brain -cognitive impairment

Which nursing intervention does the nurse include in the care plan for a child with chronic osteomyelitis? encorage frequent ambulation Administer pain medication with meals Move and turn the child carefully and gently Providing active range-of-motion exercises of the affected extremity

Move and turn the child carefully and gently Rationale Osteomyelitis is extremely painful. Movement is carried out only as needed and then carefully and gently. Active range-of-motion exercises are contraindicated until pain has subsided. Pain medication should be administered as needed. Ambulation is contraindicated until pain has subsided.

Which assessment finding is appropriate with a comminuted fracture? Irregular fragments of broken bones Single fracture line and soft-tissue swelling Multiple associated linear fractures Bleeding around the eyes (raccoon eyes)

Multiple associated linear fractures Rationale Comminuted fractures consist of multiple associated linear fractures as a result of intense impact from repeated blows against an object. In a depressed fracture, the bone is locally broken, usually into several irregular fragments that are pushed inward, causing pressure on the brain. The child's head appears misshapen. A linear fracture is a single fracture line that starts at the point of maximum impact but does not cross suture lines. Most linear skull fractures are associated with an overlying hematoma or soft-tissue swelling. Basilar fractures involve the basilar portion of the frontal, ethmoid, sphenoid, temporal, or occipital bones. Clinical features may include bleeding around the eyes.

Which treatment is appropriate for the child with sickle cell anemia whose transcranial Doppler test indicates abnormal intracranial vascular flow? Splenectomy surgery Multiple blood transfusions High doses of opioids to prevent pain Oxygen therapy to prevent hypoxia

Multiple blood transfusions Rationale If the child with sickle cell anemia has an abnormal intracranial blood flow, the child is at high risk of developing a stroke. In this case the child needs to be given chronic transfusion therapy. If a child has recurrent sequestration of the spleen, a splenectomy must be performed to save the child's life. Based on the knowledge that the child has abnormal intracranial vascular flow, it cannot be determined that the child has severe pain. Oxygen therapy is given to patients with sickle cell anemia only if they have severe hypoxia.

Why are infants particularly vulnerable to acceleration-deceleration head injuries? The anterior fontanel is not yet closed. The nervous tissue is not well developed. The scalp of the head has extensive vascularity. Musculoskeletal support of the head is insufficient.

Musculoskeletal support of the head is insufficient. Correct The relatively large head size coupled with insufficient musculoskeletal support increases the risk to infants. The anterior fontanel, nervous tissue, and scalp of the head do not have an effect on this type of injury.

Which findings in cerebrospinal fluid analysis in aseptic meningitis differ from those of bacterial meningitis? Select all that apply. Negative Gram stain Elevated opening pressure Decreased glucose content Increased lymphocyte count Reduced white blood cell count

Negative Gram stain Increased lymphocyte count Rationale In aseptic or viral meningitis, the cerebrospinal fluid analysis shows negative Gram stain while in bacterial meningitis, it shows positive Gram stain. Lymphocytes are increased in aseptic meningitis but not in bacterial meningitis. Opening pressure is elevated in bacterial meningitis but it remains normal in aseptic meningitis. In bacterial meningitis, the cerebrospinal fluid analysis shows decreased glucose content. In aseptic meningitis, it remains normal. In both cases, there is elevated white blood cell count. However, in aseptic meningitis, there is only slight elevation.

Match the medical conditions associated with Down syndrome to the period during which it occurs.

Neonatal Period Atrial septal defect Infancy Intestinal obstruction Childhood Acute leukemias Adolescence Intellectual disability

Myelomeningocele

Neural tube fails to close (28 days of pregnnancy) Anywhere along spinal column Lumbar and lumbosacral areas most common Diagnosed Prenatally At birth Sac: meninges, spinal fluid & nerves Easily ruptured Prone to leakage of CSF Varying & serious degrees of neurologic deficit Clinically, term spina bifida refers to myelomeningocele causes hydrocephalus Below 2nd lumbar vertebra: Flaccid paralysis of lower extremities Sensory deficit Not necessarily uniform on both sides of defect

So, what is a myelomeningocele?

Neural tube fails to close during the first 28 days of pregnancy. Sac encased in fine membrane that is prone to tears through which CSF leaks. The sac may be covered by dura, meninges, or skin. Most myelomeningoceles occur in the lumbosacral area. Anomaly most associated with myelomeningocele is hydrocephalus = Careful monitoring of head circumference, fontanels, and head US Yeah, you've got the baby is born with a sac on their lower back containing nerves and spinal fluid and membranes and meninges.

Which areas are appropriate for the senior nurse to tell the newly hired nurse who is caring for an infant who has myelomeningocele? Select all that apply. Neurologic and behavioral development and measurement of the head circumference and fontanels Meningeal sac protection and intervention plans to optimize the child's development Constraint-induced movement therapy to induce the weaker extremities to function Infection and skin breakdown occurrence and signs of urologic and bowel complications Changes in hemodynamic status, joint contractures, disuse atrophy, and obesity

Neurologic and behavioral development and measurement of the head circumference and fontanels Meningeal sac protection and intervention plans to optimize the child's development Infection and skin breakdown occurrence and signs of urologic and bowel complications Rationale The nurse caring for an infant who has myelomeningocele would assess for neurologic and behavioral development. The nurse would also measure the head circumference and assess the fontanels from time to time. For a child with myelomeningocele, it is important to protect the meningeal sac and plan appropriate interventions to optimize the child's development. It is also important for the nurse to prevent infection and skin breakdown and observe for signs of urologic and bowel complications. Constraint-induced movement therapy is not used to treat myelomeningocele. The nurse would monitor hemodynamic status, joint contractures, disuse atrophy, and obesity in case the child has Duchenne muscular dystrophy. However, monitoring is not done if the child has myelomeningocele.

Which finding is appropriate for a 2-month-old infant after a car accident whose Moro, tonic neck, and withdrawal reflexes are present? Neurologic health Decorticate posturing Severe brain damage Decerebrate posturing

Neurologic health Rationale The Moro, tonic neck, and withdrawal reflexes are usually present in infants under 3 to 4 months of age. Therefore the presence of these reflexes indicates neurologic health. The presence of the Moro, tonic neck, and withdrawal reflexes does not indicate severe brain damage. Decorticate posturing is indicative of severe dysfunction of the cerebral cortex and is not related to the presence of the Moro, tonic neck, or withdrawal reflexes. Decerebrate posturing is indicative of dysfunction at the level of the midbrain and is not related to the presence of the Moro, tonic neck, or withdrawal reflexes.

The nurse assesses the patient's breathing patterns and laboratory values. Which findings in the pediatric patient with a head injury indicate a possible brainstem herniation? Select all that apply. Elevated arterial blood gas values. Decreased arterial blood gas values. Normal arterial blood gas (ABG) values. Breathing pattern with periods of increased rate and depth followed by periods of decreased rate and depth. Breathing pattern with long periods of decreased rate and shallow depth followed by short periods of apnea

Normal arterial blood gas (ABG) values. A Cheyne-Stokes breathing pattern will be associated with normal arterial blood gas values which can help to differentiate this from other breathing patterns by patients experiencing respiratory distress. Breathing pattern with periods of increased rate and depth followed by periods of decreased rate and depth. Cheyne-Stokes is characterized by periods of increased rate and depth followed by periods of decreased rate and depth of breath, a pattern that will help differentiate from other breathing patterns and indicates imminent death as expected with a brainstem herniation.

The patient is diagnosed with Guillain-Barré syndrome. The nurse expects which findings in the cerebral spinal fluid (CSF) analysis? Normal glucose level Decreased fluid pressure Clear cerebral spinal fluid Elevated protein concentration Increased red blood cell level (RBC) Normal white blood cell level (WBC)

Normal glucose level Glucose levels are decreased in many bacterial infections as the bacteria "eat" the glucose found. GB is unique in that there may be normal glucose levels found in the CSF. Clear cerebral spinal fluid Cloudy CSF is expected in the case of bacterial meningitis. Correct Elevated protein concentration Protein is elevated due to inflammation and will be seen in the CSF of a patient with Guillain-Barré syndrome. Normal white blood cell level (WBC) White blood cell levels are not affected by Guillain-Barré syndrome. This differs from other infections where elevated white blood cell levels can be seen.

ICP monitoring

Normal value: 5-10 mmHg Recommend initiating treatment if ICP > 20 mmHG Indications for inserting ICP monitor GCS evaluation <8 GCS evaluation >8 with resp. assistance Deterioration of condition Subjective judgment based on clinical appearance Nursing interventions to dec. ICP Catheter into lateral ventricle- drains CSF, monitor levels and rate Positioning to reduce neck vein compression Elevate head of bed 15-30 degrees, head remains midline (do not turn head to side) OK for pillow, avoid flexing of head/neck, rolled pillows for allowance of normal finger position. gentle ROM, tennis shoes/ foot boards elevate heels Reduce activities that cause stress Cluster nursing activities Osmotherapy Sedation Minimize/eliminate environmental noise DO NOT SUCTION mannitol/ steroids meds used to decrease ICP

The nurse stresses to the family of a child with a chronic cardiac condition the importance of enrolling the child in school, pursuing hobbies and recreational interests, and attaining some independence. The nurse is facilitating what concept? Normalization Communication Decision making Family-centered care

Normalization Rationale Normalization refers to the efforts made to create a normal family life. For chronically ill children, such efforts include attending school, pursuing hobbies, and achieving a level of independence. These suggestions are not an example of communication, decision making, or family-centered care.

The parent of a child receiving an iron preparation tells the nurse that the child's stools are a tarry green color. What information does the nurse included in discussion with this parent? Symptom of iron-deficiency anemia Adverse effect of the iron preparation Indicator of an iron preparation overdose Normally expected change resulting from the iron preparation

Normally expected change resulting from the iron preparation Correct An adequate dosage of iron turns the stools a tarry green color. Descriptions of iron-deficiency anemia, iron preparation, and iron preparation overdose are not relevant. If the stools do not become tarry green, it may be indicative of administration issues

Which finding does the nurse look for when assessing a newborn for developmental dysplasia of the hip? Lordosis Ortolani sign Trendelenburg sign Telescoping of the affected limb

Ortolani sign Rationale The Ortolani sign is indicative of developmental dysplasia of the hip in a newborn. In an older infant lordosis and telescoping of the affected limb are signs of developmental dysplasia of the hip. In the weight-bearing child the Trendelenburg sign with lordosis is a clinical manifestation of developmental dysplasia of the hip.

A child in traction complains of tingling, intensifying pain in the extremity and requests additional pain medication. Which immediate action should the nurse take to care for this patient? Increase the pain medication as requested by the patient. Remove the patient from the traction device to relieve symptoms. Notify the health care provider that the patient has signs of compartment syndrome. Notify the health care provider that the patient has developed osteomyelitis.

Notify the health care provider that the patient has signs of compartment syndrome. The patient is displaying signs of compartment syndrome. The HCP should be notified right away and treatment initiated in order to prevent further injury and/or loss of limb.

The nurse is caring for a 9-year-old who fell over the handlebars of a bike yesterday. The abdomen is distended, and the child has blood oozing from the nose and ears. Which actions should the nurse take? Select all that apply. Notify the health care provider. Prepare to administer intravenous (IV) antibiotics. Prepare the child for transfer to the intensive care unit (ICU). Prepare to give an IV fluid bolus. Prepare to administer ibuprofen for pain.

Notify the health care provider. The health care provider should be notified immediately if the nurse notes abdominal distention and blood oozing from the ears and nose. Prepare the child for transfer to the intensive care unit (ICU). Abdominal distention and blood oozing from the ears and nose are signs of DIC. Children with DIC require close monitoring in an ICU.

Which findings are appropriate when taking the medical history of a child suffering from anemia? Select all that apply. Nutrition Eating habits Bowel habits Family history Family contact details History of chronic infection

Nutrition Eating habits Bowel habits Family history History of chronic infection Rationale Nutritional inquiry provides information about lactose intolerance or inadequate iron intake. Eating habits such as consumption of dirt, starch, or paper indicates possible nutritional deficiencies that may lead to anemia. Bowel habits like the presence of blood in the stools are indicators of chronic blood loss that can lead to anemia. Family history of sickle cell disease predisposes the child to anemia. Most chronic and recurrent infections may also lead to anemia. Contact details are essential for contacting the family, but do not yield any information of diagnostic value.

In evaluating a child with a chronic illness, which additional actions by the nurse assist in identifying the child's expected developmental process? Observe for the presence of developmental delays. Observe for age-appropriate responses by the ill child. Observe errors in the child's perception of the chronic illness. Monitor for miscommunication between family members regarding the child's illness. Evaluate the influence of the family's cultural beliefs on the child's cognitive milestone.

Observe for the presence of developmental delays. Evaluating for developmental delays provides information regarding the child's progress in growth and development. Correct Observe for age-appropriate responses by the ill child. Observing for age-appropriate responses by the child provides information regarding the child's progression in growth and development. Correct Observe errors in the child's perception of the chronic illness. Observing for errors in the child's perception of the illness identifies can be helpful for understand the child's progress in emotional growth and development. Correct Monitor for miscommunication between family members regarding the child's illness. Caring for the child will include knowing how well the family understands the illness. Any miscommunication or misunderstanding about the illness could have an impact on the child's growth and development.

Which nursing intervention is appropriate for neurologic assessment of children below 2 years of age? Collect family and health history to identify any genetic disorders. Evaluate the size and shape of the head to identify any neurologic dysfunction. Observe spontaneous and elicited reflex responses to evaluate neurologic status. Employ sophisticated techniques to assess acquisition of developmental milestones.

Observe spontaneous and elicited reflex responses to evaluate neurologic status. Rationale In infants and children below 2 years of age, most information is obtained by observing spontaneous and elicited reflex responses. Neurologic responses are primarily reflexive in nature in children under 2 years, and they cannot respond to directions designed to elicit specific neurologic responses. Family and health history is not the most appropriate intervention for neurologic assessment in this age group. Evaluating the size and shape of the head is not the most appropriate intervention. Sophisticated techniques should be employed to assess developmental milestones only after children develop increasingly complex gross and fine motor and communication skills.

A 10-year-old patient with aplastic anemia presents with the following vital signs: blood pressure of 118/70, heart rate of 112, respiratory rate of 28, temperature of 102.3° F. The nurse can anticipate which orders from the provider? Select all that apply. Obtain a blood culture. Collect a stool sample. Administer antibiotics. Administer an antipyretic. Obtain a complete blood count (CBC).

Obtain a blood culture. Patients with aplastic anemia are at increased risk for infection. A blood culture would be obtained to determine the source of infection. Administer antibiotics. Patients with aplastic anemia are at increased risk for infection. Prophylactic antibiotics should be administered to prevent or treat a potential infection. Correct Administer an antipyretic. A temperature of 102.3° F is a sign of infection. The provider would order an antipyretic to treat the fever. Correct Obtain a complete blood count (CBC). Patients with aplastic anemia have alterations in red blood cells, white blood cells, and platelets. A CBC should be obtained to determine the severity of the disease.

A paraplegic child is undergoing upper arm strengthening exercises and other techniques before wheelchair mobilization, and a tilt table is used to train the child in wheelchair sitting. The nurse understands this intervention is used to prevent which complication? Pressure injury Muscle weakness Poor trunk balance Orthostatic intolerance

Orthostatic intolerance Rationale A tilt table is used to manage orthostatic intolerance. The child may have orthostatic intolerance with wheelchair sitting, transferring from bed to wheelchair, and wheelchair mobilization. To prevent pressure injuries, the wheelchair is cushioned appropriately. Muscles are strengthened with upper arm strengthening exercises. A reclining backrest is provided if the child has poor trunk balance.

What is the most common osteoporosis syndrome in childhood? Kyphosis Osteogenesis imperfecta Legg-Calvé-Perthes disease Slipped capital femoral epiphysis

Osteogenesis imperfecta Rationale Osteogenesis imperfecta is the most common osteoporosis syndrome in children. Kyphosis (an abnormally increased lateral angulation in the convex curvature of the spine), Legg-Calvé-Perthes disease, and slipped capital femoral epiphysis are not the most common osteoporosis syndromes in childhood.

The nurse cares for a child with Guillain-Barré syndrome. The nurse notes a frequent, weak cough and decreased bilateral hand grips. What actions should the nurse take? Place a nasal cannula on the patient at 2 L/min. Obtain a pillow nurse call light for patient's use. Raise head of the bed to a semi-Fowler's position. Do not allow patient to have anything to eat or drink. Explain to patient what was assessed and the meaning.

Obtain a pillow nurse call light for patient's use. With inadequate grip strength, and Guillain-Barré, which is progressively decreasing strength from toes working toward neck, patient needs a way to call the nurse. A pillow call light allows patient to turn the head slightly to touch pad that calls the nurse. Correct Raise head of the bed to a semi-Fowler's position. The patient is having difficulty with oral and respiratory muscles. This will affect swallowing of saliva and diaphragmatic excursion. Placing the head of bed (HOB) up will decrease the patient's work and help prevent aspiration and hypoventilation. Correct Do not allow patient to have anything to eat or drink. Patient is at risk for aspiration and needs to be NPO until swallowing can be further evaluated. Correct Explain to patient what was assessed and the meaning. The patient should usually be told what is going on and what health care provider is doing. Patient's Guillain-Barré is progressing as evidenced by decreased strength in hands and inability to forcefully cough. Patient may soon require intubation and should be aware of that.

The nurse is caring for a child with hemophilia after the child fell from a bike. The child reports abdominal pain and complains of leg pain. After an initial assessment during which vital signs are noted to be within the normal range, the nurse notes bruising over the lower abdomen, abdominal distention, and hematuria. Which priority order from the provider would the nurse anticipate? Obtain complete blood count (CBC). Initiate intravenous (IV) access. Obtain an abdominal computed tomography (CT) scan. Administer pain medication.

Obtain an abdominal computed tomography (CT) scan. The priority at this time is to obtain an abdominal CT scan to determine the location and extent of injury.

On assessment of a patient with aplastic anemia, the nurse notes peripheral pulses 1+, cool extremities, and capillary refill of 4 seconds. Which actions should the nurse take? Select all that apply. Obtain vital signs. Administer an antibiotic. Administer an analgesic. Notify the health care provider. Complete a neurologic examination.

Obtain vital signs. Weak peripheral pulses, prolonged capillary refill, and cool skin indicate poor tissue perfusion. Vital signs should be obtained for more information on the patient's hemodynamic status. Notify the health care provider. Peripheral pulses, prolonged capillary refill, and cool skin indicate poor tissue perfusion. The nurse would notify the health care provider of abnormal assessment findings. Correct Complete a neurologic examination. Peripheral pulses, prolonged capillary refill, and cool skin indicate poor tissue perfusion. The nurse would perform a neurological exam to determine the effectiveness of cerebral perfusion

myositis ossifications

Occurs from deep contusion to the biceps and quadriceps muscles, resulting from restriction of the flexibility of the affected limb:

Greenstick fracture

Occurs when a bone is angulated beyond the limits of bending:

ICP

Older children = headaches, vomit and then they feel better. Setting sun sign! You may see that again. For an infant. What is one sign we only see in an infant for ICP? High pitched cry. Once you hear it you will never forget it. It is a horrible high pitched cry - intermittent.

symptoms of meningitis

One of the things you get with meningitis is nuchal rigidity. I didn't ask you about symptoms for meningitis. Anyone know what nuchal rigidity is? Can you put your head to your chest? If so, not meningitis. You can't bend your neck if you have true meningitis. Bacterial meningitis you tend to run a higher fever. Tend to be much sicker. Can't tell you how many courses of antibiotics I have had over the course of my life because I've held a kid down for a spinal tap that had bacterial meningitis. Because they tend to treat everybody that comes in contact with them.

An eight-year-old child is brought to the emergency department by his parents with signs of late hydrocephalus. The nurse manages what expected findings in this patient? Select all that apply. Setting-sun sign Ongoing seizure activity Restlessness and irritability Blood pressure 140/90 mm Hg Heart rate of 45 beats per minute

Ongoing seizure activity Seizures occur in a child with late hydrocephalus due to brain stress from excess fluid. The nurse administers benzodiazepines. Blood pressure 140/90 mm Hg An increased blood pressure will be exhibited in patient with late hydrocephalus. If parents desire more than comfort care, nurse may give medications to decrease blood pressure. Heart rate of 45 beats per minute A child with late hydrocephalus will exhibit decreased heart rate. If parents desire more than comfort care, nurse may give medications as ordered to increase heart rate.

Which is the first intervention in emergency management of a child who has been in an automobile accident? Perform cardiopulmonary resuscitation (CPR). Open the airway with a modified jaw thrust maneuver. Remove the child from the car seat and lay on a firm surface. Move the child immediately by automobile to the nearest trauma center.

Open the airway with a modified jaw thrust maneuver. Rationale Opening the airway with an appropriate method is the first step in emergency management. After assessing the child's level of consciousness, the nurse should treat the child as if spinal injury is present and open the airway with a modified jaw thrust maneuver. If the child is less than 8 years old, 2 minutes of CPR is recommended, and the nurse should activate the emergency medical services (EMS). Unless otherwise deemed necessary, the nurse should not remove the child from the car seat but should stabilize the spine. Transporting the child in an automobile is not recommended.

The nurse classifies the eight-month-old patient as having a severe injury based on what findings? Exhibits hypoxia, tachycardia, and hypotension Pushes a toy rather than picking it up to move it Unable to sit with support, does not bear weight on legs Opens eyes to pain, motor extension, and moans to pain

Opens eyes to pain, motor extension, and moans to pain This exhibits a higher score on the modified Glasgow coma scale that is used by the nurse to determine severity of head injury.

The Hematologic System and Its Function

Origin of formed elements Red blood cells (life span 120 days) White blood cells Platelets Hematopoietic organs Myeloid tissue (bone marrow) Lymphatic system (fluid and structures) Functions of cells Regulation of cell production spleen and liver for hematopoesis Red blood cells 120 days Regenerated Tissue hypoxia and eurtho in the kidney's Handsiteopetia

Which patient findings would have an associated effect on the white blood cell function? Select all that apply. Low iron Osteosarcoma Bacterial pneumonia Prolonged bleeding time Enlarged cervical lymph nodes

Osteosarcoma White blood cells are produced in the bone marrow. Conditions that affect the bone marrow can lead to impaired white blood cell production. Correct Bacterial pneumonia An infection such as bacterial pneumonia would cause an increase in production of WBC. Enlarged cervical lymph nodes White blood cells are produced in the lymphatic tissue. Enlarged cervical lymph nodes indicate an infection. A viral infection can decrease your WBC count and a bacterial infection can increase your WBC count.

Eyes Exam

PERRLA Eye movement test (doll's eyes, ice water caloric test) Ice water caloric test Irrigate auditory canal with ice water to see if eyes point to stimuli Painful and never performed if child is awake or tympanic membrane ruptured Pinpoint pupils commonly observed in poisoning or brainstem dysfunction Widely dilated and reactive pupils seen after seizures Widely dilated and fixed pupils Paralysis of CN III Unilateral fixed pupil Lesion on the same side Bilateral fixed pupil Brainstem damage if >5 mins Dilated and non-reactive pupils Hypothermia, anoxia, ischemia, poisoning, or prior instillation of mydriatic drugs Fixed and dilated pupil NEUROSURGICAL EMERGENCY

A nurse is caring for a patient who presented with a soft tissue injury. What should the nurse document as part of her assessment of neurovascular involvement? Select all that apply. Pain Pallor Pruritus Paresthesia Pulselessness

Pain Pain is evidence of neurovascular impairment. Correct Pallor Pallor is evidence of neurovascular impairment. Paresthesia Paresthesia is evidence of neurovascular impairment. Correct Pulselessness Pulselessness is evidence of neurovascular impairment.

Which physical assessment findings indicate a patient may have decreased hemoglobin? Select all that apply. Pallor Warm skin Weight gain Tachycardia Decreased level of consciousness

Pallor The nurse may observe pallor in a patient with decreased hemoglobin secondary to poor perfusion. Tachycardia Low hemoglobin levels result in decreased oxygenation. As a result, the heart rate will increase to compensate for the decrease in perfusion (oxygenation). The patient may have tachycardia. Decreased level of consciousness Low hemoglobin can indicate poor perfusion to body tissues, including the brain.

A nurse performs a neurovascular assessment on a leg in a cast. Which assessment findings warrant immediate interventions? Select all that apply. Paresthesia Normal mobility Decreased capillary refill Minor pain (relieved by analgesics) Difference between proximal and distal pulse

Paresthesia Paresthesia is a tingling or prickly sensation caused by pressure on or damage to peripheral nerves and is an indication for immediate intervention. Decreased capillary refill Decreased capillary refill means there was an increase in the time it took for the capillary to refill. This is an indication that blood flow is impaired and intervention is required. Difference between proximal and distal pulse A difference between the proximal and distal pulse indicates that there is impairment in blood flow which requires further investigation and intervention.

Which test is appropriate for the child with stiffness, tingling, and aches in the knee joints and has a tendency toward prolonged bleeding? Bleeding time Tourniquet test Clot retraction time Partial thromboplastin time

Partial thromboplastin time Rationale The symptoms hint that the child may be suffering from hemophilia. In hemophilia, factor VIII, an intrinsic clotting factor, is deficient. Partial thromboplastin time measures the activity of thromboplastin, which depends on intrinsic clotting factors. The partial thromboplastin time is increased in hemophilia. Bleeding time reflects platelet function, which is normal in hemophilia. The tourniquet test measures platelet function and capillary fragility, which are normal in hemophilia. The clot retraction test measures the degree to which a clot shrinks, and it is usually normal in hemophilia.

The nurse analyzes the laboratory results of a child with hemophilia. The nurse understands that which result will most likely be abnormal in this child? a. platelet count b. hematocrit level c. hemoglobin level d. partial thromboplastin time

Partial thromboplastin time; Hemophilia refers to a group of bleeding disorders resulting from a deficiency of specific coagulation proteins. Results of tests that measure platelet function are normal; results of tests that measure clotting factor function may be abnormal. Abnormal laboratory results in hemophilia indicate a prolonged partial thromboplastin time.

The nurse is caring for patients in the pediatric headache clinic. Which patients will the nurse assess urgently? Select all that apply. Patient reporting difficulty speaking clearly to the headache clinic receptionist. Patient diagnosed with tension headaches reporting a headache lasting three weeks. Patient diagnosed with migraine headaches reporting abdominal pain, nausea, and vomiting. Patient diagnosed with an arteriovenous malformation (AVM) reporting a brief vision loss this morning. Patient reporting pain in the neck muscles impairing movement of the head from side to side but no nuchal rigidity.

Patient reporting difficulty speaking clearly to the headache clinic receptionist. Difficulty speaking is an uncommon headache symptom and may indicate a stroke or seizure. This patient will need to be assessed urgently due to the severity of symptoms. Patient diagnosed with an arteriovenous malformation (AVM) reporting a brief vision loss this morning. An AVM causes vascular headaches and complications of the AVM include symptoms consistent with bleeding in the brain such as weakness, vision loss, difficulty speaking, and unsteadiness. This patient will need to be assessed urgently by the nurse.

Okay. What do we do for it? hip displasia

Pavlik Harness (newborn to 6 months) How long do they wear it? Do they wear it 24 hours a day? worn continuously until the hip is stable on both clinical and US exam. Usually 6-12 weeks. Surgical Rx and closed reduction recommended after 6 months to 24 months then Spica cast for 12 weeks Teach parents to never adjust the harness. They go in to see the doctor and the doctor adjusts the harness. Under the harness what are some things we teach parents for daily care? Put diaper under straps, no lotions or powder, massage under the straps a couple times daily just to increase circulation and check on that skin integrity, diaper under the straps so they don't rub and get dirty. After 6-24 months they do a closed surgical reduction after which the child is in a hip Spica cast. How long are they in the cast for? About 12 weeks and those casts are a booger if you've ever seen one. They literally have a cast around their waist and down both legs. There is an opening in the perineal area so you can keep em clean etc, but the child is literally in this body cast for 12 weeks. So, it's pretty uncomfortable for them.

An adolescent injured the knee at school and had a dislocation of the patella. Which is the priority intervention by the school nurse? Administer analgesics for pain. Advise surgical patellar reduction. Monitor for spontaneous reduction. Perform the patellar reduction manually.

Perform the patellar reduction manually. Rationale Patellar dislocation always happens laterally and is accompanied by acute pain and disability. The nurse should help bring the patella to its normal position by applying pressure. Analgesics are administered to provide relief from pain, but they do not repair the dislocation. For recurrent patellar dislocation, open surgery may be needed to repair the damage. Spontaneous reduction occurs in most cases. However, if the spontaneous dislocation does not occur immediately, the nurse should apply pressure to slide the patella back to its normal position.

A child who has been brought to the pediatrician's office complains of thigh and groin pain. What is the priority nursing action in response to the child's chief complaint? Assessing the child's reflexes Performing a complete hip examination Having the child perform tests of balance Documenting the chief complaint in the chart

Performing a complete hip examination Rationale A complete hip examination is the priority nursing action when a child presents with thigh and groin pain. Assessing the child's reflexes is not a priority nursing action when the child presents with thigh and groin pain. Having the child perform tests of balance is not a priority nursing action when the child presents with thigh and groin pain. Documenting the chief complaint in the chart is important but not a priority nursing action when the child presents with thigh and groin pain.

Which clinical symptoms are appropriate with spinal cord compression from atlantoaxial instability? Select all that apply. Loss of motor skills Persistent back pain Persistent neck pain Changes in sensation Loss of bladder control

Persistent neck pain Changes in sensation Loss of bladder control Loss of motor skills Rationale Children with Down syndrome who suffer atlantoaxial instability are at risk for spinal cord compression. Signs of spinal cord compression include persistent neck pain, changes in sensation, loss of motor skills, and loss of bladder or bowel control. Back pain is not a common symptom of spinal cord compression.

Which nursing information is appropriate for the parents of a 3-year-old child that has just undergone shunt revision for hydrocephaly about signs of shunt malformation? Select all that apply. Personality change Vomiting Bulging anterior fontanel Fever Dizziness

Personality change Vomiting Fever Rationale Personality change can be a sign of shunt malformation related to increased intracranial pressure (ICP). Vomiting can be a sign of shunt malformation related to increased ICP. Fever can be a sign of shunt malformation and is a very serious complication. The anterior fontanel closes between 12 and 18 months of age. Dizziness is difficult to assess in a 3-year-old and is not necessarily a sign of shunt malformation.

Which changes in pupils are appropriate to observe in a child with barbiturate poisoning who is brought to the emergency department? Pinpoint pupils A unilateral fixed pupil Widely dilated and fixed pupils Widely dilated and reactive pupils

Pinpoint pupils Rationale Pinpoint pupils are commonly observed in barbiturate poisoning. A unilateral fixed pupil is observed if there is a lesion on the same side. Widely dilated and fixed pupils are observed if there is a paralysis of oculomotor nerve secondary to the pressure from herniation of the brain through the tentorium. Widely dilated and reactive pupils are usually observed after seizures and may involve only one side.

A nurse is caring for an infant with developmental dysplasia of the hip (DDH). The nurse expects the infant to demonstrate which clinical manifestations? Select all that apply. Lordosis Lack of Babinski sign Presence of Ortolani and Barlow signs Asymmetries of thigh and gluteal folds Lengthening of the limb on the affected side

Presence of Ortolani and Barlow signs Asymmetries of thigh and gluteal folds Rationale Asymmetric thigh and gluteal folds are clinical manifestations of developmental dysplasia of the hip (DDH) and are seen from birth to 2 months. Positive findings on the Ortolani and Barlow tests are clinical manifestations of DDH. The Ortolani test is the abduction of the thighs to test for hip subluxation or dislocation. The Barlow test is adduction to feel whether the femoral head slips out of the socket posterolaterally. Shortening, not lengthening, of the limb on the affected side is another clinical manifestation of DDH. Lordosis is the inward curve of the lumbar spine just above the buttocks and is not a clinical manifestation of DDH. Lack of the Babinski sign is not a clinical manifestation of DDH; the Babinski sign is a neurologic reflex.

The hospitalized child with spina bifida has broken out in a rash. What actions should the nurse take? Place a precautions sign on the door and in the room. Change out the gloves in the room and outside the door. Request that the health care provider prescribe a steroid. Check the patient's vital signs for a temperature elevation. Ask the hospital's rapid response team to evaluate the child.

Place a precautions sign on the door and in the room. There is likely a latex allergy. A sign indicating the patient is allergic to latex is needed on the door and above the bed to alert other health care workers. Additionally, at a minimum, it should be listed on the patient's armband. The next exposure could induce anaphylaxis. Correct Change out the gloves in the room and outside the door. Children with spina bifida are at high risk for developing latex allergies due to chronic illness resulting in increased exposure to latex-containing products over time. If latex-containing gloves are in the room, they must be removed until it can be determined definitely whether this rash is related to latex. Correct Request that the health care provider prescribe a steroid. A steroid and/or antihistamine will calm the immune reaction to the latex. Correct Check the patient's vital signs for a temperature elevation. This rash could be related to a virus instead of latex. The nurse should begin to rule that out by assessing the patient for a fever.

A child is diagnosed with early stage hydrocephalus. What actions should the nurse perform? Place padding on all four of bed rails. Teach parents to expect high-pitched crying. Administer ondansetron (Zofran) for vomiting. Provide orientation to the room, call light, and personnel. Consult dietician for dietary supplement recommendations.

Place padding on all four of bed rails. Although seizures are a late sign, any time intracranial pressure (ICP) is increased enough to be symptomatic, seizure precautions should be instituted to protect patient. Administer ondansetron (Zofran) for vomiting. Vomiting occurs due to pressure within the brain on structures that control vomiting, however the action of vomiting increases ICP. The nurse should make every attempt to reduce actions that increase ICP. Correct Provide orientation to the room, call light, and personnel. The child may exhibit confusion, and providing orientation may reduce the severity of the confusion. Correct Consult dietician for dietary supplement recommendations. An infant with early stage hydrocephalus will present with poor feeding, and therefore the nurse may notice the child is not gaining weight.

Diagnostic eval ITP

Platelet count less than 20,200

The nurse is caring for a child who is immobilized following a motor vehicle accident and multiple fractures. As a result, the child has decreased chest expansion and reduced tidal volume. What is the appropriate intervention to aid respiration in this child? Position the child in supine position. Position the child in standing position. Position the child in semi-Fowler position. Position the child in Trendelenberg position.

Position the child in semi-Fowler position. Rationale Prolonged immobilization can increase the risk of respiratory complications such as decreased chest expansion and reduced tidal volume. The child should be positioned well to promote adequate chest expansion. Semi-Fowler position allows lung expansion and aids in respiration. Standing allows for better respiratory status and chest expansion. In this situation, it would be difficult for the child to stand, because the child is immobilized for a long time and may not be able to bear weight. The supine and Trendelenberg positions do not allow maximum lung expansion.

Which complication is appropriate for the 8-year-old boy who has sustained a basilar skull fracture and is becoming increasingly drowsy and now has a fever? Cerebral edema Epidural hemorrhage Subdural hemorrhage Posttraumatic meningitis

Posttraumatic meningitis Rationale Posttraumatic meningitis should be suspected in children with increasing drowsiness and fever who also have basilar skull fractures. Cerebral edema is not associated with drowsiness or fever. Epidural hemorrhage is bleeding between the dura and the skull that results in the formation of a hematoma; it is not associated with increasing drowsiness and a fever. Subdural hemorrhage is not associated with the development of drowsiness or fever.

mainstreaming

Practice of placing children with special needs in regular classroom settings, with the support of professionals who provide special education services paralleling normalization and home care

Which drug is appropriate to administer for the effective management of muscular dystrophy? Baclofen Prednisone Vecuronium Divalproex/valproic acid

Prednisone Rationale Muscular dystrophies are chronic genetic disorders most common in children. To date, no specific medication has been established to treat the disorders. Prednisone is the suggested corticosteroid medication for managing the ill effects of muscular dystrophy. Several clinical trials have proven that the use of corticosteroids improves muscle strength and performance in children with muscular dystrophy. Baclofen is used to treat muscular spasms. Vecuronium is a neuromuscular blocking agent used to treat seizures caused by tetanus infection. Valproic acid/divalproex is used to treat epilepsy-related disorders of cerebral palsy.

Which nursing action is appropriate when caring for a young child with a head injury whose pupils have suddenly become fixed and dilated? Reorientation of the patient Glasgow Coma Scale assessment Institution of seizure precautions Preparations for a neurologic emergency

Preparations for a neurologic emergency Rationale A neurologic emergency may be occurring in a child who exhibits suddenly fixed and dilated pupils. During a neurologic emergency the child may not be able to be reoriented; the priority is preparing for the possible consequences of a neurologic emergency. When the pupils suddenly become fixed and dilated, the priority nursing action is not a Glasgow Coma Scale assessment or institution of seizure precautions, but instead, preparations for dealing with a neurologic emergency.

Which finding is appropriate when assessing for nuchal rigidity? Select all that apply. Positive Babinski reflex Presence of Kernig sign Absence of Brudzinski sign Inability of the neck to support the head upright Inability to place the chin on chest

Presence of Kernig sign Inability to place the chin on chest Rationale Presence of the Kernig sign, that is, inability to extend the knee beyond 135 degrees with hips and knees flexed at 90 degrees, and pain in hamstrings are suggestive of nuchal rigidity. Inability to place the chin on the chest is also indicative of nuchal rigidity. A positive Babinski sign is indicative of pyramidal tract lesion, not nuchal rigidity. A positive Brudzinski sign is indicative of nuchal rigidity and involves flexion of the lower extremities on passive flexion of the neck. Inability of the neck to support the head is not indicative of nuchal rigidity.

A nurse assesses the growth of a 1-month-old infant and prepares to measure the head circumference and assess the fontanels. The nurse knows that which aspect(s) of this infant's head anatomy should be an expected finding? Closed sutures Closed foramina Presence of a posterior fontanel Presence of an anterior fontanel Anterior fossa containing the pituitary gland

Presence of a posterior fontanel The posterior fontanel should be present in a 1-month-old infant because it normally closes at 2-months of age. Correct Presence of an anterior fontanel The anterior fontanel should be present in a 1-month-old infant. This fontanel does not close until 18 months of age.

Which laboratory finding indicates hemolysis? Decreased reticulocyte count Decreased red blood cell count Fall in the serum bilirubin level Presence of nucleated red blood cells

Presence of nucleated red blood cells Rationale Hemolysis is a condition that requires a rapid increase in red blood cell production. Consequently, circulating erythrocytes may not be fully matured, and nucleated red blood cells may appear in circulation. Increased red blood cell destruction results in increased production of bilirubin. With increased demand for red blood cells, bone marrow is stimulated to produce an increased number of reticulocytes as well.

nursing Interventions

Prevent Bleeding (decreasing risk of injury - developing appropriate exercises, use protective equipemnt, avoid contact sport) Close supervision and safe environment Dental procedures in controlled situation Shave only with electric razor Superficial bleeding—apply pressure for at least 15 minutes and ice to vasoconstrict If significant bleeding occurs, transfuse for factor replacement soft toothbrush medical identification special precautions when doing IM, SQ substituted for IM when possible venipuncture for blood collection, because bleeds less than heel or finger NO ASPIRIN during bleeding episodes - joint is elevated and immobilized, pain analgesics

What results when ice is applied immediately after a soft tissue injury, such as a sprained ankle? Increases the pain threshold. Increases metabolism in the tissues. Produces deep tissue vasodilation. Leads to release of more histamine-like substances

Produces deep tissue vasodilation. Correct Nine to fifteen minutes of ice exposure produces deep tissue vasodilation without increased metabolism. Ice has a rapid cooling effect on tissues that reduces pain. The decreased temperature slows metabolism, thus reducing tissue oxygen requirements. Fewer histamine-like substances are released.

Which primary goal is appropriate when caring for the child with cognitive impairment? <p>Which <b>primary </b>goal is appropriate when caring for the child with cognitive impairment?</p> Encouraging play Developing vocational skills Promoting optimal development Helping families develop a care plan and having them stay with it

Promoting optimal development Rationale A comprehensive approach is desirable in establishing acceptable social behavior and feelings of self-worth and promoting optimal development. Providing parents with guidance for the selection of developmentally appropriate activities is only one component in a comprehensive care plan. Care for the cognitively impaired child is an ongoing process that changes as the child meets developmental milestones. The acquisition of vocational skills will be addressed as the child's capabilities develop and is one component of the comprehensive care plan.

Which areas are appropriate for the parents of a child with cerebral palsy that needs multidisciplinary care and coordination? Select all that apply. Proper handling and adapting to the home environment Medication administration and the changing role of parents and siblings Techniques for immobilizing the child to preserve muscle tone and prevent joint contractures Special medicine administration to prevent the child from gagging during meals Devising and modifying equipment and activities as per the child's ability

Proper handling and adapting to the home environment Medication administration and the changing role of parents and siblings Devising and modifying equipment and activities as per the child's ability Rationale A parents of a child with cerebral palsy need to know about proper handling of the child and how to adapt the home environment as per the child's mobility requirements. This will allow the child to move freely and become self-dependent. The medication administration process is a major concern for parents. The child with cerebral palsy will need help and will need routine administration of various medicines. The parents need to have proper knowledge about this. The parents and siblings of the child should also be informed about their changing roles as caregivers. In addition, the parents need to know how to devise and modify equipment and activities as per the child's requirements. This will allow the child to become self-dependent. It is not necessary for the parents to know how to immobilize the child. Most children with cerebral palsy do not need special medicine in order to avoid gagging during meals. However, special techniques are used to prevent problems during eating. These include providing jaw support and jaw control during feeding. When feeding, the child is kept in a semi-upright position.

Which medication is appropriate to add to the treatment plan for the child with status epilepticus who is not responding to lorazepam and fosphenytoin? Select all that apply. Propofol Ketamine Morphine Midazolam Phenobarbital

Propofol Midazolam Phenobarbital Rationale A child with status epilepticus who does not respond to lorazepam, fosphenytoin, or phenobarbital requires general anesthesia with a continuous infusion of propofol, midazolam, or phenobarbital. Although it is a general anesthetic, ketamine is not protective in the case of seizures. Morphine is an opioid drug and is not protective for seizures.

Which nursing intervention is appropriate for the child with spina bifida that has developed a latex allergy as a result of numerous bladder catheterizations and surgeries? Recommend allergy testing Provide a latex-free environment Use only powder-free latex gloves Limit the use of latex products as much as possible

Provide a latex-free environment Rationale The most important nursing intervention is to provide a latex-free environment. From birth on, limitation of exposure to latex is essential in an attempt to minimize sensitization. Latex-free catheters for self-catheterization are available. Allergy testing may expose the child to the allergen and therefore is not recommended. The gloves contain latex and will contribute to sensitization. No latex products should be used with children who have latex allergies. Latex products should be avoided at all times.

Which type of traction should the nurse anticipate for a patient with hemarthrosis? Halo traction Skin traction Femoral traction Skull tongs traction

Skin traction Skin traction is preferred for conditions in which invasive procedures are contraindicated, such as hemarthrosis (collection of blood in the joint) as a result of hemophilia.

Which nursing recommendations are appropriate when teaching the parents of a preterm infant ways to prevent possible nutritional deficiency anemia? Select all that apply. Provide breast milk as much as possible. Give citrus fruit or juice with iron supplements. Introduce fresh cow's milk by the age of 6 months. Administer iron supplements in two divided doses between meals. Start iron supplementation by the age of 2 months at a rate of 1 mg/kg/day. Limit quantities of milk and introduce solid food rich in iron, fluoride, zinc, and vitamin C.

Provide breast milk as much as possible. Give citrus fruit or juice with iron supplements. Administer iron supplements in two divided doses between meals. Limit quantities of milk and introduce solid food rich in iron, fluoride, zinc, and vitamin C. Rationale Breast milk is the best source of nutrition for the child and should be used for the first 12 months. It is better to accompany iron supplements with citrus fruits and juices. They reduce the iron to its most soluble form, which is readily absorbed in the gastrointestinal tract. Iron should be administered in two or three divided doses between meals, when the presence of free hydrochloric acid in the stomach is greatest. It helps in iron absorption from the stomach. Milk is deficient in iron, fluoride, zinc, and vitamin C. These nutrients should be given with solid food. Fresh cow's milk should be introduced after the age of 12 months, because intolerance to milk proteins may lead to gastrointestinal blood loss. Iron supplementation should be started at the age of 2 months at a rate of 2 mg/kg/day for preterm babies.

The parents of an adolescent with a newly diagnosed chronic illness state, "She was a little disrespectful and defiant before, but now things have gotten completely out of control. We feel like we really need some help coping with this." What can the nurse do to enhance coping in these parents? Instruct the parents on ways to maintain age-appropriate expectations and limitations on negative behaviors Provide education regarding support groups for families of children with special health care needs as a way to get emotional needs met Teach the parents ways to enhance the coping of the adolescent so that she is able to find more productive ways to work through her frustrations Have the parents ignore the behavior because the change in behavior is a normal adolescent reaction to the frustration of dealing with a chronic illness

Provide education regarding support groups for families of children with special health care needs as a way to get emotional needs met Parents also need an outlet for frustration, and support groups that include other parents going through similar situations may be the best solution.

A patient with an external fixation device placed in the tibia is being discharged. The parents describe their anxiety about caring for the child at home to the nurse. Which information should the nurse provide to increase their confidence? Select all that apply. Provide information regarding the removal of the fixator pins. Provide information and request return demonstration on pin entry site care. Provide information regarding when to contact the health care provider. Provide information regarding community resources (PT, OT, school personnel). Provide information and request return demonstration on how to perform the neurovascular assessment and what the findings indicate.

Provide information and request return demonstration on pin entry site care. Pin entry site care, including cleansing and observation for possible infection, should be provided. Provide information regarding when to contact the health care provider. Information regarding specific signs that require health care provider attention need to be communicated effectively. Provide information regarding community resources (PT, OT, school personnel). Provide additional resources to help the family cope with the current lifestyle change they are experiencing and to provide the best treatment options for the child. Provide information and request return demonstration on how to perform the neurovascular assessment and what the findings indicate. The neurovascular assessment should be performed frequently. The parents should be comfortable with the assessment process and understand the implications of the findings.

Parents of a 3-year-old patient are expressing fear over treatments used for their child's illness. What action by the nurse should be most effective for reducing fear in the parents? Provide information on support groups related to the illness Initiate a referral to a psychotherapist for grief counseling services Assess the parent's spiritual beliefs and practices related to the illness Provide information related to the illness and current treatment modalities

Provide information related to the illness and current treatment modalities Education about the child's condition and treatment often reduces fear.

The nurse on a home visit finds that a person has sustained a severe sprain. Which nursing interventions are most appropriate? Provide limb movements, apply ice, and avoid splinting. Provide warmth, splint, and keep the affected limb elevated. Provide rest, apply ice, and provide compression. Provide rest and warmth, and apply compression.

Provide rest, apply ice, and provide compression. Rationale First aid in cases of sprains or suspected fractures involves interventions that can be remembered with the acronym RICE. R stands for rest; I stands for ice; C stands for compression; E stands for elevation. Therefore the nurse suggests that caregivers provide rest, apply ice, and provide compression to the affected limb. Providing limb movements would worsen the sprain and may lead to complete rupture of the ligament. There should be no warm application for sprains, because it can worsen the edema associated with sprains.

A child with clubfoot experiences discomfort every time TAL (tendoachillis lengthening) is required and care is implemented. Which actions can the nurse take to help the child feel more comfortable? Select all that apply. Provide soft music Dim the lights and reduced loud sound Administer IV pain medication as ordered Pre-medicate with local anesthetic if required Have the parents hold the child during treatment

Provide soft music Providing soft music can help to reduce the discomfort experienced by the child. Dim the lights and reduced loud sound This action will help to reduce the child's discomfort and make the atmosphere calm. Pre-medicate with local anesthetic if required TAL (tendoachillis lengthening) can be painful and uncomfortable. Local anesthetic can be applied do reduce the pain. Have the parents hold the child during treatment Having the parents involved in the procedure will help to relieve the discomfort experienced by the child.

A child has a skeletal growth disorder. Which disorder occurs when the femoral neck is displaced from the femoral head? Scoliosis Osteomyelitis Legg-Calvé-Perthes disease Slipped Capital Femoral Epiphysis (SCFE)

Slipped Capital Femoral Epiphysis (SCFE) Increased body weight and height place more stress on the epiphyses, causing a relative displacement (slip) of the femoral neck from the femoral head.

Parents of a child with asthma are feeling overwhelmed and worried about how to access care to meet their child's health care needs. What should the nurse do to help the parents in this situation? Select all that apply. Provide the parent with the clinic hours of operation. Provide the parent with information about after-hour clinic provider coverage. Gently remind the parents that excessive worry can have a negative effect on the child. Utilize nursing resources to provide information on asthma and when to seek medical attention. Provide the parent with the address and telephone number of the closest emergency department.

Provide the parent with the clinic hours of operation. Providing the parents with clinic hours of operation will decrease problems accessing the health care system and should decrease frustration. Provide the parent with information about after-hour clinic provider coverage. Providing the parents with after-hour clinic and provider coverage decreases problems accessing the health care system and should decrease frustration. Utilize nursing resources to provide information on asthma and when to seek medical attention. Utilizing nursing resources to provide information on asthma for the parents may decrease their frustration as they will know when medical care is necessary. Provide the parent with the address and telephone number of the closest emergency department. Providing information regarding emergency access to health care services assists in eliminating a barrier to health care service

A child underwent a below-the-knee amputation. Although keeping the stump elevated decreases swelling, the nurse keeps the stump elevated for a limited period of time only for the first 24 hours. Which complication is the nurse trying to prevent? Hemorrhage Pressure sore Phantom limb pain Proximal joint contractures

Proximal joint contractures Rationale The stump should not be kept elevated for more than 24 hours. Otherwise, the child may develop proximal joint contractures, which can seriously hamper ambulation. The nurse should closely monitor for hemorrhage. A figure-eight compression bandage is used to control hemorrhage. Positioning changes from side to side and from front to back help in preventing pressure sores. Phantom limb pain is an expected experience. Positioning has no effect on phantom limb pain.

While assisting with a lumbar puncture, the nurse places the highest priority on monitoring which physiologic parameters? Select all that apply. Pulse rate Temperature Pulse oximetry Blood pressure Respiratory rate

Pulse rate The pulse rate of the child needs to be monitored by the nurse during a lumbar puncture. A rapidly increasing or decreasing heart rate indicates a problem that needs to be addressed immediately. Pulse oximetry Part of continually assessing the patient's cardiorespiratory status is ensuring that the respiratory rate is providing adequate oxygenation while the child's torso is in a position that restricts expansion of the thorax. Respiratory rate The positioning required during a lumbar puncture constricts normal lung movement, air intake, and the ability of the nurse to visualize that the patient is breathing adequately so monitoring respiratory rate is a priority.

The nurse is caring for a child with immune thrombocytopenic purpura (ITP) who has intravenous (IV) fluids running at 25 mL/hr. The insertion site shows no sign of bleeding. The provider orders intramuscular (IM) ceftriaxone to treat a suspected bacterial infection. Which action is most important for the nurse to take? Administer the IM ceftriaxone. Assess for signs of abdominal bleeding. Stop the IV fluids while the medication is administered. Question the order for IM injection and have it changed to IV injection.

Question the order for IM injection and have it changed to IV injection. When caring for a child with ITP, the nurse should avoid invasive procedures. The nurse should question the order for IM injection and have it changed to IV or oral administration to prevent bleeding.

therapeutic managemtn/ sprain strain

R rest I ice ( cloth under to protect skin) no longer than 30 min C compression E elevation - several inches above heart I ice Ccompression E elevation S support 6-12 crusial

Function of hematological components

RBC carry oxygen you breathe from your lungs to rest of body WBC keep germs from causing infection Platelets small parts of cells that help make bleeding stop by forming a clot over the injured area Plasma liquid portion of blood, which has clotting factors that help make bleeding stop "Shift to the left" indicates infection

What is a nursing intervention to prevent that pressure in the brain going higher?

Raise HOB. How high? No more than 30 degrees. About 15-30 degrees no higher than that. Low stimulation. Try and cluster your care. You don't want to go in and out of the room. Door shut, dim lights, cluster care, whatever you can do to decrease that stimulation. Anyone with ICP- meningitis you should do same.

An adolescent has heatstroke while vigorously exercising outside in the sun. Which is the priority intervention for this patient? Supplemental oxygen administration Cautious fluid and electrolyte replacement Antipyretic administration on an immediate basis Rapid cooling until core temperature reaches 102 oF

Rapid cooling until core temperature reaches 102 oF Rationale The adolescent's clothing should be removed and cool water should be applied. Rapid cooling is necessary until the core temperature reaches 102 oF. Careful monitoring of the temperature is the first priority. Supplemental oxygen administration and cautious fluid and electrolyte replacement are supportive care needed at the time of heatstroke management. Antipyretics should be avoided because antipyretics are metabolized by the liver, which is already not functioning well due to the heatstroke.

The nurse is doing a neurologic assessment on a child whose level of consciousness has been variable since sustaining a cervical neck injury 12 hr ago. What is the most essential in this assessment? Reactivity of pupils Doll's head maneuver Oculovestibular response Funduscopic examination to identify papilledema

Reactivity of pupils Correct Pupil reactivity is an important indication of neurologic health. The pupils should be assessed for no reaction, unilateral reaction, and rate of reactivity. The doll's head maneuver should not be performed if there is a cervical spine injury. The oculovestibular response is a painful test that should not be done on a child who is having variable levels of consciousness. Papilledema does not develop until 24 to 48 hr into the course of unconsciousness.

An overweight adolescent complains of knee pain and limps while walking. Radiographs have been obtained to confirm the condition. The nurse would anticipate educating the patient on which condition? A. Scoliosis B. Osteomyelitis C. Legg-Calvé-Perthes disease D. Slipped Capital Femoral Epiphysis (SCFE)

Slipped Capital Femoral Epiphysis (SCFE) The classic symptoms of SCFE include a limp and pain. Radiographs confirm the diagnosis.

In which ways can a nurse help to relieve anxiety in an adolescent having surgery for a spine abnormality? Select all that apply. Reassure the adolescent to allay fears. Allow the adolescent to have quite time for reflection. Discuss activity limitations and provide ideas for alternatives. Correct any misunderstandings the adolescent has about the surgery. Determine the adolescent's need for specific information, particularly about postoperative care.

Reassure the adolescent to allay fears. Patient's anxiety is reduced when fears are addressed by the nurse. Discuss activity limitations and provide ideas for alternatives. The patient will have limited mobility and therefore activity limitations after surgery. Discussing alternatives for activities requiring mobility can put the patient at ease and provide a sense of control. Correct any misunderstandings the adolescent has about the surgery. When able to ask questions, it helps the adolescent to think through the process and deal with the anxiety. Ensuring the patient has factual information is key to decreasing the patient's stress. Determine the adolescent's need for specific information, particularly about postoperative care. Being able to determine the adolescent's understanding of what is taking place will give the ability to assist in decreasing anxiety. When interventions are explained, the adolescent's understanding increases.

Which major goal of therapy is appropriate for children with cerebral palsy (CP)? Curing the underlying defect causing the disorder Reversing degenerative processes that have occurred Preventing spread to individuals in close contact with the children Recognizing the disorder early and promoting optimal development

Recognizing the disorder early and promoting optimal development Rationale Because cerebral palsy (CP) is currently a permanent disorder, the goal of therapy is the promotion of optimal development. This is done through early recognition and commencement of therapy. It is difficult to reverse the degenerative processes associated with CP. The underlying defect(s) associated with the development of CP cannot be cured. CP is not contagious.

A parent confides in the nurse and states, "I believe it's my fault that my child is sick." The parent is crying and also admits to not sleeping well at night. Which actions by the nurse are most appropriate? Select all that apply. Discuss the concept of "survivor guilt" with the parent. Educate the parent on over-the-counter sleep medicine. Recommend a referral to a psychologist specializing in grief therapy. Talk with the child's health care provider regarding the parent's statements. Provide the parent with the date and time of the next grief support meeting in the area.

Recommend a referral to a psychologist specializing in grief therapy. A referral to a psychologist is appropriate based on the statements of the parent, the crying, and lack of sleep expressed by the parent. Talk with the child's health care provider regarding the parent's statements. Talking with the health care provider will alert the provider to check on the parent's coping with the child's illness and provides the opportunity for addressing the parent's concerns through a variety of treatment modalities. Provide the parent with the date and time of the next grief support meeting in the area. Grief support groups provide a way for the parent to express grief. Expressing emotions is beneficial for most individuals.

A young boy has just been diagnosed with pseudohypertrophic (Duchenne) muscular dystrophy. What is included in his plan of care for his family? Recommend genetic counseling. Explain that the disease is easily treated. Suggest ways to limit the use of muscles. Assist the family in finding a nursing facility to provide his care.

Recommend genetic counseling. Correct Pseudohypertrophic (Duchenne) muscular dystrophy is inherited as an X-linked recessive gene. Genetic counseling is recommended for parents, female siblings, maternal aunts, and their female offspring. No effective treatment exists at this time for childhood muscular dystrophy. Maintaining optimum function of all muscles for as long as possible is the primary goal. It has been found that children who remain as active as possible are able to avoid the need for a wheelchair for a longer time. Finding a nursing facility is inappropriate at the time of diagnosis. When the child becomes increasingly incapacitated, the family may consider home-based care, a skilled nursing facility, or respite care to provide the necessary care.

A mother treated her 6-year-old child's fever with aspirin. The child was brought to the emergency department and diagnosed with a varicella infection. The nurse assesses that the patient has impaired consciousness. What are the next steps in nursing management? Record intake and output every 1-2 hours. Administer a broad-spectrum antibiotic as prescribed. Continue to treat the fever with aspirin every 4-6 hours. Monitor ammonia levels, liver enzymes, and coagulation studies. Monitor the child's oxygen saturation and heart rate continuously.

Record intake and output every 1-2 hours. Maintenance of fluid and electrolyte balance is essential in this child with increased intracranial pressure (ICP) cause by Reye's syndrome. Monitor ammonia levels, liver enzymes, and coagulation studies. Reye syndrome can cause liver dysfunction, affecting ammonia levels, liver function, and coagulation abilities. The nurse monitors laboratory studies for intervention. Correct Monitor the child's oxygen saturation and heart rate continuously. Because of the child's impaired consciousness, the cardiorespiratory status needs to be monitored due to the risk of rapid deterioration.

The nurse is caring for a patient with Von Willebrand disease. The patient mentions his catch in the last football game. Which action is most important for the nurse to take? Select all that apply. Perform thorough neurological exam. Administer prophylactic DDAVP intravenously as ordered. Refer the child to the hospital social worker for follow-up. Provide the family and child with information to cope with the diagnosis. Discuss the importance of avoiding contact sports with the child and family.

Refer the child to the hospital social worker for follow-up. A social work referral may be needed for parents or a child who refuse(s) to avoid contact sports. Provide the family and child with information to cope with the diagnosis. A child who continues to play contact sports after receiving patient teaching about avoiding them may need further resources to cope with the diagnosis. Discuss the importance of avoiding contact sports with the child and family. Patients should be encouraged to avoid contact sports, such as football, which can cause injuries and lead to excessive bleeding.

An adolescent has an abnormal scoliosis screening at school. On investigation, the adolescent has a scoliosis curve of 18 degrees. What is the most appropriate therapeutic management in this case? A brace is prescribed. Chiropractic treatment is suggested. Surgical intervention for realignment is advised. Regular clinical and radiographic evaluation is advised.

Regular clinical and radiographic evaluation is advised. Rationale The adolescent with a scoliosis curvature of 18 degrees does not need any intervention unless it is progressive. School scoliosis screenings are often controversial and often report false positives. Monitoring with clinical and radiographic evaluation is needed to determine if the curvature is progressive. A brace is not needed with a curvature of less than 25 degrees. Chiropractic treatment is not helpful in treating scoliosis curvature. Surgical intervention for realignment is needed for a progressive and severe scoliosis, usually 45 degrees or more.

Which similarity is appropriate for both the diagnostic procedures of the subdural tap and the ventricular puncture? Help to rule out subdural effusions Carry a risk of intracerebral hemorrhage Remove cerebrospinal fluid to relieve pressure Involve inserting a needle into the anterior fontanel

Remove cerebrospinal fluid to relieve pressure Rationale Both procedures, subdural tap and ventricular puncture, remove cerebrospinal fluid to relieve spinal fluid pressure. Subdural tap helps to rule out subdural effusions. Ventricular puncture carries a risk of intracerebral hemorrhage. While subdural tap involves inserting a needle into the anterior fontanel, ventricular puncture involves inserting a needle into the lateral ventricle.

Which treatment is appropriate for hemophilia? Exercise Corticosteroids Pain management Replacement of missing clotting factor

Replacement of missing clotting factor Rationale Replacement of the missing clotting factor is the primary therapy for hemophilia. Exercise is important but not the primary therapy for hemophilia. Corticosteroids are helpful for hematuria and chronic synovitis but are not the primary therapy for hemophilia. Treatment of pain is important but not the primary therapy for hemophilia.

A child in halo traction reports severe and unrelenting pain, is very pale, and reports a tingling feeling. Which should be the nurse's response to these findings? A. Obtain an order for IV fluids for the patient. B. Document complaints and reassess the patient in an hour. C. Encourage the patient to use guided imagery to decrease pain. D. Report the findings to the health care provider immediately.

Report the findings to the health care provider immediately. Prompt referral to a health care provider and intervention is crucial if neurovascular impairment is to be prevented.

Which concept regarding pain is appropriate for a comatose child with multiple injuries? Cannot occur if the child is comatose May occur if the child regains consciousness Requires astute nursing assessment and management Is best assessed by family members who are familiar with the child

Requires astute nursing assessment and management Rationale Because the child cannot communicate pain through one of the standard pain rating scales, the nurse must focus on physiologic and behavioral manifestations to accurately assess pain. Pain can occur in the comatose child. The child can be in pain while comatose. The family can provide insight into the child's different responses, but the nurse should be monitoring physiologic and behavioral manifestations.

The nurse is caring for a child with multiple injuries who is comatose. What information is accurate related to pain in this child? Cannot occur if the child is comatose. May occur if the child regains consciousness. Requires astute nursing assessment and management. Is best assessed by family members who are familiar with the child.

Requires astute nursing assessment and management. Correct Because the child cannot communicate pain through one of the standard pain rating scales, the nurse must focus on physiologic and behavioral manifestations. The child can be in pain while comatose. The family can provide insight into different responses, but the nurse should monitor physiologic and behavioral manifestations.

What is the most comprehensive way to treat an overuse syndrome? Activity and acetaminophen Rest and nonsteroidal antiinflammatory drugs Activity and nonsteroidal antiinflammatory drugs Rest, physical therapy, and nonsteroidal antiinflammatory drugs (NSAIDs)

Rest, physical therapy, and nonsteroidal antiinflammatory drugs (NSAIDs) Rationale Rest, physical therapy, and NSAIDs are the recommended way to treat an overuse syndrome. Rest and NSAIDs are appropriate, but not comprehensive. Activity and acetaminophen are not recommended for the therapeutic management of overuse syndrome.

Which nursing interpretation is appropriate for the 2-week-old infant with Down syndrome whose mother comments her infant is difficult to hold and does not cuddle up like her other babies did? Sign of maternal deprivation Sign of detachment and rejection Sign of autism associated with Down syndrome Result of the physical characteristics of Down syndrome

Result of the physical characteristics of Down syndrome Rationale Lack of clinging (or molding) between child and mother is a result of the muscle hypotonicity and hyperextensibility of the joints associated with Down syndrome. Mothers may have difficulty with attachment to their child as a result of the lack of clinging or molding behavior that is characteristic of Down syndrome. The nurse would recommend swaddling and wrapping the baby before picking her up. There is no indication of maternal deprivation. A lack of clinging or molding is not symptomatic of detachment and rejection. These physical signs are characteristic of Down syndrome; autism is not associated with Down syndrome.

manifestations hemophilia

SQ and IM bleedings are often most common form of internal bleeding- bleeding into the joint cavities (hemarthrosis) ( knees, elbows, anckles), leading to bone changes signs of hemarthrosis - feeling stiffness, tingling, ache in the affected joints, decreased ability to move the joint, warmth, redness, swelling, severe pain intracranial hemmorhage can be fatal Bleeding tendencies range from mild to severe Symptoms may not occur until 6 months of age Mobility leads to injuries from falls and accidents Hemarthrosis Bleeding into joint spaces of knee, ankle, elbow, leading to impaired mobility Ecchymosis Epistaxis Bleeding after procedures Minor trauma, tooth extraction, minor surgeries Large subcutaneous and intramuscular hemorrhages may occur Bleeding into neck, chest, mouth may compromise airway

late signs of increased intracranial pressure in infants andchildren

bradycardia decreased motor response to commands decreased sensory response to paiful stimuli alterations in pupil size and reactivity extension or flexion posturing cheyene stokes respiratins papilledema decreased consciousness coma

complicated fracture

broken bone has injured an internal organ

nursing activities for neurologically compromised child

Sandbags or other support devices can help maintain correct head position. The child can be propped to one side or the other, and the use of a pressure-relieving or pressure-decreasing mattress decreases the chance of prolonged pressure to vulnerable skin areas. Frequent clinical assessment of the child cannot be replaced by an ICP monitoring device. goalsL 1. maintaining CPP 2. controlling ICP, cerebral edema, seizures, fever 3. maintaining hemodynamic stability minimize environmental stimuli range of motion exercise no suction and percussion IV fluids and calories fluids amount depends on edema nutrition through NG or gastrostomy tube avoid overfeeding to prevent comiting -altered pituitary secretion: monitor I &O-- inappropriate ADH, DI - fluid restriction inflammatory confitions & edema - costecosteroids cerebral edema - osmotic diuretics midazolam and morphine

A nurse is performing a systematic assessment of a child who was admitted to an emergency facility after a road accident. The nurse is searching for any additional injuries that were not addressed in the primary survey. Which is the most appropriate procedure to follow while performing a secondary survey of the child? Search in the direction of head to toe. Search in the direction of toe to head. Ask the child about any pain the child feels. Ask the attendant for a first-hand experience.

Search in the direction of toe to head. Rationale To perform a systematic assessment of a child, it is preferred to search in the direction of toe to head. This helps the nurse to gain the child's trust as the survey progresses. Otherwise, a general systemic survey is done in the direction of head to toe. Rather than asking a child who is in trauma, the nurse should do a visual inspection to observe for areas of deformity, edema, ecchymosis, bleeding, hematoma, paralysis, or pain. The nurse should also collect a first-hand experience from a witness.

Which clinical manifestations are appropriate for the child with Down syndrome? Small nose, large ears, and hypertonia Large nose, large ears, and short, broad neck Short stature, protruding tongue, and hypotonia Short stature, narrow space between the big and second toes, and hypotonia

Short stature, protruding tongue, and hypotonia Rationale Short stature, protruding tongue, and hypotonia are common clinical manifestations of Down syndrome that the nurse is likely to observe during an assessment. A small nose, rather than a large one, and small ears, rather than large ones, with a short, broad neck are common clinical manifestations of Down syndrome. A large space, rather than a narrow one, between the big and second toes, along with hypotonia are common clinical manifestations of Down syndrome.

The nurse in a pediatric hematology unit admits a patient with sickle cell anemia, transferred from the emergency department. Which finding warrants immediate additional assessment by the nurse? Knee pain and a hemoglobin of 10.1 Abdominal pain and serum glucose level of 86 Anorexia and urinalysis that is positive for protein Shortness of breath and a white blood cell (WBC) count of 16,000

Shortness of breath and a white blood cell (WBC) count of 16,000 Shortness of breath can indicate respiratory compromise in patients with sickle cell crisis, and a WBC count of 16,000 indicates a possible infection. These findings warrant immediate assessment at this time.

Which nursing information is appropriate when discussing long-term care with the parents of a child who has a ventriculoperitoneal shunt to correct hydrocephalus? Most usual childhood activities must be restricted Cognitive impairment is to be expected with hydrocephalus Shunt malfunction or infection requires immediate treatment Parental protection is essential until the child reaches adulthood

Shunt malfunction or infection requires immediate treatment Rationale Because of the potentially severe sequelae, symptoms of shunt malfunction or infection must be assessed and treated immediately if present. Limits would be appropriate to the child's developmental age. Except for contact sports, the child will have few restrictions. The development of cognitive impairment depends on the extent of damage before the shunt was placed. Limits would be appropriate to the child's developmental age.

Match the fracture type with the clinical presentation.

Simple Fracture is diagonal through the bone Compound Fracture is horizontal, completely through the bone Greenstick Bone is fractured but did not pierce skin Comminuted Fractured bone is protruding from the skin

The nurse is caring for a 7-year-old child with a suspected left radius/ulna fracture who presents to the pediatric emergency room. The nurse notes a painful, bruised, edematous area on the left lower arm, but the assessment is otherwise normal. Which provider orders would the nurse anticipate? Select all that apply. Computed tomography of left arm Administration of intravenous fluids Single view radiograph of the left arm Assess pain level every 1 hour and as needed Notify social services of a consult for suspected non-accidental injury

Single view radiograph of the left arm A single view radiography would be an order the nurse would anticipate. Correct Assess pain level every 1 hour and as needed The child reports pain at the injured site, therefore, the nurse should anticipate an order to assess pain level in the child.

A nurse receives orders that state a 9-year-old patient should be placed in traction after returning to the unit. Surgery was performed for a femoral fracture. Based on this information, which type of traction should the nurse prepare? Select all that apply. A. Skin traction B. Skeletal traction C. Continuous traction D. Intermittent traction E. Cervical (halo) traction

Skeletal traction Skeletal traction is indicated because the patient is older than 3 years and has had a bone fracture. Correct Continuous traction The nurse should always assume that traction is continuous unless otherwise stated by the health care provider.

The parent of a young obese child reports that 1 week ago the child started limping on the right leg and complaining of pain in the groin and knee. The parent also notes that the right leg seems shorter than the left. The nurse anticipates which diagnosis? Lordosis Kyphosis Legg-Calvé-Perthes disease Slipped capital femoral epiphysis

Slipped capital femoral epiphysis Rationale Slipped capital femoral epiphysis is associated with obesity and presents with limping on the affected side, with the affected leg seeming shorter than the unaffected leg, and pain in the groin and knee. Lordosis is an accentuation of the cervical or lumbar curvature beyond physiologic limits and isn't associated with obesity or limping. Kyphosis is an abnormally increased lateral angulation in the convex curvature of the spine and isn't associated with obesity, limping, or pain on the affected extremity. Legg-Calvé-Perthes disease has an insidious onset, marked by the appearance of a limp over an extended period.

The twelve-year-old patient with spina bifida exhibits learning delays. What other assessment findings does the nurse anticipate? Select all that apply. Slow to follow directions Difficulty swallowing foods Upper limb discoordination Frequent respiratory infections Bowel and bladder incontinence

Slow to follow directions If the child's spine is affected high enough to result in learning delays, additional processing delays, such as following directions, might be expected. Difficulty swallowing foods The child's cognitive delays are related to the height of the neural tube defect, however difficulty swallowing may occur and can also indicate the child also has Chiari II malformation. Upper limb discoordination The child with learning delays secondary to spina bifida will also have difficulty with gross and fine motor skills using the arms and hands. Bowel and bladder incontinence Even fairly low neural tube defects can result in incontinence. A defect as high as this one definitely leads the nurse to anticipate incontinence.

Types of CP types of movement disorder

Spastic Athetoid/dyskinetic Ataxic Mixed/dystonic

Which condition is appropriate for the child that tends to walk on the toes with flexion at the knees and hips, and also walks stiffly, taking slow and deliberate steps? Ataxia Cerebellar gait Extrapyramidal gait Spastic paraplegic gait

Spastic paraplegic gait Rationale Spastic paraplegic gait is a narrow-based gait with a tendency to walk on toes, along with flexion at knees and hips. The patient walks stiffly, taking slow and deliberate steps. In ataxia, there is an impaired ability to coordinate movements with staggering gait and postural imbalance. Cerebellar gait is a staggering, unsteady, and wide-based gait with a tendency to veer in one lateral direction. It is often accompanied by swaying of the trunk. In extrapyramidal gait, there is rigidity, few automatic movements, and bradykinesia with associated bending of the trunk and head.

The parents of a 10-year-old child with Down syndrome tell the pediatric nurse that the child struggles to dress and feed himself, so they perform these tasks for him. They also state that they tie the child's shoes and the child cannot zip his jacket. Which action by the nurse is appropriate? Speak with the parents regarding purchasing clothing and shoes that are easy to put on by the child. Speak with the parents about providing opportunities for the child to attempt to complete the task independently. Speak with occupational therapy regarding skill building and possible adaptive devices to help the child self button, zip, dress, and feed. Speak with physical therapy regarding muscle strengthening exercises to increase the child's ability to self zip, button, dress and feed.

Speak with occupational therapy regarding skill building and possible adaptive devices to help the child self button, zip, dress, and feed. Due to the level of coordination, dexterity, and muscle strength, children with DS may not be able to button or zip or feed themselves without the use of adaptive devices.

Let's move on to hip dysplasia. What is hip dysplasia?

Spectrum of disorders related to abnormal development of the hip that may occur at any time during fetal life, infancy or childhood. Three types or degrees: acetabular dysplasia (mildest), subluxation (largest # of cases = incomplete dislocation), dislocation (femoral head loses contact with acetabulum)

Which neural tube defect is not visible externally in the lumbosacral area called? Meningocele Myelomeningocele Spina bifida cystica Spina bifida occulta

Spina bifida occulta Rationale Spina bifida occulta is completely enclosed. Often this disorder will not be noticed. A clue to the presence of this internal disorder is a dimple or tuft of hair in the lumbosacral area. A meningocele contains meninges and spinal fluid but no neural tissue and is evident at birth as a sac in the lumbosacral area. Transillumination of light will be present. A myelomeningocele is a neural tube defect that contains meninges, spinal fluid, and nerves and is evident at birth as a sac in the lumbosacral area. Transillumination of light will not be present. Spina bifida cystica is a cystic formation with an external saclike protrusion.

The nurse is caring for patients on a pediatric neurology unit. Which patient should the nurse assess first? The patient diagnosed with a brain mass reporting a sudden loss of vision. The patient with a head injury after falling off a bicycle, reporting nausea and abdominal cramping. The patient admitted with meningitis, with a recently recorded temperature of 101.2 Fahrenheit. The patient diagnosed with a seizure disorder with the parent reporting the child has significant loss of appetite.

The patient diagnosed with a brain mass reporting a sudden loss of vision. A sudden loss of vision indicates rapidly increasing ICP, probable bleeding, and needs immediate evaluation.

Which nursing action is appropriate for the 3-year-old diagnosed with cerebral palsy who is having orthopedic surgery and has difficulty swallowing, cannot hold a utensil to self-feed, and is slightly underweight for height? Placing the child in a well-supported semi-reclining position to make use of gravity flow Bottle- or tube-feeding the child with a specialized formula until sufficient weight is gained Placing the child in a sitting position with the neck hyperextended to make use of gravity flow Stabilizing the child's jaw with one hand (either from a front or side position) to facilitate swallowing

Stabilizing the child's jaw with one hand (either from a front or side position) to facilitate swallowing Rationale Because the jaw is compromised, more normal control can be achieved if the feeder provides stability. Manual jaw controls assist with head control, correction of neck and trunk hyperextension, and jaw stabilization. The child is too old to be bottle-fed. The neuromuscular compromise of the jaw interferes with the child's ability to eat. The child should be sitting up for meals to prevent aspiration. For swallowing, the neck should not be hyperextended.

early intervention programs

Statewide integrated developmental services available to families with children up to 3 years of age whose healthy development is compromised.

Which type of seizure is indicated when a child has a seizure lasting 35 minutes and loss of consciousness? Status epilepticus An absence seizure A generalized seizure A simple partial seizure

Status epilepticus Rationale Status epilepticus is a generalized seizure that lasts more than 30 minutes. This is considered a medical emergency and requires immediate treatment. Absence seizures are generalized seizures that are characterized by brief loss of consciousness, blank staring, and fluttering of the eyelids. Generalized seizures are the most common form of seizures. They include tonic-clonic (grand mal) seizures and absence (petit mal) seizures. Tonic-clonic seizures involve tonic-clonic activity and loss of consciousness and affect both hemispheres of the brain. Simple partial seizures are characterized by varying sensations and motor behaviors.

Which nursing action is appropriate for the child that is receiving a blood transfusion and begins having dyspnea, hypertension, and precordial pain? Stop the transfusion. Give antihypertensive medication. Give epinephrine immediately. Insert catheter and monitor hourly outputs.

Stop the transfusion. Rationale If during blood transfusion a patient has dyspnea, hypertension, precordial pain, and the nurse suspects circulatory overload, the priority action is to stop the transfusion. Giving antihypertensive medication is not the priority action, because the nurse needs to stop what is causing the symptoms. Giving epinephrine is an action related to allergic reactions. Inserting a catheter and monitoring hourly for output may be a nursing action later but is not the priority and is most related to actions connected to hemolytic reactions.

Which nursing action is appropriate when a child receiving a blood transfusion is experiencing an adverse reaction? Taking vital signs Stopping the transfusion Diluting the infusing blood Notifying the health care provider

Stopping the transfusion Rationale Stopping the transfusion, obtaining new tubing, and maintaining a patent IV line with normal saline solution are the priority nursing actions. The nurse should take vital signs and notify the health care provider as appropriate after priority responsibilities have been fulfilled. If an adverse reaction is occurring, it is essential to minimize the amount of blood that is infused, so stopping the transfusion, rather than diluting the blood is appropriate.

A nurse is caring for an infant with a clubfoot. When planning care, what should the nurse consider regarding the Ponseti casting method? Select all that apply. Stretch the misaligned bones on a weekly basis. Forced stretching is performed to soften the tissue. After weekly manipulation, apply a long-leg plaster cast. Limit manipulation to allow the foot to contract before applying the cast. Surgery is performed 3 weeks after manipulation of the misaligned bones to prepare the foot for casting.

Stretch the misaligned bones on a weekly basis. The misaligned bones are stretched on a weekly basis to aid in realigning the bones. After weekly manipulation, apply a long-leg plaster cast. The misaligned bones are manipulated on a weekly basis followed by the application of a well-molded long-leg plaster cast to hold the bones in place.

Which nursing action is appropriate when educating parents on how to promote mobilization in the infant with cerebral palsy? Using wheeled go-carts Ankle-foot orthoses Strollers with custom seats Power-operated wheelchairs

Strollers with custom seats Rationale Normalization and the ability to care for oneself are the primary goals of therapy for patients with cerebral palsy. The strategy used for infants is the use of strollers with customized seats. Because infants have poorly developed motor activity, making use of strollers promotes dependent mobilization as a part of illness management. Wheeled go-carts are used to promote mobilization in toddlers, preschoolers, and school-age children. Ankle-foot orthoses are used to prevent deformity of the limbs in older children with cerebral palsy. They are generally not worn by infants. Power-operated wheelchairs are mostly used for adolescents to promote independent mobility.

Which finding is appropriate when diagnosing a child with an intellectual disability? An intelligence quotient (IQ) of 70 or less An IQ of 75 or less Subaverage intellectual function, deficits in adaptive skills, and onset at any age Subaverage intellectual function, deficits in adaptive skills, and onset before 18 years of age

Subaverage intellectual function, deficits in adaptive skills, and onset before 18 years of age Rationale The diagnosis of intellectual disability is made with the presentation of subaverage intellectual functioning, deficits in adaptive skills, and onset before age 18. IQ is only one component of the diagnosis of intellectual disability. The onset of the deficit in adaptive skills and subaverage intellectual function must occur before age 18 to meet the diagnosis of intellectual disability.

down syndrome treatment

Support family at the time of diagnosis, assist the family in preventing physical problems, assist in prenatal diagnosis and genetic counseling. They can run into a lot of cardiac issues, lung issues, they can end up with the hydrocephalus, respiratory infections. They get a lot of secretions. They have a lot of trouble swallowing and getting rid of those secretions. One of the things you can teach mom and dad to do is in their bedroom at night when they are a baby is have cool mist humidifier it will help loosen those secretions and thin them out because they get very sick at times. You gotta keep those secretions thin so cool mist, not warm. Moisture in the air helps loosen up those secretions and makes it easier for them to get rid of them. Makes it easier for them to breathe. Especially with INFANTS. Okay and again the goal is pretty much the same as with any chronically ill child.

The nurse is caring for an immobilized preschool child. Which action is most helpful during this period of immobilization? Encouraging the child to wear pajamas Letting the child have few behavioral limitations Taking the child for a "walk" by wagon outside the room Keeping the child away from other immobilized children if possible

Taking the child for a "walk" by wagon outside the room Rationale It is important for children to have activities outside the room if possible. This gives them opportunities to meet their normal growth and developmental needs. The child should be encouraged to wear street clothes during the day. Limit-setting is necessary with all children. There is no reason to segregate children who are immobilized unless there are other medical issues that need to be addressed.

Which nursing action is appropriate for the cognitively impaired child before a feeding program is instituted? Food diary Task analysis Physical assessment Evaluation of the home environment

Task analysis Rationale Before beginning a self-feeding program, the nurse would perform a task analysis in which the child is observed in a particular situation, such as eating, to determine what skills are possessed and the child's developmental readiness to learn the task. The family's level of readiness is as important for the nurse to assess as the child's. Performing a physical assessment prior to beginning a self-feeding program is not a priority nursing action to the self-feeding program. Although evaluating the home environment and obtaining a food diary may be helpful, neither is a priority nursing action.

Parents of a two-year-old child with type 1 diabetes are concerned about their child's diabetes and its impact over time. How should the nurse respond to their concerns? Select all that apply. Teach and help the parents adjust and cope with the child having type 1 diabetes. Formulate a plan of care based on the manifestations and pathophysiology of type 1 diabetes. Inform the parents how treatment for type 1 diabetes will be incorporated into their daily lives. Inform the parents that care needs and approaches will likely change as the child grows and develops. Help prepare the parents to care for the child by letting them know what future school accommodations to expect with type 1 diabetes.

Teach and help the parents adjust and cope with the child having type 1 diabetes. Teaching the parents provides information that will help them adjust and cope with the child's chronic illness. This creates the necessary psychosocial environment that will positively influence the child's growth and development. Inform the parents how treatment for type 1 diabetes will be incorporated into their daily lives. Informing the parents of the treatment regime for type I diabetes and how it fits into their lives allows for normalization of the family unit and maximized function throughout the illness. Inform the parents that care needs and approaches will likely change as the child grows and develops. Because type 1 diabetes is a chronic illness and will be present for the rest of the child's life, care must incorporate changes in the child's growth and development. Help prepare the parents to care for the child by letting them know what future school accommodations to expect with type 1 diabetes. Informing the parents regarding what to expect when the child enters school with type 1 diabetes allows them to develop a plan of care and aids in achieving and maintaining the highest level of health and social functioning for the child.

What is an important nursing consideration when caring for a child with juvenile idiopathic arthritis? Apply ice packs to relieve stiffness and pain. Administer acetaminophen to reduce inflammation. Teach the child and family the correct administration of medications. Encourage range of motion exercises during periods of inflammation.

Teach the child and family the correct administration of medications. Correct The management of juvenile idiopathic arthritis is primarily pharmacologic. The family should be instructed regarding administration of medications and the value of a regular schedule of administration to maintain a satisfactory blood level in the body. They need to know that nonsteroidal antiinflammatory drugs should not be given on an empty stomach and to be alert for signs of toxicity. Warm, moist heat is best for relieving stiffness and pain. Acetaminophen does not have antiinflammatory effects. Range of motion exercises should not be done during periods of inflammation.

The health care provider examines a 7-year-old child, revealing increased deep tendon reflexes, hypertonia, flexion, and a scissors gait. Which intervention does the nurse include in this patient's plan of care? Give the parents teaching pamphlets about antiseizure medication. Ensure the child has a hospital bed and bedside commode for use at home. Provide the child with a diet plan that includes reduced-fat milk and cheeses. Teach the child and parents how to monitor for and address learning difficulties.

Teach the child and parents how to monitor for and address learning difficulties. Children with cerebral palsy tend to have learning disabilities and poor attention spans. Educating parents on how to seek help for these problems as they arise is an essential part of this child's care plan.

A 4-year-old child is newly diagnosed with Legg-Calvé-Perthes disease. Nursing considerations include what? Encourage normal activity for as long as possible. Explain the cause of the disease to the child and family. Prepare the child and family for long-term, permanent disabilities. Teach the family the care and management of the corrective appliance.

Teach the family the care and management of the corrective appliance. Correct The family needs to learn the purpose, function, application, and care of the corrective device and the importance of compliance to achieve the desired outcome. The initial therapy is rest and non-weight-bearing activity, which help reduce inflammation and restore motion. Legg-Calvé-Perthes is a disease of unknown etiology. A disturbance of circulation to the femoral capital epiphysis produces an ischemic aseptic necrosis of the femoral head. The disease is self-limiting, but the ultimate outcome depends on early and efficient therapy and the age of the child at onset.

Which nursing recommendation is appropriate for the 6-year-old child who was born with a myelomeningocele, has a neurogenic bladder, and whose parents have been performing clean intermittent catheterization? Teaching the child to perform self-catheterization Teaching the child the appropriate bladder control Continuing to have the parents do the catheterization Encouraging the family to consider urinary diversion

Teaching the child to perform self-catheterization Rationale At 6 years of age this child should have the dexterity to perform intermittent catheterization. This will give the child more control and mastery over the disability. Bladder control cannot be taught in a child with a neurogenic bladder. A urinary diversion is not necessary for a neurogenic bladder.

Which signs are appropriate when monitoring for signs of developing hydrocephalus in a toddler with spina bifida? Temperature instability, irritability, lethargy, and elevated intracranial pressure Intactness of the membranous cyst, anal reflex inactivity, and motor or sensory impairment Behavioral instability, and inactivity in spinal cord reflex and limb movement with stimuli Cognitive impairment, pain, and tension or bulging in any part of the body

Temperature instability, irritability, lethargy, and elevated intracranial pressure Rationale Early signs of hydrocephalus include signs of infection, such as temperature instability (axillary), irritability, lethargy, and elevated intracranial pressure. Children with spina bifida are placed in an incubator so their temperature can be maintained without clothing. Signs of intactness of the membranous cyst, anal reflex inactivity, and motor or sensory impairment leading to immobility are not signs of developing hydrocephalus. Similarly, behavioral instability, impaired limb movement in conjunction with stimuli, and spinal cord reflex inactivity also are signs of developing hydrocephalus. In addition to this, cognitive impairment, pain, and tension or bulging in any part of the body are also looked for. These are not signs of developing hydrocephalus.

The nurse has assessed a child with acute osteomyelitis. How does the nurse document the local clinical manifestations in this patient? Select all that apply. Irritability Tenderness Restlessness Increased warmth Diffuse swelling over the involved bone

Tenderness Increased warmth Diffuse swelling over the involved bone Rationale Tenderness, increased warmth, and diffuse swelling over involved bone are local clinical manifestations of acute osteomyelitis. Irritability and restlessness are general manifestations of acute osteomyelitis.

Which immediate nursing intervention is appropriate for the patient who has sustained a head injury and exhibits rhinorrea? Reassure the patient, because it is an insignificant finding. Test the discharge for presence of glucose. Sedate the patient and administer antihistamine. Ask the patient to report immediately if the nose bleeds.

Test the discharge for presence of glucose. Rationale Patients with head injury may have leakage of cerebrospinal fluid. The watery nasal discharge is tested for presence of glucose to rule out cerebrospinal fluid (CSF) leakage. Reassurance is given only after excluding CSF leakage. Sedating by administering an antihistamine is not appropriate in managing rhinorrhea of head injury. The patient is asked to report nasal bleeding, but priority is given to check for CSF leakage.

An adolescent presents with a severe knee sprain accompanied by edema and pain. Based on the severity of the injury and involvement of the knee ligaments, surgery is required. What should the nurse include in the preoperative teaching? Select all that apply. To anticipate being out of bed while in recovery That bearing weight is gradually increased as pain subsides The 5 P's so they can assess for complications at home Stretching and strengthening exercises to regain mobility To only ask for pain medication as needed to reduce risk of addiction

That bearing weight is gradually increased as pain subsides As the patient goes through the healing process, the pain will subside and therefore increase tolerance for more mobility. The 5 P's so they can assess for complications at home If the child is being discharged to home shortly after surgery, the parents should be taught about the signs of neurovascular impairment (the 5 P's) so they can monitor for complications after surgery at home. Stretching and strengthening exercises to regain mobility Stretching and strengthening exercises are recommended to help the child regain mobility and strength of the affected joint.

The nurse cares for patients in a pediatric headache clinic. Which patient should the nurse assess first? The 4-year-old child with abdominal pain and nausea. The 10-year-old child vomiting in the examination room. The 12-year-old child who reports smelling smoke and fire. The 6-year-old who screams when someone touches the child's head.

The 10-year-old child vomiting in the examination room. This child with vomiting is the most serious of these patients, indicating either a migraine in need of medicating or increased intracranial pressure (ICP).

The 12-year-old sibling of an infant born with Down syndrome has been increasingly difficult and lashing out on family and friends. The parents are spending time with the 12-year-old child and do not understand why the child's behavior has changed so much. What information should the nurse provide? The family is likely spending too much time with the infant and not enough time with the 12-year-old. Regression is expected from a sibling when any new infant is brought into the family because the child feels jealous. The 12-year-old is likely experiencing anger as part of a grief response to the infant being born with a chronic illness. A 12-year-old does not understand that the infant has a chronic condition, so these behaviors are likely from a lack of discipline.

The 12-year-old is likely experiencing anger as part of a grief response to the infant being born with a chronic illness. Any family member can experience grief when a child has a chronic illness. A 12-year-old may not know how to express this grief and could have difficulties navigating the grieving process. Anger is part of the natural grieving process.

A 3-year-old child with Down syndrome is hospitalized with pneumonia. The child becomes agitated when the parents leave the room and often picks at his arms. He doesn't respond to the nurse's questions and stumbles often when walking in his room and the hallway. What does this behavior suggest to the nurse? Select all that apply. The behavior suggests that the child may have sensory deficits. The behavior suggests that the child may have inadequate social skills. The behavior suggests that the child is trying to cope with his hospitalization. The behavior suggests that the child has a diminished emotional response. The behavior suggests that the child may have problems with balance and coordination.

The behavior suggests that the child may have sensory deficits. The child with DS often has hearing and vision deficits. It is common for the child to not respond when spoken to. The behavior suggests that the child is trying to cope with his hospitalization. Self-stimulating behaviors are often used as coping strategies in children with DS. The behavior suggests that the child may have problems with balance and coordination. The information provided suggests that the child has a potential problem with balance and coordination due to the stumbling gate.

A patient with a purpuric rash and platelet count of 55,000 has just undergone a bone marrow biopsy. Which diagnostic test results would be expected from the biopsy for a diagnosis of immune thrombocytopenic purpura (ITP)? Increased granulocyte production Decreased erythrocyte production Impaired lymphocyte production The bone marrow results will be normal

The bone marrow results will be normal ITP is caused by destruction of platelets outside the bone marrow. Bone marrow aspirate results will be normal.

The nurse is forming the plan of care for a child with cystic fibrosis. The child has a productive cough of thick mucus, oxygen saturation levels of 95% on room air, and has lost 4 pounds from the past clinic visit. Which expected outcome is the priority for the nurse? The child gains a half a pound of body weight per week. The child remains free of signs of pneumonia or bronchitis. The child regularly consumes a high calorie supplement between meals. The child remains able to cough up secretions with clear lung sounds bilaterally.

The child remains able to cough up secretions with clear lung sounds bilaterally. Being able to effectively clear the airway is important for a child with CF due to the thick pulmonary secretions.

A child with fragile X syndrome is unable to walk without stumbling, has uncoordinated limb movement, fails to respond to his name, and does not track eye and hand movement together. Which therapeutic intervention is most appropriate for the patient? The child requires special education services to improve sensory functioning. The child requires occupational therapy sessions to improve limb movement. The child requires sensorimotor integration therapy to decrease the symptoms. The child requires safety assessment to address issues related to safety and mobility.

The child requires sensorimotor integration therapy to decrease the symptoms. Sensorimotor integration therapy enhances motor planning, joint stability, coordination, and integration of visual, auditory, and tactile information. It is considered the therapy of choice for children with learning disabilities.

A nurse is caring for a child with impaired physical mobility. What is an appropriate outcome for this child? The child has decreased mobility and strength in unaffected joints. The child will maintain mobility but will have decreased strength in the unaffected joints. The child will not be able to maintain strength in unaffected joints but will tolerate activity restriction. The child will maintain mobility and strength of all unaffected joints and tolerate activity restrictions.

The child will maintain mobility and strength of all unaffected joints and tolerate activity restrictions. Maintaining mobility and strength of all unaffected joints and tolerating activity restrictions is the expected outcome of a child with an impaired physical mobility diagnosis. Working towards this goal will help to improve the physical mobility of the child.

The emergency department nurse triages several patients with Von Willebrand disease (vWD). Which patient should the nurse see first? The child with a cough and temperature of 102.4° F. The child with a headache after rolling off the couch who is lethargic. The child who complains of nausea and abdominal pain and is actively vomiting. The child with a platelet count of 125,000 who underwent femur fracture repair 3 days ago.

The child with a headache after rolling off the couch who is lethargic. A patient with vWD and a head injury should be seen first.

The nurse receives a report from the day shift on the assigned children with hemophilia. Which patient should the nurse see first? The child with influenza and a platelet level of 115,000. The child with gastroenteritis and a serum potassium level of 3.1. The child with a right arm fracture and a factor activity level of 48%. The child with hemoglobin of 9 after tonsillectomy.

The child with a right arm fracture and a factor activity level of 48%. The child with a right arm fracture who has a factor activity level of 48% is at increased risk for bleeding and should be seen first.

The nurse auscultates the lungs of a child with cystic fibrosis and notes coarse crackles and rhonchi. The patient also has a productive cough with thick yellow mucus. Which considerations does the nurse undertake in planning before implementing care for the child? Select all that apply. The child's pulse oximetry needs to be measured frequently. A barrel chest and digital clubbing indicate long-standing respiratory difficulty. The child may have pneumonia or bronchitis, and gas exchange may be impaired. The child may demonstrate orthopnea and activity intolerance related to the respiratory symptoms. The child may have a prolonged inspiratory phase of respiration due to long standing respiratory difficulties.

The child's pulse oximetry needs to be measured frequently. Pulse oximetry levels should be monitored frequently in the child with cystic fibrosis because of the possibility of hypoxia related to pneumonia and bronchitis. A barrel chest and digital clubbing indicate long-standing respiratory difficulty. A barrel chest and digital clubbing may be observed in the child with cystic fibrosis due to long-standing respiratory difficulty. The child may have pneumonia or bronchitis, and gas exchange may be impaired. Coarse crackles and rhonchi with a productive cough are signs of pneumonia or bronchitis, which is common in the child with cystic fibrosis. The child may demonstrate orthopnea and activity intolerance related to the respiratory symptoms. Orthopnea and activity intolerance are associated with pneumonia and bronchitis related to cystic fibrosis.

facture imterventions

The method of fracture reduction is determined by the child's age, degree of displacement, amount of overriding, amount of edema, condition of the skin and soft tissues, sensation, and circulation distal to the fracture. Simple nondisplaced fractures in children may be managed with immobilization in a cast or splint for 4 to 6 weeks. Open, displaced, and compound fractures require surgical intervention

The nurse is working with a family that has two parents and three children. The oldest child was just diagnosed with diabetes mellitus. The parents want to know how they should schedule their health care provider's appointments. What information should the nurse provide? The child who is dealing with a chronic illness requires extra attention from the parents and siblings, so everyone should attend the appointments. The family should try to schedule health care appointments at times that do not interfere with any of the children's school or extracurricular activities. The child's medical needs are top priority. Cancelling other activities or engagements for the children is an acceptable adjustment for the oldest child's health. The family needs to maintain a balance, so some scheduled appointments may be missed to allow the parents and child not to miss any extracurricular or school activities.

The family should try to schedule health care appointments at times that do not interfere with any of the children's school or extracurricular activities. Normalcy is important to the family, and each child needs to feel like their activities are important. Whenever possible, appointments should be scheduled around activities.

A child with clubfoot was treated with Ponseti casting. Following completion of the casting treatments, the patient has the final cast removed. The parents are upset because the foot appears overcorrected. Which explanation can the nurse provide regarding the rationale for the overcorrection? A. Overcorrection is necessary so that surgery will be possible. B. The foot is purposefully overcorrected and a brace will be used to prevent the recurrence of the deformity. C. Unfortunately, the overcorrection was not intended and an additional cast will be used to fix the problem. D. The overcorrection will prevent recurrence and is the last step before initiating the French physiotherapy treatment.

The foot is purposefully overcorrected and a brace will be used to prevent the recurrence of the deformity. The foot is overcorrected and the correction is maintained by using abduction orthosis full-time for 3 weeks, followed by 12-hour nightly use until at least 2 years of age.

A 1-year-old infant is seen in the pediatric clinic. The parents report that the infant does not babble or coo like other infants. They state that the infant doesn't attempt to form words or point at objects. In observing these behaviors, which action does the nurse consider next? The infant should begin sensorimotor integration therapy. The infant should have a speech and language evaluation as soon as possible. The nurse should facilitate DNA testing to confirm the diagnosis of fragile X syndrome. The nurse should assess the infant for the physical features associated with fragile X syndrome.

The infant should have a speech and language evaluation as soon as possible. An evaluation of speech and language abilities will provide information that could demonstrate the need for DNA testing to confirm the presence of fragile X syndrome.

Which reason is appropriate when explaining to parents why infants are particularly vulnerable to acceleration-deceleration head injuries? The anterior fontanel is not yet closed. The nervous tissue is not well developed. The infant scalp has extensive vascularity. The musculoskeletal support of the head is insufficient.

The musculoskeletal support of the head is insufficient. Rationale The infant's relatively large head size, coupled with insufficient musculoskeletal support, increases the risk to infants of acceleration-deceleration head injuries. The lack of closure of the anterior fontanel, lack of well-developed nervous tissue, and vascularity of the scalp are not relevant to the risk for acceleration-deceleration injuries in infants.

A term newborn female is brought to the newborn nursery for evaluation. The nurse suspects that the newborn has Down syndrome. Which clinical manifestations support the nurse's suspicion? Select all that apply. The newborn has a protruding tongue. The newborn has fused fingers and toes. The newborn has inner epicanthal folds. The newborn presents with a flat nasal bridge. The newborn has a single transverse crease across the palmar surface.

The newborn has a protruding tongue. Clinical manifestations of Down syndrome include a protruding tongue, and therefore this manifestation would support the nurse's suspicion The newborn has inner epicanthal folds. Inner epicanthal folds are a clinical manifestation of Down syndrome, and therefore this manifestation would support the nurse's suspicion. The newborn presents with a flat nasal bridge. Clinical manifestations of Down syndrome include a flat nasal bridge, and therefore this manifestation would be expected in a patient who the nurse suspects has Down syndrome. The newborn has a single transverse crease across the palmar surface. A single transverse crease across the palmar surface is a clinical manifestation of Down syndrome, and therefore this manifestation would support the nurse's suspicion.

The nurse evaluates a three-year-old child for developmental delays. When the nurse notes that the child has difficulty maintaining balance while walking, what other assessments does the nurse perform? The nurse assesses overall muscle tone and strength. The nurse assesses for speech impairments and delays. The nurse assesses deep tendon and primitive reflexes. The nurse assesses level of consciousness and orientation. The nurse assesses for developmental milestone variances.

The nurse assesses overall muscle tone and strength. Cerebral palsy (CP) is characterized by abnormal muscle tone and therefore the nurse will assess the overall tone and strength at this time. Correct The nurse assesses for speech impairments and delays. Impaired speech is a characteristic developmental impairment found in CP and therefore this would be assessed at this time. Correct The nurse assesses deep tendon and primitive reflexes. A child with CP may have persistent primitive reflexes, which should have disappeared in infancy. In assessing a suspected neurologic disorder, the nurse should assess deep tendon reflexes as well. The nurse assesses for developmental milestone variances. The child with cerebral palsy may not have reached other milestones at appropriate ages, and therefore should be assessed for these at this time.

Which factor is appropriate to consider when administering iron? To administer with ascorbic acid Can be given into a small muscle May be mixed with yogurt for administration to small children Would be injected deeply into a large muscle mass using an air-lock method

To administer with ascorbic acid Rationale Iron should be given with ascorbic acid (Vitamin C). Iron would be given into large muscles to prevent skin staining and irritation. Iron would be injected deeply into a large muscle with the Z-track, not the air-lock, method to prevent skin staining and irritation.

Neutropenia

is a reduction in the number of circulating neutrophils and is usually defined as an absolute neutrophil count (ANC) of less than 1000/mm3 in infants 2 weeks to 1 year of age or less than 1500/mm3 in children older than 1 year of age. Therapy to increase the ANC is rarely required. Children who have recurrent or severe infections, however, may benefit from the administration of granulocyte colony-stimulating factor.

The purpose of the neurologic examination

is to establish an accurate, objective baseline of neurologic information. Complete neurologic examination includes level of consciousness; posture; motor, sensory, cranial nerve, and reflex testing; and vital signs.

To reduce anxiety in the child undergoing cast removal, which of the following nursing interventions would the nurse expect to be least effective? a. Demonstrate how the cast cutter works before beginning the procedure. b. Desxribe cast saw as feeling "tickly" and warm c. Explain that it will take only a few minutes. d. Continue to reassure that all is going well and that the child's behavior us accepted during the removal process.

c. Explain that it will take only a few minutes.

approach behaviors

coping mechanisms resulting in movement toward adjustment and resolution of the crisis

Which effect does immobilization have on the cardiovascular system? Lessened need for oxygen No specific primary effect Negative nitrogen balance Venous stasis and dependent edema

Venous stasis and dependent edema Rationale Immobilization physically affects the cardiovascular system by causing venous stasis and dependent edema. Decreased need for oxygen is how immobilization affects the respiratory system, not the cardiovascular system. Immobilization has very specific effects on the cardiovascular system. Negative nitrogen balance is how immobilization affects metabolism, not the cardiovascular system.

Which nursing management goal is appropriate for an infant born with a myelomeningocele? To manage bowel control To preserve renal function To achieve optimal urinary continence To provide antibiotic therapy to prevent infections

To preserve renal function Rationale The priority goal in the management of genitourinary function for an infant with a myelomeningocele is preservation of renal function. Bowel control is not a priority goal in the management of genitourinary function for an infant with a myelomeningocele. Achieving optimal urinary continence is a priority goal in the management of genitourinary function for an older child. Providing antibiotic therapy to prevent infection is not the priority nursing goal.

Immobilization causes what effects on the cardiovascular system? Venous stasis Increased vasopressor mechanism Normal distribution of blood volume Increased efficiency of orthostatic neurovascular reflexes

Venous stasis Correct The physiologic effects of immobilization, as a result of decreased muscle contraction, include venous stasis. This can lead to pulmonary emboli or thrombi. A decreased vasopressor mechanism results in orthostatic hypotension, syncope, decreased cerebral blood flow, and tachycardia. An altered distribution of blood volume is found with decreased cardiac workload and exercise tolerance. Immobilization causes a decreased efficiency of orthostatic neurovascular reflexes with an inability to adapt readily to the upright position and with pooling of blood in the extremities in the upright position.

The parent of a toddler diagnosed with cystic fibrosis tells the nurse that her child continues to lose weight. The toddler has lost 5 lbs. in the last 3 months and takes pancreatic enzymes at every meal. What does the nurse consider before managing this patient? Select all that apply. The nurse should ask the child what is the favorite food is. The child requires a low protein, high carbohydrate diet to gain weight. The nurse should encourage high-calorie supplements between meals. The pancreatic enzymes should be administered 30 minutes before meals and snacks to be effective. The pancreatic enzymes should be mixed in food that is high in protein if the capsules can't be swallowed whole.

The nurse should ask the child what is the favorite food is. Including the child's favorite food will assist in providing a well-balanced high calorie diet. Children will eat more food if it is a favorite. The nurse should encourage high-calorie supplements between meals. To encourage optimal nutrition, the nurse should encourage high-calorie supplements to increase the caloric intake. Correct The pancreatic enzymes should be administered 30 minutes before meals and snacks to be effective. Pancreatic enzymes must be given 30 minutes before meals or snacks and not with hot food, otherwise they are inactivated by heat.

A pediatric patient who has fragile X syndrome is seen in the clinic. The parents tell the nurse that their child displays the following behaviors at home: gaze avoidance, hand flapping, echolalia, and abnormal speech patterns, restlessness, and agitation. Which action is most appropriate for the nurse to take? The nurse should initiate a speech pathology referral for the child. The nurse should initiate a physical therapy referral for the child. The nurse should initiate an occupational therapy referral for the child. The nurse should discuss the need for a referral to remediation services for the child with the health care provider.

The nurse should discuss the need for a referral to remediation services for the child with the health care provider. Remediation services include behavioral, speech, language and motor therapies for the child. These comprehensive services address all of the symptoms displayed by the child.

An adolescent has had a lower leg amputation after a motorcycle accident and is complaining of pain in the missing extremity. How does the nurse interpret this complaint? The patient has a narcotic addiction. The patient has phantom limb sensation. The patient has nerve damage in the stump. The patient has a need for psychologic counseling

The patient has phantom limb sensation. Rationale Phantom limb sensation is an expected experience because the nerve-brain connections are still present, although they gradually fade. This should be discussed before surgery with the child. There is no indication of narcotic addiction by the adolescent complaining of pain in the amputated extremity. Phantom limb pain is expected after an amputation; psychologic counseling is not required for the adolescent experiencing it. Phantom limb pain is not suggestive of nerve damage.

Blood Transfusion Therapy

Verify identity of recipient and donor's blood group-Type & Cross, Need order, need consent Monitor VS Use appropriate filter Use blood within 30 minutes of arrival Infuse over 4 hours maximum Reactions Hypoethlic is most common Vitals signs Airway is priority Cant be hanging no more than 4 hours

Since Madison's parents are divorced, which parent should the nurse try to contact first?

The parent who has been assigned legal custody of Madison by the court.

Which nursing recommendation is appropriate for the parents of a child with Down syndrome whose genetic testing shows that the disorder was caused by chromosomal translocation? The mother should be tested if she is over age 35. No further genetic testing of the family is indicated. The parents should be tested because the translocation might be inherited. The child should be retested to confirm the diagnosis of Down syndrome.

The parents should be tested because the translocation might be inherited. Rationale The parents and any siblings should be tested. Down syndrome resulting from a translocation may be inherited. This type of chromosomal abnormality presents issues for future pregnancies. This type of chromosomal abnormality occurs in children of parents of all ages. The child does not require further genetic testing, but the parents and siblings should be further evaluated with genetic testing. There is no need to retest the child at a later date, because the diagnosis has been validated with chromosomal testing.

Cerebrospinal fluid analysis of bacterial meningitis

WBC - elevated, increased neurophils protein content - elevated glucose content - decreased gram stain, bacteria culture - positive color - turbid or cloudy opening pressure - elevarted

A 3-year-old child is hospitalized after submersion injury. The child's mother complains to the nurse, "This seems unnecessary when he is perfectly fine." What is the appropriate response by the nurse? "He still needs a little extra oxygen." "I'm sure he is fine, but the doctor wants to make sure." "It is important to observe for possible physical reasons for the accident." "The reason for hospitalization is that complications could still occur."

The reason for hospitalization is that complications could still occur." Correct Complications such as respiratory compromise and cerebral edema can occur 24 hr after the incident. If the child needed oxygen, the mother would not state the child is perfectly fine. Telling the mother that the doctor wants to make sure the child is fine minimizes the role of the nurse and the need for observation for potential life-threatening complications. Physiologic causes may need to be identified in the case of a submersion injury, but it is not the reason for hospitalization.

A teenaged patient with leukemia is frustrated by the amount of time spent in the hospital and expresses anger to the nurse. Which nursing intervention is most appropriate? The teen should be encouraged to calm down and decrease anger. The teen should be allowed to express anger and frustration through talking. The teen should be left alone to process feelings until frustration and anger is eliminated. The teen should be allowed to leave the hospital because hospitalization is the source of the frustration.

The teen should be allowed to express anger and frustration through talking. Encouraging discussion of difficult topics like those that trigger anger and frustration provide an appropriate outlet for feelings and decrease those feelings with support and subsequent intervention by the nurse.

A patient is admitted for surgery due to complication of osteomyelitis. As the nurse monitors the wound after surgery, which assessment would cause the nurse to be concerned that the antibiotic is not effective? A. Scab formation is noted at the affected site. B. Thick discharge is present at the affected site. C. The patient complains of discomfort at the affected site. D. A temperature change to 99.9° F from a previous recording of 101.8° F.

Thick discharge is present at the affected site. Redness, inflammation, and discharge indicate worsening of an infection. This would indicate the antibiotics are not effective.

What is the cause for the signs and symptoms when hemoglobin falls sufficiently to produce clinical manifestations? Phagocytosis Tissue hypoxia Pulmonary hypertension Depressed bone marrow

Tissue hypoxia Correct The signs and symptoms (e.g., weakness, fatigue, and a waxy pallor in severe anemia) are caused by tissue hypoxia. Phagocytosis is a function of white blood cells used in prevention of infection. Pulmonary hypertension is not associated with anemia. Severe anemia may contribute to cardiac compensation. Depressed bone marrow may be the cause of the low hemoglobin.

Which is an appropriate reason why a patient with an alarming rise in intracranial pressure would be induced into a coma using barbiturates? Select all that apply. To increase cerebral metabolic rate To maintain cardiac system functioning To maintain respiratory system functioning To reduce the intracranial pressure of the brain To protect the brain from the effects of low brain perfusion

To reduce the intracranial pressure of the brain To protect the brain from the effects of low brain perfusion Rationale Barbiturates reduce intracranial pressure when all other measures have failed. Barbiturates protect the brain during times of reduced cerebral perfusion pressure. Barbiturates decrease the cerebral metabolic rate for oxygen. Barbiturates do not maintain cardiac function; they decrease it, which requires intensive cardiac monitoring. Barbiturates do not maintain respiratory function; they reduce it, which requires intensive monitoring.

Which nursing advice is appropriate for the parents of a baby with Down syndrome regarding solid feedings? Tongue thrusting is a physiologic response to feeding. A protruding tongue is an indication that the child does not like the food. Use a small, short, straight-handled spoon to place food in the front of the mouth. Use a small, short, round-handled spoon to place food in the side of the mouth.

Tongue thrusting is a physiologic response to feeding. Rationale The parents of a baby with Down syndrome need to know that tongue thrusting is a physiologic response to feeding that does not indicate that the child likes or dislikes the food. Parents need to also understand that a small, long, straight-handled spoon should be used to place food in the back and side of the mouth to encourage adequate nutritional intake.

clinical manifestation of CP

UNIVERSAL MANIFESTATION - delayed gross motor development ABNORMAL MOTOR PERFORMANCE - very earlyy unilateral hand preference, abnormal and assymetric crawl, standing or walking on toes, uncoordinated or involuntary movements, poor sucking, feeding difficulties, persistent tongue thrust ALTERATION OF MUSCLE TONE - increased or decreased resistance to passive movements, arching back, feel stiff on handling or dressing, difficulty diapering, rigid and unbending at the hip and knee joints when pulled to sitting position (early sign) ABNORMAL POSTURE - maintain hip higher than trank in prone position, scissoring and extension of legs with feet plantar flexed in supine position, persistent infintile resting and sleeping position, arms abducted at shoulders, elbows flexed, hands fisted REFLEX ABNORMALITIES - persistence or primitive infintile reflexes, tonic neck reflex at any age, nonpersistence beyond 6 mo, persistence of hyperactivity of Moro, plantar and palmar grasp, hyperreflexia, ankle clonus, ASSOCIATED DISABILITIES - altered learning and reasoning, seizures, impaired behavioral and interpersonal relationships, sensory impairment ( vision, hearing), marked distractability, poor attention span, defects of integration Excessive irritability No smiling by age 3 months Feeding difficulties Persistent tongue thrusting Frequent gagging or choking with feeds, aspiration risk difficulty brushing teeth - bad teeth, as well as gingivits ( also due to antiepileptic drugs like phentoin) constipation - poor position on toilet, poor liquid intake, decreased mobility

The parent of a child being screened for developmental dysplasia of the hip (DDH) asks the nurse why ultrasonography, not radiography, is being used. Which statement could the nurse use to explain the reason for selecting ultrasonography? A. Ultrasonography is more sensitive because it uses low dose radiation. B. Radiography is more sensitive because it uses high intensity radiation. C. Radiography is more sensitive because it can be used to view the dislocation at any age. D. Ultrasonography is more sensitive because it allows for a penetrating look at hip abnormalities.

Ultrasonography is more sensitive because it allows for a penetrating look at hip abnormalities. Ultrasonography provides the ability to view the entire anatomy. Current research supports the concept that ultrasonography is a more sensitive indicator of abnormalities of the infant hip than radiography.

The charge nurse is assisting a new nurse to prepare a toddler for skin traction. What interventions will help prevent skin damage? Select all that apply. A. Use an egg-crate-type mattress for comfort. B. Use alcohol to clean areas that are prone to irritation. C. Use lotion, powder, or talc on areas prone to irritation. D. Provide a trapeze to facilitate independent repositioning. E. Applying tincture of benzoin to the intact skin that has potential for irritation.

Use an egg-crate-type mattress for comfort. Egg-crate-type or sheepskin mattress provides comfort under the back and lower legs. Additionally, it will wick moisture from the skin to decrease skin irritation. Provide a trapeze to facilitate independent repositioning. Providing a trapeze to facilitate independent repositioning would be an appropriate intervention in this situation. Correct Applying tincture of benzoin to the intact skin that has potential for irritation. Applying tincture of benzoin to the intact skin before the traction is applied may protect against skin irritation.

The nurse is educating a group of people about the first aid to be given in cases of fractures. A person questions how to splint a leg fracture when there is no ready-made splint available. Which statement by the nurse is most appropriate? "You need not splint the leg." "Just bandage the affected limb." "Use a smaller splint if available." "Use the patient's other leg as a splint."

Use the patient's other leg as a splint." Rationale A splint offers support and rest to the injured body part. If a ready-made splint is not available, the patient's other leg can be used as a splint to support the injured part. A splint should cover the joint above and below the fracture. Using a smaller splint or bandaging the limb may be insufficient to immobilize the fractured leg. Splinting the leg is very important, because movement in the fractured limb may further worsen the fracture.

Pavlik Harness

Used for childern with hip dysplasia, its a abduction harness that sometimes its worn for 23 hours a day even when you shower - Needs to be adjusted by doctor ever 1-2 weeks - Harness may be wron for 3-5 months - Baby wears knee socks to prevent skin break down doesnt immobilize, but prevents from hip extension and adduction

An arterial blood gas is drawn on a patient and it shows a decrease in the arterial partial pressure of carbon dioxide (PaCO2). The nurse should expect which response of the cerebral vasculature? Vasodilation, increased blood flow Vasodilation, decreased blood flow Vasoconstriction, increased blood flow Vasoconstriction, decreased blood flow

Vasoconstriction, decreased blood flow A decrease in the partial pressure of carbon dioxide will result in vasoconstriction and a decrease in cerebral flood flow.

Suspect for child abuse.

When the child's injuries are not congruent with the parent's description of the incident = chart will read NAT (not accidental trauma) - When the child's behaviors (fear, lack of crying) are not consistent with expected behavior - When radiographs show multiple healed fractures - Bumps, bruises, breaks, burns, anything that happened in the bathroom

The parents of a child hospitalized with sickle cell anemia tell the nurse that they are concerned about narcotic analgesics causing addiction. The nurse would explain what concerning narcotic analgesics? They are often ordered but not usually needed. When they are medically indicated, children rarely become addicted. They are given as a last resort because of the threat of addiction. They are used only if other measures, such as ice packs, are ineffective

When they are medically indicated, children rarely become addicted. Correct Pain is the most common and debilitating symptom experienced by patients with sickle cell disease. The chronic nature of this pain can greatly affect the child's development. A multidisciplinary approach is best for its management. Patient-controlled analgesia or continuous intravenous administration is usually effective. Pharmacologic intervention is necessary for the pain of sickle cell crisis.

Which factors are appropriate when determining the degree of neurologic dysfunction that occurs with spina bifida occulta? Where the sac protrudes through the vertebrae, the anatomic level of the defect, and the amount of neurologic tissue involved How much the sac protrudes through the vertebrae, the anatomic level of the defect, and the amount of muscle tissue involved Where the sac protrudes through the vertebra, the anatomic level of the defect, and the point in the gestation at which the defect occurred How much the sac protrudes through the vertebrae, the anatomic level of the defect, and the point in the gestation at which the defect occurred

Where the sac protrudes through the vertebrae, the anatomic level of the defect, and the amount of neurologic tissue involved Rationale The degree of neurologic dysfunction that occurs with spina bifida occulta depends on where the sac protrudes through the vertebrae, the anatomic level of the defect, and the amount of nerve tissue involved. How much the sac protrudes through the vertebrae does not affect the degree of neurologic dysfunction. The amount of neurologic tissue involved, rather than the point in the gestation at which the defect occurred, affects the degree of neurologic dysfunction that occurs.

An 8-year-old patient comes to the emergency department with a femur fracture. The health care provider has obtained x-rays of the fractured femur and the unaffected femur. The nurse knows that both x-rays need to be done for the patient for which reason? X-rays of both extremities allows for observation of bone growth. X-rays of both extremities is part of the hospital protocol for children. X-rays of both extremities allows for the comparison between the extremities. X-rays of both extremities allows the health care provider to observe inflammation.

X-rays of both extremities allows for the comparison between the extremities. A radiograph of the unaffected extremity may be obtained for comparison purposes, especially when trying to determine whether a line on the radiograph represents a fracture or merely an epiphyseal line.

is socialazation important or not?

YES

Which statement is appropriate when explaining the significance of white blood cells to a 10-year-old patient? They help in phagocytizing bacteria. White blood cells help keep germs from causing infections. They give immunity against bacterial infections. White blood cells carry oxygen from the lungs to all parts of the body.

White blood cells help keep germs from causing infections. Rationale It is important for a nurse to avoid using medical jargon when talking to children. She would explain medical terms in simple words that a child easily understands. Therefore the most appropriate response of the nurse is to say that white blood cells help keep germs from causing infections. Although it is correct that white blood cells help in phagocytizing bacteria and give immunity against bacterial infections, these sentences contain medical jargon that is difficult for a child to understand. It is the function of red blood cells, not white blood cells, to carry oxygen from the lungs to all parts of the body.

Infantile spinal muscular atrophy (SMA type 1) a. may be diagnosed prenatally by genetic analysis of circulating fetal cells in maternal blood or amniotic fluid b. can be cured with surgery performed in utero c. is associated with nutritional failure to thrive, the most serious complication, often leading to the infant;s early death d. is associated with children who are intellectually delayed

a. may be diagnosed prenatally by genetic analysis of circulating fetal cells in maternal blood or amniotic fluid

Commotio cordis: a. occurs after a blunt, non penetrating blow to the chest, which produces ventricular fibrillation. b. rarely causes death. c. occurs almost exclusively in athletes with hypertrophic cardiomyopathy. d. occurs in athletes who have a history of sudden death in a relative under the age of 50 years.

a. occurs after a blunt, non penetrating blow to the chest, which produces ventricular fibrillation.

Nursing care directed toward nonsurgical management in a teenager with scoliosis primarily includes: a. promoting self-esteem and positive body image b. preventing immobility c. promoting adequate hydration d. preventing infection

a. promoting self-esteem and positive body image

Johnny, a 12-year-old with fracture of the femur, has developed sudden chest pain and shortness of breath. The astute nurse suspects: a. pulmonary embolism. b. compartment syndrome. c. myocardial infarction. d. pneumonia

a. pulmonary embolism.

tentorium

a segment of dura that separates the cerebellum from the occipital lobe of the cerebrum

falx cerebri

a segment of the sheeet of dura that separates the cerebral hemispheres

falx cerebelli

a segment of the sheet of dura that separates the cerebellar hemispheres

russell traction

a sling is positioned under the knee which suspends the distal thigh above the bed two pulls - one longitudal, one perpendicular realignment of the lower extremity and immobilizes hip and knee in aflexed position Nurse must monitoring the position of ttraction and appropriate amount of hip flexion is maintained to prevent peroneal nerve damage and footdrop

Research has shown that maternal supplementation of folic acid can reduce the recurrence rates of spina bifida, anencephaly, or encephalocele. How should this supplement be administered? a. Daily folic acid dose of 4 mg beginning 1 month before conception and during the first trimester b. Daily folic acid dose of 0.4 mg as soon as pregnancy is confirmed c. Daily folic acid does of 4 mg given through the use of multivitamin preparations beginning 1 month before conception and throughout the first trimester d. Daily folic acid does of 4 mg beginning with the confirmation of pregnancy and continuing throughout pregnancy.

a. Daily folic acid dose of 4 mg beginning 1 month before conception and during the first trimester

Children with CP often have manifestations that include alterations of muscle tone. Which of the following is an example of a finding in a child with altered muscle tone? a. Demonstrates increased or decreased resistance to passive movements b. Develops hand dominance by the age of 5 months c. Has an asymmetric crawl d. When placed in prone position, maintains hips higher than trunk, with legs and arms flexed or drawn under the body

a. Demonstrates increased or decreased resistance to passive movements

Which of the following is a complication of immobility that is easily prevented with appropriate nursing intervention? a. Disuse atrophy & loss of muscle mass b. Constipation c. Hypocalcemia d. Pain

a. Disuse atrophy & loss of muscle mass

The nurse knows that the etiology of CP is most commonly related to which of the following? a. Existing prenatal brain abnormalities/injury b. Maternal asphyxia c. Childhood meningitis d. Preeclampsia

a. Existing prenatal brain abnormalities

Zac, a 16 year old football star at the local high school, is at the school nurse practitioner's office for acne that is not clearing with over the counter medications. During the physical examination, the nurse notes that Zac has achieved a marked increase in muscle and strength in a very short time. Which of the following would the nurse suspect caused these changes? a. Use of an ergogenic acid, anabolic steroids b. more frequent and more strenuous workouts in the gym c. increased protein and vitamins in the diet d. use of methylphenidate (Ritalin) or phenmetrazine (Preludin)

a. Use of an ergogenic acid, anabolic steroids

Osteomyelitis resulting from a blood-borne bacterium that could have developed from an infected lesion is termed; a. acute hematogenous osteomyelitis b. exogenous osteomyelitis c. subacute osteomyelitis d. chronic osteomyelitis

a. acute hematogenous osteomyelitis

Slipped capital femoral epiphysis is suspected when: a. an adolescent or preadolescent begins to limp and complains of continuous or intermittent pain in the hip. b. an examination reveals no restriction on internal rotation or adduction but restriction on external rotation. c. referred pain goes into the sacral and lumbar areas. d. all of the above occur.

a. an adolescent or preadolescent begins to limp and complains of continuous or intermittent pain in the hip.

The priority nursing consideration for the child in the acute phase of GBS is: a. careful observation for difficulty in swallowing and respiratory involvement b. prevention of contractures c. prevention of bowel and bladder complications d. prevention of sensory impairment

a. careful observation for difficulty in swallowing and respiratory involvement

Which of the following nursing goals is most appropriate for the child with juvenile idiopathic arthritis? a. child will exhibit signs of reduced joint inflammation and adequate joint function. b. child will exhibit no signs of impaired skin integrity due to rash. c. child will exhibit normal weight and nutritional status. d. child will exhibit no alteration in respiratory patterns or respiratory tract infection.

a. child will exhibit signs of reduced joint inflammation and adequate joint function.

cerebral blood flow, oxygen consumption, and brain growth are all: a. less in adults than in children b. greater in adults than in children c. greater in adukts than in infunts d. less in infants thann in children

a. less in adults than in children

Immediate treatment of sprains and strains include: a. rest and cold application b. disregarding the pain and "working out" the sprain or strain c. rest, elevation and pain medication d. compression of the area and heat application

a. rest and cold application

hydrocephalus surgical interventions

a. the goal of surgical treatment is to prevent further CSF accumulation by bypassing the blockage and draining the fluid from the ventricles to a location where it may be reabsorbed b. in a ventriculoperitoneal shunt, CSF drains into the peritoneal cavity from the lateral ventricle 3. in a ventriculoatrial shunt, CSF drains into the right atrium of the heart from the lateral ventricle, bypassing the obstruction (used in older children and in children with pathological conditions of the abdomen) 4. shunt revision may be necessary as the child grows 5. an alternative to shunt placement is endoscopic third ventriculostomy in which a small opening in the floor of the third ventricle is made that allows CSF to bypass the fourth ventricle and return to the circulation to be absorbed; this treatment may not be appropriate for some types of hydrocephalus.

nonbacterial (aseptic) meningitis/ treatment

acetominophen for headache and muscle pain maintenance of hydration positioning for comfort antimicrobial agents until definite diagnosis isolation enforced course is shorter without significant complications

What action should the nurse take in response to how they are handling the care of the cast if they lifting the cast with the palms of their hands

acknowledge that the parents have correctly learned how to move the cast while it is wet

Madison is going to a surgery in an hour she had last dose 3 hrs ago What action should the nurse implement?

administer another dose of morphine immediately

major priority of nursing care in child with meningitis

administer antibiotics as soon as they are ordered respiratory isolation for at least 24 hrs after initiation of antimicrobial therapy

Methotrexate

adverse effects - liver disease, bone marrow supression, GI disturbances, teratogenic effects, carcinogenesis improves uvelitis important to monitor liver enzymes levels and blood count daily folic acid supplement to reduce oral ulcers take med at night to reduce nausea AVOID PREGNANCY AND PREGNANT CAREGIVERS SHOULD BE WARNED alcohol increases risk of hepatotoxicity not to take methotrexone during some illnesses because it supresses immune response

decreased muscle tone in down syndrome

affects gastric motility predisposing to constipation increase fluid, fiber, prevent obesity

bone healing/stage 5/consolidation and remodeling

after 9 months - bone marrow cavity is restored -compact bone forms according to stress patterns - remodeling occurs according to Wolff's law - fracture line is always visible on radiograph

traction choice depends on

age condition of soft tissue type and degree of displacement of the fracture

Anemia caused by

an inadequate supply of dietary iron is the most prevalent nutritional disorder in the United States and the most common mineral disturbance. Iron-deficiency anemia can be caused by any number of factors that decrease the supply of iron, impair its absorption, increase the body's need for iron, or affect the synthesis of hemoglobin. Children 12 to 36 months of age are at risk for anemia because cow's milk is a staple of their diet

Headaches

are a common complaint of children and are associated with different pathologic conditions, including extracranial disease, intracranial disease, vascular abnormalities, psychogenic disorders, or a combination of these. Migraine headaches occur in children as well as adults. Typical symptoms include nausea, vomiting, and abdominal pain, which are relieved by sleep. Migraine headaches are managed with general measures (education, a headache diary to identify and eliminate precipitating factors, and documented response to treatment), abortive treatment, and prophylactic treatment. At the onset of the headache, the child should rest or sleep in a quiet, dark room when feasible. Migraine therapy, if administered early in the course of the headache, may provide rapid relief. Acetaminophen or ibuprofen is often effective if given early.

The clinical manifestations of iron-deficiency anemia

are directly attributable to the reduction in the amount of oxygen available to the tissues and resemble those seen in any type of anemia. Usually the signs are insidious and obscure, and the severity is directly related to the duration of the dietary deficiency.

The goals of therapy for children with CP

are early recognition and promotion of an optimum developmental course to enable affected children to attain their potential within the limits of their dysfunction. The broad aims of therapy are to (1) establish locomotion, communication, and self-help, (2) gain optimum appearance and integration of motor functions, (3) correct associated defects as early and effectively as possible, (4) provide educational opportunities adapted to the individual child's needs and capabilities, and (5) promote socialization experiences with other affected and unaffected children.

Acquired hip deformities are managed

are managed with non-weight-bearing devices (for Legg-Calvé-Perthes disease) or surgical stabilization (for treatment of slipped capital femoral epiphysis). Legg-Calvé-Perthes disease is a self-limiting disorder in which there is aseptic necrosis of the femoral head. The aims of treatment are to eliminate hip irritability; restore and maintain adequate range of hip motion; prevent capital femoral epiphyseal collapse, extrusion, or subluxation; and ensure a well-rounded femoral head at the time of healing.

Causes of severe C

are primarily genetic, biochemical, and infectious. Whereas mild impairment is associated primarily with familial, social, and environmental causes, severe disability is more likely to be associated with specific syndromes

Muscular dystrophies

are the greatest and most important cause of neuromuscular dysfunction of childhood. Duchenne muscular dystrophy is the most severe and the most common muscular dystrophy of childhood. It is inherited as an X-linked recessive trait, and the single-gene defect is located on the short arm of the X chromosome. Major complications of Duchenne muscular dystrophy include joint contractures, disuse atrophy, obesity, cardiomyopathy, sleep-disordered breathing, and respiratory failure as the condition progresses. Children with neuromuscular disease require careful monitoring of respiratory function and prompt interventions to prevent hypoxia, hypercarbia, and eventual respiratory failure.

emergency treatment of fracture

assess 5 Ps ( pain, pulselessnesss, pallor, parethsesia, paralysis) - determine mechanism of injury - move injured part as little as possible - cover open wound with sterile or clean dressing - immobiliza the limb with the splint - reassess neurovascular status - apply manual traction if circulatory compromise if present - elevate the injured limb is possible - apply cold to the injured area ( no longer than 20 min each time) - call emergency services

abnormal involuntary muscular movements

ataxia - gross incoordination that may become worse with the eyes closed spasm - involuntary contraction of muscles spasticity - prolonged and steady contractions of a muscle, charaterized by clonus ( altertion relaxation and contraction of the muscle and exagerrated reflexes rigidity - inability to flex or extend a joint tremors - constant small involuntary movements twitching - spasmotic moevments of short duration tic - involuntary, compulsive, stereotyped movement of an associated group of muscles choreiform movements - quick, jerky, grossly uncoordinated, irregular movements that may disappear on relaxation athetosis - slow writhing, wormlike, constant grossly uncoordinated movements that increase on voluntary activity and decrease on relaxation dystonia - slow twisting movements of limbs and trunks associated movements - voluntary movement of one muscle accompanied by involuntary movement of another muscle mirrowing movements - same as associated movements axcept with symmetric muscle group

abnormalities of gait that indicate cerebral dysfunction

ataxia - impaired ability to coordinate moevemnts, staggering gait and postural imbalance spastic paraplegic gait - narroe based gait with a tendency to walk on toes, along with flexion of knees and hips and shuffling, scissoring position spastic hemiplegic gait - leg extended, circumducted, plantar flexion, affected arm flexed and doesn't swing extrapyramidal gait - rigidity, few automatic movements, sloweness of movements

Nurses are positioned to help parents cope with their child's chronic illness or disability in many ways. Nurses must be

attentive, offer humanistic support, solicit suggestions for care, facilitate communication, allow verbalization of feelings, and refer to volunteer and community agencies.

Marilyn, age 12, has been diagnosed with scoliosis and placed in a thoracolumbosacral orthotic (TLSO) brace. Which of the following information provided by the nurse to Marilyn is correct? a. "The brace will cure your curvature." b. "The brace is an underarm brace made of plastic that will be molded and shaped to your body to correct the curvature." c. "The brace includes a neck ring to extend the neck." d. The brace will only be worn in bed, since it prevents walking because of the severity of the trunk bend."

b. "The brace is an underarm brace made of plastic that will be molded and shaped to your body to correct the curvature."

Approach behaviors are coping mechanisms that result in a family's movement toward adjustment and resolution of the crisis of having a child with a chronic illness or disability. What is considered an approach behavior in parents? a. Are unable to adjust to a progression of the disease or condition b. Anticipate future problems and seek guidance and answers c. Look for new cures without a perspective toward possible benefit d. Fail to recognize seriousness of child's condition despite physical evidence

b. Anticipate future problems and seek guidance and answers

A nurse discovers 5-year-old Jimmy, a neighbor, lying in the street next to his bicycle. The nurse sends another witness to activate the Emergency Medical Services (EMS) while the nurse begins a primary assessment of Jimmy. Which one of the following best describes the primary assessment and its correct sequence? a. Body inspection, level of consciousness, head-to-toe survey, and airway patency b. Level of consciousness, airway patency, respiratory effectiveness, circulatory status c. Open airway, head-to-toe assessment for injuries, and chest compressions d. Weight estimation, symptom analysis, blood pressure measurement

b. Level of consciousness, airway patency, respiratory effectiveness, circulatory status

The nurse is caring for 7-year-old Charles after application of skeletal traction. Which of the following is contradicted? a. clean pin sites per hospital policy b. Release the traction when repositioning Charles in bed. c. Inspect pin sites for bleeding or infection. d. Assess for alterations in neurovascular status.

b. Release the traction when repositioning Charles in bed

The nurse is assessing Carol, age 8, for complications relating to her recent fracture and application of a fiberglass cast to her forearm and elbow. Carol is crying with pain, the nurse is unable to locate pulses in the affected extremity, and there is lack of sensitivity to the area as well as some edema. Which of the following would the nurse suspect as most likely to be occurring? a. Normal occurrence for the first few hours after application of traction. b. Volkmann contracture. c. Nerve compression syndrome. d. Epiphyseal damage.

b. Volkmann contracture.

In discussing sexuality with the teenager who has a spinal injury, the nurse correctly includes which of the following in the discussion? a. Development of secondary sexual will be delayed b. Well-motivated young people can look forward to successful participation in marital and family activities c. If injury occurs before onset of menstruation, ovulation and conception are not possible. d. Females can easily experience vaginal or clitoral orgasms

b. Well-motivated young people can look forward to successful participation in marital and family activities

The care plan for the child during the acute phase of osteomyelitis always includes: a. performing wound irrigations b. ensuring administration of antibiotics c. isolating the child d. incorporating passive range of motion exercises for the affected area

b. ensuring administration of antibiotics

The major anomaly most frequently associated with myelomeningoocele is: a. Chiari malformation b. hydrocephalus c. clubfoot d. developmental dysplasia of the hips

b. hydrocephalus

Diagnostic evaluation of the child who is seen with a spinal cord injury includes a complete neurologic examination. Motor system evaluation is done by: a. stimulating peripheral receptors by eliciting reflex such as the patellar b. observing gait if the child is able to walk; noting balance maintenance; and assessing the ability to life, flex, and extend extremities c. testing all 12 cranial nerves d. using the blunt end of a safety pin and the sharp point to test each dermatome

b. observing gait if the child is able to walk; noting balance maintenance; and assessing the ability to life, flex, and extend extremities

The goal of therapeutic management for the child with CP is: a. assisting with motor control of voluntary muscle b. promoting an optimal developmental course to enable children to achieve their maximal potential c. delaying the development of sensory deprivation d. surgically correcting deformities

b. promoting an optimal developmental course to enable children to achieve their maximal potential

Nursing considerations for the patient diagnosed with osteogenesis imperfecta include: a. preventing fractures by holding onto the child's ankle when changing diapers. b. providing nonjudgemental support while parents may be dealing with accusations of child abuse. c. providing guidelines to the parents in avoiding all exercise and sports for the child. d. educating parents that the use of braces and splints can increase the rate of fracture.

b. providing nonjudgemental support while parents may be dealing with accusations of child abuse.

Which of the following statements made to the athlete by the nurse is correct? a. it is more important to replace sodium and chloride than water. b. recommended dietary energy intake for adolescents involved in sports is 50% of caloric intake from carbohydrates. c. iron replacement is necessary only for the female athlete. d. energy for prolonged exercise is best obtained from high-carbohydrate foods eaten 2 hours before the event.

b. recommended dietary energy intake for adolescents involved in sports is 50% of caloric intake from carbohydrates.

Therapeutic management that provides the most favorable morbidity and mortality outcomes for the child born with myelomeningocele is: a. early physical therapy b. surgical closure of the defect within the first 24 to 72 hours c. vigorous antibiotic therapy d. splint application to lower extremities

b. surgical closure of the defect within the first 24 to 72 hours

When an older child is sitting on a booster seat, where is the best place to locate the seat?

back seat of the car with lap and shoulder belts.

Which of the following is a true statement about Guillian-Barre syndrome (GBS)? a. GBS is an autosomal recessive inherited disease b. GBS is more likely to affect children than adults, with children under the age of 4 years having the higher susceptibility c. GBS is an acute demyelinating polyneuropathy with a progressive, usually ascending, flaccid paralysis d. GBS is an autoimmune disorder associated with the attack of circulating antibodies on the acetylcholine receptors.

c. GBS is an acute demyelinating polyneuropathy with a progressive, usually ascending, flaccid paralysis

Ben, a 15 year old high school student, is at a track event. He has been running multiple events. He was feeling unwell before the event and had been vomiting. Now he is complaining of thirst, headache, fatigue, dizziness, and nausea. He seems to be somewhat disoriented and is sweating. Ben's temperature is normal. Which of the following is the most likely to describe Ben's condition? a. Heat cramps b. Dehydration c. Heat exhaustion d. Heatstroke

c. Heat exhaustion

Which one of the following is not included in the teaching plan of a child with a brace or prosthesis? a. Frequent assessment of all areas in contact with a brace for signs of skin irritation b. Assessment of the stump area before application of the prosthesis c. Removal of the prosthesis limited to bedtime unless skin breakage occurs d. Use of protective clothing under the brace

c. Removal of the prosthesis limited to bedtime unless skin breakage occurs

The disease inherited only as an autosomal recessive trait and characterized by progressive weakness and wasting of skeletal muscles caused by degeneration of anterior horn cells is: a. cerebral palsy b. Kugelberg-Welander disease c. infantile SMA type 1 (Werdnig-Hoffmann disease) d. Guillian-Barre syndrome (GBS)

c. infantile SMA type 1 (Werdnig-Hoffmann disease)

Children with spina bifida who are confined to a wheelchair are at increased risk for: a. tethered cord syndrome b. Chiari malformation c. skin breakdown d. orthopedic deformities

c. skin breakdown

physical characteristics of kid with down syndrome

can cause protruding tongue, low-set ears, short stature almond eyes

main complication of anemia

cardiac decompensation fromexcessive demands on the heart due to increased metabolic needs or cardiac overload symptoms of HF - tachycardia, dyspnea, rales, moist respiration, cough, sweating

Care guide

care of client t- temperature extremity or infection r- ropes hang freely a- alignment c- circulation check (5 P's) t- type and location of fracture i- increases fluid intake o- overhead trapeze n- no weights on bed or floor

Altered Pituitary Secretion—SIADH

cerebral dysfunction - often with hyperthermia, so reduce temp ( antipyuretics), cooling devices elimination - urinary catheter or weighted diapers for long term care, intermitent catherization, stool softeners, record daily bowel activity maintain skin integrity - matress, lotion massage, change position, inspect area, clothing dry and free of wrincles oral care twice daily eyes examined and artificial tears are placed The unconscious child is positioned to minimize ICP and to prevent aspiration of saliva, nasogastric secretions, and vomitus. The head of the bed is elevated, and the child is placed in a side-lying or semiprone position. A small, firm pillow is placed under the head, and the uppermost limbs are flexed and supported with pillows. gentle ROM, tactile when tolerated = held and touched

clinical manifestations of bacterial meningitis/neonates/specific signs

child well at birth but within a few days begins to look and behave poorly - refuses feeding - poor sucking ability - vomiting or diarrhea - poor tone - lack of movement - weak cry - full, tense, and bulging fontanel may appear late in course - neck usually supple

fracture complications

circulatory impairment nerve compression syndrome compartment syndrome physeal damage nonunion malunion infection kidney stones pulmonary emboli

hydrocephalus types

communicating a.) hydrocephalus occurs as a result of impaired absorption within the subarachnoid space. b. interference of the cerebrospinal fluid in the ventricular system does not occur noncommunicating: obstruction of cerebrospinal fluid flow in the ventricular system does occur

nursing care management/ cast

complete water evaporation from cast 24-48 hrs cast must remain uncovered, suppored on pilllow turn child every 2 hrs use hairdryer on cool setting avoid swelling - elevate body part, observe use specially designed car seats bath only if cast out of water child may have problem with self-feeding - prone position with small table, or semisitting position on a wheelchail they cannot use toilet - bedpen use, waterproofing bed, protective skin barrier

brainstem

connected to the hemispheres by thick bunches of nerve fibers; all nerve fibers traverse through this structure as they pass from hemispheres to the cerebellum and the spinal cord; extends from the base of the hemispheres through the foramen magnum, where it is continuous with the spinal cord

The syndrome known as female athlete triad

consists of amenorrhea, osteoporosis, and decreased energy availability. The phenomenon has been attributed to a complex interplay of physical, genetic, hormonal, nutritional, psychologic, and environmental factors that include the stress of competition, decreased protein consumption, and altered lean-to-fat body ratio.

Abnormalities that come from the embryonic neural tube, neural tube defects,

constitute the largest group of congenital anomalies with multifactorial inheritance. The most common are spina bifida and myelomeningocele. Myelomeningocele involves the protrusion of a sac in the lumbar or lumbosacral area that contains meninges, spinal fluid, and nerves. Care of the infants and children with myelomeningocele is directed toward protecting the meningeal sac, preventing infection and skin breakdown, observing for signs of urologic and bowel complications, promoting early parent-infant interaction, and planning appropriate interventions to optimize the child's development. Associated problems include infection, impaired genitourinary and bowel function, musculoskeletal impairment, and latex allergy.

which of the following statements about "nursemaid's elbow" is correct a. the most common partial dislocation of the radial head of the elbow is usually found in children ages 1 to 4 years old b. this condition is caused by a sudden pull of the wrist whilet hte arm is fully extended and the forearm is pronated c. the longer the dislocation is present, the longer it takes the child to recover mobility after treatment d. all of the above statements are correct

d. all of the above statements are correct

Bob, age 7, is diagnosed with Legg-Calve-Perthes disease. Which of the following manifestations is not consistent with this diagnosis? a. intermittent appearance of a limp on the affected side. b. hip soreness, ache, or stiffness that can be constant or intermittent. c. pain and limp most evident on arising and at the end of a long day of activities. d. specific history of injury to the area

d. specific history of injury to the area

symptoms and signs of anemia

decreased red blood cell production - pallor, tachycardia, fatigue, headache, muscle weakness, systolic heart murmur, frontal bossing INCREASED RED BLOOD CELL DESTRUCTION - jaundice, fatigue, headache, tachycardia, dark urine, splenomegaly, hepatomegaly, low blood pressure INCREASED RED BLOOD CELL LOSS - pallor, fatigue, headache, muscle weakness, cool skin, tachycardia, decreased peripheral pulses, low blood pressure

passive anger

decreased visiting refusal to beleive how sickthe child is inability provide comfort

Which technique is most beneficial when preparing a preschooler for a procedure that may be frightening?

describe what the child will experience shortly before the procedure takes place

The broad term developmental dysplasia of the hip (DDH)

describes a spectrum of disorders related to abnormal development of the hip at any time during fetal life, infancy, or childhood. DDH appears to be related to intrauterine, genetic, and postnatal factors. Subluxation and the tendency to dislocate can be demonstrated by the Ortolani and Barlow tests; such tests should be performed only by an experienced clinician to prevent damage to the hip. Treatment of DDH involves maintaining the head of the femur correctly positioned in the acetabulum by means of an external device, usually the Pavlik harness. Older children may require treatment with a hip spica cast, traction, or closed surgical reduction with casting.

adjustment process culmination

development of realistic expectations reintegration of family life with the illness resolution is not possible until child dies or leaves home as independent adult includes social intgration chronic sorrow experience

hypotonicity of muscles in down syndrome

difficulty positioning lack of molding to parent's bodies due to flaccicity wrap in snuggly blanket

how cerebral palsy affects the child

difficulty with gross motor movement, can need braces to walk affects mitility motor developmental delay can cause seizures problems with cognition, speech, fluid in the brain Delayed gross motor development, abnormal motor performance, alterations of muscle tone, abnormal posture, reflex abnormalities, associated disabilities.

early signs suggestive of cognitive impairment

dysmorphic syndroms 2. irritability or nonresponsiveness to environment 3. major organ system dysfynction 4. gross motor delay 5. fine motor delay 6. language difficulties or delay 7. behavior difficulties

bacterial meningitis patients outcomes

early recognition of signs and symptoms 2. antibiotics administered as sson asdiagnosis is established 3. cerebral edema is prevented 4. exposure prevented by early isolation 5. side effects managed 6. neurologic sequelae prevented

Promoting the family's adaptation to the day-to-day management of the child's condition involves

education about the child's condition, general health care, and developmental needs and about realistic goal setting.

normalization

effort family members make to create a normal family life, their perception of the consequences of theses efforts such as attending school, pursuing hobbies and recreational interests, achieving employment and a level of independence

nutrition during bone healing

eleminate soda, because phosphoric acid interferes with calcium absorption

Education of children with intellectual disability

emphasizes sensory and verbal discrimination, improvement of short-term memory, motivation, and technologic support. E

substance use in athletes

ergogenic aids - increase strength and indurance, increase ability to concentrate, decrease sensetivity to pain

bracing and exercise in scoliosis

for moderate curves 25-45 degrees it is not curative, butto slow the progression boston and wilmington braces, milwaukee brace is rarely, charlston night time only when child is in bed because it prevents walking

malunion

fracture union with increased angulation or deformity at the fracture site reason - unsatisfactory reduction, cast or splint that allows fracture movement exsessive deformity - needs reallignment and reimmobilization nothing done - if degree of deformity, probability of spontaneous alignemnt correction is done when healing is almost finished

underdeveloped nasal bone

leads to chronic inadequate drainage of mucus constantly stuffy nose forces child to breathe by mouth, which dries the oropharyngial membranes, leading to upper respiratory tract infection clear nose with bulb syringe, rinse mouth with water after feeding, increase fluid intake, cool mist voporized to keep membranes moist hand hygiene, change position of child often, proper disposal of foiled articles, full course of antibiotics

children's fractures

fractures not observed in adults include the presence of growth plates, thicker and stronger periosteum, bone porosity, more rapid healing, and less joint stiffness. The five Ps of ischemia from a vascular injury that should be included in the assessment are pain, pallor, pulselessness, paresthesia, and paralysis. The goals of fracture management are to regain alignment and length of bony fragments, retain alignment and length, and restore function to injured parts.

consciousness levels

full consciousness awake and alert, oriented time, place and person; behavior appropriate for age confusion impaired decision making disorientation confusion regarding time, place; decreased level of consciousness lethargy limited spontaneous movement, sluggish speech, drowsiness obtundation arousable with stimulation stupor remaining in a deep sleep, responsive only to vigorous and repeated stimulation coma no motor or verbal response to noxious (painful) stimuli persistent vegetative state (PVS) permanently lost function of the cerebral cortex. Eyes follow objects only be reflex or when attracted to the direction of loud sounds; all four limbs are spastic but can withdraw from painful stimuli; hands show reflexive grasping and groping; the face can grimace, some foods may be swallowed, and the child may groan or cry but utter no words

if immobilized child ask crackers What action should the nurse take?

give Madison a few crackers and stay with her while she eats them

thrust maneuver to open airway

grasp angles of the victim's lower jaw and lifting with both hands, one on each side, and displacing the mandible upward and outward without tilting the head

Immobility consequences

has a profound effect on all aspects of a child's growth and development. Because of the recognized problems with immobility, efforts are directed toward early ambulation. The major physical consequences of prolonged immobilization are loss of muscle strength, endurance, and muscle mass; bone demineralization; loss of joint mobility; and contractures. Secondary effects include alterations in the skin, cardiovascular, respiratory, metabolic, and renal systems. The three major cardiovascular consequences of immobility are orthostatic intolerance, increased workload of the heart, and thrombus formation. Deep vein thrombosis (DVT) involves the formation of a thrombus in a deep vein such as the iliac and femoral veins and can cause significant morbidity if it remains undetected and untreated. Prolonged immobility, friction, moisture, and impaired circulation also place children at greater risk for tissue ischemia and pressure ulcer development.

epitaxis emergency treatment

have child sit with neck forward or erect ( lo lie down) apply continuous pressure to tip of nose with thumb and forefinger for 10 min do not insert anything in nose ( tissue) or blow nose, because may dislodge the clot apply ice or cold cloth to bridge keep child calm and quite

Dislocation DDH

head loses contact with acetabulum and is displaced posteriorly and superiorly. ligamentum teres is elongated and taut

Neurologic physical examination

includes obsevation of the size and shae of the head, spontaneous activity, postural reflex activity, sensory responses, symmetry of movement

complications of ventriculoperitoneal shunt

infection malfunction - mechanical obstruction ( kinking, plugging, separation, migration of tubing) common complication - functional obstructionof a shunt's antisiphon device signs of shunt obstruction - increasd ICP, nausea, vomiting, irritability, bulging fontanle, worsening neurologic status most commn and serious complication - shunt infection - sepsis, bacterial endocarditis, wound infection, shunt nephritis, meningitis, ventriculitis, brain absess Tx - antibiotics IV or intracathetal, so use antibiotics imregnanted shunts another complication - subdural hematoma - too rapid reduction of ICP other complications - peritonitis, abdominal absesses, perforation of abdominal organs bu catheter, fistulas, hernias,

Bone healing sequence

inflammatory restorative remodelling

How should the nurse respond if pafrent say that she uses adult seat for a 5 yo child and 50 lb

instruct this parent that the child's age and size still require the use of a safety seat

intellectual disability components

intellectual functioning (measured by the intellegence quotient) adaptive behavior (inabilty to reason, plan, solve problems, think abstructly, comprehend complex ideas, learn from experience that is age appropriate, unable t meet standards for culturally appropriate demands of daily life) age younger than 18 at the time of diagnosis

inadequate drainage of mucus in down syndrome

interferes with feeding child may gag on solid clear nose before feeding, give small frequent feedings, allow rest during meals

Nursing care of a child with hemophilia

involves preventing bleeding by decreasing the risk of injury, recognizing and managing bleeding with factor replacement, preventing the disabling effects of joint degeneration, and preparing and supporting the child and family for home care.

Congenital clubfoo

is a complex deformity of the ankle and foot that includes forefoot adduction, midfoot supination, hindfoot varus, and ankle equinus. Treatment of clubfoot involves manual overcorrection of the deformity, maintenance of the correction until normal muscle balance is gained, and follow-up observation to detect possible recurrence of the deformity. To accomplish these goals, serial casting, manipulation, and a Denis Browne splint are used. Failure to achieve normal alignment by 3 months may require surgical correction and casting. Treatment of clubfoot consists of manipulation and serial casting to correct the deformity, maintenance of the correction, surgical heel cord release, and prevention of possible recurrence of the deformity.

what is the earliest indicator of change in neurologic status

level of consciousness, change in behavior that is often simple, happy/cooperative to grumpy and combative

Idiopathic thrombocytopenic purpura (ITP)

is an acquired hemorrhagic disorder characterized by (1) excessive destruction of platelets (thrombocytopenia), (2) purpura (discoloration caused by petechiae beneath the skin), and (3) normal bone marrow with a usual increase in large, young platelets. Management of ITP is primarily supportive because the disease is usually self-limiting. Activity is restricted at the onset while the platelet count is low and while active bleeding or progression of lesions is occurring. Treatment for acute presentation is symptomatic and has included prednisone, IV immune globulin (IVIG), and anti-D antibody. These are not curative therapies. Some experts suggest that no therapy is necessary for asymptomatic patients because the recovery time of platelet counts is the same with or without treatment. Nursing care is largely supportive and includes teaching on the possible side effects of therapy and limitation in activities while the child's platelet count is lower than 100,000/mm3. Children with ITP should not participate in any contact sports, bike riding, skateboarding, in-line skating, gymnastics, climbing, or running.

Bacterial meningitis

is an acute inflammation of the meninges and cerebrospinal fluid. Acute bacterial meningitis is a medical emergency that requires early recognition and immediate therapy to prevent death and avoid residual disabilities. The child is isolated from other children, usually in an intensive care unit for close observation. An intravenous (IV) infusion is started to facilitate administration of antimicrobial agents, fluids, antiepileptic drugs, and blood, if needed. The child is placed in respiratory isolation. Many different viruses cause aseptic meningitis. The onset may be abrupt or gradual. The initial manifestations are headache, fever, photophobia, and nuchal rigidity. Diagnosis is based on clinical features and cerebral spinal fluid findings. Treatment is primarily symptomatic.

The most characteristic clinical feature of vWD

is an increased tendency toward bleeding from mucous membranes. The most common symptom is frequent nosebleeds followed by gingival bleeding, easy bruising, and excessive menstrual bleeding (menorrhagia) in women. Unlike hemophilia, vWD affects both males and females because its inheritance shows an autosomal dominant pattern. However, the treatment and final outcome are similar in both disorders. Treatment of bleeding is with 1-deamino-8-d-arginine vasopressin (DDAVP) or a specially concentrated clotting factor known as Humate-P (or both)

Osteomyelitis

is an infectious process involving the bone which may occur at any age but most frequently is seen in children 10 years of age or younger. Staphylococcus aureus is the most common causative organism. In children, severe pain, fever, irritability, and tenderness with or without local signs of inflammation suggest osteomyelitis. The treatment involves administration of antibiotics.

Juvenile idiopathic arthritis (JIA)

is chronic childhood arthritis. A group of heterogeneous chronic autoimmune diseases, JIA causes inflammation in the synovium, joints, and surrounding tissue. The cause of JIA is unknown. The major goals of therapy are to control pain, preserve joint range of motion and function, minimize the effects of inflammation such as joint deformity, and promote normal growth and development.

Anemia

is defined as reduction of red blood cells (RBCs) or hemoglobin concentration to levels below normal for age. Anemias are classified in relation to (1) etiology or physiology, manifested by erythrocyte or hemoglobin depletion, and (2) morphology, the characteristic changes in RBC size, shape, or color. The basic physiologic defect caused by anemia is a decrease in the oxygen-carrying capacity of blood and consequently a reduction in the amount of oxygen available to the cells.

cerebral edema

is expected after craniocerebral trauma. Cerebral edema peaks at 24 to 72 hr after injury and may account for changes in a child's neurologic status. Cerebral edema associated with traumatic brain injury may be a result of two different mechanisms, cytotoxic edema or vasogenic edema.

The main nursing goal in prevention of nutritional anemia

is parent education regarding correct feeding practices. Use only breast milk or iron-fortified formula (containing 7 to 12 mg/l for full-term infants and 15 mg/l for preterm infants of iron) for the first 12 months. Provide iron supplementation of 1 mg/kg/day by 4 to 6 months of age in full-term infants and 2 mg/kg/day by 2 months of age in preterm infants. Administer iron drops at a dosage of 2 to 3 mg/kg/day to a maximum of 15 mg/day of elemental iron to breastfed preterm infants after 2 months of age and give iron-fortified infant cereal when solid foods are introduced. Limit the amount of formula to no more than 1 l/day to encourage intake of iron-rich solid foods.

Osteogenesis imperfecta (OI)

is the most common osteoporosis syndrome in children, characterized by excessive fractures and bone deformity. Clinical features include varying degrees of bone fragility, deformity, and fracture; blue sclerae; hearing loss; and dentinogenesis imperfecta (hypoplastic discolored teeth). The treatment for OI is primarily supportive. Bisphosphonate therapy with pamidronate, olpadronate, neridronate, or alendronate to promote increased bone density and prevent fractures has become standard therapy for many children with OI.

Level of consciousness

is the most important indicator of neurologic health. Various levels include full consciousness, confusion, disorientation, lethargy, obtundation, stupor, coma, and persistent vegetative state. An altered state of consciousness may be the outcome of several processes that affect the central nervous system (CNS). Impaired neurologic function can result from a direct or indirect cause. Some altered states, such as the diffuse changes observed in encephalitis, are directly related to cerebral insult. Others are the result of dysfunction in other organs or processes. For example, biochemical changes can impair neurologic function without morphologic findings, as in hypoglycemia.

Respiratory effectiveness

is the primary concern in the care of an unconscious child, and establishment of an adequate airway is always the first priority. Carbon dioxide has a potent vasodilating effect and will increase cerebral blood flow and intracranial pressure (ICP). Cerebral hypoxia at normal body temperature that lasts longer than 4 minutes nearly always causes irreversible brain damage.

The nurse's role in treatment of anemia

is to assist in establishing a diagnosis, prepare the child for laboratory tests, administer prescribed medications, decrease tissue oxygen needs, implement safety precautions, and observe for complications.

postoperative care/scoliosis

log rolled to prevent damage sometimes immobilization skin care pressure releiving matress assessment of wound, circulation, vital signs, neurologic status, patient's extremities (watch for paralysis), adequate urinary output, fluid and electrolyte balance common problems - hypotention from blood loss, wound infection, atelectasis, pneumothorax ileus, delayed neurologic injury, implanted hardware complication, inappropriate antidiuretic hormone opioid on a regular basis, for older children PCA walking asap (2-3 day)

Nursing care of DDH

maintenance of the device adaptation nurturing activities to meet child's needs teaching parents ( main goal) to apply and maintain the reduction device or brace skin care ( check frequently (2-3 times a day) for red areas under straps; gently massage healthy skin under straps once a day to stimulate circulation; avoid lotion and powders; always place diaper under the straps not removed for daycare and bathing the difficulties for parents - difficulty reapplying harness, carrying the child, skin crease dermatitis, feet slipping out of harness, difficulty in clothing

nursing care management DMD

major role - 1. to assist the child and family in coping with the progressive, incapacitating and fatsl nature of the disease 2. to help design a programmthat will afford a greater degree of independence and reduce the predictable and preventabe disabilities associated with the disorder 3. help the child and family deal constructively with the limitations the disease imposes on the daily life 4. assist the family in developing the child's self-help skills 5. help parents to develop balance between limiting activity and allowing the child to accomplish things alone 6. assist with adjusting housing, transportation, mobility, diet, home care, social activities 7. help with psychiatric, psychologic counseling, involving in support groups

shock and senial

met initial diagnosis , defence mechanism, a cushing to prevent disintergation and normal response lasts days to months and longer 1. physical shopping 2. attributing the symptoms of the actual illness to a minor condition 3. refusing to beleive the diagnostic test 4. delaying consent for treatment 5. acting happy and optimistic despite the diagnosis 6. refusing to talk or tell anyone about condition 7. insisting that noone is telling the truth, regardless of other's attempts to do so 8. denying the reson for admission 9. asking no questions about the diagnosis, treatment, prognosis

clinical classification of cerebral palsy

motor abnormalities associated impairement - seizures, hearing, vision, behavioral, communicative, motor and speech function anatomic and radiologic findings - white matter lesion or brain anomaly noted on ct or mri causation and timing - identification of clearly identified cause such as postnatal event (meningitis, traumatic brain injury)

treatments, goals for CP

muscle strength PT learn ADL to keep the functioning as long as possble the goal is to give them as notmal life as possible

Tuberculous meningitis

must be considered in children who travelled, lived in developing countries most likely very young and immunosuppressed children ischemic infarction is complication clinical findings - fever, altered consciousnesss, cranial nerve involvement, seizures, focal neurologic deficit early diagnosis prevents disability from hydrocephalus nursing care - administration of medications, support of the child, control of pain, neurologic monitorig

Leading causes of injury to children

nclude falls, being struck by or against an object, motor vehicle accidents, fires, drowning, pedestrian-vehicle accidents, and firearms. Falls are the leading cause of nonfatal injury among children age 0 to 15 years. Trauma from motor vehicle crashes is the leading cause of nonfatal injury in adolescents ages 15 to 19 years of age.

Treatment of Duchenne Muscular Dystrophy

no cures corticosteroids for muscle bulk and power (prednisone) side effects - weight gain and cushingoid facial appearance primary goal - to maintain optimum function in all muscles for as long as possible secondary - prevent contractures

respiratory care for DMD

noninvasive ventilation mechanically assisted coughing tracheostomy couging and suctioning devices during sleep - pulse oxymetry manual cough-assisting techniques - glossopharengyal breathing or air stacking ( frog breathing); abdominal thrust, manual hyperinflation with a self-inflating rescucitating bag without oxygen

plastic deformation

occurs when the bone is bent but not broken

nonbacterial (aseptic) meningitis

onset of memingeal symptoms, fever abd pleocytosis without bacterial growth from CSF cultures cause by viruses, including arbovirus, enterovirus, herpes simplex, cytomegalovirus, HIV abrupt onset symptoms - headache, fever, photophobia, nuchal rigidity

Meromelia

partial absence of a limb

assessment of headache

pattern - acute, acute recurrent, chronic progressive, chronic nonprogressive presence - seizures, ataxia, lethergy, nausea, vomiting, weakness, personality chnages

promoting normalization

preparation - prepare child in advance for changes that may occur from the chronic or complex condition ( side effects of therapy) PARTICIPATION - include child in as many decisions as possible, especially those relating to his or her care regimen (responsible for taking meds) SHARING - allow bioth family emmbers and child's peers to be a part of the care regimen whenever possible ( give child meds, when other child receives vitamins) CONTROL - identify areas where child can be in control so that feelings of uncertainty, passivity and helplessness are decreased ( the child identifies activities that are appropriate to his or her energy level and chooses to rest when fatigued) EXPECTATIONS - apply the same family rules to the child with complex chronic illness as to the well sibling ( has to fulfill household responsibilities)

nurses's role in childrens sport

preparation and evaluation of children for activities counseling about nutrition prevention of injuries treatment of injuries rehabilitation after injuries teachin parents about injuries and treatment

buckle fracture (torus fracture)

produced by compression of the porous bone Buckle/torus: Raised or bulging projection R/T compression of a super porous part of a bone

preoperative care/scoliosis

radiographic images, including bending or traction spine films - pulmonary function studies - serologic labs - prothrombin, partial thromboplastin, blood count, electrolyte levels, urinalysis, urine culture, blood levels of med collect blood from patient before surgery - erythropoetic administration monitor blood pressure - preoperative teaching -bring personal stuff for after surgery - may be meet with peers in similar situation

symptoms of ischemia

reactive hyperemia, increase in temp, blistering, swelling, dark purple or black areas

goals of fracture managemrnt

reestablish alignment and length of the bony fragments 2. to retain alignment and length 3. to restore function to the injured part 4. to prevent further injury and deformity

A family-centered approach to care of the child and family facing life-threatening illness or death

respects the central role of the family and emphasizes communication, collaboration, and cultural sensitivity.

diseases of anterior horn cells

result of disruption or atrophy of the anterior horn of the spinal column along with inability to transfer impulses from sensory neurons to motor neurons

home care

return to a system and set of priorities in which family values are as important in the care of a child with a chronic health problem as they are in the care of other children.

care of prothesis

routinely wash the socket with water and mild soap, rinse, dry - check straps and rubber bands with each application - check joints to ensure that they operate smoothely - replace worn or brokwn parts use 100% cotton stump socks to absorb perspiration, prevent skin friction, provide comfort - change socks daily, wash dry according to the insrructions

scheuermann kyphosis

scheuermann kyphosis - thoracic curve of greater than 45 degrees with wedging of more than 5 degrees of at least three adjacent vertebral bodies and vertebral irregularity onset 10-12yo pain along with visible kyphosis rigidity and lack of spine extension flexibility tx - physical therpay to increase spine felxibility and strength

skin care of TLSO

skin at the edges should be padded and inspected red marks that don't face within 20 min shoudl be reported child should wear cotton undershirt under braces keep area clean dry and free of wrinckles - clean braces on weekly basis, water and soap, throughly dry - avoid leaving in hot places

cerebral palsy types

spastic - jerky movements, have seizures, almost in spasm, hard for them to control ataxic - they are floppy, reflexes are greatly delayed

radiographic stages of legg-calve-perthes disease

stage 1 - aseptic necrosis or infarction of the femoral capital epiphysis with degenerative changes producing flattening of the upper surface of the femoral head ( the avascular stage) stage 2 - capital bone resorption and revasculization with fragmentation that gives a mottled appearance on radiographs ( the fragmentation or revasculization stage) stage 3 - new bone formation, which is represented on radiographs as calcification and ossification or increased density in the areas of radiolucency ( the reparative stage) stage 4 - gradual reformation of the head of the femur without radiolucency and it is hoped to a sperical form ( the regenerative stage)

maintenance of function include /Duchenne muscular dystrophy

stretching exercse, strength and muscle training breathing exercise use of incentive spirometry airway clearance range of motion exercise surgery to release contracture deformities bracing performace of ADL knee-ankle-foot orthoses serial casting of ankles encourage vaccines ( flu and pneumonia) avoid people with respiratory gract infection long term ventilation regular test pulmonary function, ECG, check cardiac and respiratory during wake sleep cycles if child has sleep disordered breathing - polysomnography ( once daily), positive pressure ventilation

Crutches, Canes, and Walkers

support balance

pearson attachment

supports lower leg, and provides increased stability in the overall traction setup, it does not prevent flexion deformities in the ankle or foot, not attached to skeletal pin

Stress fractures

symptoms -result of repeated muscle contraction and occur most often in sports involving repetitive weight bearing such as running, gymnastics - Sx: Sharp, persistent, progressive pain or a deep, persistent dull ache over the bone - Tx: Rest, alleviating whatever repetitive stress caused the symptoms, alt. exercise because continued mobility and training is good cryotherapy, cold whirpool bath, taping, bracing, splinting, NSAID

signs of exertion

tachycardia palpitation tachypnea dyspnea hyperpnea dizzines lightheadedness diaphoresis change in skin color fatigued child - sagging limp posture, slow strained movements, inability to tolerate additionla activity

guidelines to transfusion

take vital signs before administering blood, 15 min after beginning, hourly while infusing and on completion 2. check blood type and group of the recepient against donor's 3. administer the first 50 ml or initial 20% slowly and stay with the child 4. administer with normal saline in a piggyback 5. administer blood through appropriate filter to eleminate particles 6. use blood within 30 min of its arrival from the blood bank, if not used return to blood bank 7. infuse a unit of blood within 4 hrs 8. if a reaction suspected, stop the infusion, take vitals, maintain patent IV line with normal saline, notify HCP and do not restart until condition reevaliated

clinical manifestation of increased intracranial pressure in infants

tense bulging fontanels separated cranial suturs macewen (cracked pot) sign irritability and restlessness frowsiness increased sleeping high pitched cry increased frontoocipital circumference distended scalp veins poor feeding crying when disturbed setting sun sign

The child's reaction to illness or disability depends on

the child's developmental level, coping mechanisms, reactions of others, and the illness itself

tentorial hiatus

the large gap through which the brainstem passes; the site of herniation in untreated increased intracranial pressure

corpus callosum

the largest fiber bundle in the brain, the central part of the cerebral hemispheres, interconnects cortical areas of the right and left hemispheres

complications of bacterial meningitis

thick pus, fibrin, or adhesions may occlude the narrow passages, obstructing the flow of CSF, causing obstructive hydrocephalus subdural effusion thrombosis in meningeal veins brain absesses extention of infectiom to the cranial nervs compression necrosis from increased pressure -> deafness, blindness, weakness or paralysis of facial or other muscles of the head/neck meningococcal sepsis/ meningococcemia SIADH subdurasl effusion seizures cerebral edema herniation hydrocephalus

occupational therapy in CP

to train skills of ADL according to normal development incorporated into play indoor and outdoor activities (sports) provide rest due to spent much energy to accomplush ADLs ensure adequate caloric intake, they need more calories due to spasticity and spent energy for it (oral or NG) safety measures to prevent falls, like helmets immunizations

leading cause of death in children older than 1

trauma

child's response to trauma depends on

undetected congenital anomaly temperature unstable due to large surface area rapid metabilic chnages small volume of blood 1. ABC 2. toe-to head systemic assessent

ilizarov external fixator

wires, rings telescoping rods that permit the lim lengthening by manual distraction corrects angular or rotational deformities immobilize fractures


Kaugnay na mga set ng pag-aaral

A - Min Net Capital Dollar Requirements

View Set

Behaviors- Interpersonal Violence

View Set